CBSE Question Breakdown

¡Supera tus tareas y exámenes ahora con Quizwiz!

unstable angina

chest pain that occurs while a person is at rest and not exerting himself

A 44-year-old man with HIV comes to the physician for a routine follow-up examination. He has been noncompliant with his antiretroviral medication regimen for several years. He appears chronically ill and fatigued. CD4+ T-lymphocyte count is 405/mm3. Further evaluation of this patient is most likely to show which of the following findings?

This patient has a CD4 T-cell count below 500/mm3, but greater than 200/mm3, putting him at risk for several opportunistic infections, including oral hairy leukoplakia, oral thrush, HHV-8 infection, and squamous cell carcinoma Violaceous lesions on skin exam 37% Violaceous skin lesions in a patient with HIV describe Kaposi sarcoma. HIV patients are at a higher risk of Kaposi sarcoma at variable CD4 counts, even above 400/mm3.

Giemsa stain

Chlamydia, Borrelia, Rickettsia, Trypanosomes, Plasmodium

Freidrich ataxia genetics

Friedreich ataxia An autosomal recessive disorder involving trinucleotide repeat expansion that leads to progressive neurodegeneration. It affects multiple spinal cord tracts, causing muscle weakness and impaired coordination of all limbs. A staggering gait in childhood is the resulting main symptom. Other features include skeletal abnormalities, cardiomyopathy, and diabetes.

A 67-year-old woman who was recently diagnosed with Crohn disease comes to the physician for evaluation of her immunosuppressive therapy. She has had recurrent flares since her diagnosis. Physical examination shows two shallow ulcers on her oral mucosa. The physician considers adding azathioprine to her medication regimen. A deficiency of which of the following enzymes would diminish the therapeutic effect of this drug?

Hypoxanthine-guanine phosphoribosyl transferase (HGPRT) converts free purine bases (guanine and hypoxanthine) into purine nucleotides (GMP and IMP). It also converts the azathioprine metabolite 6-MP into thio-inosine monophosphate, which is the molecule directly responsible for the drug's cytotoxic effects. A deficiency of HGPRT would lead to a decreased conversion of 6-MP into its active metabolite, thus limiting the effectiveness of the drug.

A 32-year-old woman comes to the physician because of a 4-day history of low-grade fever, joint pain, and muscle aches. The day before the onset of her symptoms, she was severely sunburned on her face and arms during a hike with friends. She also reports being unusually fatigued over the past 3 months. Her only medication is a combined oral contraceptive pill. Her temperature is 37.9°C (100.2°F). Examination shows bilateral swelling and tenderness of the wrists and metacarpophalangeal joints. There are multiple nontender superficial ulcers on the oral mucosa. The detection of antibodies directed against which of the following is most specific for this patient's condition

Nuclear Sm proteins Antibodies to nuclear Sm proteins (i.e., anti-Smith antibodies) are a highly specific marker for SLE. However, they are only positive in 30% of patients, meaning they are not a very sensitive marker. For this reason, the diagnosis of SLE is always based on clinical features as well as immunologic test results. Another highly specific antibody for the diagnosis of SLE targets double-stranded DNA. Anti-dsDNA antibodies are present in 70% of patients with SLE (high sensitivity), correlate with disease activity, and are associated with lupus nephritis.

carotid body

chemoreceptor located in the carotid artery that detects changes in oxygen, carbon dioxide, and blood acid levels

Stable angina

chest pain that occurs when a person is active or under severe stress

Akimbo test

Deltoid muscle test

Hemangioma

hemangioma or haemangioma is a usually benign vascular tumor derived from blood vessel cell types.

P wave

atrial depolarization

Lacunar stroke

microinfarctions; due to hyaline arteriolosclerosis (hypertension, diabetes)

Violaceous

Purple or violet discoloration

Escitalopram

SSRI

Sertraline

Zoloft SSRI

Adenovirus

common cold

corticospinal tract

motor

Taenia solium

pork tapeworm

rhinorrhea

runny nose

Hematoma

tumor of blood

Ghon complex

TB granulomas (Ghon focus) w/ lobar and perihilar lymph node involvement. Reflects primary infxn or exposure.

maldorous

having an offensive odor

myalgia

muscle pain

preeclampsia

a complication of pregnancy characterized by hypertension, edema, and proteinuria

primigravida

a woman who is pregnant for the first time

transmural

affecting the entire thickness of a wall

APGAR scoring

an assignment of numbers to assessment parameters of a newborn at 1 and 5 minutes after birth, named after its developer, Virginia Apgar

adenopathy

disease of a gland

lymphadenopathy

disease of the lymph nodes

Proptosis

displacement of the eye from the orbit

Councilman bodies

dying hepatocytes Councilman body An eosinophilic remnant of apoptotic hepatocytes seen on liver biopsy. Associated with yellow fever and viral hepatitis.

Olecranon

elbow

blastic

embryonic

polyhydramnios

excessive amniotic fluid

Astrocytes

form blood brain barrier

Chloride

gaba

zona fasciculata

glucocorticoids

aortic valve

heart valve between the left ventricle and the aorta

Fasiculations

involuntary twitching of muscle fibers

ataxia

lack of muscle coordination

Agnosis

loss of ability to recognize importance of sensory impressions

fundal height

measurement of the size of the uterus

erythrocyte sedimentation rate

measures time it takes for erythrocytes to settle to the bottom of a test tube

pallor

paleness

Tibia

shin bone

-gravida

term gravida comes from the Latin word gravidus. It is used to describe a woman who is pregnant and is also a medical term for the total number of confirmed pregnancies a woman has had, regardless of the outcome of the pregnancy.

blanch

to whiten; to make pale

Tachypneic

very rapid respiration

endoscopy

visual examination within a hollow organ

Astroglial cells

Astrocytes help form the physical structure of the brain, and are thought to play a number of active roles, including the secretion or absorption of neural transmitters and maintenance of the blood-brain barrier.[13] The concept of a tripartite synapse has been proposed, referring to the tight relationship occurring at synapses among a presynaptic element, a postsynaptic element and a glial element.[14] Structural: They are involved in the physical structuring of the brain. Astrocytes get their name because they are "star-shaped". They are the most abundant glial cells in the brain that are closely associated with neuronal synapses. They regulate the transmission of electrical impulses within the brain. Glycogen fuel reserve buffer: Astrocytes contain glycogen and are capable of gluconeogenesis. The astrocytes next to neurons in the frontal cortex and hippocampus store and release glucose. Thus, astrocytes can fuel neurons with glucose during periods of high rate of glucose consumption and glucose shortage. A recent research on rats suggests there may be a connection between this activity and physical exercise.[15] Metabolic support: They provide neurons with nutrients such as lactate. Glucose sensing: normally associated with neurons, the detection of interstitial glucose levels within the brain is also controlled by astrocytes. Astrocytes in vitro become activated by low glucose and are in vivo this activation increases gastric emptying to increase digestion.[16] Blood-brain barrier: The astrocyte end-feet encircling endothelial cells were thought to aid in the maintenance of the blood-brain barrier, but recent research indicates that they do not play a substantial role; instead, it is the tight junctions and basal lamina of the cerebral endothelial cells that play the most substantial role in maintaining the barrier.[17] However, it has recently been shown that astrocyte activity is linked to blood flow in the brain, and that this is what is actually being measured in fMRI.[18][19] Transmitter uptake and release: Astrocytes express plasma membrane transporters such as glutamate transporters for several neurotransmitters, including glutamate, ATP, and GABA. More recently, astrocytes were shown to release glutamate or ATP in a vesicular, Ca2+-dependent manner.[20] (This has been disputed for hippocampal astrocytes.)[21] Regulation of ion concentration in the extracellular space: Astrocytes express potassium channels at a high density. When neurons are active, they release potassium, increasing the local extracellular concentration. Because astrocytes are highly permeable to potassium, they rapidly clear the excess accumulation in the extracellular space.[22] If this function is interfered with, the extracellular concentration of potassium will rise, leading to neuronal depolarization by the Goldman equation. Abnormal accumulation of extracellular potassium is well known to result in epileptic neuronal activity.[23] Modulation of synaptic transmission: In the supraoptic nucleus of the hypothalamus, rapid changes in astrocyte morphology have been shown to affect heterosynaptic transmission between neurons.[24] In the hippocampus, astrocytes suppress synaptic transmission by releasing ATP, which is hydrolyzed by ectonucleotidases to yield adenosine. Adenosine acts on neuronal adenosine receptors to inhibit synaptic transmission, thereby increasing the dynamic range available for LTP.[25] Vasomodulation: Astrocytes may serve as intermediaries in neuronal regulation of blood flow.[26] Promotion of the myelinating activity of oligodendrocytes: Electrical activity in neurons causes them to release ATP, which serves as an important stimulus for myelin to form. However, the ATP does not act directly on oligodendrocytes. Instead, it causes astrocytes to secrete cytokine leukemia inhibitory factor (LIF), a regulatory protein that promotes the myelinating activity of oligodendrocytes. This suggests that astrocytes have an executive-coordinating role in the brain.[27] Nervous system repair: Upon injury to nerve cells within the central nervous system, astrocytes fill up the space to form a glial scar, and may contribute to neural repair. The role of astrocytes in CNS regeneration following injury is not well understood though. The glial scar has traditionally been described as an impermeable barrier to regeneration, thus implicating a negative role in axon regeneration. However, recently, it was found through genetic ablation studies that astrocytes are actually required for regeneration to occur.[28] More importantly, the authors found that the astrocyte scar is actually essential for stimulated axons (that axons that have been coaxed to grow via neurotrophic supplementation) to extend through the injured spinal cord.[28] Astrocytes that have been pushed into a reactive phenotype (termed astrogliosis, defined by upregulation of GFAP expression, a definition still under debate) may actually be toxic to neurons, releasing signals that can kill neurons.[29] Much work, however, remains to elucidate their role in nervous system injury. Long-term potentiation: Scientists debate whether astrocytes integrate learning and memory in the hippocampus. Recently it has been shown that engrafting human glial progenitor cell in the nascent mice brains will cause the cells to differentiate into astrocytes. After differentiation these cells increase LTP and improve memory performance in the mice.[30] Circadian clock: Astrocytes alone are sufficient to drive the molecular oscillations in the SCN and circadian behavior in mice, and thus can autonomously initiate and sustain complex mammalian behavior.[31] The switch of the nervous system: Based on the evidence listed below, it has been recently conjectured in,[32] that macro glia (and astrocytes in particular) act both as a lossy neurotransmitter capacitor and as the logical switch of the nervous system. I.e., macroglia either block or enable the propagation of the stimulus along the nervous system, depending on their membrane state and the level of the stimulus.

vestibular nerve

CN VIII, nerve that conducts impulses related to maintaining balance to the brain

Achilles tendon reflex

S1

Fibula

The lateral and smaller bone of the lower leg

induration

abnormal hard spots

Naproxen

Aleve Nonsteroidal anti-inflammatory drug It can treat fever and pain.

pulse oximetry

An assessment tool that measures oxygen saturation of hemoglobin in the capillary beds.

Metanephrine

Product of epinephrine and norepinephrine metabolism

Glucose challenge

Screening test for gestational diabetes

Infliximab

"Inflicts pain on TNFalpha" Really very similar to Etanercept, but this actually binds TNFalpha itself Chimeric aby that binds TNFa-->inhibits binding of TNFa with its receptor-->decreased other cytokines released-->anti-inflammation Uses = autoimmune diseases (RA, IBD, etc) SE = increased infection susceptibility Adalimumab--same really, just monoclonal not chimeric

thrill

A thrill is a vibratory sensation felt on the skin overlying an area of turbulence and indicates a loud heart murmur usually caused by an incompetent heart valve.

Stroma

A type of cell that makes up certain types of connective tissue (supporting tissue that surrounds other tissues and organs).

A 71-year-old man comes to the physician because of involuntary shaking and gait abnormalities for the past year. He also reports depressed mood and diminished smell and taste. Examination shows a shuffling gait, resting tremor in the left hand, and rigidity in both arms. This patient most likely has structural changes in which of the following labeled areas of the brain?

B 78% The substantia nigra pars compacta contains dopaminergic neurons that transmit signals involved in movement regulation and is the part of the basal ganglia affected in Parkinson disease. Progressive degeneration of these neurons results in a clinical syndrome characterized by bradykinesia/akinesia, resting tremor, rigidity, and gait abnormalities, as seen in this patient. Additional findings include sensory deficits, personality changes, and dementia.

Haptoglobin

Binds free hemoglobin after RBC lysis to be destroyed by spleen, decreased levels in sickle cell, thalessemia, lead poisoning,etc.

Fick principle

CO = rate of O2 consumption / (arterial O2 content - venous O2 content)

Carbidopa/Levodopa

Levodopa is converted to dopamine via the action of a naturally occurring enzyme called DOPA decarboxylase. This occurs both in the peripheral circulation and in the central nervous system after levodopa has crossed the blood brain barrier. Activation of central dopamine receptors improves the symptoms of Parkinson's disease; however, activation of peripheral dopamine receptors causes nausea and vomiting.

Rhonci

Rhonchi are continuous low pitched, rattling lung sounds that often resemble snoring. Obstruction or secretions in larger airways are frequent causes of rhonchi. They can be heard in patients with chronic obstructive pulmonary disease (COPD), bronchiectasis, pneumonia, chronic bronchitis, or cystic fibrosis.

Teres minor

axillary nerve Rotates arm laterally

ptosis

drooping

friable

easily crumbled

Uriticaria

hives

fundus of uterus

Most superior & widest portion of uterus

A 62-year-old woman comes to the physician for decreased vision and worsening headaches since this morning. She has hypertension and hypercholesterolemia. Pulse is 119/min and irregular. Current medications include ramipril and atorvastatin. Ocular and funduscopic examination shows no abnormalities. The findings of visual field testing are shown. Which of the following is the most likely cause of this patient's symptoms?

Occlusion of the posterior cerebral artery 76% Occlusion of the posterior cerebral artery, usually due to emboli from the vertebral vessels, aorta, or heart, results in contralateral homonymous hemianopia with macular sparing. The macula is spared because the macular tracts have additional blood supply from the middle cerebral arteries.

fremitus

Rhonchal fremitus, also known as bronchial fremitus, is a palpable vibration produced during breathing caused by partial airway obstruction. The obstruction can be due to mucus or other secretions in the airway, bro

A 51-year-old man comes to the physician because of a 4-day history of fever and cough productive of foul-smelling, dark red, gelatinous sputum. He has smoked 1 pack of cigarettes daily for 30 years and drinks two 12-oz bottles of beer daily. An x-ray of the chest shows a cavity with air-fluid levels in the right lower lobe. Sputum culture grows gram-negative rods. Which of the following virulence factors is most likely involved in the pathogenesis of this patient's condition?

This patient's sputum culture with gram-negative rods and history of foul-smelling, dark red, gelatinous sputum (currant jelly sputum) is consistent with a pulmonary infection caused by Klebsiella pneumoniae. Klebsiella pneumoniae possess a capsular polysaccharide that acts as an antiphagocytic virulence factor and allows bacteria to evade host immune defenses. This pathogen is part of the natural flora of the gastrointestinal tract. In patients who are immunocompromised and patients who have an increased risk for aspiration events (e.g., due to chronic alcohol use), aspiration of Klebsiella can lead to destruction of alveoli, resulting in bloody sputum with a currant jelly appearance.

slit-lamp examination

stereoscopic magnified view of the anterior eye structures in detail, which includes the cornea, lens, iris, sclera, and vitreous humor

ataxia

the loss of full control of bodily movementsf

photocoagulation

the use of lasers to treat some forms of wet macular degeneration by sealing leaking or damaged blood vessels

Hypotonia/flaccidity

Loose, flexible; weak muscle tone

Barium esophagography

Viewing upper GI w/ contrast, use ____.

icterus

jaundice

spondylarthropathy

joint disease of the vertebrae

Merkel's disks

pressure, position receptors contrast with meissner's, pacinian rapidly adapting responds to most changes

osteopenia

thinner than average bone density

parietal lobe

receives sensory input for touch and body position

polysomnography

recording multiple aspects of sleep

hydrocele

sac of clear fluid in the scrotum

Romberg test

-ask client to stand with feet at comfortable distance apart, arms at sides, and eyes closed -expected finding: client should be able to stand with minimal swaying for at least 5 seconds

Normal levels AST ALT

AST 10-40 ALT 7-56 Mild elevation is 2-3 times higher than normal levels

subacute combined degeneration SCD • Classic presentation • Problems walking • Positive Romberg • Spastic paresis in legs • Lower extremity hyperreflexia • Positive Babinski

B12 deficiency → degeneration of dorsal columns, LCST, spinocerebellar tracts (abnormal myelin from lack of methylmalonyl-CoA mutase activity)

Carbamazepine

Carbamazepine is a sodium channel blocker.[33] It binds preferentially to voltage-gated sodium channels in their inactive conformation, which prevents repetitive and sustained firing of an action potential. Carbamazepine has effects on serotonin systems but the relevance to its antiseizure effects is uncertain. There is evidence that it is a serotonin releasing agent and possibly even a serotonin reuptake inhibitor.[34][35][36]

Colchine

Colchicine is a classical anti-mitotic drug which blocks mitotic cells in metaphase. It binds to soluble tubulin to form tubulin-colchicine complexes in a poorly reversible manner, which then binds to the ends of microtubules to prevent the elongation of the microtubule polymer

Tabes dorsalis

Degeneration of dorsal columns and dorsal roots due to tertiary syphilis, resulting in impaired proprioception and locomotor ataxia. Associated with charcot's joints, shooting (lightning pain) (see image 12 in First Aid), Argyll Robertson upils (Reactive to accommodation but not light), absence of DTRs, positive romberg, and sensory ataxia at night.

dilated cardiomyopathy

Dilated cardiomyopathy (DCM) is a condition in which the heart becomes enlarged and cannot pump blood effectively.

Brushfield spots (on the iris)

Down's syndrome

Erythema multiforme

Erythema multiforme is a skin reaction that can be triggered by an infection or some medicines. It's usually mild and goes away in a few weeks. There's also a rare, severe form that can affect the mouth, genitals and eyes and can be life-threatening. This is known as erythema multiforme major. The cause of erythema multiforme is unknown, but it appears to be an allergic reaction that occurs in response to medications, infections, or illness. As noted above, it often appears in association with herpes simplex virus or with infectious organisms such as Mycoplasma pneumoniae.

satellite phenomenon

Haemophilus influenzae can grow on blood agar media with Staphylococcus aureus which can provide factor V as it is called "Satellite phenomenon

Grade 3/6 murmur

III/VI: Loud murmur without a palpable thrill

Atropine

In general, atropine counters the "rest and digest" activity of glands regulated by the parasympathetic nervous system. This occurs because atropine is a competitive, reversible antagonist of the muscarinic acetylcholine receptors (acetylcholine being the main neurotransmitter used by the parasympathetic nervous system). Atropine is a competitive antagonist of the muscarinic acetylcholine receptor types M1, M2, M3, M4 and M5.[28] It is classified as an anticholinergic drug (parasympatholytic).

Spongiotic dermatitis

Inercellular epidermal edema that increases width between cells - eczema

nystagmus

Involuntary rapid eye movements

leukocyte esterase

Leukocyte esterase (LE) is an esterase (a type of enzyme) produced by leukocytes (white blood cells). A leukocyte esterase test (LE test) is a urine test for the presence of white blood cells and other abnormalities associated with infection.

Angiotensin-converting enzyme (ACE) inhibitors help relax your veins and arteries to lower your blood pressure. ACE inhibitors prevent an enzyme in your body from producing angiotensin II, a substance that narrows your blood vessels. This narrowing can cause high blood pressure and force your heart to work harder.

Lisonopril or the prils

Baum's loop

Part of visual pathway In Parietal lobe Damage--> quadrantic anopia (Pie in the floor)

chocolate agar

Neisseria and Hemophilus

neocortex (cerebral cortex)

Newest, outer layer ("new bark") of the forebrain, composed of about six layers of gray matter; creates our reality.

Lateral decubitus position

Patient lying on the affected side Detects small pleural effusions

Pilocytic astrocytomas

Pilocytic astrocytoma is a benign brain tumor that arises from astrocytes, the supportive cells in the nervous system. These slow-growing tumors usually occur in children, and are considered the most benign type of astrocytoma.

Metanephrine

Product of epinephrine and norepinephrine metabolism Metanephrines are made when your body breaks down hormones called catecholamines. These hormones are made by the adrenal glands. Catecholamines help your body respond to stress. They are sometimes called "fight or flight" hormones. They also include epinephrine, norepinephrine, and dopamine.

Sarcoidosis

Sarcoidosis is a disease involving abnormal collections of inflammatory cells that form lumps known as granulomata.

Sialometry

Sialometry is a measure of saliva flow. Many different techniques have been devised, yet none are perfect. Four major salivary glands and innumerable minor salivary glands generate saliva.

pudendal nerve

Supplies sensation to external genitalia and perineum (external hemorrhoids, below dentate line) The pudendal nerve is the main nerve of the perineum. It carries sensation from the external genitalia of both sexes and the skin around the anus and perineum, as well as the motor supply to various pelvic muscles, including the male or female external urethral sphincter and the external anal sphincter.

muscle tapping

Tapping is the use of a light force applied manually over a tendon or muscle belly to facilitate a voluntary contraction". Tapping is used to assess reflex activity with a normal response being a brisk muscle contraction. Rood recommended three to five taps over the muscle belly to be facilitated.

Doxycycline

Tetracycline Doxycycline is a broad spectrum antibiotic. It inhibits the synthesis of bacterial proteins by binding to the 30S ribosomal subunit, which is only found in bacteria

ST segment

The ST segment is the flat, isoelectric section of the ECG between the end of the S wave (the J point) and the beginning of the T wave. The ST Segment represents the interval between ventricular depolarization and repolarization.

abductor pollicis brevis

The abductor pollicis brevis is a flat, thin muscle located just under the skin. It is a thenar muscle, and therefore contributes to the bulk of the palm's thenar eminence.

internal pundendal artery

The internal pudendal artery is one of the three pudendal arteries that branches off the internal iliac artery, providing blood to the external genitalia.

pronator teres

The pronator teres is a muscle (located mainly in the forearm) that, along with the pronator quadratus, serves to pronate the forearm (turning it so that the palm faces posteriorly when from the anatomical position).

hydrochlorothiazide

Treats hypertension at the distal convoluted tubule resulting in sodium and water secretion to decrease blood pressure

macular sparing

Vascular lesion of PCA or branches which causes loss of vision except for macula due to sufficient collateral blood flow from the MCA.

Neutropenia

a decreased number of neutrophils

aphasia

impairment of language, usually caused by left hemisphere damage either to Broca's area (impairing speaking) or to Wernicke's area (impairing understanding).

cranial nerve 3

oculomotor nerve

monomeric

one polypeptide chain

A 72-year-old man comes to the physician with chills, nausea, and diffuse muscle aches for 3 days. His niece had similar symptoms 2 weeks ago and H3N2 influenza strain was isolated from her respiratory secretions. He received his influenza vaccination 2 months ago. His temperature is 38°C (100.4°F). A rapid influenza test is positive. Which of the following mechanisms best explains this patient's infection despite vaccination?

Antigenic drift is most likely the cause of the this patient's infection despite appropriate vaccination. It is typically the cause of influenza epidemics. Random point mutations within viral genome Random point mutations within the viral genome are responsible for antigenic drift, which creates a new virus strain. In some cases, although the patient has been vaccinated against the previous strain, the new strain will have different antigenic properties that can evade the host immune response and cause an infection, as observed in this patient

Inducers of CYP450

Rifampin, phenobarbital, carbamazepine, phenytoin, St. Johns wort, Theophylline, Nafcillin, and Modafinil inducer of CYP450. When taken concurrently with phenytoin, patients will have subtherapeutic levels of phenytoin and likely present with poorly controlled seizures.

heterophile agglutination test

The mononuclear spot test or monospot test, a form of the heterophile antibody test,[1] is a rapid test for infectious mononucleosis due to Epstein-Barr virus (EBV).

A 2-year-old boy with a history of multiple hospitalizations for fever and infection undergoes immunologic evaluation. Serum CH50 assay shows inappropriately low erythrocyte lysis and further workup confirms C8 deficiency. This patient is at increased risk for recurrent infections with which of the following pathogens?

Vaccination and prophylactic antibiotics are the recommended treatment for patients with terminal complement deficiencies (C5-C9). Neisseria species Terminal complement deficiencies (C5-C9) impede the formation of the membrane attack complex, which renders affected patients more susceptible to Neisseria bacteremia (e.g., N. meningitidis and N. gonorrhoeae pyogenic infections).

disseminated intravascular coagulation

abnormal activation of the proteins involved in blood coagulation, causing small blood clots to form in vessels and cutting off the supply of oxygen to distal tissues Under homeostatic conditions, the body is maintained in a finely tuned balance of coagulation and fibrinolysis. The activation of the coagulation cascade yields thrombin that converts fibrinogen to fibrin; the stable fibrin clot being the final product of hemostasis. The fibrinolytic system then functions to break down fibrinogen and fibrin. Activation of the fibrinolytic system generates plasmin (in the presence of thrombin), which is responsible for the lysis of fibrin clots. The breakdown of fibrinogen and fibrin results in polypeptides called fibrin degradation products (FDPs) or fibrin split products (FSPs). In a state of homeostasis, the presence of plasmin is critical, as it is the central proteolytic enzyme of coagulation and is also necessary for the breakdown of clots, or fibrinolysis.[citation needed] In DIC, the processes of coagulation and fibrinolysis are dysregulated, and the result is widespread clotting with resultant bleeding. Regardless of the triggering event of DIC, once initiated, the pathophysiology of DIC is similar in all conditions. One critical mediator of DIC is the release of a transmembrane glycoprotein called tissue factor (TF

dorsum of hand

top of hand

Acantholysis

Acantholysis is the loss of intercellular connections, such as desmosomes, resulting in loss of cohesion between keratinocytes, seen in diseases such as pemphigus vulgaris. It is absent in bullous pemphigoid, making it useful for differential diagnosis.

nystagmus

involuntary, jerking movements of the eyes An involuntary eye movement which may cause the eye to rapidly move from side to side, up and down, or in a circle, and may slightly blur vision.

effusion

outpouring

A-alpha fibers

proprioception, somatic motor

somnolent

sleepy

sonography

ultrasound imaging

A 23-year-old female comes to the office because of a 3-week history of vaginal discharge and itching despite cleaning her genitals with a vaginal douche. Her last menstrual period was one week ago. She is sexually active with her new boyfriend. She has an intrauterine device and does not use barrier protection. She was treated for a sore throat infection one month ago. Speculum examination shows erythema around the vaginal introitus and copious white discharge. Vaginal pH is 4.3 and a KOH test shows multiple pseudohyphae on microscopy. Which of the following is the strongest predisposing factor for this patient's condition?

Multiple pseudohyphae on wet mount preparation are characteristic of a Candida infection. Suppression of vaginal bacterial flora 80% Systemic antibiotic treatment (e.g., oral, intravenous) causes suppression of normal vaginal bacterial flora, which promotes C. albicans overgrowth. Antibiotics increase the incidence of symptomatic vulvovaginitis and vaginal colonization with C. albicans in asymptomatic patients. A normal vaginal pH (< 4.5) and pseudohyphae on microscopic examination of a KOH test confirms the diagnosis. This patient was likely treated with antibiotics for bacterial pharyngitis.

Epilepsy

seizure disorder Normally brain electrical activity is non-synchronous, as neurons do not normally fire in sync with each other, but rather fire in order as signals travel throughout the brain.[2] Its activity is regulated by various factors both within the neuron and the cellular environment. Factors within the neuron include the type, number and distribution of ion channels, changes to receptors and changes of gene expression.[55] Factors around the neuron include ion concentrations, synaptic plasticity and regulation of transmitter breakdown by glial cells.[55][56] Chronic inflammation also appears to play a role.[57]

gall bladder

stores bile

stridor

strained, high-pitched sound heard on inspiration caused by obstruction in the pharynx or larynx

Aldesleukin

Proleukin Antineoplastic agent, biological response modulator, interleukin-2 recombinant

CH50 assay

test for the overall deficiencies of the classical pathway

perineum

the area between the anus and the scrotum or vulva.

epitheloid cells

the characteristic giant cell found in a tuberculosis lesion modified macrophages

thenar eminence

the fleshy mass at the base of the thumb

ablation

the removal of a body part or the destruction of its function

fremitus

vibration of the chest wall produced by vocalization.

hemiparesis

weakness on one side of the body

macula

yellowish region on the retina lateral to and slightly below the optic disc

Right Flank Region

• Right lateral region • Contents include parts of small and large intestine, and right kidney

Sideroblast

Sideroblasts (sidero- + -blast) are nucleated erythroblasts (precursors to mature red blood cells) with granules of iron accumulated in the mitochondria surrounding the nucleus. Normally, sideroblasts are present in the bone marrow, and enter the circulation after maturing into a normal erythrocyte. Ring sideroblasts are named so because iron-laden mitochondria form a ring around the nucleus. It is a subtype of basophilic granules of the erythrocyte, but which can only be seen in bone marrow.

Gallant reflex

Stroking along one side of the spine while newborn is in ventral suspension (face down) causes lateral flexion of lower body toward stimulated side Galant reflex, or truncal incurvation reflex, is a newborn reflex, named after neurologist Johann Susmann Galant. It is elicited by holding the newborn in ventral suspension (face down) and stroking along the one side of the spine. The normal reaction is for the newborn to laterally flex toward the stimulated side. This is one of the reflexes tested in newborns to help rule out brain damage at birth.

pectinate line

Structure that separates internal and external hemorrhoids

Rhabditiform larvae

noninfective, feeding, first-stage larvae; the larvae have an hourglass-shaped esophagus The rhabditid nematode (roundworm) Strongyloides stercoralis is the major causative agent of strongyloidiasis in humans. Rarer human-infecting species of Strongyloides are the zoonotic S. fuelleborni (fülleborni) subsp. fuelleborni and S. fuelleborni subsp. kellyi, for which the only currently known host is humans. Strongyloides spp. are sometimes called "threadworms" (although in some countries this common name refers to Enterobius vermicularis).

transilluminate

pass strong light through (an organ or part of the body) in order to detect disease or abnormality.

PCR uses

DNA dependent DNA polymerases Clone DNA for recombination Amplify DNA to detectable levels Sequence DNA Diagnose genetic disease Detect pathogens

A 1-week-old male newborn is brought to the physician for a follow-up examination after the results of newborn screening showed an increased serum concentration of phenylalanine. Genetic analysis confirms a diagnosis of phenylketonuria. The physician counsels the patient's family on the recommended dietary restrictions, including avoidance of artificial sweeteners that contain aspartame. Aspartame is a molecule composed of aspartate and phenylalanine and its digestion can lead to hyperphenylalaninemia in patients with phenylketonuria. Which of the following enzymes is primarily responsible for the breakdown of aspartame?

Dipeptidase is a brush border enzyme in the human duodenum, jejunum, and ileum. It is a type of exopeptidase that hydrolyzes dipeptides such as aspartame.

missense mutation

A base-pair substitution that results in a codon that codes for a different amino acid.

ureter

A duct leading from the kidney to the urinary bladder.

temporal lobe

A region of the cerebral cortex responsible for hearing and language.

-ptosis

Abnormal low-lying or drooping upper eyelid.

Tachypnea

Abnormal rapid breathing

Bibasilar

Both lungs

A 52-year-old woman with HIV infection is brought to the emergency department 20 minutes after she had a generalized tonic-clonic seizure. She appears lethargic and confused. Laboratory studies show a CD4+ count of 89 cells/μL (N > 500). A CT scan of the head with contrast shows multiple ring-enhancing lesions in the basal ganglia and subcortical white matter. An India ink preparation of cerebrospinal fluid is negative. Which of the following is the most likely diagnosis?

Cerebral toxoplasmosis In an immunocompromised patient, multiple ring enhancing lesions within the subcortical white matter, basal ganglia, and/or thalamus are highly suggestive of cerebral toxoplasmosis. Toxoplasma gondii is an obligate intracellular parasite, contracted via ingestion of cysts in raw meat or cat feces. In immunocompetent patients, the presentation is usually asymptomatic. Cerebral toxoplasmosis is treated with a combination of sulfadiazine and pyrimethamine.

A 49-year-old woman is admitted to the hospital for the evaluation of postprandial colicky pain in the right upper quadrant of the abdomen. Abdominal ultrasound shows multiple round, hyperechoic structures within the gallbladder lumen. She undergoes a cholecystectomy. A photograph of the content of her gallbladder is shown. This patient is most likely to have which of the following additional conditions?

Chronic hemolytic anemia Black pigment gallstones are associated with chronic hemolytic anemia (e.g., due to hereditary spherocytosis or beta thalassemia major), in which an elevated red blood cell breakdown leads to an increased serum concentration of indirect bilirubin. Excess bilirubin is excreted by the liver into the bile. The excess bilirubin then precipitates within the gallbladder due to supersaturation and reacts with calcium salts to form black calcium bilirubinate stones. Black pigment stones account for approx. 10% of all gallstones.

etoposide

Etoposide forms a ternary complex with DNA and the topoisomerase II enzyme (which aids in relaxing negative or positive supercoils in DNA), prevents re-ligation of the DNA strands, and by doing so causes DNA strands to break.[7] Cancer cells rely on this enzyme more than healthy cells, since they divide more rapidly. Therefore, this causes errors in DNA synthesis and promotes apoptosis of the cancer cell.[5][8]

A 21-year-old woman comes to the physician because of hair loss on her frontal scalp over the past year. Menses have occurred at irregular 40- to 60-day intervals since menarche at the age of 17 years. She has no history of serious illness and takes no medications. She is 162 cm (5 ft 3 in) tall and weighs 73 kg (158.7 lb); BMI is 28 kg/m2. Her pulse is 75/min and blood pressure 130/76 mm Hg. Physical examination shows scattered pustules on her face and patches of velvety hyperpigmentation on her axilla and groin. Her morning serum cortisol concentration is 18 μg/dL. This patient's condition is most likely associated with increased stimulation of which of the following types of cells?

Frontal hair loss, menstrual irregularities, overweight, acne vulgaris (scattered facial pustules), and acanthosis nigricans (velvety hyperpigmentation on axilla and groin) suggest polycystic ovary syndrome.

first dorsal interosseous

Function. The dorsal interossei abduct the index, middle, and ring fingers. The first dorsal interosseous is also able to rotate the index finger slightly at the metacarpophalangeal joint and assist adductor pollicis in thumb adduction.

A 22-year-old man comes to the physician because of headaches and blurry vision for the past 6 months. He also reports frequent episodes of vomiting over the last month. His father has died of renal cell carcinoma at the age of 37 years. Examination shows 20/40 vision bilaterally. Fundoscopic examination shows bilateral optic disc swelling and growth of capillary vessels in the temporal peripheral retina. An MRI of the brain shows an infratentorial mass. The patient undergoes surgical resection of the mass. A photomicrograph of the resected specimen is shown. Which of the following is the most likely diagnosis? This patient presents with an infratentorial brain tumor and abnormal capillary growth in the retina. In combination with a family history of renal cell carcinoma, this raises suspicion for von Hippel-Lindau disease. Von Hippel-Lindau disease (VHL), also known as Von Hippel-Lindau syndrome, is a rare genetic disorder with multisystem involvement.[2] It is characterized by visceral cysts and benign tumors with potential for subsequent malignant transformation. It is a type of phakomatosis that results from a mutation in the Von Hippel-Lindau tumor suppressor gene on chromosome 3p25.3.[3][4][5]

Hemangioblastomas are rare, highly vascularized, infratentorial brain tumors, that may occur sporadically or as a manifestation of von Hippel-Lindau (VHL) disease. As in this case, patients often present with features of increased intracranial pressure (e.g., headache, vomiting, blurred vision, papilledema). Hemangioblastomas are further associated with polycythemia, caused by paraneoplastic erythropoietin production. Histology shows densely packed, thin-walled capillary vessels surrounded by large stromal cells with intracytoplasmic lipid inclusions, as seen here. VHL-associated tumors most commonly occur in early adulthood and include hemangioblastomas of the spine, cerebellum, and retina (commonly bilateral and peripherally located, as seen here), renal cell carcinomas, pheochromocytomas, and pancreatic cysts.

A 35-year-old woman comes to the physician because of a 3-month history of facial hair growth, acne, and irregular menses. Her friends have told her that her voice sounds lower than usual. Physical examination shows pustular acne and dark hair growth along the jawline. Serum studies show elevated testosterone levels and normal inhibin levels. An ultrasound of the pelvis shows a left-sided ovarian mass. Microscopic examination of the resected ovarian mass shows pale, testosterone-positive staining cells with cytoplasmic Reinke crystal inclusions. These abnormal cells are homologous to which of the following physiological cell type in females?

In combination with an ovarian mass, hirsutism and virilization due to elevated testosterone levels indicate a testosterone-producing tumor. Reinke crystals are eosinophilic cytoplasmic inclusions that are found in Leydig cells. Hirsutism A condition of excessive male pattern hair growth in women (e.g., on the chin, above the upper lip, and around the umbilicus) that is most commonly idiopathic but associated with excess androgen in 10% of cases. Virilization The appearance of male secondary sexual characteristics (e.g., voice deepening, hirsutism, male-pattern hair loss, increased muscle mass, clitoromegaly) in a female individual. Can develop in any condition that causes hyperandrogenism. Theca interna cells 64% Leydig cells are homologous to the female theca interna cells, which cover the granulosa cell layer of developing ovarian follicles and also secrete testosterone. Theca cells express luteinizing hormone (LH) receptors and, when stimulated, convert cholesterol into androstenedione with the enzyme desmolase. After rupture of the mature ovarian follicle, theca interna cells differentiate into theca-lutein cells, which continue secreting androgens along with progesterone.

An 8-year-old boy is brought to the emergency department because of a 4-day history of severe, left-sided ear pain and purulent discharge from his left ear. One week ago, he returned with his family from their annual summer vacation at a lakeside cabin, where he spent most of the time outdoors hiking and swimming. Examination shows tragal tenderness and a markedly edematous and erythematous external auditory canal. Audiometry shows conductive hearing loss of the left ear. Which of the following is the most likely cause of this patient's symptoms?

Infection with Pseudomonas aeruginosa 82% Pseudomonas aeruginosa is the most common causal pathogen of otitis externa (commonly following swimming activities). Other common causal pathogens of otitis externa include Staphylococcus epidermidis and Staphylococcus aureus. The patient has ear pain and purulent discharge after returning from a vacation that involved outdoor swimming. Examination shows an inflamed external auditory canal and conductive hearing loss. These features are consistent with otitis externa. Otitis externa Swimmer's ear Abbreviation: OE A condition of inflammation of the external auditory canal, which is most often the result of local bacterial infection. Risk factors include injury to the skin of the external auditory canal and/or exposure to water.

A 55-year-old man comes to the physician because of swelling and unusual heaviness of his scrotum that he first noticed a few weeks ago. The symptoms improve when he is lying in bed. He also reports a 5-kg (11-lbs) weight loss and increasing fatigue over the past 6 months. He has smoked 1 pack of cigarettes daily for the past 15 years. Physical examination shows tenderness to palpation of his right flank. There are soft strands palpable in the right hemiscrotum superior of the right testis. The strands increase with Valsalva maneuver. Applying a bright light to the right hemiscrotum shows no transillumination. Obstruction of venous drainage into which of the following vessels is most likely responsible for this patient's scrotal enlargement?

Inferior vena cava 69% Unilateral right-sided varicoceles arise due to obstruction of right pampiniform venous plexus drainage via the testicular vein into the inferior vena cava (IVC). Varicoceles are more commonly left-sided, because the left testicular vein drains into the relatively high-pressure left renal vein at a 90-degree angle, while the right testicular vein drains into the low-pressure IVC. Unilateral right-sided varicoceles should raise suspicion for a mass in the retroperitoneal space (e.g., renal cell carcinoma, lymphoma, Ormond disease) blocking the venous blood return. This patient's history of weight loss, fatigue, smoking history, and right flank tenderness is concerning for an underlying right-sided renal cell carcinoma obstructing the right testicular vein and causing the varicocele.

A 65-year-old woman with osteoarthritis comes to the physician because of severe lower back and left leg pain. She has chronic lower back pain that is usually well-controlled with ibuprofen, but 3 hours ago her back pain acutely worsened after she picked up her 3-year-old granddaughter. The pain radiates from her lower back over her left outer thigh and knee towards the top of her big toe. Physical examination shows a diminished posterior tibial reflex on the left side. Muscle strength is 5/5 in all extremities and there are no sensory deficits. Steroid injection into which of the following anatomical locations is most likely to relieve her symptoms?

Intervertebral foramen 38% Disc herniation can cause nerve root compression, as the root exits the central nervous system through the intervertebral foramen. Steroid injection at the intervertebral foramen helps reduce inflammation and edema around the nerve root and spinal cord and can relieve nerve root compression and alleviate symptoms of disc herniation. Acute worsening of back pain in a 65-year-old woman after lifting something should raise suspicion for lateral disc herniation. The radiation of her pain to the lateral left leg suggests compression of the L5 nerve root.

A 3-year-old boy is brought to the emergency department after losing consciousness. His parents report that he collapsed and then had repetitive, twitching movements of the right side of his body that lasted approximately one minute. He recently started to walk with support. He speaks in bisyllables and has a vocabulary of almost 50 words. Examination shows a large purple-colored patch over the left cheek. One week later, he dies. Which of the following is the most likely finding on autopsy of the brain?

Leptomeningeal vascular malformation 69% This patient has several characteristics of Sturge-Weber syndrome, a congenital disorder that affects the capillaries and commonly manifests with developmental delay and port-wine stain. Sturge-Weber syndrome is associated with vascular malformations in the choroid and CNS, including leptomeningeal angiomas, which are typically located ipsilateral to the port-wine stain and can cause seizures. Congenital vascular disorder characterized by sturge weber port wine stain (capillary malformation of the face, nevus flammeus) primarily in V1 or V2 distribution of the trigeminal nerve capillary-venous malformation (eye and brain) Etiology GNAQ gene somatic mosaic mutation GNAQ gene product is involved in regulating intracellular signaling pathways this is a non-inherited disease Associated with: leptomeningeal angioma usually ipsilateral to port wine stain seizures capillary-venous malformation intellectual disability behavioral problems ophthalmologic involvement glaucoma visual field defect

An 81-year-old man is brought to the physician by his daughter after he was found wandering on the street. For the last 3 months, he often has a blank stare for several minutes. He also claims to have seen strangers in the house on several occasions who were not present. He has hypertension and hyperlipidemia, and was diagnosed with Parkinson disease 8 months ago. His current medications include carbidopa-levodopa, hydrochlorothiazide, and atorvastatin. His blood pressure is 150/85 mm Hg. He has short-term memory deficits and appears confused and disheveled. Examination shows bilateral muscle rigidity and resting tremor in his upper extremities. He has a slow gait with short steps. Microscopic examination of the cortex of a patient with the same condition is shown. Which of the following is the most likely diagnosis?

Lewy body dementia Histopathological findings of intracellular Lewy bodies in the cortex are found in patients with Parkinson disease or some Parkinson-plus syndromes. New-onset dementia, early onset of gait abnormalities, and a poor response to levodopa in a patient with Parkinsonism should raise suspicion of a Parkinson-plus syndrome. This patient's visual hallucinations are especially consistent with Lewy body dementia, the second most common form of neurodegenerative dementia. Another Parkinson-plus syndrome associated with Lewy bodies is multiple system atrophy.

A 25-year-old man is brought to the emergency department 30 minutes after he was involved in a motorcycle collision. Physical examination shows a deep laceration on the volar surface of the distal left forearm. Neurological examination shows loss of abduction and opposition of the left thumb. The radial and ulnar pulses are palpable. Based on these findings, which of the following nerves is most likely injured in this patient?

Median nerve 77% Laceration of the median nerve at the wrist causes loss of thumb abduction and opposition, as seen in this patient. Additional deficits include reduced or absent sensation in the thumb, index, middle, and radial-sided ring fingers. The laceration could be through the median nerve proper or through the recurrent branch of the median nerve. The presence of sensory disturbances would be the determining factor, as the recurrent branch solely provides motor innervation.

A 35-year-old woman comes to the physician because of a 1-month history of double vision, difficulty climbing stairs, and weakness when trying to brush her hair. She reports that these symptoms are worse after she exercises and disappear after she rests for a few hours. Physical examination shows drooping of her right upper eyelid that worsens when the patient is asked to gaze at the ceiling for 2 minutes. There is diminished motor strength in the upper extremities. The remainder of the examination shows no abnormalities. Which of the following is the most likely diagnosis?

Myasthenia gravis 89% Myasthenia gravis (MG) is the most likely diagnosis in a patient with diplopia, worsening proximal muscle weakness on exertion, and ptosis. While skeletal muscle is affected, deep tendon reflexes and the pupillary reflex are normal. In MG, autoantibodies against the post-synaptic acetylcholine (ACh) receptors compete competitively with ACh in the synaptic cleft. After a period of rest, the ACh levels build up, allowing for stimulation of the motor end plate. As a result, symptoms worsen with exercise or constant use and improve with rest. The diagnosis of MG is based on these clinical findings, detection of antibodies (acetylcholine receptor antibodies in 90% of patients), and electrodiagnostic tests (such as repetitive nerve stimulation).

Phentolamine

Non-selective alpha antagonist Its primary action is vasodilation due to α1 blockade.[3 Antihypertensive drug It can prevent and control high blood pressure during surgery. It can treat and prevent skin injury caused by norepinephrine injection. It is also used to diagnose an adrenal gland tumor (pheochromocytoma).

A 47-year-old woman comes to the physician because of a 3-month history of recurrent episodes of nausea and feeling like the room is spinning. Each episode lasts less than a minute and usually occurs when she gets out of bed in the morning or suddenly turns her head. She has not vomited and does not have headaches. She is asymptomatic between episodes. Which of the following findings most closely correlates with this patient's symptoms?

Nystagmus 63% Vertigo and nausea are common symptoms of dysfunctions of the vestibular system, which often also present with pronounced nystagmus. This patient's episodic vertigo triggered by specific types of head movement suggests benign paroxysmal positional vertigo (BPPV), which is confirmed by observing nystagmus during a provoked maneuver such as the Dix Hallpike test.

Moro reflex

The Moro reflex is an infantile reflex that develops between 28-32 weeks of gestation and disappears between 3-6 months of age. It is a response to a sudden loss of support and involves three distinct components:[1] spreading out the arms (abduction) pulling the arms in (adduction) crying (usually) It is distinct from the startle reflex.[2] Unlike the startle reflex, the Moro reflex does not decrease with repeated stimulation.[3]The primary significance of the Moro reflex is in evaluating integration of the central nervous system.[

Ecchymosis

bruise

ecchymosis

bruise

pruritus

itching

Pruritic

itchy

Auscultation

listening to sounds within the body

somnolent

sleepy, drowsy

dysarthria

slurred speech

Agnosia

the inability to recognize familiar objects.

thymoma

tumor of the thymus gland

Ataxic gait

walking abnormally and/or unsteadily

Fc portion

what part of the antibody determines its isotope (IgM/D, etc.)?

xerosis cutis

xerosis cutis, or simply dry skin, is a skin condition characterized by excessively dry skin.[2] The medical term xeroderma is derived from the Greek words meaning dry skin.

Guillian-Barre Syndrome (GBS)

Acute inflammatory condition involving the spinal nerve roots, peripheral nerves and possibly cranial nerve.

A 65-year-old man comes to the physician because of a 1-month history of progressive back pain. He has also had a 5-kg (11-lb) weight loss over the past 3 months. His only medications are a daily multivitamin and ibuprofen, which he takes daily for the back pain. Physical examination shows tenderness to palpation over the lower spine and the left iliac crest. His hemoglobin concentration is 9.3 g/dL, his serum calcium concentration is 12 mg/dL, and his serum creatinine concentration is 2.1 mg/dL. A bone marrow biopsy shows 21% plasma cells. A diagnosis of multiple myeloma is established. In preparation for an autologous hematopoietic stem cell transplantation, the patient receives a myeloablative treatment regimen that includes busulfan. Which of the following drugs acts via a similar mechanism of action to busulfan?

Busulfan is an alkylating chemotherapeutic agent of the alkyl sulfonate subgroup. It inhibits DNA replication by creating cross-links between DNA strands. Lomustine 57% Lomustine is an alkylating chemotherapeutic agent of the nitrosourea subgroup; similar to busulfan, it inhibits DNA replication by creating cross-links between DNA strands. These modifications cannot be repaired, which results in cell apoptosis. Busulfan is used as part of a conditioning regimen prior to autologous hematopoietic stem cell transplantation, as seen in this patient. Important adverse effects include myelosuppression, pulmonary fibrosis, and cutaneous hyperpigmentation. Lomustine is used to treat brain tumors and Hodgkin lymphoma. Important adverse effects include nausea, vomiting, and central nervous system dysfunction (e.g., ataxia, lethargy).

A 4-year-old boy is brought to the physician because of a 3-day history of fever and left ear pain. Examination of the left ear shows a bulging tympanic membrane with green discharge. Gram stain of the discharge shows a gram-negative coccobacillus. The isolated organism grows on chocolate agar. The causal pathogen most likely produces a virulence factor that acts by which of the following mechanisms?

Chocolate agar is a growth medium enriched with hemin (factor X) and nicotinamide adenine dinucleotide (factor V). It is used to isolate fastidious organisms like Haemophilus influenzae, a gram-negative coccobacillus that is a typical causative pathogen of otitis media. Cleavage of secretory immunoglobulins 62% Hemophilus influenza produces IgA protease, a virulence factor that mediates cleavage of secretory immunoglobulins (IgA antibodies). IgA protease thereby facilitates adherence of the pathogen to mucosal membranes (e.g., the middle ear mucosa and tympanic membrane), which are normally protected by IgA antibodies.

Two weeks after undergoing an emergency cardiac catheterization for unstable angina pectoris, a 65-year-old man has decreased urinary output. He takes naproxen for osteoarthritis and was started on aspirin, clopidogrel, and metoprolol after the coronary intervention. His temperature is 38.1°C (100.5°F), pulse is 96/min, and blood pressure is 128/88 mm Hg. Examination shows mottled, reticulated purplish discoloration of the feet and ischemic changes on the right big toe. His leukocyte count is 16,500/mm3 with 56% segmented neutrophils, 12% eosinophils, 30% lymphocytes, and 2% monocytes. His serum creatinine concentration is 4.5 mg/dL. A photomicrograph of a kidney biopsy specimen is shown. Which of the following is the most likely cause of this patient's presentation?

Cholesterol embolization 54% Cholesterol embolization syndrome is a known complication of cardiac catheterization, vascular interventions, and anticoagulant therapy. Cholesterol is released into the circulation, where it can travel to many organs, especially the kidneys (as they receive a large amount of the blood supply). This patient presents with the classic symptoms of blue toe syndrome, livedo reticularis, and acute kidney failure. Eosinophilia and eosinophiluria in the setting of cholesterol embolism are very suggestive findings. The kidney biopsy shows intravascular cholesterol deposits (spindle-shaped vacuoles) with multiple intra-arterial 'cholesterol clefts' and associated giant-cells, which confirms the diagnosis.

estrogen receptor positive

Describes cells that have a protein that binds to the hormone estrogen. Cancer cells that are estrogen receptor positive may need estrogen to grow. These cells may stop growing or die when treated with substances that block the binding and actions of estrogen. Also called ER positive.

An investigator is developing a drug that selectively inhibits the retrograde axonal transport of rabies virus towards the central nervous system. To achieve this effect, this drug must target which of the following?

Dynein is an ATP-dependent motor protein responsible for the retrograde migration and movement of cellular organelles and proteins along microtubules in axons. After rabies exposure, the rabies virus binds to acetylcholine receptors and enters the motor and sensory nerves of the peripheral nervous system, spreading in a retrograde direction to the CNS. To inhibit the migration of the virus and subsequent brain infection, the new drug must therefore target dynein motors.

A healthy 29-year-old nulligravid woman comes to the physician for genetic counseling prior to conception. Her brother has a disease that has resulted in infertility, a right-sided heart, and frequent sinus and ear infections. No other family members are affected. The intended father has no history of this disease. The population prevalence of this disease is 1 in 40,000. Which of the following best represents the chance that this patient's offspring will develop her brother's disease?

The constellation of infertility, a right-sided heart, and recurrent upper respiratory tract infections is consistent with Kartagener syndrome, which is an autosomal recessive condition. Since the patient's parents are asymptomatic, they must both be carriers for this trait (heterozygous). Mendelian principles are used to calculate the patient's probability of being a heterozygous carrier. The Hardy-Weinberg principle is used to calculate the husband's probability of being a heterozygous carrier. 0.2% 48% An offspring of two heterozygote parents has a ½ chance of being a carrier of the mutation (heterozygote), a ¼ chance of having an autosomal recessive disease, and a ¼ chance of not inheriting the mutation at all. Since this patient does not have the disease, she cannot carry two recessive traits. Therefore, she has a ⅔ chance of being a carrier, and a ⅓ chance of not inheriting the mutation. The chance of the intended father being a heterozygous carrier = 2pq, where (q) is the diseased allele frequency and (p) is the normal allele frequency. If the population prevalence (q2) is 1/40,000, then the diseased allele frequency (q) = √(1/40,000) = 1/200. The normal allele frequency (p) = 1-(diseased allele frequency) = 1-q = 1-(1/200) = 199/200. Therefore, the partner's chance of being a heterozygous carrier is 2pq = 2*(199/200)*(1/200) = 199/20,000 = .00995. To determine the chance of having an affected offspring, the probability of each parent passing on the affected gene (50%) is multiplied by the probability that each patient is a heterozygous carrier. For this case, the calculation would be (1/2)(2/3) * (1/2)(199/20,000) = ∼ 0.2%.

Midbrain

The midbrain or mesencephalon is the forward-most portion of the brainstem and is associated with vision, hearing, motor control, sleep and wakefulness, arousal (alertness), and temperature regulation.[2] The name comes from the Greek mesos, "middle", and enkephalos, "brain".[3] A small part of the brain above the pons that integrates sensory information and relays it upward.

Dix-Hallpike test

Useful in diagnosis of Benign Paroxysmal Vertigo (BPV). -- negative if no nystagmus after 45 secs -- nystagmus most likely torsional/rotational Epley Maneuver = particle repositioning procedure = SEE YOUTUBE

internal iliac lymph nodes

Where does lymph from the rectum and the anal canal (above the pectinate line) drain?

glial cells

cells in the nervous system that support, nourish, and protect neurons

verrucous

wart-like

carotid sinus

A dilation of a common carotid artery; involved in regulation of systemic blood pressure.

An otherwise healthy 25-year-old woman comes to the physician because of a 2-day history of pain and swelling of her right knee joint and left wrist. She went camping with her new boyfriend 3 weeks ago but does not recall any tick bites. Her temperature is 37.8°C (100.0°F). Examination of the right knee shows swelling, warmth, and tenderness on passive movement. There is a tender pustule on the sole of the left foot. Arthrocentesis of the right knee joint yields 8 mL of cloudy fluid with a leukocyte count of 45,000/mm3 (90% segmented neutrophils with intracellular organisms). Which of the following is the strongest risk factor for this patient's condition?

A positive nucleic acid amplification test (NAAT) would confirm the diagnosis in this case. Nucleic acid amplification test Nucleic acid test Abbreviation: NAAT A test that detects a particular pathogen in a specimen of blood, tissue, or bodily fluid by amplifying the RNA or DNA of the pathogen. Examples include polymerase chain reaction and ligase chain reaction. Sexually transmitted infection 73% Gonorrhea is a sexually transmitted infection caused by Neisseria gonorrhoeae, which can either manifest in a genitourinary, often asymptomatic form or as disseminated disease with arthritis-dermatitis syndrome or purulent arthritis. Arthritis-dermatitis syndrome typically presents with migratory, asymmetric arthritis, tenosynovitis, and vesicular, pustular, or maculopapular lesions on the trunk and extremities (sometimes involving the palms and soles), as seen in this patient. Synovial fluid analysis showing an increased neutrophil-dominated leukocyte count with intracellular organisms further supports the diagnosis. Septic arthritis Infectious arthritis A joint infection that causes severe pain, swelling, erythema, warmth, and loss of function of the affected joint. Also frequently causes systemic signs such as fever.

A 27-year-old man comes to the physician because of a 4-month history of unintentional weight gain, fatigue, and decreased sexual desire. There is no personal or family history of serious illness. His blood pressure is 149/88 mm Hg. Physical examination shows central obesity and abdominal striae. He has a prominent soft tissue bulge at the dorsum of his neck. Laboratory studies show a 24-hour urinary free cortisol of 200 μg (N < 50) and a morning serum ACTH of 1 pg/mL (N = 7-50). Which of the following tests is most likely to confirm the underlying etiology of this patient's symptoms?

Abdominal CT 47% This patient presents with symptoms and laboratory findings consistent with ACTH-independent Cushing syndrome due to autonomous overproduction of cortisol by the adrenal gland. Once the Cushing's syndrome has been determined to be ACTH-independent, a thin-section CT or MRI of the adrenal glands should be performed to evaluate for adrenal mass. Common etiologies for excess adrenal cortisol production include adrenal adenoma, adrenal carcinoma, and adrenal hyperplasia. ACTH stimulation test 9% ACTH stimulation test is used to test the response of the adrenal glands to ACTH in patients suspected of having adrenal insufficiency (e.g., hypocortisolism), such as Addison disease. Since this patient presents with hypercortisolism, this test would not be indicated.

Microglia

Act as phagocytes, eating damaged cells and bacteria, act as the brains immune system

A 62-year-old woman is brought to the emergency department after briefly losing consciousness while walking her dog. She spontaneously regained consciousness 20 seconds later. She has a history of atrial fibrillation. Current medications include metoprolol. She reports that she forgot to take her medication the day before and took double the dose this morning instead. A decrease in which of the following most likely contributed to this patient's episode? This patient takes metoprolol, a selective beta-1 adrenergic receptor antagonist that is often used to control the ventricular rate in atrial fibrillation. An increased dose as in this patient can cause bradycardia and subsequent syncope.

Activity of adenylyl cyclase in cardiomyocytes 67% Because the Gs protein of β receptors (G protein-coupled receptors) normally activates adenylyl cyclase, beta blockers lead to a decreased activity of adenylyl cyclase in cardiomyocytes. Inhibited adenylyl cyclase activity results in decreased downstream activation of cAMP and, subsequently, of protein kinase A. Decreased activation of protein kinase A can lead to bradycardia and decreased contractility, which causes syncope at toxic levels. This patient's symptoms are likely due to metoprolol toxicity. She may benefit from glucagon, which activates β receptors, thereby countering the effects of metoprolol.

A 13-year-old girl is brought to the physician by her parents because of a 1-week history of fever and fatigue. During this period, she has also had decreased appetite and a 1.5-kg (3.5-lb) weight loss. Her temperature is 39.2°C (102.5°F), pulse is 118/min, respirations are 16/min, and blood pressure is 110/70 mm Hg. Physical examination shows cervical lymphadenopathy. Laboratory studies show a hemoglobin concentration of 15 g/dL, a leukocyte count of 21,000/mm3, and a platelet count of 320,000/mm3. A peripheral blood smear is shown. Which of the following is the most likely cause of the findings on this patient's peripheral blood smear? The peripheral blood smear shows neutrophilia. Neutrophilia Neutrophilic leukocytosis, Neutrocytosis An increased neutrophil fraction (normal range: 50-70% of leukocytes). Most commonly caused by bacterial infection, inflammation, or increased levels of cortisol and epinephrine.

Acute bacterial infection 47% This patient has many nonspecific clinical features of acute infection, including fever, fatigue, decreased appetite, weight loss, tachycardia, lymphadenopathy, and leukocytosis. The peripheral blood smear finding of neutrophilia raises suspicion for a bacterial etiology. In response to bacterial infection, neutrophils demarginate from the vascular endothelium and are released from the bone marrow. This results in an increased concentration of neutrophils in the blood (i.e., neutrophilia) as they migrate to sites of infection to clear bacteria via phagocytosis. The bone marrow also increases neutrophil production, which can result in the presence of immature predecessors, such as band cells, in the blood (i.e., left shift

A 34-year-old woman comes to the physician because she has not had her period for 4 months. Menses had previously occurred at regular 28-day intervals with moderate flow. A home pregnancy test was negative. She also reports recurrent headaches and has noticed that when she goes to the movies she cannot see the outer edges of the screen without turning her head to each side. This patient's symptoms are most likely caused by abnormal growth of which of the following?

Adenohypophysis 82% Abnormal growth of the adenohypophysis is the most likely underlying cause of this patient's symptoms, which are consistent with a pituitary adenoma. Adenomas of the pituitary gland are most commonly due to aberrant proliferation of prolactin-producing cells in the adenohypophysis. The resulting hyperprolactinemia eventually causes suppression of gonadotropins and subsequent secondary amenorrhea, as seen in this patient. Bitemporal hemianopsia (due to compression of the optic chiasm) and headaches may develop as the adenoma grows.

True A 4-year-old boy is brought to the emergency department because of fever, nausea, and headache for 1 day. His temperature is 39.7°C (103.5°F). Examination shows involuntary flexion of the knees and hips when the neck is flexed. A lumbar puncture is performed and cerebrospinal fluid (CSF) analysis shows numerous segmented neutrophils and a decreased glucose concentration. Gram stain of the CSF shows gram-negative diplococci. This patient is at increased risk for which of the following complications?

Adrenal insufficiency 73% Waterhouse-Friderichsen syndrome is a rare but potentially lethal complication of meningococcal meningitis. It results from an endotoxin-triggered coagulopathy, and is characterized by adrenal hemorrhage and subsequent primary adrenal insufficiency (low cortisol, elevated ACTH). In addition to adrenal insufficiency, further signs may include petechiae, purpura, disseminated intravascular coagulation, respiratory failure, and shock. Waterhouse-Friderichsen syndrome (WFS) is defined as adrenal gland failure due to bleeding into the adrenal glands, commonly caused by severe bacterial infection. Typically, it is caused by Neisseria meningitidis.[1]

A 31-year-old man comes to the physician because of severe muscle pain and fever for 4 days. He likes to go hunting and consumed bear meat 1 month ago. Examination shows periorbital edema and generalized muscle tenderness. His leukocyte count is 12,000/mm3 with 19% eosinophils. The release of major basic protein in response to this patient's infection is most likely a result of which of the following?

Antibody-dependent cell-mediated cytotoxicity 53% Antibody-dependent cell-mediated cytotoxicity is critical for defense against helminthic infections. IgE molecules coat pathogens and are subsequently recognized by Fc receptor-bearing granulocytes, including eosinophils. The interaction of the IgE molecule and the receptor stimulates the release of cytotoxic granules such as major basic protein, which ultimately leads to the destruction of the pathogen. This man has periorbital edema, symptoms of myositis, fever, and eosinophilia after consuming bear meat. This suggests the diagnosis of trichinellosis. Major basic protein plays an important role in defending against helminthic infections. Trichinellosis Trichinosis An infection caused by Trichinella species (typically T. spiralis). Transmitted via ingestion of encysted larvae in undercooked meat (especially pork). Infection can be subclinical or manifest with abdominal pain, nausea, vomiting, and diarrhea. In severe infections, larvae disseminate into the skeletal muscle and cause fever, pain, weakness, periorbital edema, and eosinophilia.

Vinca Alkaloids MOA

Arrests mitotic division at the metaphase of cell division The mechanism of action of vinca alkaloids is to arrest dividing cells in metaphase by binding tubulin and preventing its polymerization into microtubules. This is also the proposed mechanism of causing neuropathy by inhibiting anterograde and retrograde axonal transport, thereby causing axonal degeneration.

Arthoroplasty

Arthroplasty is a surgical procedure to restore the function of a joint. A joint can be restored by resurfacing the bones. An artificial joint (called a prosthesis) may also be used. Various types of arthritis may affect the joints.

costovertebral angle

angle formed by the 12th rib and the vertebral column on the posterior thorax, overlying the kidney

C4d staining

C4d staining Featured snippet from the web C4d is a classical pathway complement degradation product that, when detected in peritubular capillaries, correlates with antibody-mediated rejection, and is associated with poor renal allograft outcome.

During a study on the immune system, an investigator isolates and labels T cells from the cortex of the thymus. The T cells that do not bind cortical epithelial cells expressing MHC molecules undergo apoptosis within 3-4 days. Which of the following best describes the T cells during this phase of differentiation?

CD4+ and CD8+ 74% Positive selection is the process described above. It takes place in the thymic cortex and ensures that new double-positive (CD4+/ CD8+) T cells can adequately bind MHC I (CD8) and MHC II (CD4). T cells that can bind receive a survival signal and those that do not undergo apoptosis. The surviving cells go on to negative T cell selection, where they are tested for autoimmunity.

Cefalexin

Cefalexin, also spelled cephalexin, is an antibiotic that can treat a number of bacterial infections.[4] It kills gram-positive and some gram-negative bacteria by disrupting the growth of the bacterial cell wall.[4] Cefalexin is a beta-lactam antibiotic within the class of first-generation cephalosporins.[4] It works similarly to other agents within this class, including intravenous cefazolin, but can be taken by mouth.[5]

A 27-year-old woman comes to the physician for a 1-week-history of painful urination and urinary frequency. She has no history of serious illness and takes no medications. She is sexually active with her boyfriend. Her temperature is 36.7°C (98.1°F). There is no costovertebral angle tenderness. Urine dipstick shows leukocyte esterase. A Gram stain does not show any organisms. Which of the following is the most likely causal pathogen?

Chlamydia trachomatis 65% This patient likely has Chlamydia trachomatis infection. Although most cases are asymptomatic, patients can present with urethritis and/or vulvovaginitis, as seen here. Chlamydiae do not gram stain well because they are obligate intracellular bacteria that lack muramic acid in their cell wall. In the diagnosis of chlamydia infection, Giemsa stain or fluorescent antibody stain can be used to identify reticulate bodies (i.e., cytoplasmic inclusions.) PCR (nucleic acid amplification) is the gold standard diagnostic test for chlamydiae.

A 48-year-old man comes to the physician because of a 3-month history of worsening shortness of breath and cough productive of frothy, whitish sputum. One year ago, he had a similar episode lasting 6 months. He has smoked a pack of cigarettes daily for 25 years. Physical examination shows bluish discoloration of the tongue and lips. Scattered expiratory wheezing and rhonchi are heard throughout both lung fields. Further evaluation of this patient is most likely to show which of the following findings?

Chronic bronchitis with symptoms of central cyanosis indicates chronic hypoxemia. Reduced ventilation despite adequate perfusion results in a ventilation-perfusion mismatch, which is classically seen in chronic bronchitis. Increased serum hematocrit is seen in chronic hypoxemia, which stimulates the release of erythropoietin (EPO) from renal interstitial cells. This process enhances erythropoiesis in bone marrow, resulting in secondary polycythemia.

A 43-year-old man is brought to the physician for a follow-up examination. He has a history of epilepsy that has been treated with a stable dose of phenytoin for 15 years. He was recently seen by another physician who added a drug to his medications, but he cannot recall the name. Shortly after, he started noticing occasional double vision. Physical examination shows slight vertical nystagmus and gait ataxia. Which of the following drugs was most likely added to this patient's medication regimen? (This patient has been safely treated with phenytoin for many years but is now presenting with subtle signs of phenytoin toxicity (diplopia, nystagmus, and ataxia) shortly after having been prescribed a new drug. Since phenytoin is metabolized by the hepatic cytochrome-P450 system (CYP450), the toxicity is likely due to the addition of a CYP450 inhibitor.)

Cimetidine is an inhibitor of CYP450 and a commonly prescribed drug for relief of GERD symptoms. Concurrent intake of cimetidine and phenytoin will lead to increased serum levels of phenytoin that are causing the toxicity seen in this patient. There are numerous drugs that affect CYP450 and careful revision of a patient's medication list is imperative prior to any changes.

A 62-year-old man is brought to the emergency department by his wife because she thinks he has had a stroke. He has hypertension and type 2 diabetes mellitus. Current medications include enalapril and metformin. He has smoked 1 pack of cigarettes per day for the past 35 years. His blood pressure is 162/95 mm Hg. A CT scan of the brain shows a lacunar stroke involving the left subthalamic nucleus. The patient most likely presented with which of the following findings on physical examination?

Contralateral hemiballismus and choreiform movements are common symptoms of a lacunar infarct of the subthalamic nucleus. They are caused by a reduction in the activity of the indirect pathway of the basal ganglia. Lacunar infarcts are noncortical infarcts, characterized by the absence of cortical signs. Subthalamic strokes are usually ischemic in etiology, and uncontrolled hypertension, diabetes mellitus, and a significant smoking history are known risk factors. The patient's wife could have been injured if she had been standing nearby when the patient was having the stroke.

A previously healthy 24-year-old woman comes to the physician because of a 1-day history of nausea and weakness. She is sexually active with 2 male partners and uses an oral contraceptive; she uses condoms inconsistently. Her last menstrual period was 4 days ago. Her temperature is 38.4°C (101°F). Physical examination shows right costovertebral angle tenderness. Pelvic examination is normal. Which of the following is the most likely cause of this patient's condition?

Costovertebral angle tenderness, nausea, and fever are classic characteristics of pyelonephritis. Pyelonephritis An infection of the renal pelvis and parenchyma that is usually associated with an ascending bacterial infection of the bladder. Typically presents with flank pain, costovertebral angle tenderness, fever, and features of cystitis (e.g., dysuria, frequency). Ascending bacteria from the bladder 83% Ascending bacteria from the bladder, typically due to underlying cystitis, is the most likely cause of this patient's pyelonephritis. Other risk factors include pregnancy, urinary tract obstruction (e.g., catheter, BPH, VUR), and recurrent urinary tract infections. Cystitis An inflammation of the bladder wall typically caused by a lower urinary tract infection. Less common causes include radiation or cyclophosphamide induced cystitis, interstitial cystitis, schistosomiasis, and tuberculosis.

Brown-Sequard Syndrome Brown-Sequard Syndrome • Weak side = side with lesion • UMN signs below • 1: Level of lesion • LMN signs • Loss of all sensation • If above T1 Horner's • Constricted pupil, eyelid droop • 2: Loss of motor, posterior columns • 3: Loss of pain/temp

Damage to half of spinal cord Loss of pain and temperature sensation on contralateral side of body Loss of proprioception and discriminatory touch on ipsilateral side of body.

A 26-year-old woman comes to the physician because of painful paresthesia in her right foot. Examination shows decreased sensation in the first interdigital space and a hallux valgus deformity. This patient's paresthesia is most likely caused by compression of which of the following nerves?

Deep peroneal nerve 62% The deep peroneal nerve provides sensory innervation to the skin between the first two toes. It also provides motor innervation to the muscles of the anterior compartment of the leg (e.g., tibialis anterior, extensor hallucis longus, extensor digitorum longus), which dorsiflex the toes and ankle joint; thus, injury to the nerve can result in a high-stepping gait. Deep peroneal nerve compression can be caused by tight-fitting shoes, which are suggested by this patient's hallux valgus deformity (bunion). It may also occur following trauma, especially navicular fracture.

An investigator studying protein synthesis in human stem cells isolates tRNA molecules bound to mRNA molecules. The isolated tRNA molecules have inosine in the 5' position of the anticodon; of these, some are bound to adenine, some to cytosine, and some to uracil at the 3' position of the mRNA codon. Which of the following properties of the genetic code is best illustrated by this finding?

Degeneracy 54% Degeneracy (genetic code) refers to the redundancy of codon base pairing; multiple codons can specify one single amino acid. During translation, some tRNA molecules can recognize multiple codons because of flexibility between the pairing of the 3' nucleotide of the codon (the wobble position) with the 5' nucleotide of the anticodon, which usually contains inosine (hypoxanthine nucleoside) instead of the usual nucleotides. Degeneracy reduces the extent of damage caused by DNA mutations.

A 33-year-old woman comes to the physician because of a 3-day history of dry cough and low-grade fever. Four months ago, she was diagnosed with major depressive disorder and started treatment with fluoxetine. Physical examination shows no abnormalities. A diagnosis of upper respiratory infection is made and a medication is prescribed to relieve her symptoms. A drug with which of the following mechanisms of action should be avoided in this patient?

Dextromethorphan, an antitussive with serotonergic effects, increases the risk of life-threatening serotonin syndrome in patients taking fluoxetine. Inhibition of NMDA glutamate receptors 48% Dextromethorphan is an NMDA glutamate receptor antagonist that desensitizes the medullary cough center to signaling from cough receptors in the respiratory tract. It is also a serotonin receptor agonist and has nonspecific serotonin-norepinephrine reuptake inhibitor effects. For this reason, it increases the risk of serotonin syndrome when combined with other serotonergic drugs (e.g., fluoxetine) and should be avoided in this patient.

effaced

During the first stage of labor, the cervix opens (dilates) and thins out (effaces) to allow the baby to move into the birth canal. In figures A and B, the cervix is tightly closed. In figure C, the cervix is 60 percent effaced and 1 to 2 cm dilated. In figure D, the cervix is 90 percent effaced and 4 to 5 cm dilated. The cervix must be 100 percent effaced and 10 centimeters dilated before a vaginal delivery.

A 5-year-old boy is brought to the emergency department by his mother because of a sudden loss of consciousness. He has asthma and has been hospitalized multiple times. His mother has type 2 diabetes mellitus. He is somnolent and diaphoretic. Serum studies show a glucose concentration of 22 mg/dL and a potassium concentration of 2.4 mEq/L. A dextrose infusion is administered, after which his glucose concentration normalizes and his symptoms improve. He is admitted to the hospital for further observation. Overnight, he has another episode of decreased consciousness. Serum studies taken during the episode show a glucose concentration of 19 mg/dL, an insulin concentration of 108 mIU/L (N=2.6-24.9), and a C-peptide concentration of 0.3 ng/mL (N = 0.8-3.1). Which of the following is the most likely diagnosis?

Endogenous insulin release involves the cleavage of proinsulin into insulin and C-peptide. Factitious disorder imposed on another 83% Factitious disorder imposed on another (FDIA) is a psychiatric condition in which an individual intentionally falsifies manifestations of disease in another individual (e.g., their child or elderly dependent) for primary gain. This boy's mother, who has type 2 diabetes mellitus, has given him exogenous insulin, which causes hyperinsulinemia, hypoglycemic episodes, and decreased serum C-peptide concentrations (because exogenous insulin does not contain C-peptide). The primary gain in this scenario includes the attention, sympathy, and care given to sick patients and their families.

Erythema multiforme is a skin immune reaction that an infection or medication can trigger. Its name combines the Latin "erythema" (redness), "multi" (many), and "forme" (shapes) and describes the main symptom, which is a rash on the body where each mark resembles a bullseye Erythema multiforme is a hypersensitivity reaction usually triggered by infections, most commonly herpes simplex virus (HSV

Erythema multiforme 74% This patient with recurrent genital HSV and a painful exanthem with target lesions most likely has Erythema multiforme (EM). EM is a type IV hypersensitivity reaction of the skin that can be triggered by certain infections (e.g., HSV, Mycoplasma pneumonia) and medications (e.g., beta-lactam antibiotics, sulfonamides, phenytoin). EM is characterized by lesions of varying morphology (e.g., macules, papules, vesicles) that typically progress to target lesions and spread proximally from the backs of the hands and feet, as seen here. Nikolsky sign is negative and the disease course is self-limiting in most cases. Nikolsky's sign The formation of a cutaneous blister upon stroking the skin. Classically seen in blistering skin disorders that are associated with suprabasal acantholysis and intraepidermal bullae formation: pemphigus vulgaris, staphylococcal scalded skin syndrome, bullous impetigo, Stevens-Johnson syndrome (or toxic epidermal necrolysis), and certain forms of epidermolysis bullosa.

A 4-year-old girl is brought to the physician because of diarrhea and vomiting for 5 days. Vaccinations are up-to-date. She appears pale and irritable. Her vital signs are within normal limits. Examination shows petechiae on her trunk and extremities. Abdominal examination shows diffuse abdominal tenderness with hyperactive bowel sounds. The remainder of the exam shows no abnormalities. Laboratory studies show: Hemoglobin8 g/dLLeukocyte count17,000/mm3Platelet count49,000/mm3SerumCreatinine1.6 mg/dLLactate dehydrogenase300 U/L Coagulation studies are normal. A peripheral blood smear is shown. Which of the following is the most likely underlying cause of these findings?

Escherichia coli infection This child presents with the triad of thrombocytopenia, signs of hemolysis (anemia, ↑ LDH, schistocytes), and renal dysfunction, which is indicative of hemolytic uremic syndrome (HUS). The Shiga-toxin-producing E. coli (STEC) is the most common cause of pediatric HUS. The illness is often preceded by a prodrome of vomiting and diarrhea, decreased urine output, and lethargy. Hemolytic uremic syndrome Abbreviation: HUS A condition in which microthrombi occlude the arterioles and capillaries, which results in microangiopathic hemolytic anemia, thrombocytopenia, and acute kidney injury. Predominantly affects children and most commonly occurs following dysentery due to Shiga toxin-producing enterohemorrhagic Escherichia coli (EHEC), typically serotype O157:H7.

A 70-year-old woman comes to the physician for a follow-up examination 2 months after undergoing a total hip replacement surgery. She reports that she has persistent difficulty in walking since the surgery despite regular physiotherapy. Examination of her gait shows sagging of the left pelvis when her right leg is weight-bearing. Which of the following nerves is most likely to have been injured in this patient?

Examination in this patient shows a positive Trendelenburg sign, which is caused by weakness of the hip abductors (i.e., the gluteus medius and minimus). Trendelenburg's sign A physical examination finding exhibited when a patient stands on one leg and the pelvis tilts downward towards the contralateral side. Caused by ipsilateral neurological weakness of the gluteus medius and minimus due to malfunction of the ipsilateral superior gluteal nerve. Right superior gluteal nerve 64% The right superior gluteal nerve innervates the right gluteus medius and minimus, which are responsible for abducting the right hip. Damage of the right superior gluteal nerve results in lowering of the left hip while standing on the right foot (positive Trendelenburg sign). This patient's symptoms suggest that she suffered damage to the right superior gluteal nerve during total hip replacement surgery.

A 60-year-old man is brought to the emergency department because of a 1-hour history of disorientation and slurred speech. He has a 10-year history of hypertension and hypercholesterolemia. His blood pressure is 210/110 mm Hg, and pulse is 90/min. Once the patient is stabilized, an MRI of the brain is performed, which shows an infarct of the left precentral gyrus involving the region that supplies the facial nerve. Given the MRI findings, which of the following neurological findings would most be expected?

Flattening of the right nasolabial fold 45% Flattening of the right nasolabial fold would be expected in central facial nerve palsy. The facial motor nucleus has two regions: one that supplies the muscles of the forehead and a second that supplies the rest of the face. The branches of the facial nerve that innervate the forehead are activated by both cerebral hemispheres, whereas the branches innervating the lower portion of the face are only activated by the contralateral cerebral hemisphere. As a result, the forehead is spared with unilateral infarction of the precentral gyrus, while the contralateral lower face becomes paralyzed Decreased lacrimation of the left eye 8% Lacrimation of the eye is modulated by the greater petrosal nerve, a branch of the facial nerve that carries parasympathetic fibers, and could therefore be impaired by peripheral facial nerve palsy. Decreased lacrimation of the left eye would be expected in left-sided peripheral facial palsy. However, central facial nerve palsy causes contralateral deficits, and a lesion to the precentral gyrus, the location of the primary motor cortex, does not affect lacrimation. Inability to raise the right eyebrow 19% Inability to raise the right eyebrow is seen in ipsilateral peripheral facial nerve palsy, which is commonly caused by trauma (e.g., zygomatic fractures, lacerations), infection (e.g., herpes simplex virus, varicella-zoster virus), tumors (e.g., neuromas, parotid gland tumors), or is idiopathic (Bell palsy). In central facial nerve palsy, drooping of the eyebrow would not be expected, since the forehead is innervated by fibers from both cerebral hemispheres.

Two weeks after being hospitalized for acute pancreatitis, a 36-year-old man comes to the physician for a follow-up examination. Multiple family members have coronary artery disease. Physical examination shows multiple, yellow papular lesions on both upper eyelids. Fasting serum lipid studies show: Total cholesterol280 mg/dlHDL-cholesterol45 mg/dlLDL-cholesterol185 mg/dlTriglycerides1080 mg/dl Treatment with gemfibrozil is initiated. The expected beneficial effect of this drug is most likely due to which of the following mechanisms of action?

Gemfibrozil is a fibrate. Fibrates such as gemfibrozil activate peroxisome proliferator-activated receptor alpha (PPAR-α), and thereby, increase the synthesis of high-density lipoproteins. Upregulation of lipoprotein lipase 54% Gemfibrozil upregulates lipoprotein lipase, which hydrolyzes triglycerides into free fatty acids (FFA). Adipose tissue subsequently removes FFAs from the blood and stores them. This process reduces the concentration of triglycerides in serum, thereby reducing the morbidity associated with triglyceride deposition in vessels and soft tissues. Fibrates also induce HDL synthesis through peroxisome proliferator-activated receptor alpha (PPAR-α) activation.

A 36-year-old woman is fasting prior to a religious ceremony. Her only oral intake in the last 36 hours has been small amounts of water. A metabolic enzyme that plays an important role in maintaining normal blood glucose in this patient is located exclusively within the mitochondria. An increase in which of the following substances is most likely to increase the activity of this enzyme?

Hepatic gluconeogenesis is the primary source of glucose after 24 hours of fasting. Pyruvate carboxylase is the only gluconeogenic enzyme that is located in the mitochondria. Acetyl coenzyme A 31% Acetyl-CoA activates pyruvate carboxylase, a mitochondrial-specific enzyme that catalyzes oxaloacetate production. Oxaloacetate is converted to phosphoenolpyruvate (PEP) in the cytosol by the GTP-dependent enzyme PEP carboxykinase. PEP then undergoes essentially a reversed process of glycolysis, which results in glucose formation. Partitioning of the gluconeogenic pathway between the cytosol and mitochondria allows for greater control and regulation of gluconeogenesis

A 17-year-old girl comes to the physician because of a sore throat, fevers, and fatigue for the past 3 weeks. Her temperature is 37.8°C (100°F), pulse is 97/min, and blood pressure is 90/60 mm Hg. Examination of the head and neck shows cervical lymphadenopathy, pharyngeal erythema, enlarged tonsils with exudates, and palatal petechiae. The spleen is palpated 2 cm below the left costal margin. Her leukocyte count is 14,100/mm3 with 54% lymphocytes (12% atypical lymphocytes). Results of a heterophile agglutination test are positive. This patient is at increased risk for which of the following conditions?

Hodgkin lymphoma Hodgkin lymphoma (HL) is a potential complication of infectious mononucleosis caused by Epstein-Barr virus (EBV). Additional complications of EBV infection include Non-Hodgkin lymphoma (e.g., Burkitt lymphoma, diffuse large B cell lymphoma, and primary CNS lymphoma in HIV-positive patients) and nasopharyngeal carcinoma. The pathophysiology of EBV-associated malignancy includes virus-induced expression of latent membrane protein 1 (LMP1), which constitutively activates NF-κB and kinases that drive cell proliferation. Monospot test Heterophile agglutination test, Heterophile antibody test A laboratory test in which nonhuman erythrocytes (e.g., from sheep or horses) are exposed to human serum. Aggregation of erythrocytes indicates the presence of heterophile antibodies (e.g., as a result of infectious mononucleosis).

A 23-year-old woman with asthma is brought to the emergency department because of shortness of breath and wheezing for 20 minutes. She is unable to speak more than a few words at a time. Her pulse is 116/min and respirations are 28/min. Pulse oximetry on room air shows an oxygen saturation of 92%. Examination of the lungs shows decreased breath sounds and scattered end-expiratory wheezing over all lung fields. Treatment with high-dose continuous inhaled albuterol is begun. This patient is at increased risk for which of the following adverse effects?

Hypokalemia 54% Hypokalemia is a common adverse effect of continuous albuterol therapy. It is caused by β2-agonist stimulation of the Na+/K+-ATPase, which leads to an intracellular K+ shift. Additionally, β2-mediated stimulation of the liver elevates cAMP levels, thereby increasing glycogenolysis and hyperglycemia. Hyperglycemia stimulates insulin secretion, which also activates the Na+/K+-ATPase and further potentiates the intracellular K+ shift. Hypokalemia can cause EKG changes, such as T wave flattening, ST depression, and U waves.

A 25-year-old man comes to the physician because of a 2-week history of numbness in his left lower extremity. One month ago, he sustained a fracture of the neck of the left fibula during soccer practice that was treated with immobilization in a plaster cast. Physical examination of the left lower extremity is most likely to show which of the following findings?

Impaired dorsiflexion of the foot 76% Dorsiflexion of the foot is accomplished through contraction of muscles in the anterior compartment of the leg, including the tibialis anterior (main dorsiflexor), extensor hallucis longus, extensor digitorum longus, and peroneus tertius. These muscles are innervated by the deep peroneal nerve, which branches from the common peroneal nerve. Injury to the common peroneal nerve, either from the initial fibula fracture or, more likely, compression from the plaster cast, explains this patient's numbness and would result in impaired dorsiflexion and eversion on motor examination. Patients with injury to the common peroneal nerve typically present with steppage gait. Inability to stand on tiptoes 6% Standing on one's tiptoes requires contraction of the muscles in the superficial posterior compartment of the leg, gastrocnemius, and soleus. Direct injury to these muscles or damage to the tibial nerve, which innervates these muscles, would make it impossible to stand on one's tiptoes, but this patient's history of isolated fracture of the fibular neck does not suggest these pathologies.

A previously healthy 67-year-old man comes to the physician for routine health maintenance evaluation. He works at a community center and volunteers at a local homeless shelter. A tuberculin skin test shows an induration of 14 mm. An x-ray of the chest is normal. Treatment with an antimycobacterial drug is initiated. Two months later, he has numbness and burning over both feet and an unsteady gait. Physical examination shows decreased sensation to light touch extending from the soles of the feet to the mid-shin bilaterally. This patient's condition is most likely to result in which of the following? Isoniazid is a commonly used monotherapy for latent tuberculosis. Adverse effects of isoniazid include peripheral polyneuropathy due to pyridoxine deficiency.

Impaired synthesis of neurotransmitters 36% Pyridoxine deficiency causes impaired synthesis of neurotransmitters, as pyridoxine is an important cofactor for multiple enzymes, including those that take part in the synthesis of GABA, serotonin, epinephrine, norepinephrine, and dopamine. Decreased levels of GABA can cause hyperirritability and seizures. In addition, vitamin B6 is an important cofactor of the enzyme cystathionine synthase, which is needed for the conversion of homocysteine to cysteine. When this enzyme is impaired, (e.g., during isoniazid treatment), homocysteine is increasingly metabolized to S-adenosylmethionine (SAM) via methionine synthase. SAM inhibits the synthesis of sphingolipids and myelin, which can lead to irreversible nerve damage.

C8 deficiency

Increased susceptibility to Neisseria infections

Kinesins

Kinesins are biological motor proteins that are ATP-dependent and function to assist cells with the transport of molecules along microtubules. Simply put, these proteins, function as highways within cells as they allow for the transport of all sorts of cellular cargo.

A 35-year-old man comes to the physician for evaluation of a neck mass and hoarseness. He has no history of major medical illness. Physical examination shows a 2.5-cm fixed, irregular thyroid nodule. His serum calcitonin concentration is elevated. The nodule is most likely composed of cells that are embryologically derived from which of the following structures?

Lateral endodermal anlage 23% The two lateral thyroid anlagen differentiate from ultimobranchial bodies (UBB), the most caudal, or ultimate, structures derived from the fourth branchial pouch. The lateral anlagen then fuse with the median thyroid anlage to form the thyroid gland. The parafollicular C cells are thought to be derived from the endoderm of the lateral thyroid anlage and secrete calcitonin. Medullary thyroid cancer arises from these cells and can manifest as a firm, painless thyroid nodule, as well as hoarseness and elevated serum calcitonin levels, which are all seen in this case. The findings of recent experimental studies argue strongly against the previous hypothesis that parafollicular C cells are derived from the neural crest. Lateral thyroid anlage Paired endodermal cell thickenings derived from the 4th branchial pouch. Located lateral to the median thyroid anlage. Fuse with the median thyroid anlage to form the parafollicular C cells of the thyroid gland.

A 40-year-old man comes to the physician because of a 1-month history of a painless lump on his neck. Two years ago, he underwent surgery for treatment-resistant hypertension, episodic headaches, and palpitations. Physical examination shows a firm, irregular swelling on the right side of the neck. Ultrasonography of the thyroid gland shows a 2-cm nodule with irregular margins and microcalcifications in the right thyroid lobe. Further evaluation of this patient is most likely to show increased serum concentration of which of the following substances? This patient likely has a history of pheochromocytoma (treatment-resistant hypertension, episodic headaches, palpitations) that was treated with adrenalectomy. His painless, irregular thyroid nodule should raise concern for medullary thyroid carcinoma as a feature of MEN2 syndrome.

Medullary thyroid carcinoma is a malignant neoplasm of the parafollicular cells (C cells) that often arises sporadically but is also associated with multiple endocrine neoplasia type 2A and type 2B (MEN2). Since parafollicular cells produce the hormone calcitonin, increased serum calcitonin would be expected. Measuring the serum calcitonin concentration is helpful to support the diagnosis of medullary thyroid carcinoma and is also used to monitor for residual disease or recurrence after thyroidectomy.

Miosis and Mydriasis

Miosis is constriction of pupils Mydriasis is dilation of pupils

A 19-year-old man with a history of generalized tonic-clonic seizures comes to the physician for a routine health maintenance examination. He is a known user of intravenous cocaine. His vital signs are within normal limits. Physical examination shows multiple hyperpigmented lines along the forearms. Oral examination shows marked overgrowth of friable, ulcerated gingival mucosa. Which of the following is the most likely cause of this patient's oral examination findings?

Overgrowth of the gingival tissue is also known as gingival hyperplasia. Phenytoin 81% Phenytoin is used in the prophylaxis of generalized tonic-clonic seizures, as seen in this patient. The agent has many side effects, including gingival hyperplasia (drug-induced gingival overgrowth), hirsutism, Stevens-Johnson syndrome, megaloblastic anemia, nystagmus, ataxia, and peripheral neuropathy. If possible, the drug should be discontinued and the patient should be advised to adhere to strict oral hygiene. If the gingival overgrowth does not improve or infection occurs, excision of excessive gingival tissue should be considered.

Two weeks after undergoing an allogeneic skin graft procedure for extensive full-thickness burns involving his left leg, a 41-year-old man develops redness and swelling over the graft site. He has not had any fevers or chills. His temperature is 36°C (96.8°F). Physical examination of the left lower leg shows well-demarcated erythema and edema around the skin graft site. The graft site is minimally tender and there is no exudate. Which of the following is the most likely underlying mechanism of this patient's skin condition?

Pain and edema within 6 months of receiving an allogeneic skin graft are signs of acute graft rejection. Acute transplant rejection A rejection of transplanted tissue that occurs within 6 months after transplantation (usually within days to weeks) and causes ~ 50% of post-transplant organ dysfunction. Due to allorecognition (T-lymphocyte-induced cell-mediated and/or humoral immunity). Manifests with fever, edema, graft pain, and organ dysfunction. Th1-induced macrophage activation Th1 cell-induced macrophage activation is responsible for this patient's acute graft rejection. This type IV hypersensitivity reaction is triggered by foreign antigens from the allogeneic skin graft, which incite host T-cell sensitization. Continued exposure to the foreign antigen causes the pre-sensitized T cells to activate macrophages and thus induce an inflammatory reaction in the tissue. In addition, this exposure leads to antibody-mediated complement activation (similar to hyperacute transplant rejection), during which IgM and IgG antibodies develop in response to foreign antigens from an allogeneic graft. This reaction most commonly presents as redness, swelling, itching, and tissue sloughing at the graft site.

Biliary colic

Pain that occurs when a gallstone is being passed and is blocking a bile duct, typically comes and goes in a fairly regular pattern.

A 36-year-old man is admitted to the hospital because of a cocaine overdose. At the time of admission, he is agitated and his blood pressure is 210/150 mm Hg. The physician writes an order in the patient's chart for "5.0 mg phentolamine" and "2.5 mg diazepam" to be administered intravenously every 15 minutes. Three hours later, the patient loses consciousness. His blood pressure is 70/50 mm Hg. The physician determines that the patient received ten times the prescribed dose of one of the medications as a result of an error. Which of the following measures would have most likely prevented the medication error? (Medication errors can occur at the stages of ordering, transcribing, dispensing, and administering a drug. This patient's medication error is most likely due to faulty transcribing and led to the misinterpretation of a trailing zero.)

Paper-based systems, in which medication orders are handwritten, are prone to transcribing errors, which can result in the recipient of the order not being able to properly read and interpret the prescription. In this case, as the physician determined that the patient received ten times the prescribed dose, the transcribing error was most likely due to the misinterpretation of a trailing zero after a decimal point. Trailing zeros are part of the Joint Commission's "Do Not Use" list, which provides recommendations on information management. To prevent transcribing errors, most hospitals have adopted computerized provider order entry (CPOE) systems, which also have the advantage of helping to prevent ordering and dispensing errors because they are usually combined with a clinical decision support system. If a CPOE system has not been adopted and the health care facility maintains a paper-based system, the usage of standardized abbreviations/writing styles and having an intermediary (like a clinical pharmacist or a pharmacy technician) read and interpret the prescriptions can also help to prevent transcribing errors.

Uhthoff's phenomenon

Peripheral nerve studies have shown that even a 0.5 °C increase in body temperature can slow or block the conduction of nerve impulses in demyelinated nerves. With greater levels of demyelination, a smaller increase in temperature is needed to slow down the nerve impulse conduction. Exercising and performing activities of daily living can cause a significant increase in body temperature in individuals with MS, especially if their mechanical efficiency is poor due to the use of mobility aids, ataxia, weakness, and spasticity

A 38-year-old woman comes to the physician because of a 4-month history of crampy abdominal pain, recurrent watery diarrhea, and a 2.5-kg (5.5-lb) weight loss. Her husband has noticed that after meals, her face and neck sometimes become red, and she develops shortness of breath and starts wheezing. Examination shows a grade 3/6 systolic murmur heard best at the left lower sternal border. The abdomen is soft, and there is mild tenderness to palpation with no guarding or rebound. Without treatment, this patient is at greatest risk of developing which of the following conditions? Watery diarrhea, crampy abdominal pain, episodic cutaneous flushing triggered by eating or emotional events, asthma-like attacks, and a murmur indicative of tricuspid regurgitation are highly suggestive of carcinoid syndrome. The neuroendocrine tumor is highly metabolically active and secretes excess amounts of serotonin. Carcinoid syndrome is a paraneoplastic syndrome comprising the signs and symptoms that occur secondary to carcinoid tumors. The syndrome includes flushing and diarrhea, and less frequently, heart failure, vomiting and bronchoconstriction.[3] It is caused by endogenous secretion of mainly serotonin and kallikrein. Most dietary niacin is in the form of nicotinic acid and nicotinamide, but some foods contain small amounts of NAD and NADP. The body also converts some tryptophan, an amino acid in protein, to NAD, so tryptophan is considered a dietary source of niacin.

Pigmented dermatitis 30% Dermatitis is a characteristic feature of pellagra, or niacin deficiency; other key findings include dementia and diarrhea (i.e., the "three Ds"). Dermatitis in patients with pellagra is characterized by a symmetrical, pigmented, scaly thickening of the skin in sun-exposed areas, such as the limbs and around the neck (casal necklace). The increased metabolism of serotonin in carcinoid tumors depletes the amount of tryptophan precursors available for other organic compounds, including niacin. A lack of niacin leads to decreased production of the coenzymes NAD+ and NADP+, which are essential in redox reactions throughout cellular metabolism. Diagnosis is confirmed via 24-hour urine testing for 5-hydroxyindoleacetic acid (5-HIAA), the end product of serotonin metabolism.

23-year-old woman comes to physician for an annual health maintenance examination. She feels well. She is 155 cm (5 ft 1 in) tall and weighs 79 kg (174 lb); BMI is 33 kg/m2. Examination shows a skin rash over both axillae. A photograph of her left axilla is shown. This patient's skin finding is most likely associated with which of the following conditions?

Polycystic ovarian syndrome 68% Acanthosis nigricans is commonly seen in disorders characterized by insulin resistance and hyperinsulinemia (e.g., polycystic ovarian syndrome, obesity, diabetes mellitus, acromegaly). Elevated levels of insulin stimulate keratinocyte and dermal fibroblast proliferation via interaction with insulin-like growth factor 1, resulting in epidermal hyperplasia and plaque-like lesions, as seen in this patient. In addition, insulin resistance in PCOS is responsible for hyperandrogenism (leading to acne, hirsutism) and anovulation (causing amenorrhea and impaired fertility). Acanthosis nigricans A skin disorder characterized by patches of velvety hyperpigmentation in the axilla, groin, and/or neck. Skin tags can also develop in affected areas. Classified as benign (e.g., from endocrine or drug adverse effects) or malignant (e.g., as a paraneoplastic syndrome).

An investigator is studying the incidence of sickle cell trait in African American infants. To identify the trait, polymerase chain reaction testing is performed on venous blood samples obtained from the infants. Which of the following is required for this laboratory technique?

Polymerase chain reaction testing (PCR) amplifies a specific region of interest on a DNA strand. Initial sequence of the 3' end of a DNA strand 54% The polymerase chain reaction (PCR) test detects and multiplies a specific target segment of a DNA strand using primers and the enzyme Taq DNA polymerase. The first step of PCR involves heating the test sample (venous blood, in this case) to denature the double-stranded DNA into single strands (denaturation step). Primers, composed of nucleotides that are complementary to the 3′ end of the target DNA sequence nucleotides, bind to the 3′ end of the target DNA (hybridization step). This converts the single-stranded DNA segment into a double-stranded structure, which allows the enzyme DNA polymerase to synthesize new complementary DNA strands (amplification step).

A 33-year-old woman comes to the physician because of a 4-day history of fever, anterior neck pain, and throat swelling. She has no history of serious illness. Her temperature is 38.1°C (100.6°F) and pulse is 109/min. Physical examination shows diaphoresis and a fine tremor of the outstretched hands. The thyroid gland is enlarged, firm, and tender to palpation. Serum thyroid stimulating hormone level is 0.06 μU/mL and erythrocyte sedimentation rate is 65 mm/h. 123I scan shows an enlarged thyroid gland with diffusely decreased uptake. Histologic examination of a thyroid biopsy specimen is most likely to show which of the following findings?

Signs of hyperthyroidism, a tender and enlarged thyroid gland, and an elevated ESR in a young female patient are suggestive of De Quervain thyroiditis, a type of subacute thyroiditis. Noncaseating granulomas with multinucleated giant cells 44% De Quervain thyroiditis is a transient and self-limited disease characterized by patchy granulomatous inflammation of the thyroid. This disease is often preceded by a viral upper respiratory tract infection and typically presents initially with hyperthyroidism (as seen in this patient), followed by hypothyroidism. Classic signs include thyroid tenderness, increased ESR, and jaw pain. Subacute granulomatous thyroiditis Giant cell thyroiditis A transient and self-resolving patchy inflammation of the thyroid gland that is associated with granuloma formation. Often occurs after a viral upper respiratory infection and is more common among women. The clinical course is typically triphasic, beginning with hyperthyroidism, followed by hypothyroidism, and finally a return to the euthyroid state. Classically presents with tender goiter, elevated ESR, and jaw pain.

Spherocytes

Spherocytosis is the presence in the blood of spherocytes, i.e erythrocytes (red blood cells) that are sphere-shaped rather than bi-concave disk shaped as normal. Spherocytes are found in all hemolytic anemias to some degree.

A 68-year-old man is brought to the emergency department by his wife because of a 2-week history of progressive disorientation and a 1-day history of left-sided weakness and difficulty speaking. The wife reports that the patient had a minor fall 4 months ago, during which he may have hit his head. He has hypertension and hyperlipidemia. He drinks 3-4 bottles of beer daily. He is only oriented to person. Neurological examination shows moderate spastic weakness, decreased sensation, and increased deep tendon reflexes in the left upper and lower extremities. A CT scan of the head is shown. Which of the following is the most likely cause of this patient's condition?

Tearing of bridging veins 80% This CT scan shows a classic subdural hematoma. The history of minor head trauma followed by the insidious onset of unilateral neurological symptoms (spastic paresis, hyperreflexia, decreased sensation) is consistent with a chronic subdural hematoma. This type of bleeding results from tears in the bridging veins that cross the subdural space.

A 54-year-old man with hypertension and congenital blindness comes to the physician because he is unable to recognize objects by touch with his right hand. The symptoms started about 2 hours ago. When given a house key, he can feel the object in his right hand but is not able to identify what it is. This patient's condition is most likely caused by a lesion in which of the following locations?

The ability to recognize objects by touch is called stereognosis. Tactile information is processed by the secondary somatosensory cortex (somatosensory association cortex). Contralateral superior parietal lobule 70% The somatosensory association cortex is located in the superior parietal lobule and is involved in tactile information processing and spatial orientation. A lesion in the left superior parietal lobule can cause right-sided astereognosis, as seen in this patient. Hemispatial neglect can also result from parietal lobe damage; however, it is more characteristic of right parietal lobe lesions, rather than the left parietal lobe lesion seen here.

flexor pollicis longus

The flexor pollicis longus (/ˈflɛksər ˈpɒlɪsɪs ˈlɒŋɡəs/; FPL, Latin flexor, bender; pollicis, of the thumb; longus, long) is a muscle in the forearm and hand that flexes the thumb. It lies in the same plane as the flexor digitorum profundus. This muscle is unique to humans, being either rudimentary or absent in other primates. The flexor pollicis longus is supplied by the anterior interosseous(C8-T1) branch of the median nerve (C5-T1).

Endoneurium

The endoneurium (also called endoneurial channel, endoneurial sheath, endoneurial tube, or Henle's sheath) is a layer of delicate connective tissue around the myelin sheath of each myelinated nerve fiber in the peripheral nervous system. Its component cells are called endoneurial cells.

A 67-year-old man with hypertension comes to the physician because of a 5-month history of a facial rash. He occasionally feels burning or stinging over the affected area. His only medication is lisinopril. Physical examination shows the findings in the photograph. Which of the following is the strongest predisposing factor for this patient's skin condition?

This patient presents with facial erythema, telangiectasia, papules, and pustules, which are findings characteristic of rosacea. Alcohol consumption While the etiology of rosacea remains uncertain, there are several known triggers that precipitate or exacerbate symptoms, including stress, exercise, hot weather, and alcohol consumption. These triggers are believed to induce local inflammation, which causes vasodilation of cutaneous capillaries and skin flushing.

A previously healthy 16-year-old girl is brought to the physician by her parents because of behavior changes and involuntary limb movements over the past 2 days. She also has a 2-week history of fever, headache, and fatigue. Her temperature is 38°C (100.4°F), pulse is 110/min, respirations are 20/min, and blood pressure is 102/72 mm Hg. Mental status examination shows impaired speech and a disorganized thought process. Muscle strength is 5/5 in all extremities. Urine toxicology screening is negative. Cerebrospinal fluid analysis shows a leukocyte count of 70 cells/mm3 (90% lymphocytes) and a protein concentration of 51 mg/dL. Abdominal ultrasound shows a large right adnexal mass. The patient's symptoms are most likely caused by antibodies against which of the following?

This young girl with an adnexal mass has features of paraneoplastic encephalomyelitis such as cognitive defects, personality changes, speech impairment, involuntary limb movements, and autonomic instability (tachycardia, hypotension) Glutamate receptors 31% NMDA glutamate receptors are targeted by antibodies in anti-NMDA encephalitis, which is a type of paraneoplastic encephalomyelitis (PEM) that is typically associated with ovarian teratoma. Patients with PEM can have a prodrome of fever, headache, and fatigue lasting weeks to months before the onset of features of CNS damage. CSF analysis of patients with PEM can show lymphocytic pleocytosis with increased protein concentration. Another cause of PEM is autoantibodies against neuronal Hu antigens (anti-Hu encephalitis), which typically occur in patients with small cell lung cancer.

A 35-year-old woman comes to the emergency department because of a 3-day history of pain in the left cheek. The pain occurs every few hours, lasts 30-60 seconds, and is aggravated by chewing and brushing. She has a history of pain with a vesicular rash in the right axillary area 1 year ago. She had an upper respiratory infection 2 weeks ago. Physical examination shows no abnormalities. Which of the following is the most likely diagnosis

Trigeminal neuralgia is a chronic pain syndrome characterized by transient (< 1 minute), recurrent, severe shooting, stabbing, and/or paroxysmal pain that can occur up to 100 times per day in the distribution of the trigeminal nerve. It most commonly manifests with pain shooting from the mouth to the angle of the jaw. It can be triggered by a variety of stimuli, including brushing, chewing, cold, and touch. It is not related to this patient's prior rash and upper respiratory infection.

Supraspinatus

abduction of shoulder Empty can test

ecchymoses

bruising

Acetazolamide

carbonic anhydrase inhibitor (zolamide suufix indicates ca

excoriated vesicles

damage or remove part of the surface of (the skin).

myeloid

derived from bone marrow

gravida

pregnant woman

Gliadin

sticky fraction of gluten

central pallor

the central pale area of the RBC that represents the thinnest part of the biconcave disc Red blood cells have a characteristic pink appearance due to their high content of hemoglobin, which is basic. The central pale area of each red blood cell is due to the concavity of the disc. Also visible in this slide are several platelets, which play a crucial role in the blood clotting cascade.

The height of American adults is expected to follow a normal distribution, with a typical male adult having an average height of 69 inches with a standard deviation of 0.1 inches. An investigator has been informed about a community in the American Midwest with a history of heavy air and water pollution in which a lower mean height has been reported. The investigator plans to sample 30 male residents to test the claim that heights in this town differ significantly from the national average based on heights assumed be normally distributed. The significance level is set at 10% and the probability of a type 2 error is assumed to be 15%. Based on this information, which of the following is the power of the proposed study?

0.85 The power of the proposed study refers to the probability of correctly rejecting the null hypothesis. By convention, most studies aim to achieve 80% statistical power. The statistical power can be calculated as follows: Power = 1 - β, where β is the probability of a type 2 error. In this question, β is 15%. Therefore, the power of this study would be 0.85 (Power = 1 - 0.15 = 0.85 or 85%).

5 days after receiving chemotherapy for ovarian cancer, a 74-year-old woman comes to the physician for a follow-up examination. She feels well and has no complaints. Her leukocyte count is 3,500/mm3 (11% neutrophils and 89% lymphocytes). This patient's profound granulocytopenia is most likely to predispose her to infection with which of the following organisms?

Absolute neutrophil count (ANC) is the product of leukocyte count and percent neutrophils. A leukocyte count of 3,500/mm3 with 11% neutrophils corresponds to an ANC of 385 cells/μL. An ANC below 500 cells/μL is classified as severe neutropenia. Enterococcus faecalis 49% Patients with severe neutropenia (including this patient with chemotherapy-induced neutropenia) are at increased risk of developing pyogenic and/or enteric bacterial infections. Enterococcus faecalis is an enteric gram-positive organism that is part of the normal GI flora and can cause serious nosocomial urinary tract and biliary tract infections, bacteremia, and infective endocarditis in neutropenic patients. Prolonged neutropenia (> 7 days) can predispose to fungal infections as well (especially Candida and Aspergillus species)

A 71-year-old woman comes to the physician for evaluation of a pigmented lesion on her left foot. She has had decreased appetite and a 6-kg (13-lb) weight loss in the past 2 months. Physical examination shows that on the left medial ankle there is a 2-cm reddish-black nodule with a verrucous surface. A full-thickness excisional biopsy of the lesion shows S100-protein-positive epithelioid cells. A subsequent CT scan of the chest, abdomen, and pelvis shows metastases to the liver; a diagnosis of metastatic melanoma is made. Chemotherapy with aldesleukin is initiated. Which of the following is the mechanism of action for this drug?

Aldesleukin is recombinant interleukin-2 (IL-2), a cytokine that is secreted by all T-cell subtypes. IL-2 triggers the destruction of tumor cells via the activation of cell types that recognize MHC I receptors. Activating natural killer cells and cytotoxic T cells IL-2 stimulates the activation of natural killer cells (NK cells) and cytotoxic T cells. These activated cells recognize the absence of MHC I receptors on infected or malignant cells and consequently destroy them. Aldesleukin is used in the therapy of both metastatic melanoma and renal cell carcinoma.

substantia nigra

An area of the midbrain that is involved in motor control and contains a large concentration of dopamine-producing neurons

A 47-year-old woman comes to the physician for a 2-month history of fatigue, intermittent left-sided flank pain, and diffuse extremity pain. She was treated for a prolactinoma 5 years ago and has had recurrent antral and duodenal peptic ulcers despite medical therapy. Her only medication is omeprazole. Physical examination shows a moderately distended abdomen that is diffusely tender to palpation. There is mild left-sided costovertebral angle tenderness. Serum studies show a calcium concentration of 12 mg/dL, phosphorus concentration of 2 mg/dL, and parathyroid hormone level of 426 pg/mL. An ultrasound of the abdomen shows multiple kidney stones in the left ureteropelvic junction. This patient is most likely to have which of the following protein abnormalities?

Altered menin protein 62% Altered menin protein function resulting from a mutation in the MEN1 gene is seen in multiple endocrine neoplasia type 1. MEN1 is characterized by the 3 P's: pituitary tumors (prolactinoma), parathyroid gland tumors (resulting in primary hyperparathyroidism), and pancreatic tumors (such as gastrinoma or insulinoma). MEN1 is also associated with angiofibromas and meningiomas. This patient has evidence of hyperparathyroidism (hypercalcemia, elevated PTH level) and a history of recurrent ulcers, which suggests Zollinger-Ellison syndrome. The constellation of parathyroid, pituitary, and pancreatic disease should raise suspicion for multiple endocrine neoplasia type 1 (MEN1). Gastrinoma Zollinger-Ellison syndrome Abbreviation: ZES A gastrin-secreting neuroendocrine tumor that is most often localized to the duodenum and pancreas. May occur as part of multiple endocrine neoplasia (MEN) syndrome. Malignant in more than half of patients. Typically presents with recurrent, therapy-resistant peptic ulcer disease and diarrhea due to elevated serum gastrin and low gastric pH.

A 78-year-old left-handed woman with hypertension and hyperlipidemia is brought to the emergency room because of sudden-onset right leg weakness and urinary incontinence. Neurologic examination shows decreased sensation over the right thigh. Muscle strength is 2/5 in the right lower extremity and 4/5 in the right upper extremity. Strength and sensation in the face are normal but she has difficulty initiating sentences and she is unable to write her name. The most likely cause of this patient's condition is an occlusion of which of the following vessels?

Anterior cerebral artery (ACA) strokes manifest with contralateral sensorimotor deficits (more pronounced in the lower extremity than in the upper extremity), urinary incontinence, and, less commonly, speech and language deficits (typically transcortical motor aphasia). Limb apraxia and abulia can also be seen. This patient's right-sided sensorimotor deficits, incontinence, and aphasia are consistent with a left ACA stroke. This patient's lower extremity deficits, urinary incontinence, and aphasia raise concern for a stroke in a specific vascular distribution.

A 74-year-old woman is brought to the physician by her daughter for worsening memory for the past 1 month. She can no longer manage her bills and frequently forgets the names of her children. Her daughter is also concerned that her mother has a urinary tract infection because she has had increased urinary urgency and several episodes of urinary incontinence. Vital signs are within normal limits. Physical examination shows poor short-term memory recall and a slow gait with wide, short steps. Which of the following is most likely to improve this patient's condition?

Apraxic ("magnetic") gait, dementia, and incontinence are the classic triad of normal pressure hydrocephalus. Cerebral shunt placement 69% Normal pressure hydrocephalus presents a risk of permanent neurological damage, due to compression of periventricular white matter from ventricular distention. Even removal of a small amount of CSF via a lumbar puncture can be helpful but the definitive treatment of normal pressure hydrocephalus usually involves the drainage of excess CSF via a cerebral shunt, usually into the peritoneum (e.g., VP shunt). Cerebral shunts are commonly used to treat hydrocephalus, the swelling of the brain due to excess buildup of cerebrospinal fluid (CSF). If left unchecked, the cerebrospinal fluid can build up leading to an increase in intracranial pressure (ICP) which can lead to intracranial hematoma, cerebral edema, crushed brain tissue or herniation.[1] The cerebral shunt can be used to alleviate or prevent these problems in patients who suffer from hydrocephalus or other related diseases. Donepezil therapy 17% Prescription of donepezil, a cholinesterase inhibitor, is an established treatment for mild to moderate Alzheimer disease or vascular dementia. Although this patient does show signs of dementia, her cognitive symptoms only developed recently and manifest alongside urinary incontinence, which would not be expected until the late stages of Alzheimer disease. In addition, gait abnormalities are not a typical feature of Alzheimer disease.

Two weeks after starting chemotherapy with etoposide, a 66-year-old man with small cell lung cancer comes to the physician because of persistent nausea and vomiting. His symptoms improve significantly after the administration of metoclopramide. This patient's symptoms were most likely due to the effect of chemotherapy on which of the following structures?

Area postrema 81% The area postrema is an area of the medulla that contains the chemoreceptor trigger zone (CTZ) and is one of the primary sites involved in chemotherapy-induced nausea and vomiting, along with the nucleus tractus solitarius. The area postrema of the brainstem has fenestrated capillaries, which allows input from substances in the blood such as drugs (e.g., chemotherapy) and/or hormones. Metoclopramide is a D2 dopamine receptor antagonist that acts primarily by inhibiting the vomiting centers of the area postrema. Chemotherapy induces nausea by acting on the chemoreceptor trigger zone, which responds to certain chemicals in the blood. Metoclopramide is a dopamine receptor antagonist that is especially effective in chemotherapy-induced nausea and vomiting.

A 42-year-old woman comes to the physician because of a 2-week history of joint pain and fatigue. She has a history of multiple unprovoked deep vein thromboses. Physical examination shows small bilateral knee effusions and erythematous raised patches with scaling and follicular plugging over the ears and scalp. Oral examination shows several small ulcers. Laboratory evaluation of this patient is most likely to show which of the following?

Arthritis, fatigue, discoid rash, and oral ulcers are suggestive of a diagnosis of systemic lupus erythematosus (SLE). This patient's history of unprovoked DVTs should raise concern for antiphospholipid syndrome. Positive rapid plasma reagin test 47% Rapid plasma reagin test (RPR) is a nonspecific screening test for syphilis that detects anticardiolipin antibodies. This patient with features of systemic lupus erythematosus and antiphospholipid syndrome likely has antibodies against cardiolipin (a common type of antiphospholipid antibodies), which react with RPR and VDRL test, resulting in false-positive testing for infection with Treponema pallidum. False-positive RPR testing can also be seen in infectious mononucleosis, hepatitis A, malaria, and pregnancy.

A 72-year-old man with coronary artery disease comes to the physician because of intermittent episodes of substernal chest pain and shortness of breath. The episodes occur only when walking up stairs and resolves after resting for a few minutes. He is a delivery man and is concerned because the chest pain has impacted his ability to work. His pulse is 98/min and blood pressure is 132/77 mm Hg. Physical examination is unremarkable. An ECG shows no abnormalities. A drug that blocks which of the following receptors is most likely to prevent future episodes of chest pain from occurring? This patient has stable angina and will benefit from a drug that reduces myocardial oxygen demand. Aldosterone receptor blockade (e.g., spironolactone, eplerenone) decreases sodium reabsorption and inhibits hydrogen and potassium excretion in the collecting ducts of the kidneys. These agents are commonly used in CHF and are proven to decrease mortality rates. However, aldosterone antagonists are not used for symptomatic treatment of stable angina.

Beta-1 adrenergic receptor blockade decreases heart rate and cardiac contractility (negative chronotropic and inotropic effect, respectively). This, in turn, decreases blood pressure and myocardial oxygen demand, and therefore anginal symptoms. Beta-1 receptor blockade also decreases AV node conduction (increased refractory period), giving drugs targeting this receptor an antiarrhythmic effect

A 38-year-old man with HIV infection comes to the physician for a follow-up examination. The patient is homeless and reports that he has not been taking his medications regularly. Examination shows multiple large, firm violaceous nodules, which erode easily, on his arms, legs, and trunk. His CD4+ T-lymphocyte count is 90/mm3. Treatment with oral erythromycin is begun. Three months later, the lesions have resolved. Which of the following is the most likely causal pathogen?

Biopsy of the nodules would show vascular proliferation and neutrophilic infiltrate. The causal pathogen is a gram-negative rod and can be visualized on biopsy using the Warthin-Starry stain. Bartonella henselae 68% In a patient with HIV infection and a CD4+ T-lymphocyte count < 100/mm3, multiple firm violaceous nodules or papules that erode easily and resolve after prolonged treatment with erythromycin or doxycycline suggest bacillary angiomatosis (BA), which is most commonly caused by Bartonella henselae. Patients infected with B. henselae usually have a history of contact with cats because the pathogen is typically transmitted from the feces of cat fleas via a cat scratch. Immunocompetent individuals remain asymptomatic or develop only a self-limiting infection with fever and granulomatous regional lymphadenopathy (cat scratch disease). Immunocompromised patients, however, can develop BA as a result of hematogenous dissemination. Bacillary angiomatosis A condition caused by Bartonella bacteria (e.g., Bartonella henselae) that most commonly develops in patients who are immunocompromised. Causes vascular proliferation, which leads to the development of solitary or multiple, red-purple papules that bleed easily. The Warthin-Starry stain (WS) is a silver nitrate-based staining method (a silver stain) used in histology. It was first introduced in 1920 by American pathologists Aldred Scott Warthin (1866-1931) and Allen Chronister Starry (1890-1973), for the detection of spirochetes.[1][2] It has been considered a standard stain for the detection of spirochetes,[3] and is also used to stain Helicobacter pylori, Lawsonia intracellularis,[4] Microsporidia,[5][6] and particulates.[7] It is also important for confirmation of Bartonella henselae, a causative organism in cat-scratch disease.

A 62-year-old man comes to the physician because of a 5-day history of swelling in his left arm. Two months ago, he was diagnosed with a deep venous thrombosis in the left calf. He has had a 7-kg (15-lb) weight loss in the last 3 months. He has smoked 1 pack of cigarettes daily for the past 25 years. His only medication is warfarin. Physical examination shows warm edema of the left forearm with overlying erythema and a tender, palpable cord-like structure along the medial arm. His lungs are clear to auscultation bilaterally. Duplex sonography shows thrombosis of the left basilic and external jugular veins. Which of the following is the most appropriate next step to confirm the underlying diagnosis?

CT scan of the abdomen 36% A CT scan of the abdomen is most likely to confirm the diagnosis of Trousseau syndrome, a condition that is characterized by recurrent venous thrombosis in different veins with evidence of simultaneous thrombophlebitis (warm, red, swollen skin with tender cord-like structures) as seen here. This patient's older age, smoking history, and weight loss raise concern for a malignancy-induced hypercoagulable state. Trousseau syndrome is most often seen in the setting of a visceral malignancy (classically, pancreatic adenocarcinoma) due to the production of procoagulants by the malignant cells. Thrombophlebitis migrans A form of superficial thrombophlebitis that is recurrent and migratory (i.e., occurring in various locations, resolving, then appearing elsewhere). Referred to as Trousseau syndrome if related to underlying malignancy (most commonly pancreatic adenocarcinoma and bronchial carcinoma). It is also a finding of thromboangiitis obliterans and Behçet disease.

Carboplatin

Carboplatin, sold under the trade name Paraplatin among others, is a chemotherapy medication used to treat a number of forms of cancer.[1] This includes ovarian cancer, lung cancer, head and neck cancer, brain cancer, and neuroblastoma.[1] It is used by injection into a vein.[1] Two theories exist to explain the molecular mechanism of action of carboplatin with DNA: Aquation, or the like-cisplatin hypothesis. Activation, or the unlike-cisplatin hypothesis. The former is more accepted owing to the similarity of the leaving groups with its predecessor cisplatin, while the latter hypothesis envisages a biological activation mechanism to release the active Pt2+ species.

A 60-year-old woman is brought to the emergency department because of sudden, painless loss of vision in her right eye that occurred 30 minutes ago while watching TV. She has coronary artery disease, hypertension, and type 2 diabetes mellitus, and has had trouble adhering to her medication regimen. Her blood pressure is 160/85 mm Hg. Examination shows 20/50 vision in the left eye and no perception of light in the right eye. Direct pupillary reflex is present in the left eye but absent in the right eye. Accommodation is intact bilaterally. Intraocular pressure is 16 mm Hg in the left eye and 18 mm Hg in the right eye. Fundoscopic examination of the right eye shows a pale, white retina with a bright red area within the macula. The optic disc appears normal. Which of the following is the most likely diagnosis?

Central retinal artery occlusion 74% The patient has multiple risk factors for retinal artery disease, including diabetes, hypertension, and atherosclerosis. Central retinal artery occlusion (CRAO) is characterized by sudden, painless loss of vision and an afferent pupillary defect. Ophthalmoscopy findings of CRAO include retinal pallor and a cherry-red spot in the foveal region. Narrow retinal vessels with a cattle-truck appearance are another classic finding of retinal artery occlusion.

Two weeks after starting chemotherapy with etoposide, a 66-year-old man with small cell lung cancer comes to the physician because of persistent nausea and vomiting. His symptoms improve significantly after the administration of metoclopramide. This patient's symptoms were most likely due to the effect of chemotherapy on which of the following structures?

Chemotherapy induces nausea by acting on the chemoreceptor trigger zone, which responds to certain chemicals in the blood. Metoclopramide is a dopamine receptor antagonist that is especially effective in chemotherapy-induced nausea and vomiting. The area postrema is an area of the medulla that contains the chemoreceptor trigger zone (CTZ) and is one of the primary sites involved in chemotherapy-induced nausea and vomiting, along with the nucleus tractus solitarius. The area postrema of the brainstem has fenestrated capillaries, which allows input from substances in the blood such as drugs (e.g., chemotherapy) and/or hormones. Metoclopramide is a D2 dopamine receptor antagonist that acts primarily by inhibiting the vomiting centers of the area postrema.

A 68-year-old man is brought to the emergency department 25 minutes after he was found shaking violently on the bathroom floor. His wife reports that he has become increasingly confused over the past 2 days and that he has been sleeping more than usual. He was started on chemotherapy 4 months ago for chronic lymphocytic leukemia. He is confused and oriented to person only. Neurological examination shows right-sided ptosis and diffuse hyperreflexia. An MRI of the brain shows disseminated, nonenhancing white matter lesions with no mass effect. A polymerase chain reaction assay of the cerebrospinal fluid confirms infection with a virus that has double-stranded, circular DNA. An antineoplastic drug with which of the following mechanisms of action is most likely responsible for this patient's current condition?

Chronic lymphocytic leukemia Abbreviation: CLL A low-grade, non-Hodgkin, B-cell lymphoma that presents with lymphocytosis. The most common leukemia in adults, especially in advanced age. Monoclonal antibody against CD20+ 68% Rituximab is a chimeric monoclonal antibody against the B-cell marker CD20 that is used in the treatment of CLL. This drug is associated with an increased risk of PML, a demyelinating disease of the CNS caused by the destruction of oligodendrocytes secondary to reactivation of JC virus. Other drugs associated with an increased risk of PML include natalizumab, which is used to treat multiple sclerosis and Crohn disease. PML predominantly develops in patients with severe immunosuppression (e.g., patients with AIDS or those undergoing certain immunosuppressive therapy) and may present with decreased cognitive function, focal neurological deficits, and seizures, as seen here.

A 38-year-old woman comes to the physician for the first time because of a 2-year history of lower back pain and fatigue. She also says that she occasionally feels out of breath. Her symptoms are not associated with physical activity. She has seen multiple physicians over the past year. Extensive workup including blood and urine tests, abdominal ultrasound, MRI of the back, and cardiac stress testing have shown no abnormalities. The patient asks for a medication to alleviate her symptoms. Which of the following is the most appropriate response by the physician?

Chronic symptoms that do not fit a specific diagnosis, visits to multiple physicians, and negative findings after an extensive workup are suggestive of a somatic symptom disorder. Regularly scheduled office visits with the same physician are recognized as the most effective treatment for somatic symptom disorder. Psychotherapy (e.g., cognitive behavioral therapy) is an additional component of first-line treatment. This patient has already had a comprehensive medical workup, suggesting that an organic cause of her symptoms is very unlikely and that further diagnostic steps would not be appropriate.

A 63-year-old man with inoperable esophageal carcinoma undergoes palliative chemoradiotherapy. Four hours after his first infusion of carboplatin and paclitaxel, he develops nausea and 3 episodes of vomiting and dry heaving. This adverse reaction is caused by stimulation of a brain region on the floor of the fourth ventricle. Chemotherapeutic drugs are able to stimulate this region because of the absence of a cell junction that is composed of which of the following proteins?

Claudins and occludins 63% Claudins and occludins are key components of the tight junctions that form the blood-brain barrier. The tight junctions seal adjacent epithelial cells together, which prevents paracellular transport of ions and molecules. The chemoreceptor trigger zone lacks tight junctions and is thus highly permeable. As a result, several chemotherapeutic drugs can freely cross the blood-brain barrier and cause nausea.

A 23-year-old man comes to the emergency department for 2 days of episodic headaches. The pain is most intense on his left forehead and eye. He had similar symptoms last summer. He has been taking indomethacin every 6 hours for the last 24 hours but has not had any relief. He has smoked 1 pack of cigarettes daily for the past 5 years. He works as an accountant and describes his work as very stressful. He appears restless. Physical examination shows drooping of the left eyelid, tearing of the left eye, and rhinorrhea. The left pupil is 2 mm and the right pupil is 4 mm. There is localized tenderness along the right supraspinatus muscle. The remainder of the examination shows no abnormalities. Which of the following is the most likely diagnosis?

Cluster headache 83% Cluster headache is characterized by agonizing, unilateral periorbital or temporal pain. It is more common in men and attacks are commonly accompanied by lacrimation and rhinorrhea. Pupillary changes can be seen as part of partial Horner syndrome (no decreased vision). Cluster headache does not respond to NSAIDs but is treated with oxygen and triptans.

A 3000-g (6-lb 10-oz) male newborn delivered at 38 weeks' gestation develops respiratory distress shortly after birth. Physical examination shows low-set ears, retrognathia, a flattened nose, and club feet. Within a few hours, the newborn dies. Examination of the liver at autopsy shows periportal fibrosis and dilatation of the peripheral intrahepatic ducts. This patient's condition is most likely associated with which of the following?

Compression of the fetus secondary to maternal oligohydramnios results in club feet and craniofacial abnormalities such as retrognathia and low-set ears (Potter sequence). Insufficient amniotic fluid also causes pulmonary hypoplasia, which results in respiratory distress after birth. The concurrent presence of periportal fibrosis is the key to the correct answer! Pulmonary hypoplasia An underdevelopment of the lungs characterized by a decreased number of alveoli and airways. Result in an impaired gas exchange. Associated with oligohydramnios and Potter sequence. Cystic dilation of collecting ducts 25% This neonate has autosomal recessive polycystic kidney disease (ARPKD), which is caused by a mutation in the fibrocystin (PKHD1) gene on the short arm of chromosome 6. This mutation leads to cystic dilation of the distal nephron and collecting ducts. ARPKD can cause in-utero renal failure, subsequently leading to maternal oligohydramnios and Potter sequence. ARPKD is also associated with progressive hepatic fibrosis, which can cause portal hypertension and liver failure.

A 32-year-old woman comes to the physician for a screening health examination that is required for scuba diving certification. The physician asks her to perform a breathing technique: following deep inspiration, she is instructed to forcefully exhale against a closed airway and contract her abdominal muscles while different cardiovascular parameters are evaluated. Which of the following effects is most likely after 10 seconds in this position?

Decreased left ventricular stroke volume 50% A sustained Valsalva maneuver reduces the left ventricular stroke volume. When the Valsalva maneuver is initiated, four phases are typically seen. Phase 1 (onset of straining) and phase 2 (continuation of straining) describe the changes that occur during active expiration against a closed glottis, while phase 3 (release of straining) and phase 4 (recovery phase) follow the release of pressure. The increased intrathoracic pressure seen in phase 2 reduces total venous return, and increases systemic vascular resistance, both of which reduce stroke volume. Once pressure is released, i.e., in phases 3 and 4, the sudden release of pressure would increase stroke volume.

Which of the following can be found in bacterial endospores?" (Answer is dipicolinic acid

Dipicolinic acid (pyridine-2,6-dicarboxylic acid or PDC and DPA) is a chemical compound which plays a role in the heat resistance of bacterial endospores. Dipicolinic composes 5% to 15% of the dry weight of bacterial spores.[2][3] It has been implicated as responsible for the heat resistance of the endospore,[2][4] although mutants resistant to heat but lacking dipicolinic acid have been isolated, suggesting other mechanisms contributing to heat resistance are at work.[5] Two genera of bacterial pathogens are known to produce endospores: the aerobic Bacillus and anaerobic Clostridium.[6] Dipicolinic acid forms a complex with calcium ions within the endospore core. This complex binds free water molecules, causing dehydration of the spore. As a result, the heat resistance of macromolecules within the core increases. The calcium-dipicolinic acid complex also functions to protect DNA from heat denaturation by inserting itself between the nucleobases, thereby increasing the stability of DNA.[7]

An investigator is developing a drug that selectively inhibits the retrograde axonal transport of rabies virus towards the central nervous system. To achieve this effect, this drug must target which of the following?

Dynein is an ATP-dependent motor protein responsible for the retrograde migration and movement of cellular organelles and proteins along microtubules in axons. After rabies exposure, the rabies virus binds to acetylcholine receptors and enters the motor and sensory nerves of the peripheral nervous system, spreading in a retrograde direction to the CNS. To inhibit the migration of the virus and subsequent brain infection, the new drug must therefore target dynein motors. Nicotinic acetylcholine receptors (not acetylcholine itself) have been implicated as the binding site of the rabies virus to axons. A drug targeting these receptors could inhibit the entry of rabies virus into neurons, but it would not selectively impair the retrograde axonal transport towards the CNS.

A 19-year-old male soccer player undergoes an exercise tolerance test to measure his maximal oxygen uptake during exercise. Which of the following changes are most likely to occur during exercise?

Exercise increases cardiac output and pulmonary blood flow. Decreased physiologic dead space 49% Physiologic dead space consists of the anatomic dead space and the alveolar dead space. In the resting state, the majority of pulmonary blood flow goes to the base of the lungs due to gravity, and therefore alveolar dead space primarily localizes to the lung apices. During exercise, vasodilation of the pulmonary vessels in the apices occurs. Additionally, the increased right ventricular cardiac output increases pulmonary circulatory pressure, allowing the opening of apical blood vessels that were collapsed at rest. These processes increase apical lung perfusion, thereby decreasing alveolar dead space and subsequently, physiologic dead space.

A 34-year-old woman comes to the physician because of a 6-week history of depressed mood, loss of interest, and difficulty sleeping. She also has had a 4.5-kg (10-lb) weight loss during this period. She has not been as productive as before at work due to difficulty concentrating. There is no evidence of suicidal ideation. Laboratory studies including thyroid-stimulating hormone are within the reference range. The physician prescribes treatment with escitalopram. This drug targets a neurotransmitter that is produced in which of the following brain structures?

Escitalopram is a selective serotonin reuptake inhibitor (SSRI) used in the treatment of major depressive disorder. Raphe nucleus 68% The raphe nucleus in the midbrain is the primary site of serotonin production in the central nervous system. A decreased serotonin concentration is associated with depression. Therefore, increasing its concentration in the synaptic cleft by inhibiting its reuptake, as mediated by drugs of the SSRI class (e.g. escitalopram), is an effective treatment for depression. Other conditions associated with low serotonin concentrations are anxiety disorders and Parkinson disease.

A 69-year-old man comes to the physician because of a 1-week history of blood in the urine and fatigue. He also has had a 5.0-kg (11-lb) weight loss during the past month. Physical examination shows pallor and cachexia. A nontender right flank mass is palpated. A CT scan of the chest, abdomen, and pelvis shows a 5-cm right upper pole renal mass and several pulmonary lesions. A biopsy specimen of an affected area of the lung is obtained. A photomicrograph of the biopsy specimen is shown. Molecular evaluation of the specimen is most likely to show which of the following genetic changes?

Fatigue, hematuria, weight loss, and a palpable renal mass are highly suggestive of renal cell carcinoma (RCC). The lung is a common site of metastasis. VHL gene deletion 74% Deletions of the VHL tumor suppressor gene are a common cause of both sporadic and familial renal cell carcinoma (RCC). Most cases of RCC are sporadic, although a small percentage are associated with VHL gene deletions in hereditary disorders such as von Hippel-Lindau disease. Histologic examination of tissue shows polygonal cells with clear cytoplasm due to the accumulation of lipids and carbohydrates (clear cells). Gross specimens often appear yellow as a result of high lipid content. VHL gene A tumor suppressor gene on chromosome 3 that is deleted in Von Hippel-Lindau disease and in some patients with renal cell carcinoma.

Bitot's spots

Foamy plaques of the cornea that are a sign of vitamin A deficiency Bitot's spots are the buildup of keratin located superficially in the conjunctiva of human's eyes. They can be oval, triangular or irregular in shape. The spots are a sign of vitamin A deficiency and associated with drying of the cornea.

A 63-year-old woman comes to the office because of a 2-year history of upper and lower extremity weakness and neck pain that is worse with sneezing. She has had difficulty swallowing and speaking for the past 8 months. Musculoskeletal examination shows spasticity and decreased muscle strength in all extremities. There is bilateral atrophy of the trapezius and sternocleidomastoid muscles. Neurologic examination shows an ataxic gait and dysarthria. Deep tendon reflexes are 4+ bilaterally. Babinski sign is positive. Sensation is decreased below the C5 dermatome bilaterally. An MRI of the neck and base of the skull is shown. Which of the following is the most likely cause of this patient's symptoms?

Foramen magnum meningioma 67% This patient most likely has a foramen magnum meningioma, as indicated by the MRI. Although the falx cerebri, the supratentorial convex surface, and the sphenoidal wings are the most common locations of meningiomas, in rare cases they also occur at the foramen magnum. Early features include neck pain, which is typically worse with neck flexion and maneuvers that increase ICP (e.g., sneezing). Later, patients present with long tract signs (e.g., sensory and motor deficits that progress from one extremity to the others). Other late findings include spasticity (hyperreflexia, positive Babinski sign), gait disturbances, and lower cranial nerve palsies (CN IX-XII), which most likely accounts for this patient's dysarthria and atrophy of the sternocleidomastoid and trapezius muscles. Microscopic examination shows psammoma bodies and concentrically arranged spindle cells.

heart valve

Heart valves allow blood to flow in only one direction through the heart. Your heart has four valves that keep blood flowing in the correct direction. These valves include the mitral valve, tricuspid valve, pulmonary valve and aortic valve. Each valve has flaps (leaflets or cusps) that open and close once during each heartbeat

A 4-year-old girl is brought to the physician for evaluation of a rash that her mother noticed 5 months ago. The rash is not painful or itchy but she notices that her daughter sometimes picks at the "spots." The girl's 2-year-old brother has also started developing similar skin lesions over the past month. The patient has no history of serious illness and takes no medications. She is in the 75th percentile for height and 50th percentile for weight. A photograph of the patient's rash is shown. Which of the following infectious agents is the most likely cause of this patient's skin condition?

Histological evaluation of the skin lesions would show keratinocytes with eosinophilic intracytoplasmic inclusion bodies, also known as molluscum bodies. Poxvirus 84% The molluscum contagiosum virus is a DNA poxvirus that typically causes multiple nontender, skin-colored, pearly, dome-shaped papules with central umbilication, which can be located on the face, trunk, and extremities, as seen in this patient. Molluscum contagiosum most commonly affects children under 5 years of age. The virus is transmitted through direct skin contact, which explains why the patient's younger brother is also infected. Typically, the lesions are self-limiting within a few months.

A 36-year-old woman is brought to the emergency department because of lightheadedness, weakness, and abdominal pain for 6 hours. Over the past 3 days, she has also had severe nausea, vomiting, and watery diarrhea. She was diagnosed with pulmonary sarcoidosis 2 years ago. Current medications include prednisone. Her temperature is 38.9°C (102.0°F), pulse is 112/min, and blood pressure is 85/50 mm Hg. Physical examination shows a round face with prominent preauricular fat pads. Her fingerstick blood glucose concentration is 48 mg/dL. Further evaluation is most likely to show which of the following laboratory changes?

Hypotension, hypoglycemia, and abdominal pain in a patient on prolonged high-dose glucocorticoid therapy (as is evident in her Cushingoid facies) should raise concern for acute adrenal insufficiency (adrenal crisis). Adrenal crisis is a severe, acute glucocorticoid deficiency that is usually precipitated by a trigger such as infection, dehydration, trauma, or surgery. It can also occur when glucocorticoids are abruptly discontinued and/or their absorption is impaired, e.g., due to acute gastroenteritis, as is likely the case here. Decreased corticotropin-releasing hormone 58% Decreased corticotropin-releasing hormone (CRH) is caused by prolonged iatrogenic suppression of the hypothalamic-pituitary-adrenal axis from chronic glucocorticoid use. Decreased production and release of CRH leads to decreased endogenous ACTH production and, in turn, insufficient serum cortisol levels in the absence of exogenous glucocorticoids. Increased adrenocorticotropic hormone 15% Adrenocorticotropic hormone (ACTH) would be decreased rather than increased in adrenal crisis due to exogenous glucocorticoid replacement. Chronic glucocorticoid use suppresses the production and release of hormones that physiologically stimulate cortisol production in the adrenal cortex, including ACTH. In contrast, there would be a compensatory increase in ACTH secretion in the case of primary adrenal insufficiency, which occurs when the adrenal gland itself is damaged (e.g., in adrenal hemorrhage, adrenal infarction).

A 28-year-old man comes to the physician because of a four-week history of progressively worsening fatigue, nausea, and right upper quadrant pain. He has a history of intravenous heroin use. Hepatitis B surface antigen, hepatitis B envelope antigen, and hepatitis B core antibodies are positive. Further analysis of the hepatitis B core immunoglobulin is most likely to show which of the following properties?

IgM antibody to hepatitis B core antigen (IgM anti-HBc): Positivity indicates recent infection with hepatitis B virus (<6 mos). Its presence indicates acute infection. Formation of a pentamer when secreted IgM is the first immunoglobulin produced in response to an antigen. The valency of antibodies (i.e., the number of arms available to bind antigen) differs among the different classes. IgM is a monomer on B cells and functions as an antigen receptor. However, it forms a pentamer through the linkage of Fc domains via a J chain when secreted.

An investigator studying targeted therapy in patients with gastrointestinal stromal tumors requires a reliable test to determine the spatial distribution of CD117-positive cells in biopsy specimens. Which of the following is the most appropriate test?

Immunohistochemistry 47% Immunohistochemistry uses antigen-antibody interactions to identify a specific cell or tissue based on the presence or absence of a certain antigen. In this study, the CD117 (c-KIT) serves as an antigen that characterizes GIST cells. An anti-CD117-antibody that is coupled with a chromogenic substrate will bind on the cell surface and visualize the tumor cells using ordinary light microscopy. This technique is best for studying special cell populations within biopsy samples due to the preservation of tissue architecture.

A previously healthy 26-year-old woman comes to the physician for numbness and tingling in her legs for the past 2 months. Her only medication is an oral contraceptive. Examination shows conjunctival pallor and dry scaly lips with cracks at the corner of the mouth. Sensation to pinprick and light touch is decreased in both lower limbs. Her hemoglobin concentration is 10.0 g/dL and mean corpuscular volume is 76 μm3. Bone marrow biopsy shows erythroblasts with a ring of basophilic granules around the nucleus. This patient's condition is caused by the deficiency of a substrate that acts as a cofactor in which of following steps of catecholamine synthesis?

In a patient taking OCPs, clinical features of peripheral neuropathy (paresthesias, decreased sensation in the extremities) and sideroblastic anemia (microcytic hypochromic anemia with sideroblasts in the bone marrow) suggest a vitamin B6 deficiency. C 48% DOPA decarboxylase catalyzes the conversion of L-DOPA to dopamine in the presence of the cofactor pyridoxal phosphate (active form of vitamin B6), which also serves as a cofactor in multiple other reactions, such as cystathionine synthesis, heme synthesis, decarboxylation, and transamination reactions. Clinical features of vitamin B6 deficiency include glossitis, gingivostomatitis, peripheral neuropathy, and sideroblastic anemia, as observed in the patient.

A 42-year-old woman comes to the physician with acute, severe pain in the middle of her lower back. She also complains of constipation and trouble sleeping recently. Menses occur regularly at 28-day intervals. Examination shows localized tenderness to palpation over the lumbar spine. Serum calcium is 14 mg/dL and serum phosphorus is 1.5 mg/dL. An x-ray of the lumbar spine shows a compression fracture of the L4 vertebral body and osteopenia. Which of the following is the most likely underlying cause of this patient's decreased bone mineral density? Decreased ovarian estrogen production can cause postmenopausal osteoporosis, which may manifest with vertebral compression fractures. However, this patient is having regular menstrual cycles and is, therefore, not menopausal. The decreased bone mineral density seen here is caused by parathyroid hormone-mediated osteoclast activation, not decreased estrogen.

Increase in interleukin-1 secretion 42% Parathyroid hormone (PTH) induces interleukin-1 (IL-1) secretion in osteoblasts, which activates osteoclasts. This induces bone resorption and the release of calcium into circulation. In addition, PTH increases the renal excretion of phosphorus, leading to diminished serum phosphorus levels. This patient's vertebral compression fracture is likely caused by osteoclast-mediated bone resorption.

A 44-year-old woman comes to the physician because of a 6-month history of fatigue, constipation, and a 7-kg (15.4-lb) weight gain. Menses occur irregularly in intervals of 40-50 days. Her pulse is 51/min, and blood pressure is 145/86 mm Hg. Examination shows conjunctival pallor and cool, dry skin. There is mild, nonpitting periorbital edema. Serum thyroid-stimulating hormone concentration is 8.1 μU/mL. Treatment with the appropriate pharmacotherapy is initiated. After several weeks of therapy with this drug, which of the following hormonal changes is expected?

Increased reverse T3 54% Levothyroxine is a synthetic form of T4, which is converted to reverse T3 (biologically inactive) and T3 (biologically active) by a deiodinase in the blood. Since an increase of reactants leads to an increase in products, rT3 would be expected to increase after several weeks of levothyroxine therapy. T3 and T4 negatively regulate the secretion of TRH from the hypothalamus and of TSH from the anterior pituitary.

A 66-year-old woman with chronic obstructive pulmonary disease is brought to the emergency department because of fever, body aches, malaise, and a dry cough. She has smoked one pack of cigarettes daily for 30 years but quit smoking 1 year ago. She lives with her daughter and her granddaughter, who attends daycare. Her temperature is 38.1°C (101°F). Physical examination shows bilateral conjunctivitis, rhinorrhea, and erythematous tonsils without exudates. Further testing confirms infection with an enveloped orthomyxovirus. Administration of a drug with which of the following mechanisms of action is most appropriate?

Inhibition of neuraminidase 88% Neuraminidase is an enzyme expressed by influenza virus that promotes virus release from host cells during replication. Neuraminidase inhibitors, e.g., oseltamivir and zanamivir, inhibit budding and release of the progeny influenza virus and can be used for both treatment and prevention of disease. Early initiation of treatment (within 48 hours of symptom onset) can shorten the duration of disease.

A 26-year-old woman comes to the physician for a follow-up examination 1 week after undergoing rotator cuff repair surgery. Physical examination of the right shoulder shows decreased sensation to pinprick and light touch over the lateral shoulder. There is normal passive range of motion of the shoulder, but she has difficulty externally rotating and abducting her right arm. Which of the following structures was most likely injured during the surgery?

Injury to this structure can also occur after delayed reduction of an anterior shoulder dislocation. Axillary nerve 70% Injury to the axillary nerve results in impaired abduction (from 15-100°) due to deltoid paralysis, limited external rotation due to teres minor paralysis, and sensory deficits over the skin of the lateral deltoid. Because the axillary nerve circumflexes the neck of the humerus, injury to the nerve from excessive traction or direct injury can result from rotator cuff surgery, fracture of the surgical neck of the humerus, or anterior dislocation of the humerus. Axillary nerve 70% Injury to the axillary nerve results in impaired abduction (from 15-100°) due to deltoid paralysis, limited external rotation due to teres minor paralysis, and sensory deficits over the skin of the lateral deltoid. Because the axillary nerve circumflexes the neck of the humerus, injury to the nerve from excessive traction or direct injury can result from rotator cuff surgery, fracture of the surgical neck of the humerus, or anterior dislocation of the humerus.

A 70-year-old man comes to the physician because of a 4-month history of epigastric pain, nausea, and weakness. He has smoked one pack of cigarettes daily for 50 years and drinks one alcoholic beverage daily. He appears emaciated. He is 175 cm (5 ft 9 in) tall and weighs 47 kg (103 lb); BMI is 15 kg/m2. He is diagnosed with gastric cancer. Which of the following cytokines is the most likely direct cause of this patient's examination findings? Cancer anorexia-cachexia syndrome results from a cytokine that stimulates increased metabolism and the production of an acute-phase reactant.

Interleukin 1, interleukin 6, interferon-gamma, and tumor necrosis factor-alpha play an important role in the development of cancer anorexia-cachexia syndrome. These cytokines, which are secreted as a chronic inflammatory response to malignancy, increase the basal metabolic rate in cancer patients. The combination of decreased caloric intake and a hypermetabolic state results in significant weight loss from muscle atrophy.

A 12-year-old girl is brought to the physician by her mother 2 hours after cutting her hand while playing in the yard. Examination of the right hand shows a 2-cm laceration on the thenar region of the palm with some surrounding tenderness and erythema. The right palm appears mildly swollen in comparison to the left. In response to this patient's injury, the endothelial cells lining the blood vessels of the affected area express increased numbers of cellular adhesion molecules. Which of the following mediators is most likely directly responsible for the described change?

Interleukin-1 44% In the adhesion step of the acute inflammation process, IL-1 and TNF-alpha promote the upregulation of cellular adhesion molecules on the endothelium. These cellular adhesion molecules interact with integrins on leukocytes, leading to the adhesion of leukocytes to the endothelium.

A 47-year-old man is brought to the emergency department 30 minutes after he was involved in a motor vehicle collision. On arrival, he is unconscious and unresponsive to painful stimuli. His pulse is 120/min, respirations are 10/min, and his blood pressure is 88/60 mm Hg. Infusion of 0.9% saline is begun, and intubation is attempted without success. Pulse oximetry on 20 L/min of oxygen via bag-mask shows an oxygen saturation of 78%. The most appropriate next step in the management involves passing a tube through an incision in which of the following structures?

Intubation has failed in this increasingly hypoxemic patient. The next step requires establishing a surgical airway. Investing layer of deep cervical fascia 22% Incision of the investing layer of deep cervical fascia is necessary to perform the cricothyroidotomy needed to establish an airway in this patient. In emergency situations where a patient cannot be effectively ventilated by the conventional bag and mask ventilation or endotracheal intubation, a cricothyroidotomy is the surgical procedure of choice to restore oxygenation. A cricothyroidotomy is performed by vertically incising the skin (superficial cervical fascia, investing layer of deep cervical fascia, and pretracheal fascia) and then horizontally incising the cricothyroid membrane.

A 13-year-old girl is brought to the physician because of worsening fever, headache, photophobia, and nausea for 2 days. One week ago, she returned from summer camp. She has received all age-appropriate immunizations. Her temperature is 39.1°C (102.3°F). She is oriented to person, place, and time. Physical examination shows a maculopapular rash. There is rigidity of the neck; forced flexion of the neck results in involuntary flexion of the knees and hips. Cerebrospinal fluid studies show: Opening pressure120 mm H2OAppearanceClearProtein47 mg/dLGlucose68 mg/dLWhite cell count280/mm3Segmented neutrophils15%Lymphocytes85% Which of the following is the most likely causal organism?

Involuntary flexion of the knees and hips in response to forced flexion of the neck is referred to as a positive Brudzinski sign. Together with fever, headaches, photophobia, and neck stiffness, this suggests acute meningitis. CSF findings of normal opening pressure, clear appearance, slightly elevated protein, normal glucose, and lymphocytic pleocytosis are consistent with aseptic meningitis. Echovirus 56% Echovirus and other enteroviruses are typically responsible for summer outbreaks of aseptic meningitis, especially in children and teenagers. Enteroviruses are the most common cause of aseptic meningitis overall. Aseptic meningitis commonly manifests with a nonspecific rash, fever, headache, nuchal rigidity, and a positive Brudzinski sign. Herpes simplex virus 20% Herpes simplex virus (HSV) is a common cause of encephalitis and meningitis in children and teenagers. HSV encephalitis is mainly caused by HSV-1, whereas HSV meningitis is typically caused by HSV-2. Either can manifest with fever, headache, photophobia, nuchal rigidity, and lymphocytic pleocytosis with slightly elevated proteins in CSF. However, another virus is statistically more common than HSV-2 as a cause of aseptic meningitis. Furthermore, since HSV-1 mainly affects the temporal lobe, focal neurological deficits such as altered sense of smell, aphasia, memory loss, or ataxia would be expected. Other typical symptoms include seizures, altered mental status, and behavioral changes. Neisseria meningitidis 14% Neisseria meningitidis is the most common cause of meningitis in children and teenagers, as well as the main cause of meningitis on college campuses. It usually manifests with fever, headache, nuchal rigidity, and signs of meningeal irritation. A rash is present in some cases. However, bacterial meningitis would manifest more acutely and with more severe features, e.g., altered mental status or seizures. CSF would have a cloudy appearance, elevated opening pressure, neutrophilic pleocytosis, elevated protein, low glucose, and elevated lactate.

A 42-year-old woman, gravida 5, para 5, comes to the physician because of a 6-month history of occasional involuntary urine loss that is exacerbated by coughing, sneezing, and laughing. She has no urgency or dysuria. Physical examination shows normal appearing external genitalia, vagina, and cervix. There is a loss of urine with the Valsalva maneuver. The physician recommends doing Kegel exercises. Which of the following muscles is strengthened by these exercises?

Kegel exercises consist of voluntary contractions of all the pelvic floor muscles, with the intention of strengthening the striated muscles of the region. Levator ani 70% The levator ani forms a major portion of the pelvic floor, along with smaller muscles (e.g., coccygeus muscle) and connective tissue. It is comprised of three parts: the iliococcygeus, pubococcygeus, and puborectalis muscles, which all contract in synchrony. Contraction of the pelvic floor through Kegel exercises strengthens these muscles, along with the ischiocavernosus and bulbospongiosus muscles in men. Strengthening the pubococcygeus improves urinary retention in both sexes and is recommended as a first-line measure in the treatment of urinary incontinence. Strengthening of these muscles may also help men with premature ejaculation.

A 60-year-old man comes to the office because of an 8-month history of cough, night sweats, shortness of breath, and fatigue. He has also had a 9-kg (19.8-lb) weight loss during this time. He appears pale. Abdominal examination shows hepatosplenomegaly. His leukocyte count is 80,000/mm3 and his leukocyte alkaline phosphatase level is increased. A peripheral blood smear shows > 82% neutrophils with band forms and immature and mature neutrophil precursors. An x-ray of the chest shows a 9-mm right hilar nodule. Which of the following is the most likely cause of this patient's laboratory findings?

Leukemoid reaction A leukemoid reaction is characterized by profound leukocytosis with a proportionate increase in all elements of the myeloid lineage (neutrophils, immature and mature precursors) and elevated leukocyte alkaline phosphatase (LAP), as seen in this patient. B symptoms, chronic fatigue, and hepatosplenomegaly are indicative of an underlying malignancy. The pulmonary nodule on chest x-ray, along with cough and shortness of breath are highly suggestive of a leukemoid reaction secondary to lung cancer. Leukemoid reactions can mimic leukemia but are typically due to infections (e.g., C. difficile colitis, whooping cough), drugs (e.g., glucocorticoids), and certain solid tumors (e.g., kidney cancer, Hodgkin lymphoma, and melanoma). B symptoms The presence of fever, night sweats, and weight loss, which together are classically associated with lymphomas. Sometimes used to refer to the same symptoms in patients with other conditions (e.g., tuberculosis, inflammatory conditions, rheumatologic conditions).

A 35-year-old woman with irritable bowel syndrome comes to the physician because of increased diarrhea. She has not had any fever, bloody stools, nausea, or vomiting. The increase in stool frequency began when she started a new job. She is started on loperamide, and her symptoms improve. Which of the following is the primary mechanism of action of this drug?

Loperamide is in the same drug class as diphenoxylate, fentanyl, and dextromethorphan. μ-opioid receptor agonism 79% Loperamide is an antidiarrheal drug that acts directly on μ-opioid receptors in the GI tract to inhibit propulsive peristalsis and intestinal fluid secretion, and increase anal sphincter tone. Unlike other μ-opioid receptor agonists, loperamide has poor CNS penetration and therefore lacks analgesic properties

A 21-year-old man comes to the physician's office due to a 3-week history of fatigue and a rash, along with the recent development of joint pain that has moved from his knee to his elbows. The patient reports going camping last month but denies having been bitten by a tick. His past medical history is significant for asthma treated with an albuterol inhaler. His pulse is 54/min and blood pressure is 110/72. Physical examination reveals multiple circular red rings with central clearings on the right arm and chest. There is a normal range of motion in all joints and 5/5 strength bilaterally in the upper and lower extremities. Without proper treatment, the patient is at highest risk for which of the following complications?

Lyme disease is caused by Borrelia burgdorferi, which is typically transmitted by the bite of the Ixodes tick. Camping is, therefore, a risk factor for Lyme disease. Patients typically present with a painless, circular, slowly expanding red ring around the bite site after an incubation period of 7-14 days (early localized Lyme disease). However, in approx. 20% of patients the initial rash at the bite site may not be present. If left untreated, patients will develop early disseminated Lyme disease. Cranial nerve palsy, most commonly Bell palsy, is often a finding of early disseminated (stage II) Lyme disease. Other features of this stage of Lyme disease include migratory arthralgia, polyneuropathy, meningitis, and the risk of developing cardiac arrhythmia due to an atrioventricular block. This patient should have an urgent electrocardiogram to evaluate for an AV block in the setting of his bradycardia.

A 75-year-old man with a seizure disorder is brought to the emergency department by a friend because of progressive confusion over the past two weeks. He is unable to provide any history. His vital signs are within normal limits. He appears lethargic and is only oriented to person. Oral mucosa is moist. There is no jugular venous distention. A basic metabolic panel shows a serum sodium concentration of 115 mEq/L but is otherwise normal. Serum osmolality is low and antidiuretic hormone level is elevated. X-ray of the chest shows no abnormalities. Which of the following is the most likely cause of this patient's hyponatremia?

Medication effect 74% SIADH is an adverse effect of several medications, including carbamazepine, SSRIs, and cyclophosphamide and is characterized by an inappropriately elevated serum ADH despite low plasma osmolality (e.g., the patient is not hypovolemic). Risk factors for developing medication-induced SIADH include older age (> 65 years old), concomitant use of other antiseizure drugs and diuretics, and a history of hyponatremia. Other causes of SIADH include CNS disorders, pulmonary disease, and ectopic production of ADH (e.g., small cell lung cancer). Cyclophosphamide An alkylating agent used to treat many solid tumors, leukemias, lymphomas, and multiple myeloma and as an immunosuppressant to treat autoimmune diseases (e.g., rheumatoid arthritis, systemic lupus erythematosus) and severe nephrotic syndrome. Adverse effects include myelosuppression, syndrome of inappropriate antidiuretic hormone secretion (SIADH), and hemorrhagic cystitis (risk reduced with prophylactic administration of mesna and aggressive hydration).

A 74-year-old man comes to the physician because of a 2-month history of shortness of breath and leg swelling. He has trouble climbing one flight of stairs due to dyspnea. He has hypertension and was diagnosed with multiple myeloma 6 months ago. Current medications include hydrochlorothiazide. Cardiopulmonary examination shows jugular venous distention, bibasilar crackles, and pitting edema in the lower extremities. Transthoracic echocardiography shows a thickened myocardium with elevated diastolic filling pressures. Which of the following is the most likely underlying mechanism of this patient's cardiomyopathy?

Multiple myeloma is characterized by the production of monoclonal immunoglobulins. Amyloid deposition 69% Amyloid plaque is a common sequela of multiple myeloma and forms as a result of excess production of monoclonal immunoglobulins. Immunoglobulins can deposit in any organ, including the heart, in which case they cause a specific type of restrictive cardiomyopathy known as infiltrative cardiomyopathy. Thickening of the myocardium results in diastolic dysfunction and back-flow of blood causes shortness of breath, jugular venous distention, and pulmonary edema, which are all seen here.

A 21-year-old woman comes to the physician because of a 5-day history of pain with urination and vaginal itching. She is sexually active with multiple partners and uses condoms inconsistently. Pelvic examination shows erythema of the vulva and vaginal mucosa, punctate hemorrhages on the cervix, and green-yellow, malodorous discharge. A photomicrograph of the discharge is shown. Which of the following is the most likely causal organism?

Microscopic examination of a wet mount preparation of this patient's vaginal discharge shows extracellular, motile, flagellated protozoa. trichomonas vaginalis 82% Trichomonas vaginalis infection causes trichomoniasis, which manifests with vaginal pruritus, dysuria, strawberry cervix (punctate hemorrhages on the cervix), and green-yellow, malodorous vaginal discharge as seen in this patient. The presence of motile trophozoites on wet mount preparation confirms the diagnosis. Vaginal discharge typically has a pH > 4.5. Risk factors for infection include multiple sexual partners and inconsistent condom use. Trichomoniasis Trichomonal vaginitis A sexually transmitted infection of the vulva and vagina that is caused by Trichomonas vaginalis, an anaerobic, motile protozoan with flagella. Presents with foul-smelling, frothy, yellow-green, discharge as well as vulvovaginal pruritus, a burning sensation, dyspareunia, dysuria, and a "strawberry" cervix (erythematous cervical mucosa with petechiae).

A 32-year-old woman comes to the physician because of a 3-month history of recurrent headaches and nausea. The headaches occur a few times a month and alternately affect the right or left side. The headaches are exacerbated by loud sounds or bright light. She is in graduate school and has been under a lot of stress recently. She does not smoke or drink alcohol but does drink 2-3 cups of coffee daily. Her only medication is an oral contraceptive. Physical examination shows no abnormalities; visual acuity is 20/20. Which of the following is the most likely diagnosis?

Migraine headache Migraine headaches are recurrent throbbing headaches that are frequently unilateral and classically accompanied by nausea, vomiting, and sensitivity to light and sound. Migraine headaches may also localize in an alternating pattern, as seen in this patient. Risk factors include stress, poor sleep hygiene, and oral contraceptive use.

A 44-year-old man comes to the physician because of weakness in his right hand for the past 3 months. He works as a violinist in a professional orchestra. Physical examination shows atrophy of the thenar and dorsal interosseus muscles of the right hand. Sensation to pinprick and vibration is normal. A cross-section of the spinal cord at the level of C6 is shown. The most likely cause of this patient's symptoms is damage to which of the following labeled structures?

Muscle atrophy and weakness are signs of lower motor neuron damage. Structure G, the right dorsolateral column in the anterior horn, is responsible for this patient's symptoms. This patient's presentation of atrophy restricted to the thenar eminence is a common early sign of amyotrophic lateral sclerosis (ALS), known as split-hand syndrome. While the corticospinal tracts (structures C and H) are also often damaged in ALS, this patient has no signs of upper motor neuron damage (e.g., increased muscle tone and reflexes).

A 29-year-old man is brought to the emergency room 6 hours after the onset of severe epigastric pain and vomiting. His heart rate is 110/min and blood pressure is 98/72 mm Hg. He is diagnosed with acute pancreatitis, and fluid resuscitation with normal saline is initiated. Which of the following is the most likely immediate effect of fluid resuscitation in this patient? Fluid resuscitation would increase venous return and right ventricular end diastolic volume (EDV).

Myocardial oxygen demand depends on the heart rate, heart contractility, and wall tension (affected by afterload). An increase in end-diastolic volume, as occurs in fluid resuscitation, causes an increase in cardiac preload by stretching cardiac myocyte sarcomeres, which results in increased myocardial contractility based on the Frank-Starling mechanism. Increased contractility also increases cardiac output (CO) and cardiac work, and hence myocardial oxygen demand as well. Note, however, that fluid resuscitation is beneficial in hypovolemic patients because it also increases myocardial oxygen supply by improving coronary perfusion pressure.

A 23-year-old man comes to the physician for evaluation of decreased hearing, dizziness, and ringing in his right ear for the past 6 months. Physical examination shows multiple soft, yellow plaques and papules on his arms, chest, and back. There is sensorineural hearing loss and weakness of facial muscles bilaterally. His gait is unsteady. An MRI of the brain shows a 3-cm mass near the right internal auditory meatus and a 2-cm mass at the left cerebellopontine angle. The abnormal cells in these masses are most likely derived from which of the following embryological structures?

Neural crest 77% Schwann cells are formed from neural crest cells, which arise from the lateral border of the neural plate (neuroectoderm) and migrate along somites to different parts of the body. Other structures derived from neural crest cells are neurons of cranial nerves III to XII, peripheral nerve neurons (both somatic and autonomic), skull bones, pia and arachnoid mater, odontoblasts, laryngeal cartilage, adrenal medulla, melanocytes, aorticopulmonary septum, and endocardial cushion. An immunohistochemical marker of neural crest cells is S-100, which is positive in melanomas, Langerhans cell histiocytosis, and schwannomas.

A 64-year-old man who recently immigrated to the United States from Haiti comes to the physician because of a 3-week history of progressively worsening exertional dyspnea and fatigue. For the past few days, he has also had difficulty lying flat due to trouble breathing. Over the past year, he has had intermittent fever, night sweats, and cough but he has not been seen by a physician for evaluation of these symptoms. His temperature is 37.8°C (100°F). An x-ray of the chest is shown. Further evaluation of this patient is most likely to show which of the following findings?

New-onset dyspnea, orthopnea, and fatigue and pericardial calcifications on x-ray indicate constrictive pericarditis. This patient who hails from Haiti and has a prolonged history of night sweats, fever, cough most likely has tuberculosis, which can cause constrictive pericarditis. Constrictive pericarditis A condition characterized by chronic pericardial inflammation with subsequent thickening and rigidity of the pericardium, which results in decreased ventricular compliance and diastolic heart failure. Causes include idiopathic or viral pericarditis, radiotherapy, cardiac surgery, connective tissue diseases, and tuberculosis. Manifestations include fatigue, dyspnea, jugular vein distention (with a prominent y descent), peripheral edema, Kussmaul sign, pulsus paradoxus, and a pericardial knock. Kussmaul sign A sign characterized by distention of the jugular veins during inspiration (due to elevation of jugular venous pressure). Can be seen in patients with constrictive pericarditis, restrictive cardiomyopathy, right ventricular infarction, and tricuspid stenosis. Jugular venous distention on inspiration 44% Normally, the decrease in intrathoracic pressure during inspiration expands the compliant right ventricle resulting in a decrease in jugular venous pressure. In constrictive pericarditis, compliance of the right ventricle is decreased and the right ventricle fails to expand during inspiration. Therefore, the jugular venous pressure, which might already be increased in constrictive pericarditis due to restricted right ventricular filling during late diastole, would increase further during inspiration. This phenomenon, which is called Kussmaul sign, can also be seen in patients with restrictive cardiomyopathy and right atrial or ventricular tumors.

A 52-year-old man with Crohn disease comes to the physician because of a 1-week history of dysuria and urinary urgency. He had been treated for urinary tract infections twice in the past 6 months. He is currently taking infliximab. Cystoscopy shows pearl-like plaques on the bladder wall. Biopsy of these lesions shows a focal layer of stratified squamous epithelium with hyperkeratosis. If the patient's bladder finding is caused by a vitamin deficiency, which of the following features is also most likely to be seen? The biopsy finding of stratified squamous epithelium with hyperkeratosis suggests a diagnosis of squamous metaplasia of the bladder urothelium. Vitamin A is required for proper differentiation of specialized epithelial cells (e.g., columnar cells of the pancreatic duct, respiratory tract, and conjunctiva, or urothelial cells). Vitamin A deficiency can result in metaplasia of these cells to keratinizing cells.

Night blindness 78% Vitamin A deficiency can cause ocular disorders such as xerophthalmia, Bitot spots, corneal xerosis, keratomalacia, and night blindness. Other signs of vitamin A deficiency include poor bone growth, hyperkeratosis, xeroderma, immunosuppression, and squamous metaplasia, as seen in this patient. Vitamin A deficiency has a higher prevalence in patients with disorders of fat malabsorption, such as Crohn disease.

A 62-year-old woman comes to the physician for decreased vision and worsening headaches since this morning. She has hypertension and hypercholesterolemia. Pulse is 119/min and irregular. Current medications include ramipril and atorvastatin. Ocular and funduscopic examination shows no abnormalities. The findings of visual field testing are shown. Which of the following is the most likely cause of this patient's symptoms?

Occlusion of the posterior cerebral artery 76% Occlusion of the posterior cerebral artery, usually due to emboli from the vertebral vessels, aorta, or heart, results in contralateral homonymous hemianopia with macular sparing. The macula is spared because the macular tracts have additional blood supply from the middle cerebral arteries. Visual field testing shows the inability to see the right visual field in either eye (right homonymous hemianopia) with macular sparing, which most commonly results from decreased blood flow to the left occipital lobe.

A 73-year-old woman recently diagnosed with colonic adenocarcinoma comes to the physician because of a 1-week history of nausea and multiple episodes of vomiting. These symptoms started shortly after her first infusion of oxaliplatin and fluorouracil. The patient is started on an appropriate medication. Three weeks later, at a follow-up appointment, she states that she has developed headaches and constipation. The patient was most likely treated with a drug with which of the following mechanisms of action?

Ondansetron is a common antiemetic drug, with adverse effects that include headache and constipation. 5-HT3 receptor antagonist 60% 5-HT3 antagonists, such as ondansetron, have a central-acting antiemetic effect and cause peripheral inhibition of the intestinal tract's vagal nerve signals. They are especially effective in relieving chemotherapy-induced and postoperative nausea and vomiting and are commonly used as first-line antiemetics. Possible side effects include headaches, constipation or diarrhea, and elevated liver enzymes. In severe cases, they may also cause QT interval prolongation (which may lead to torsades de pointes) and serotonin syndrome; caution should therefore be exercised in patients receiving frequent doses.

A 53-year-old man is brought to the emergency department because of wheezing and shortness of breath that began 1 hour after he took a new medication. Earlier in the day he was diagnosed with stable angina pectoris and prescribed a drug that irreversibly inhibits cyclooxygenase-1 and 2. He has chronic rhinosinusitis and asthma treated with inhaled β-adrenergic agonists and corticosteroids. His respirations are 26/min. Examination shows multiple small, erythematous nasal mucosal lesions. After the patient is stabilized, therapy for primary prevention of coronary artery disease should be switched to a drug with which of the following mechanisms of action?

P2Y12 receptor antagonists (e.g., clopidogrel) irreversibly inhibit platelet aggregation by preventing the ADP-mediated formation of the GpIIb/IIIa receptor complex. Clopidogrel is used for treatment of stable angina pectoris in patients with aspirin allergy as well as adjunctive therapy in acute coronary syndromes. Other indications include secondary prevention of recent stroke, recent myocardial infarction, and peripheral arterial disease.

Misoprostol

PGE1 derivative: orally active prostaglandin used to prevent peptic ulcers in patients taking NSAIDs for arthritis. Tox: diarrhea Misoprostol, a prostaglandin analogue, binds to myometrial cells to cause strong myometrial contractions leading to expulsion of tissue. This agent also causes cervical ripening with softening and dilation of the cervix. Misoprostol binds to and stimulates prostaglandin EP2 receptors, prostaglandin EP3 receptor and prostaglandin EP4 receptor but not Prostaglandin EP1 receptor and therefore is expected to have a more restricted range of physiological and potentially toxic actions than prostaglandin E2 or other analogs which activate all four prostaglandin receptors.[43]

involution

decrease in size

palmar grasp reflex

Palmar grasp reflex (sometimes simply grasp reflex) is a primitive reflex found in infants of humans and most primates. When any object is placed in an infant's palm, the fingers flex reflexively around the object.[1] The grip is strong but unpredictable;[2] though it may be able to support the child's weight, they may also release their grip suddenly and without warning. The reverse motion can be induced by stroking the back or side of the hand.[3] Reflex A fetus exhibits the reflex in utero by 28 weeks into gestation (sometimes, as early as 16 weeks[4]),[5][6][7] and persists until development of rudimentary fine motor skills between two to six months of age Biologists have found that the reflex is significantly more frequent in infants of fur carrying primate species. They theorize that the grasping reflex evolved as it is essential to survival in species where the young are carried in the fur. This suggests that the grasping reflex is vestigial in humans and in other non-fur carrying primates.[10]

A pathologist receives a patient sample for analysis. Cells in the sample are first labeled with fluorescent antibodies and then passed across a laser beam in a single file of particles. The light scatter and fluorescent intensity of the particles are plotted on a graph; this information is used to characterize the sample. This laboratory method would be most useful to establish the diagnosis of a patient with which of the following?

Paroxysmal nocturnal hemoglobinuria (PNH) can present with pancytopenia, deep vein thrombosis, and intermittent hemoglobinuria. It usually results from impaired synthesis of the glycosylphosphatidylinositol anchor (GPI), which links cell surface proteins like CD55 and CD59 to the plasma membrane. These surface proteins are inhibitors of complement activation; their absence permits inappropriate complement-mediated destruction of cells. Flow cytometry is used to detect the reduced expression of CD55 and CD59 on RBCs and confirm the diagnosis of PNH.

A 27-year-old man is brought to the emergency department because of a knife wound to his back. His pulse is 110/min, respirations are 14/min, and blood pressure is 125/78 mm Hg. Examination shows a 5-cm deep stab wound at the level of T9. He withdraws the right foot to pain but is unable to sense vibration or whether his right toe is flexed or extended. Sensation in the left leg is normal. Motor strength is 5/5 in all extremities. Rectal tone is normal. Which of the following spinal column structures was most likely affected?

Posterior spinal artery 51% Injury to the right posterior spinal artery leads to posterior spinal artery syndrome due to ischemia and subsequent infarction of the posterior column, which is directly supplied by this artery. Affected patients present with absent ipsilateral proprioception and vibration sensation below the level of the wound. In most cases, sensation to light touch is preserved. This patient's normal motor function and sensation to pain rule out any further injury to the corticospinal tract and spinothalamic tract. A unilateral loss of proprioception and vibration with preservation of motor function and pain sensation indicates isolated damage to this patient's dorsal column.

A 15-year-old girl is brought to the physician because of a 8-month history of fatigue, intermittent postprandial abdominal bloating and discomfort, foul-smelling, watery diarrhea, and a 7-kg (15-lb) weight loss. She developed a pruritic rash on her knees 3 days ago. Physical examination shows several tense, excoriated vesicles on the knees bilaterally. The abdomen is soft and nontender. Her hemoglobin concentration is 8.2 g/dL and mean corpuscular volume is 76 μm3. Further evaluation of this patient is most likely to show which of the following findings?

Postprandial abdominal discomfort and bloating, chronic diarrhea, and severe microcytic anemia (likely secondary to iron deficiency) point to a malabsorptive syndrome. The additional presence of dermatitis herpetiformis (several tense, grouped subepidermal blisters) is strongly suggestive of celiac disease. IgA tissue transglutaminase antibodies 84% Testing for IgA tissue transglutaminase antibodies is the gold standard for diagnosing celiac disease and is also useful in monitoring a patient's response to treatment, as antibody levels start to decline and usually normalize 3-12 months following the introduction of a gluten-free diet. Although celiac disease can occur at any age, it most commonly manifests either in early childhood or between 20-40 years of age.

pudendal nerve compression

Pudendal nerve entrapment syndrome is caused by compression of the pudendal nerve as it leaves or enters the pelvis in various tunnels created by adjacent muscles, tendons or bony and ligamentous tissues. In this condition the nerve is most commonly compressed at: The space between sacrotuberous and sacrospinous ligaments (~70% cases) Within the pudendal canal of Alcock (~20% cases) While straddling of the falciform process of the sacrotuberal ligament by the pudendal nerve and its branches Anywhere along the course of the pudendal nerve or its branches It is thought that changes in the shape and position of the ischial spine occur in young cyclists. This predisposes them to pudendal nerve entrapment in later years especially if they continue to cycle for prolonged periods. The most common causes for pudendal nerve entrapment syndrome include: Repeated mechanical injury (eg, sitting on bicycle seats for prolonged periods over many years or months) Trauma to the pelvic area, for example during childbirth Damage to the nerve during surgical procedures in the pelvic or perineal regions Compression from lesions or tumours arising in the pelvis Any cause for the development of peripheral neuropathy (eg, diabetes or vasculitis).

A 62-year-old woman comes to the physician because of worsening mental status over the past month. Her husband reports that she was initially experiencing lapses in memory but has recently started having difficulties performing activities of daily living. She appears withdrawn and avoids eye contact. Examination shows diffuse involuntary muscle jerking that can be provoked by loud noises. A cerebrospinal fluid analysis shows elevated concentration of 14-3-3 protein. Four months later, the patient dies. Pathologic examination of the brain on autopsy is most likely to show which of the following findings?

Rapidly progressive mental deterioration (dementia, behavioral abnormalities, altered mood), startle myoclonus, and elevated 14-3-3 protein concentration in the cerebrospinal fluid are highly suggestive of Creutzfeldt-Jakob disease. Spongiform vacuolation of the cortex 76% The presence of intracytoplasmic vacuoles within neurons of the cerebral and cerebellar cortex (spongiform encephalopathy) is the characteristic histological feature of Creutzfeldt-Jakob disease (CJD) and other prion diseases. In addition to the classic features displayed by this patient, CJD can also lead to akinetic mutism, seizures, and neuropsychiatric symptoms (e.g., visual hallucinations, depression). The disease has a rapid course that often leads to death within 12 months. Deposits of amyloid beta peptides 8% Extracellular plaques composed of amyloid beta peptides and intracellular accumulation of hyperphosphorylated tau protein (neurofibrillary tangles) are typical histological features of Alzheimer disease (AD). Although AD initially manifests with short-term memory loss, which is seen in this patient, other classic findings are absent here, e.g., impaired naming and language comprehension or temporospatial disorientation. Most importantly, AD has an insidious onset and progresses slowly over several years, whereas this patient's symptoms progressed rapidly within 4 months. In addition, elevated levels of 14-3-3 protein in CSF are not characteristic of AD.

A group of researchers conducted a study to determine whether there is an association between folic acid supplementation before pregnancy and autism spectrum disorder (ASD) in offspring. The researchers retrospectively surveyed 200 mothers with children diagnosed with ASD during the first 4 years of life and 200 mothers with healthy children. All participants were interviewed about their prenatal consumption of folic acid using standardized questionnaires. A 94% response rate was obtained from the surveys. The study ultimately found that folic acid supplementation was associated with lower rates of ASD in offspring (OR = 0.3, p < 0.01). Which of the following type of bias is most likely to have influenced these results? A significant period of time (4 years) has passed between the prenatal exposure being studied here and the administration of the questionnaire. FEEDB

Recall bias 88% Recall bias is a major concern in case-control studies, as subjects with a particular outcome may unintentionally or intentionally recall past exposures in more detail than those without the outcome. This difference in recall may lead to misclassification of exposure and false conclusions about the relationship between exposure and outcome.

Babinski reflex

Reflex in which a newborn fans out the toes when the sole of the foot is touched The Babinski sign can indicate upper motor neuron lesion constituting damage to the corticospinal tract

carotid sinus

Respond to blood pressure, Carotid Body responds to partial pressures of O2/CO2, supplied by glossopharyngeal nerve

A 22-year-old man comes to the physician because of abdominal pain, diarrhea, and weight loss that started after a recent backpacking trip in Southeast Asia. He does not smoke or drink alcohol. His leukocyte count is 7,500/mm3 (61% segmented neutrophils, 13% eosinophils, and 26% lymphocytes). Stool microscopy shows rhabditiform larvae. This patient is most likely to develop which of the following?

Rhabditiform larvae on stool microscopy, eosinophilia, and a recent travel history to Southeast Asia are consistent with Strongyloides stercoralis (threadworm) infection. erianal serpiginous rash 35% Strongyloides stercoralis is a nematode endemic to Southeast Asia that is transmitted via larval penetration of intact skin (usually when bare feet come into contact with contaminated soil). Larvae travel to the alveoli through the bloodstream, where they ascend the pulmonary system before being coughed up and swallowed. Once in the GI tract, they become adult female worms and reproduce via parthenogenesis, with the eggs hatching as rhabditiform larvae. Rhabditiform larvae then become infective filariform larvae, penetrating intestinal mucosa or the skin of the perianal region, which completes the process of autoinfection. This results in a rapidly migrating (5 cm or more per hour) pink, raised track, referred to as larva currens; this is pathognomonic of Strongyloides infection.

A 47-year-old woman comes to the physician because of a 5-month history of insomnia. She frequently experiences leg discomfort when trying to fall asleep that is relieved temporarily by movement. Her husband tells her that she frequently flexes her ankles upward when she sleeps. She appears fatigued and anxious. Physical examination shows no abnormalities. Laboratory studies including a complete blood count and iron studies are within the reference range. Which of the following is the most appropriate pharmacotherapy?

Ropinirole and other dopamine agonists are considered first-line pharmacological agents for restless leg syndrome. Pharmacologic therapy is reserved for cases of RLS in which symptoms result in functional impairment and sleep disturbance, or when symptoms persist despite behavioral interventions (e.g., sleep hygiene, leg massage) and/or correction of any iron deficiency. Dopamine receptor agonists A class of drugs that activate dopamine D1 and/or D2 receptors. Typically used to treat Parkinson disease, prolactinoma, or restless legs syndrome. Adverse effects include nausea, drowsiness, orthostatic hypotension, confusion, hallucinations, impulse control disorders, and compulsive drug use (dopamine dysregulation syndrome).

A 60-year-old man is brought to the emergency room because of fever and increasing confusion for the past 2 days. He has paranoid schizophrenia treated with chlorpromazine. He appears diaphoretic. His temperature is 40°C (104°F), pulse is 130/min, respirations are 29/min, and blood pressure is 155/100 mm Hg. Neurologic examination shows psychomotor agitation and incoherent speech. There is generalized muscle rigidity. His deep tendon reflexes are decreased bilaterally. Serum laboratory analysis shows a leukocyte count of 11,300/mm3 and serum creatine kinase concentration of 833 U/L. The most appropriate drug for this patient acts by inhibiting which of the following?

Ryanodine receptor on the sarcoplasmic reticulum Dantrolene is a muscle relaxant that prevents the release of calcium into the myocyte cytoplasm by inhibiting the ryanodine receptor on the sarcoplasmic reticulum. After discontinuation of the offending drug (i.e., chlorpromazine), dantrolene is indicated in the treatment of NMS because it decreases muscle rigidity and thereby body temperature and the risk of rhabdomyolysis. Although dantrolene is most commonly given, alternative pharmacotherapy includes bromocriptine, amantadine, and possibly apomorphine. Benzodiazepines are also usually given to patients suffering from NMS with psychomotor agitation, as seen here. Neuroleptic malignant syndrome (NMS) is a rare reaction to antipsychotic drugs that treat schizophrenia, bipolar disorder, and other mental health conditions. It affects the nervous system and causes symptoms like a high fever and muscle stiffness. The condition is serious, but it's treatable.

A 62-year-old man comes to the physician for a follow-up examination. One month ago, therapy with lisinopril was initiated for treatment of hypertension. His blood pressure is 136/86 mm Hg. Urinalysis shows a creatinine clearance of 92 mL/min. The patient's serum creatinine concentration is most likely closest to which of the following values?

Since creatinine is freely filtered and minimally secreted or reabsorbed, creatinine clearance can be used to estimate GFR. A glomerular filtration rate (GFR) of 90 mL/min/1.73 m2 or above is considered normal for a male. 1.1 mg/dL 45% Creatinine is produced by myocytes and serum levels are maintained via continuous renal excretion. Since creatinine is freely filtered and minimally secreted or reabsorbed, creatinine clearance can be used for the estimated glomerular filtration rate (eGFR). The Cockcroft-Gault equation is used to calculate serum creatinine from the eGFR, although it slightly overestimates the serum creatinine because it ignores minimal creatinine secretion at the proximal tubule. As a general rule, every time GFR halves, serum creatinine roughly doubles. Given this patient's creatinine clearance of 92 mL/min, we can estimate that he has a normal GFR (> 90 mL/min/1.73m2 for a male), which means his serum creatinine is in the normal range (0.6-1.2 mg/dL for males).

A 46-year-old man comes to the emergency department because of a 10-day history of right upper quadrant abdominal pain. He has also been feeling tired and nauseous for the past 6 weeks. On examination, scleral icterus is present. Abdominal examination shows tenderness to palpation in the right upper quadrant. The liver edge is palpated 2 cm below the right costal margin. Laboratory studies show: Aspartate aminotransferase1780 U/LAlanine aminotransferase2520 U/LHepatitis A IgM antibodynegativeHepatitis B surface antigennegativeHepatitis B surface antibodynegativeHepatitis B core IgM antibodypositiveHepatitis C antibodypositiveHepatitis C RNAnegative Which of the following is the most appropriate treatment for this patient?

Sudden onset of nausea, jaundice, fever, and right upper quadrant pain with massively elevated serum transaminase levels (> 1000 U/L) suggests a diagnosis of acute viral hepatitis. The presence of anti-HBc IgM antibodies without concurrent HBsAg or anti-HBs antibodies indicates that this patient is in the window period of acute hepatitis B infection. Supportive therapy 33% Supportive therapy is all that is required for a patient with acute hepatitis B infection. A negative HBsAg with positive anti-HBc IgM antibodies indicate that the infection is resolving. Anti-HBs antibody titers may not increase for up to 1-2 weeks following the disappearance of HbsAg from serum (window period), which is why measuring anti-HBc antibody titers in patients with suspected acute hepatitis B infection is important.

A 64-year-old man with osteoarthritis of the knee comes to the physician for evaluation of weakness in his foot. Physical examination shows a swelling in the popliteal fossa. There is marked weakness when attempting to invert his right foot. He is unable to curl his toes. Further evaluation of this patient is most likely to show decreased sensation over which of the following locations?

Swelling of the popliteal fossa can lead to compression of the inlying structures, including the tibial nerve. Compression of the tibial nerve can result in neuropraxia, causing marked motor weakness of foot inversion and toe plantarflexion, as well as sensory deficits in the tibial nerve distribution. Sole of the foot 61% The sole of the foot, lateral foot, and posterolateral leg are innervated by sensory branches of the tibial nerve (nerve stem from L4-S3). The posterior compartment of the leg, responsible for foot inversion and plantarflexion of the toes and ankle, is innervated by motor branches of the tibial nerve. This patient's neuropraxia of the tibial nerve secondary to popliteal swelling would diminish the sensory and motor functions of these regions.

Horner's syndrome

Sympathetic trunk/cervical ganglion damage, results in ptosis, pupil constriction, vasodilation, and absence of sweating Horner syndrome is due to a deficiency of sympathetic activity. The site of lesion to the sympathetic outflow is on the ipsilateral side of the symptoms.

Giant cell arteritis (GCA), also called temporal arteritis, is an inflammatory disease of large blood vessels.[4][7] Symptoms may include headache, pain over the temples, flu-like symptoms, double vision, and difficulty opening the mouth.[3] Complication can include blockage of the artery to the eye with resulting blindness, aortic dissection, and aortic aneurysm.[4] GCA is frequently associated with polymyalgia rheumatica.[4] Polymyalgia rheumatica (PMR) is a syndrome with pain or stiffness, usually in the neck, shoulders, upper arms, and hips, but which may occur all over the body. The pain can be very sudden, or can occur gradually over a period. Most people with PMR wake up in the morning with pain in their muscles;

Temporal arteritis

Sideroblastic anemia

defect in mitochondrial heme synthesis producing ringed sideroblasts Sideroblastic anemia is a group of blood disorders characterized by an impaired ability of the bone marrow to produce normal red blood cells . In this condition, the iron inside red blood cells is inadequately used to make hemoglobin, despite normal amounts of iron.

An investigator is studying the mechanism of HIV infection in cells obtained from a human donor. The effect of a drug that impairs viral fusion and entry is being evaluated. This drug acts on a protein that is cleaved off of a larger glycosylated protein in the endoplasmic reticulum of the host cell. The protein that is affected by the drug is most likely encoded by which of the following genes?

The structural gene env encodes for the proteins gp120 and gp41. These two proteins are required for initial infection and entry into host cells by the HIV virion. The virus attaches to CD4+ receptors on T cells via its gp120 glycoprotein, while fusion and entry are mediated via the gp41 glycoprotein. These vital proteins have become a target for drugs like enfuvirtide, a fusion inhibitor that competitively binds to the viral protein gp41 and thereby prevents fusion of HIV with its target cell.

right upper quadrant

liver, gallbladder

A 35-year-old woman comes to the emergency department because of a 3-day history of pain in the left cheek. The pain occurs every few hours, lasts 30-60 seconds, and is aggravated by chewing and brushing. She has a history of pain with a vesicular rash in the right axillary area 1 year ago. She had an upper respiratory infection 2 weeks ago. Physical examination shows no abnormalities. Which of the following is the most likely diagnosis?

Trigeminal neuralgia is a chronic pain syndrome characterized by transient (< 1 minute), recurrent, severe shooting, stabbing, and/or paroxysmal pain that can occur up to 100 times per day in the distribution of the trigeminal nerve. It most commonly manifests with pain shooting from the mouth to the angle of the jaw. It can be triggered by a variety of stimuli, including brushing, chewing, cold, and touch. It is not related to this patient's prior rash and upper respiratory infection.

A 58-year-old man is brought to the emergency department because of sudden-onset right-sided body numbness for 1 hour. He reports that he has a 15-year history of hypertension treated with hydrochlorothiazide. He is alert and oriented to time, place, and person. Neurological examination shows decreased sensation to light pinprick and temperature on the right side of the face and body. Motor strength is 5/5 and deep tendon reflexes are 2+ bilaterally. Perfusion of which of the following structures of the brain is most likely impaired in this patient?

Ventral thalamus 51% A pure sensory stroke is caused by diminished perfusion of the ventral posterolateral and ventral posteromedial nucleus of the thalamus, which relays sensory information of proprioception, pain, pressure, and temperature to the primary somatosensory cortex. Affected individuals generally present with one-sided numbness of the face and body (contralateral to the affected side of the thalamus), as observed in this patient. Chronic hypertension and the resulting lipohyalinosis of small vessels are major risk factors for lacunar infarctions which include thalamic strokes. Pure sensory stroke A clinical variant of a lacunar stroke, usually caused by lesions in the thalamus or posterior limb of the internal capsule. Patients present solely with contralateral numbness and paresthesia of the face, arm, and leg. This patient's symptoms and risk factor (hypertension) suggest that he has a pure sensory stroke.

Volar surface

Volar: Pertaining to the palm or the sole. For example, the volar surface of the forearm is the portion of the forearm that is on the same side as the palm of the hand.

intrasellar mass

What is Intrasellar? Within the sella turcica. Extended Definition: The sella turcica is the bony structure at the base of the skull in which the pituitary gland rests

Kluver-Bucy syndrome

a condition, brought about by bilateral amygdala damage, that is characterized by dramatic emotional changes including reduction in fear and anxiety Klüver-Bucy syndrome is a syndrome resulting from bilateral lesions of the medial temporal lobe (including amygdaloid nucleus).[1] Klüver-Bucy syndrome may present with compulsive eating, hypersexuality, insertion of inappropriate objects in the mouth (hyperorality), visual agnosia, and docility.

Gerstmann syndrome

agraphia, aclaculia, finger agnosia, left-right disorientation. Damage to dominant- parietal temporal cortex. Gerstmann syndrome is characterized by four primary symptoms, collectively referred to as a tetrad: Dysgraphia/agraphia: deficiency in the ability to write[3][4] Dyscalculia/acalculia: difficulty in learning or comprehending mathematics[3][4] Finger agnosia: inability to distinguish the fingers on the hand[3][4] Left-right disorientation[3][4]

prolapse

an organ slips out of its normal position

Sarcoidosis

autoimmune disease with fibrous lesions forming in lymph nodes, liver, skin, lungs, spleen, eyes, and small bones of hands and feet

melena

blood in stool

cardiopulmonary resuscitation

car·di·o·pul·mo·nar·y re·sus·ci·ta·tion noun a medical procedure involving repeated compression of a patient's chest, performed in an attempt to restore the blood circulation and breathing of a person who has suffered cardiac arrest.

glial cells

cells in the nervous system that support, nourish, and protect neurons macroglial astrocytes, oligodendrodendrycytes, ependydemal cells

Purkinje cells

cerebellum

Anaplastic

characterized by a loss of differentiation of cells and their orientation to one another; a characteristic of malignant tumors

DNA viruses

enter the host cell's nucleus and are replicated and assembled there

euchromatin vs heterochromatin

euchromatin: uncoiled genetic material (lighter stained) where mRNA is transcribed heterochromatin: coiled genetic material that is transcriptionally inactive

Gonioscopy

examination of the angle of the anterior chamber of the eye to determine ocular motility and rotation and diagnose and manage glaucoma

hyperacusis

excessively sensitive hearing

extensor pollicis brevis

extends thumb radial nerve

Vulva

external female genitalia; includes the labia, hymen, clitoris, and vaginal orifice The vulva is the outer part of the female genitals. The vulva includes the opening of the vagina (sometimes called the vestibule), the labia majora (outer lips), the labia minora (inner lips), and the clitoris. Around the opening of the vagina, there are 2 sets of skin folds.

Valsalva maneuver

forcible exhalation against a closed glottis, resulting in increased intrathoracic pressure The Valsalva maneuver is performed by moderately forceful attempted exhalation against a closed airway, usually done by closing one's mouth, pinching one's nose shut while expelling air out as if blowing up a balloon.

tonic-clonic seizure

generalized seizure in which the patient loses consciousness and has jerking movements of paired muscle groups

Adductor pollicis

is the most powerful of the intrinsic muscles of the hand. Its main function is the adduction of the thumb which is the movement of the thumb towards the index finger from an abducted position. This action is essential for functions that require pinching and gripping. ulnar nerve The adductor pollicis is innervated by the deep branch of the ulnar nerve (C8-T1). Between the oblique and transverse heads is a thin fibrous arcade which the nerve passes as it traverses the palm laterally. The nerve is accompanied by the deep palmar arch.

Leukocytosis

increase in the number of white blood cells

Leukomoid reaction

increased alk phos, increased white blood cell count

hyperemia

increased blood flow

Spasticity

increased muscle tone

C-peptide

indicates amount of endogenous insulin formed

Atropine

muscarinic antagonist In general, atropine counters the "rest and digest" activity of glands regulated by the parasympathetic nervous system. This occurs because atropine is a competitive, reversible antagonist of the muscarinic acetylcholine receptors (acetylcholine being the main neurotransmitter used by the parasympathetic nervous system). Atropine is a competitive antagonist of the muscarinic acetylcholine receptor types M1, M2, M3, M4 and M5.[28] It is classified as an anticholinergic drug (parasympatholytic). In cardiac uses, it works as a nonselective muscarinic acetylcholinergic antagonist, increasing firing of the sinoatrial node (SA) and conduction through the atrioventricular node (AV) of the heart, opposes the actions of the vagus nerve, blocks acetylcholine receptor sites, and decreases bronchial secretions. In the eye, atropine induces mydriasis by blocking contraction of the circular pupillary sphincter muscle, which is normally stimulated by acetylcholine release, thereby allowing the radial iris dilator muscle to contract and dilate the pupil. Atropine induces cycloplegia by paralyzing the ciliary muscles, whose action inhibits accommodation to allow accurate refraction in children, helps to relieve pain associated with iridocyclitis, and treats ciliary block (malignant) glaucoma. The vagus (parasympathetic) nerves that innervate the heart release acetylcholine (ACh) as their primary neurotransmitter. ACh binds to muscarinic receptors (M2) that are found principally on cells comprising the sinoatrial (SA) and atrioventricular (AV) nodes. Muscarinic receptors are coupled to the Gi-protein; therefore, vagal activation decreases cAMP. Gi-protein activation also leads to the activation of KACh channels that increase potassium efflux and hyperpolarizes the cells.

Muscular hypotonia

rag doll posture and pendular reflexes Weak muscle tone

serpiginous rash

rash as a response to allergic reaction as cutaneous larva migrans migrate through skin.

Palpation

to examine by touch

Meissner's corpuscles

touch receptors located near the surface of the skin concentrated in sensitive areas like fingers -glaborous (hairless skin) A alpha 9large myelinated fibers)

Arsenic poisoning

toxicity can result fromexposure to pesticides/insecticides (so can Ach overexposure so keep that in mind too). Presents witih abdominal pain, vomitting, diarrhea, hypotension (from volume loss), and a garlic breath odor. Tx: Dimercaprol (pharm) Arsenic is used as a doping agent in semiconductors (gallium arsenide) for solid-state devices. It is also used in bronzing, pyrotechnics and for hardening shot. Arsenic compounds can be used to make special glass and preserve wood. Arsenic binds with sulfhydryl groups and disrupts sulfhydryl containing enzymes. It replaces the stable phosphorus anion in phosphate with the less stable As (V) anion, leading to rapid hydrolysis of high energy bonds in compounds such as ATP.

eclampsia

true toxemia of pregnancy characterized by high blood pressure, albuminuria, edema of the legs and feet, severe headaches, dizziness, convulsions, and coma

A 28-year-old man comes to the physician for a pre-employment examination. He has no history of serious illness and takes no medications. A screening blood test is performed in which peptides are added to the sample to stimulate in vitro production of interferon-gamma, which is then measured using an enzyme-linked immunosorbent assay. This test is most likely to be helpful in diagnosing infection with which of the following pathogens?

75% The IGRA test for latent Mycobacterium tuberculosis relies on the fact that IFN-γ is released by T lymphocytes when they are exposed to mycobacterial antigens. IGRA is preferred in patients who have received Bacillus Calmette-Guérin (BCG) vaccine, as the test is not dependent on the patient's vaccination status (the PPD test may show a false positive in these patients). If an IGRA test is positive, further evaluation for active TB (e.g. chest x-ray) is indicated Bacillus Calmette-Guérin (BCG) vaccine is a vaccine primarily used against tuberculosis (TB).[4] In countries where tuberculosis or leprosy is common, one dose is recommended in healthy babies as close to the time of birth as possible.[

A 59-year-old woman comes to the physician because of upper extremity weakness and fatigue for the past 4 months. She has had difficulty combing her hair and lifting objects. She has also had difficulty rising from her bed in the mornings for 2 months. Over the past month, she started using over-the-counter mouth rinses for dry mouth. She has smoked 1 pack of cigarettes daily for 40 years. Examination shows decreased deep tendon reflexes. Repetitive muscle tapping shows increased reflex activity. There are no fasciculations or muscle atrophy. A low-dose CT scan of the chest shows a 3-cm mass with heterogeneous calcifications in the center of the right lung. Which of the following is the most likely underlying mechanism responsible for this patient's current symptoms?

A CT scan showing a 3-cm pulmonary mass with irregular calcifications in a patient with a 40 pack-year history of cigarette smoking is suspicious for lung malignancy. Autoimmunity 74% Lambert-Eaton myasthenic syndrome (LEMS) is a paraneoplastic autoimmune disease, most commonly associated with small cell lung cancer. In LEMS, antibodies against presynaptic voltage-gated calcium channels impair acetylcholine (ACh) release in the neuromuscular junction. This leads to proximal muscle weakness, autonomic dysfunction (e.g., dry mouth, constipation), and decreased deep tendon reflexes, as seen in this patient. A unique feature of LEMS is an improvement in strength and reflex activity with repeated use/stimulation. Since postsynaptic ACh receptors remain intact in LEMS, once enough ACh has built up in the postsynaptic cleft from repeated stimulation, muscle contraction can return.

Nissl body

A Nissl body, also known as Nissl substance and Nissl material, is a large granular body found in neurons. These granules are of rough endoplasmic reticulum (RER) with rosettes of free ribosomes, and are the site of protein synthesis. The functions of Nissl bodies is thought to be the same as that of the rest of the ER and the Golgi apparatus: the manufacture and release of proteins and amino acids

pulmonary embolism

A blood clot that breaks off from a large vein and travels to the blood vessels of the lung, causing obstruction of blood flow.

An otherwise healthy 49-year-old man comes to the physician because of a 1-month history of worsening headaches and increasing irritability. Examination shows bilateral papilledema. An MRI of the brain shows a mass in the right temporal lobe with heterogeneous peripheral rim enhancement that crosses the midline. He undergoes surgical resection of the mass. Gross examination shows a poorly-demarcated mass with areas of hemorrhage and necrosis. Microscopic examination of the mass will most likely show which of the following?

A brain tumor in the temporal lobe that crosses the midline and displays irregular rim enhancement is suggestive of glioblastoma, the most common type of malignant brain tumor in adults. The gross pathology findings of a poorly-demarcated mass with areas of hemorrhage and necrosis are consistent with this diagnosis. Pleomorphic cells forming pseudopalisades 78% Pleomorphic cells arranged in pseudopalisades are a hallmark of glioblastoma multiforme, a highly malignant brain tumor that arises from astrocytes. Pseudopalisades are arrays of tumor cells migrating away from an area of central necrosis. Histological findings in glioblastoma multiforme vary greatly but would most likely also show microvascular proliferation as well as necrosis and hemorrhage (as already seen on gross examination).

bruit

A bruit is an audible vascular sound associated with turbulent blood flow. Although usually heard with the stethoscope, such sounds may occasionally also be palpated as a thrill.

A 57-year-old man comes to the physician because of a 3-month history of fatigue, difficulty swallowing, and weight loss. He has smoked 1 pack of cigarettes daily for 30 years. He is 173 cm (5 ft 8 in) tall, and weighs 54 kg (120 lb); BMI is 18 kg/m2. Upper gastrointestinal endoscopy shows an exophytic tumor at the gastroesophageal junction. The patient is diagnosed with advanced esophageal adenocarcinoma. Palliative treatment is begun. Two months later, he complains of difficulty sleeping. His husband says that the patient does not get out of bed most days and has lost interest in seeing his friends. Mental status examination shows a blunted affect, slowed speech, and poor concentration. This patient is at increased risk of developing which of the following findings on polysomnography?

A cancer diagnosis is a life-changing event that can significantly increase a patient's stress levels. Stress that exceeds a patient's coping mechanisms may result in major depressive episodes. REM sleep Rapid eye movement sleep A stage of deep sleep that lasts 10-30 minutes, characterized by predominant beta waves on EEG. During REM sleep there is a high waking threshold, maximum muscle relaxation with simultaneously rapid eye movement (REM), activation of autonomic functions, and high dream activity. Decreased REM sleep latency 51% REM sleep latency is the period between the onset of sleep and the first REM sleep. In a healthy patient, this period lasts 90-120 minutes. The shorter REM sleep latency in depressed patients leads to a decrease in the amount of time in non-REM sleep, thus lowering the quality of sleep. Prolonged periods of damaged sleep architecture exacerbate mood dysfunction, decrease concentration, and impair memory, all of which worsen the patient's depression.

Circle of Willis

A circle of arteries at the base of the brain that supply blood to the brain

A 35-year-old woman that is currently being treated for chronic hepatitis C comes to the physician because of progressive fatigue for 10 days. Previous attempts to treat her hepatitis C infection have been unsuccessful. Examination shows pale conjunctivae. Her hemoglobin concentration is 10.1 g/dL, serum total bilirubin concentration is 1.9 mg/dL, and LDH is 259 U/L. A Coombs test is negative. Assuming her current condition is an adverse drug effect, which of the following mechanisms most likely contributes to the pharmacologic efficiency of this drug?

A combination of decreased hemoglobin, elevated bilirubin, and elevated LDH indicates hemolytic anemia, which is a known side effect of ribavirin. Ribavirin Antiviral drug that is used to treat hepatitis C, RSV infection, and viral hemorrhagic fever (e.g., Lassa fever, Hantavirus infection). It is a nucleoside analog that can act on both RNA and DNA viruses. For hepatitis C, it is always used in combination with other medications such as simeprevir or PEG-interferon-α. Adverse effects include nausea, irritability, hemolytic anemia, and severe teratogenicity. Decreased synthesis of guanine nucleotides 35% Decreased synthesis of guanosine monophosphate (GMP), a guanine nucleotide, through the competitive inhibition of IMP dehydrogenase is one of the mechanisms of action of ribavirin. GMP is used to create viral RNA; therefore, decreased GMP synthesis leads to impaired replication of HCV. In addition, ribavirin also binds to the nucleotide-binding site of RNA polymerase, thereby further inhibiting viral replication. Ribavirin-induced hemolytic anemia results from the accumulation of phosphorylated forms of ribavirin within red blood cells; since RBCs lack dephosphorylating enzymes, ribavirin accumulates within cells, leading to hemolysis. Because of the advent of newer HCV medications, ribavirin is typically used as an adjunct in patients with refractory hepatitis C infection

congestive heart failure

A condition resulting from the heart's inability to pump out all the blood that returns to it; blood backs up in the veins leading to the heart, causing an accumulation of fluid in various parts of the body A chronic condition in which the heart doesn't pump blood as well as it should. Heart failure can occur if the heart cannot pump (systolic) or fill (diastolic) adequately.

A 1-year-old girl is brought to the physician for a well-child examination. She has no history of serious illness. She receives a vaccine in which a polysaccharide is conjugated to a carrier protein. Which of the following pathogens is the most likely target of this vaccine?

A conjugate vaccine contains capsular polysaccharide antigens that are conjugated to carrier proteins, which enhances immunogenicity by promoting T-cell activation. Polysaccharide conjugate vaccine A type of vaccine in which a weakly immunogenic antigen (usually a polysaccharide) is chemically linked to a strongly immunogenic antigen (usually a protein). This combination dramatically increases the immune response to the polysaccharide, especially in infants and toddlers, in whom the polysaccharide alone would not produce sufficient immunization. Examples of polysaccharide vaccines are: Haemophilus influenzae B, meningococcal, and pneumococcal conjugate vaccines Streptococcus pneumoniae 67% Conjugate polysaccharide vaccines are used to target encapsulated bacteria, such as Streptococcus pneumoniae. In young children (< 2 years), capsular polysaccharide antigens do not produce a sufficient immune response. Therefore, conjugate vaccines are administered to ensure sufficient immunization via activation of T cells, which stimulate a more rapid and long-lasting immune response. In addition to the PCV13, other conjugate vaccines include the Hib vaccine and meningococcal vaccine.

A research group investigates the safety and toxicity profile of a new drug. Data about the length of time from the administration of the drug until the onset of adverse effects is collected for a period of 20 days. Administration of the drug is terminated for any patient experiencing adverse effects. The collected data is shown. Which of the following statements about this data set is most accurate?

A data set that follows a right-skewed (positively skewed) distribution is clustered on the left side of the scale with a long tail on the right (positive direction of the scale), as seen here. Such a distribution has the following inequality between the measures of central tendency: mean > median > mode.

A 38-year-old woman comes to the physician because of a 3-month history of moderate abdominal pain that is unresponsive to medication. She has a history of two spontaneous abortions at 11 and 12 weeks' gestation. Ultrasound examination of the abdomen shows normal liver parenchyma, a dilated portal vein, and splenic enlargement. Upper endoscopy shows dilated submucosal veins in the lower esophagus. Further evaluation of this patient is most likely to show which of the following findings?

A dilated portal vein, splenic enlargement, and dilated lower esophageal submucosal veins with an ultrasound showing normal liver parenchyma suggests portal hypertension due to portal vein thrombosis. Portal vein thrombosis Abbreviation: PVT A thrombus in the portal vein, which can cause complete or partial closure of the vein. Most commonly occurs as a complication of liver cirrhosis. Can lead to portal hypertension. Thrombocytopenia 28% Thrombocytopenia is a common presentation in patients with an enlarged spleen because enlargement can increase splenic activity (hypersplenism), leading to the rapid clearance of platelets from the blood stream. Splenic enlargement and esophageal varices (collateral circulation) are seen in all forms of portal hypertension. However, unlike post-hepatic and hepatic causes of portal hypertension (e.g., Budd-Chiari syndrome, cirrhosis), pre-hepatic causes of portal hypertension such as splenic or portal vein thrombosis are usually not associated with ascites. Repeated spontaneous miscarriages in patients with thrombophilia, as suggested here by the diagnosis of portal vein thrombosis, is suspicious of antiphospholipid syndrome (APS)

small cell carcinoma

A fast-growing type of lung cancer commonly caused by smoking. This aggressive form of lung cancer most commonly occurs in smokers. It usually starts in the breathing tubes (bronchi) and grows very quickly, creating large tumors and spreading (metastasizing) throughout the body.

A 16-year-old girl is brought to the physician by her father because of concerns about her behavior during the past 2 years. She does not have friends and spends most of the time reading by herself. Her father says that she comes up with excuses to avoid family dinners and other social events. She states that she likes reading and feels more comfortable on her own. On mental status examination, her thought process is organized and logical. Her affect is flat. Which of the following is the most likely diagnosis?

A flat affect is the loss or lack of emotional responses to a situation or event that would typically elicit emotion. People with flat affect may appear to be completely unemotional or apathetic. Other associated symptoms include speaking in a monotone voice and a reduction in facial expressions. Schizoid personality disorder 78% This patient has signs of social withdrawal, a preference for solitary activities, and a flattened affect, all of which are features of schizoid personality disorder. Individuals with this personality disorder neither desire nor enjoy close relationships. They show little interest in interpersonal or sexual contact and often appear indifferent or emotionally cold towards others. Of note, the DSM-5 states that personality disorders can be diagnosed in patients < 18 years of age if features have been present for ≥ 1 year. The only exception to this rule is antisocial personality disorder, which can only be diagnosed in patients ≥ 18 years of age.

An investigator studying the molecular characteristics of various malignant cell lines collects tissue samples from several families with a known mutation in the TP53 tumor suppressor gene. Immunohistochemical testing performed on one of the cell samples stains positive for desmin. This sample was most likely obtained from which of the following neoplasms?

A hereditary mutation in TP53 is seen in Li-Fraumeni syndrome. This condition is associated with various malignancies that occur at an early age, one of which arises in a type of tissue that routinely expresses desmin, a type of intermediate filament. Li-Fraumeni syndrome Sarcoma, Breast, Leukemia, and Adrenal Gland cancer syndrome A syndrome caused by an autosomal dominant inherited mutation of the tumor suppressor gene TP53. Characterized by multiple malignancies (sarcoma, breast carcinoma, leukemia, adrenocortical carcinoma) that occur at an early age. Rhabdomyosarcoma 73% Desmin is an intermediate filament protein of the cytoskeleton of muscle cells. As desmin is specific to muscle, it is only expressed in tumors composed of muscle cells such as rhabdomyosarcomas (striated muscle) or leiomyomas (smooth muscle). Rhabdomyosarcoma is one of the more common malignancies that arise in Li-Fraumeni syndrome. Here, the loss-of-function mutation in a tumor suppressor gene (TP53) results in a loss of heterozygosity and an increased risk for the development of cancer. Rhabdomyosarcoma A rare, malignant, mesenchymal tumor of primitive muscle cells that primarily occurs in the orbital cavity but can also arise in the head, neck, or genital regions. Most commonly affects children in the first decade of life.

A 2550-g (5 lb 10 oz) female newborn is delivered at term to a 27-year-old woman, gravida 1, para 1. Labor and delivery were uncomplicated but the mother did not receive prenatal care. Examination of the newborn shows hypotonia in the lower extremities and a red, fleshy swelling without overlying skin in the lumbosacral region. Clear yellow fluid leaks from the superior margin of the swelling. Ultrasonography of the swelling shows a fluid-filled sac containing herniated neural tissue. Which of the following best explains the pathogenesis of this patient's condition?

A lumbosacral swelling that is not covered by skin, leaks CSF, and contains neural tissue is suggestive of myelomeningocele. Myelomeningocele A neural tube defect in which the meninges and spinal cord herniate through an abnormal vertebral opening. Myelomeningocele is a severe form of spina bifida in which the spinal cord and nerves develop outside of the body and are contained in a fluid-filled sac that is visible outside of the back area. These babies typically have weakness and loss of sensation below the sac. Failure of posterior neuropore closure 89% Myelomeningocele is a disorder of primary neurulation that results from the failure of the posterior (caudal) neuropore to close at 3-4 weeks gestation. While myelomeningoceles can usually be detected on routine prenatal ultrasound or laboratory studies (i.e., serum α-fetoprotein), this patient's mother did not receive any prenatal care. Moreover, a folate deficiency due to a lack of prenatal vitamins is associated with an increased risk of neural tube defects. Symptoms including lower extremity paralysis and sensory deficits, bowel and bladder dysfunction, and cognitive impairment are often present from birth, as seen here. Additional evaluation, including a head MRI, should be performed because approximately 95% of patients with myelomeningocele have an associated Chiari II malformation.

lumpectomy

A lumpectomy is the surgical removal of a cancerous or noncancerous breast tumor. A lumpectomy also includes removing a small amount of normal breast tissue around a cancerous tumor. Other names for breast lumpectomy include partial mastectomy, breast-conserving surgery, breast-sparing surgery, and wide excision.

A 56-year-old man is brought to the emergency department by his neighbor 2 hours after ingesting an unknown substance in a suicide attempt. He is confused and unable to provide further history. His temperature is 39.1°C (102.4°F), pulse is 124/min, respiratory rate is 12/min, and blood pressure is 150/92 mm Hg. His skin is dry. Pupils are 12 mm and minimally reactive. An ECG shows no abnormalities. Which of the following is the most appropriate treatment for this patient's condition?

A patient with elevated body temperature, tachycardia, dry skin, mydriasis with cycloplegia, and confusion after ingesting an unknown substance likely has anticholinergic toxicity. Physostigmine 58% Physostigmine is the first-line treatment for anticholinergic syndrome, as it readily enhances parasympathetic signaling and is able to cross the blood-brain barrier. In addition to the classic findings seen in this patient, overdose of anticholinergic drugs can also cause urinary retention, decreased bowel sounds, and erythematous skin (due to compensatory cutaneous vasodilation from the hyperthermia).

peripheral pulses

A peripheral pulse refers to the palpation of the high-pressure wave of blood moving away from the heart through vessels in the extremities following systolic ejection. Carotid, radial, brachial, femoral, posterior tibial, and dorsalis pedis pulses should be routinely examined bilaterally to ascertain any differences in the pulse amplitude, contour, or upstroke.

A 32-year-old man comes to the physician because of a 1-month history of intermittent tingling of his hand. He is an avid cyclist and has recently started training for a cycle marathon. Physical examination shows decreased grip strength in the right hand and wasting of the hypothenar eminence. On asking the patient to grasp a piece of paper between his right thumb and right index finger in the first web space, there is hyperflexion of the right thumb interphalangeal joint. Which of the following additional findings is most likely in this patient?

A positive Froment sign (hyperflexion of the thumb due to paralysis of the adductor pollicis and unopposed action of flexor pollicis longus) and wasting of the hypothenar eminence are suggestive of ulnar nerve entrapment. Inability to extend the little finger at the proximal interphalangeal joints 29% This patient's physical exam is suggestive of long-term ulnar nerve compression at the level of Guyon canal (Guyon canal syndrome), which is common in avid cyclists due to long-term pressure of the wrist against bike handlebars. This compression disrupts innervation to the ulnar innervated intrinsic hand muscles, including the lumbricals, which are responsible for little finger extension at the proximal interphalangeal joint. Additional physical exam findings typically seen in this syndrome include numbness of the palmar surface of the small and ring fingers. Lumbrical A set of muscles that extend the digits. The lumbricals of the hand are innervated by the median and ulnar nerves are facilitate metacarpophalangeal joint flexion and interphalangeal joint extension. The lumbricals of the feet are innervated by the medial and lateral plantar nerves and facilitate metatarsophalangeal joint flexion and interphalangeal joint extension.

skin turgor

A reflection of the skin's elasticity, measured by monitoring the time it takes for the skin on he back of the hand to return to position after it is lightly pinched between the examiner's thumb and forefinger. Normal turgor is a return to normal contour within a few seconds; if the skin remains elevated (tented) more than a few seconds, turgor is decreased. Skin turgor is the skin's elasticity. It is the ability of skin to change shape and return to normal.

A 31-year-old man comes to the physician because of worsening abdominal pain, an inability to concentrate at work, and a general lack of motivation over the past several months. He has a history of spontaneous passage of two kidney stones. His father and uncle underwent thyroidectomy before the age of 35 for thyroid cancer. Physical examination shows diffuse tenderness over the abdomen. Serum studies show: Na+142 mEq/LK+3.7 mEq/LGlucose (fasting)83 mg/dLCa2+12.3 mg/dLAlbumin4.1 g/dLParathyroid hormone350 pg/mL Further evaluation is most likely to show elevated levels of which of the following?

A young patient with primary hyperparathyroidism (difficulty concentrating, lethargy, recurrent kidney stones, elevated PTH levels, and increased serum calcium) and a family history of thyroid cancer should raise suspicion for multiple endocrine neoplasia type 2A (MEN 2A) syndrome. Urine metanephrines 56% Increased levels of catecholamine metabolites such as metanephrines in the urine or serum are diagnostic of pheochromocytoma, a catecholamine-secreting neuroendocrine tumor typically located in the adrenal medulla. Pheochromocytomas may be a sign of genetic syndromes such as MEN 2A and MEN 2B due to an autosomal dominant mutation of the RET gene.

A researcher is investigating compounds that modulate the cell cycle as possible chemotherapeutic agents against peripheral T-cell lymphoma. The researcher discovers a group of natural compounds with inhibitory activity against histone deacetylases, a class of enzymes that remove acetyl groups from the lysine residues of histones. A histone deacetylase inhibitor most likely causes which of the following?

Acetyl groups carry a negative charge. Removal of an acetyl group from histones results in a more positively charged molecule. Relaxation of DNA coiling 56% Histone deacetylase removes acetyl groups from histones. This exposes positively-charged lysine residues that attract negatively-charged DNA. As a result, DNA binds histones more tightly, which restricts transcription. The novel compound described in the stem inhibits histone deacetylase, which leads to increased histone acetylation. This, in turn, relaxes DNA coiling and facilitates transcription.

An otherwise healthy 62-year-old woman comes to the physician because of a 3-year history of hearing loss. To test her hearing, the physician performs two tests. First, a vibrating tuning fork is held against the mastoid bone of the patient and then near her ear, to which the patient responds she hears the sound better on both sides when the tuning fork is held near her ear. Next, the physician holds the tuning fork against the bridge of her forehead, to which the patient responds she hears the sound better on the right side than the left. The patient's examination findings are most consistent with which of the following conditions?

Acoustic neuromas, which commonly arise from the internal acoustic canal, often exert a mass effect on the vestibulocochlear nerve (CN VIII). The resulting unilateral sensorineural hearing loss, as seen in this patient, is the most common early symptom of this condition. Other symptoms include tinnitus due to pressure on CN VIII as well as dizziness/vertigo and gait instability due to vestibular deficits. In this patient, the results of the Rinne test are positive, indicating normal conductive hearing. In combination with a Weber test that lateralized to the right, this indicates left-sided sensorineural hearing loss. Otosclerosis on the left Otosclerosis is characterized by conductive hearing loss (more in the lower frequencies than in the higher), typically bilateral. It most commonly manifests in the third or fourth decade of life. If otosclerosis were present on the left side, the patient would have a negative Rinne test on the left and a positive test on the right. The Weber test would lateralize to the left.

A previously healthy 53-year-old man is brought to the emergency department 45 minutes after the onset of a severe headache. He returned from a vacation in the mountains 2 weeks ago, where he camped near a lake. On arrival, he is confused. His temperature is 39°C (102.2°F) and blood pressure is 105/68 mm Hg. Neurologic examination shows diffuse hyperreflexia. An MRI of the brain shows asymmetrical, bitemporal hyperintensities. A lumbar puncture is performed. Cerebrospinal fluid analysis shows: Leukocyte count120/mm3Segmented neutrophils10%Lymphocytes90%Erythrocyte count15/mm3Glucose58 mg/dLOpening pressure130 mm HgProtein75 mg/dL Which of the following is the most likely causal pathogen?

Acute illness associated with headache, fever, altered mental status, and CSF showing lymphocytic pleocytosis with normal glucose levels and elevated protein levels is indicative of acute viral encephalitis. The localized temporal abnormalities seen on MRI are virtually pathognomonic of this pathogen. Herpes simplex virus is the most common cause of viral encephalitis. MRI of the brain classically shows temporal lobe involvement, possibly including hemorrhagic changes. Damage to the temporal lobe is also responsible for some of the other clinical features that may be seen in herpes simplex encephalitis such as olfactory hallucinations, aphasia, and focal seizures.

A 9-year-old boy is brought to the physician for evaluation of a 3-day history of fever, sore throat, and itchy, red eyes. His symptoms began while he was away at summer camp. His immunizations are not up-to-date. He appears ill. His temperature is 39.1°C (102.3°F). Physical examination shows erythema and edema of the conjunctivae and posterior pharyngeal wall. There is bilateral, tender, preauricular lymphadenopathy. Further evaluation shows infection with a DNA virus. Which of the following is the most likely causal pathogen?

Adenovirus is a nonenveloped, double-stranded DNA virus with a linear structure and the most likely underlying cause of this patient's symptoms. Conjunctivitis and febrile pharyngitis occurring simultaneously in a patient who has spent time in close proximity to other individuals (e.g., summer camp) is highly suspicious for adenovirus infection. Apart from pharyngoconjunctivitis, adenovirus may cause gastroenteritis, pneumonia, acute hemorrhagic cystitis, and epidemic keratoconjunctivitis.

A 7-year-old boy is brought to the physician because of decreased vision, hearing, and speaking over the past 3 months. During this time, he has also had difficulty walking, concentrating, drawing, and feeding himself. His maternal male cousin had similar complaints and died at the age of 5 years. Physical examination shows hyperpigmented mucosa and skin. His speech is dysarthric. Neurologic examination shows an ataxic gait, spasticity, and decreased muscle strength in all extremities. Fundoscopy shows optic atrophy. Serum studies show elevated concentrations of fatty acids with 22 or more carbons. A deficiency in which of the following is the most likely cause of this patient's symptoms?

Adrenoleukodystrophy is a rare X-linked neurological disease caused by an impaired peroxisomal ATP-binding cassette (ABC) transporter protein due to a mutation in the ABCD1 gene. This mutation leads to an accumulation of very long-chain fatty acids in the serum as well as the adrenal glands, testes, and white matter, and subsequent neuron demyelination. Adrenoleukodystrophy can sometimes present very similarly to other rare neurologic conditions of childhood (e.g., sphingolipidoses, mucopolysaccharidoses). Often, the main distinguishing factor is the presence of adrenal insufficiency (usually hyperpigmented skin, but also nausea, vomiting, hypotension, etc.). It is primarily diagnosed via a very long-chain fatty acid panel or mutation analysis.

Twelve days after undergoing a cadaveric renal transplant for adult polycystic kidney disease, a 23-year-old man has pain in the right lower abdomen and generalized fatigue. During the past 4 days, he has had decreasing urinary output. Creatinine concentration was 2.3 mg/dL on the second postoperative day. Current medications include prednisone, cyclosporine, azathioprine, and enalapril. His temperature is 38°C (100.4°F), pulse is 103/min, and blood pressure is 168/98 mm Hg. Examination reveals tenderness to palpation on the graft site. Creatinine concentration is 4.3 mg/dL. A biopsy of the transplanted kidney shows tubulitis. C4d staining is negative. Which of the following is the most likely cause of this patient's findings?

After renal transplant from a cadaveric donor, the serum creatinine should start to decrease by the end of the first week. Increasing creatinine levels within days to weeks of the renal transplantation is concerning for acute rejection of the renal allograft. Allorecognition with T cell activation Abdominal pain, hypertension, and an increase in creatinine within 6 months of renal transplantation are concerning for acute rejection of the renal allograft. There are two mechanisms for acute allograft rejection, acute cellular rejection by host T lymphocytes and acute humoral rejection by circulating antibodies. The absence of C4d staining (a marker of antibody-mediated damage) and the presence of inflammatory infiltrate within the wall of tubules (tubulitis) indicate that the most likely mechanism for graft rejection in this patient is acute cellular rejection.

A 48-year-old woman comes to the emergency department because of a 1-day history of fever, vomiting, and abdominal pain. Two weeks ago, while visiting Guatemala, she had an emergency appendectomy under general inhalational anesthesia. During the surgery, she received a transfusion of 1 unit of packed red blood cells. Her temperature is 38.3°C (100.9°F) and blood pressure is 138/76 mm Hg. Examination shows jaundice and tender hepatomegaly. Serum studies show: Alkaline phosphatase 132 U/L Aspartate aminotransferase 760 U/L Bilirubin Total 3.8 mg/dL Direct 3.1 mg/dL Anti-HAV IgG positive Anti-HAV IgM negative Anti-HBs positive HBsAg negative Anti-HCV antibodies negative Abdominal ultrasonography shows an enlarged liver. A biopsy of the liver shows massive centrilobular necrosis. Which of the following is the most likely underlying cause of this patient's condition?

After undergoing surgery, this patient developed jaundice, fever, vomiting, and tender hepatomegaly, with highly elevated serum transaminases, which indicates acute postoperative hepatitis. Adverse effect of anesthetic 54% This patient's acute postoperative hepatitis is highly suggestive of halothane hepatitis. Halothane hepatitis commonly leads to elevated serum aminotransferases, elevated bilirubin, eosinophilia, and slightly elevated alkaline phosphatase, as seen in this patient. This patient's symptoms began 13 days following surgery, which is consistent with the timeframe for postoperative onset of halothane hepatitis (2 days to 3 weeks). Liver biopsy is not necessary for diagnosis but typically shows massive centrilobular hepatic necrosis In rare cases, repeated exposure to halothane in adults was noted to result in severe liver injury. This occurred in about one in 10,000 exposures. The resulting syndrome was referred to as halothane hepatitis, and is thought to result from the metabolism of halothane to trifluoroacetic acid via oxidative reactions in the liver.

A 69-year-old man comes to the physician with a 2-year history of progressive hearing loss. His hearing is worse in crowded rooms, and he has noticed that he has more difficulty understanding women than men. He has no history of serious illness and does not take any medications. A Rinne test shows air conduction is greater than bone conduction bilaterally. This condition is most likely associated with damage closest to which of the following structures?

Age-related, progressive, bilateral hearing loss at higher frequencies (e.g., women's voices) that worsens in noisy environments is the key feature of presbycusis, the most common type of hearing loss in older adults. Since sound conduction is unaffected, the Rinne test is normal bilaterally (air conduction is greater than bone conduction). Basal turn of the cochlea 57% Sound waves cause vibrations that resonate with the basilar membrane of the cochlea, which converts this input into nerve signals that encode the auditory information. The sound wave's point of optimal resonance within the cochlea depends on the frequency of the sound wave (tonotopy). The base of the cochlea, including the basal turn, responds to higher frequencies. Progressive degeneration of the organ of Corti near this region causes presbycusis. Presbycusis is the loss of hearing that gradually occurs in most individuals as they grow older. Hearing loss is a common disorder associated with aging. About 30-35 percent of adults age 65 and older have a hearing loss. It is estimated that 40-50 percent of people 75 and older have a hearing loss.

A 68-year-old man is brought to the emergency department because of a 1-day history of fever and cloudy urine. He lives in a group home and has a chronic indwelling urinary catheter for benign prostatic hyperplasia. He has depression, for which he takes vilazodone. His temperature is 38.3°C (100.9°F), pulse is 89/min, and blood pressure is 110/74 mm Hg. Urine cultures grow Enterococcus faecium resistant to vancomycin, and therapy with intravenous linezolid is initiated. Eight hours later, he becomes agitated and starts to sweat profusely. His temperature is 39.6°C (103.3°F), pulse is 120/min, and blood pressure is 184/105 mm Hg. Neurologic examination shows increased deep tendon reflexes and clonus of the arms and legs. In addition to discontinuation of his current medication, administration of which of the following drugs is most appropriate in this patient?

Agitation, hyperthermia, tachycardia, hypertension, diaphoresis, and evidence of neuromuscular hyperactivity (hyperreflexia, clonus) in a patient taking vilazodone (a serotonergic drug) suggest serotonin syndrome. The administration of linezolid, an antibiotic that has mild monoamine oxidase inhibitor activity, was likely the trigger Cyproheptadine 61% Cyproheptadine is a H1-, 5-HT1A-, and 5-HT2A-receptor antagonist and is used as first-line antidote in patients with serotonin syndrome who do not respond to supportive measures and discontinuation of the offending drug. This drug competes with the excess serotonin for receptor binding, reducing pathological overstimulation. Cyproheptadine should be used in conjunction with cooling measures (e.g., application of ice packs, cold compresses) and benzodiazepines for sedation.

A 23-year-old woman with asthma is brought to the emergency department because of shortness of breath and wheezing for 20 minutes. She is unable to speak more than a few words at a time. Her pulse is 116/min and respirations are 28/min. Pulse oximetry on room air shows an oxygen saturation of 92%. Examination of the lungs shows decreased breath sounds and scattered end-expiratory wheezing over all lung fields. Treatment with high-dose continuous inhaled albuterol is begun. This patient is at increased risk for which of the following adverse effects?

Albuterol is a β2-agonist. At typical doses (e.g., from a metered-dose inhaler), albuterol does not typically have systemic effects. However, use of continuous inhaled albuterol for long periods of time can result in β2-mediated adverse effects. Beta-2 adrenergic agonists A drug that stimulates β2 adrenergic receptors, resulting in dilation of certain smooth muscles. Primarily used as bronchodilators to treat bronchial asthma and COPD. Obstetric use includes inducing tocolysis in preterm or undesired contractions. Adverse effects include tachycardia, arrhythmias, tremor, hyperglycemia, and hypokalemia. Hypokalemia 54% Hypokalemia is a common adverse effect of continuous albuterol therapy. It is caused by β2-agonist stimulation of the Na+/K+-ATPase, which leads to an intracellular K+ shift. Additionally, β2-mediated stimulation of the liver elevates cAMP levels, thereby increasing glycogenolysis and hyperglycemia. Hyperglycemia stimulates insulin secretion, which also activates the Na+/K+-ATPase and further potentiates the intracellular K+ shift. Hypokalemia can cause EKG changes, such as T wave flattening, ST depression, and U waves.

A 73-year-old man is brought to the emergency department because of fever, headaches, and confusion for the past 24 hours. Three years ago, he underwent heart transplantation because of congestive heart failure. His temperature is 38.1°C (100.5°F). He is oriented only to person. Physical examination shows nuchal rigidity. A cerebrospinal fluid culture on blood agar grows colonies of a gram-positive bacillus surrounded by a narrow transparent rim. Administration of which of the following antibiotics is most likely to be effective in the treatment of this patient's condition?

Ampicillin 56% Ampicillin is the most effective drugs for listerial CNS infection and, therefore, the treatment of choice for affected patients. In immunocompromised patients, neonates, and patients with Listeria infection affecting the endocardium or the central nervous system, gentamicin is added in empiric therapy for its synergistic effect.

A 62-year-old woman is brought to the physician because of 6 months of progressive weakness in her arms and legs. During this time, she has also had difficulty swallowing and holding her head up. Examination shows pooling of oral secretions. Muscle strength and tone are decreased in the upper extremities. Deep tendon reflexes are 1+ in the right upper and lower extremities, 3+ in the left upper extremity, and 4+ in the left lower extremity. Sensation to light touch, pinprick, and vibration are intact. Which of the following is the most likely diagnosis?

Amyotrophic lateral sclerosis 78% Amyotrophic lateral sclerosis (ALS) typically presents between 50-70 years of age with progressively worsening features of both upper motor neuron degeneration (hyperreflexia, increased muscle tone, Babinski sign) and lower motor neuron degeneration (hyporeflexia, decreased muscle tone, atrophy), as seen in this patient. While dysphagia from bulbar atrophy is present in approximately 20% of patients at disease onset, it is most often a late clinical feature and suggests a poor prognosis. Myasthenia gravis 8% Myasthenia gravis (MG) can present with generalized muscle weakness most commonly involving the extraocular or eyelid muscles that becomes worse with prolonged muscle activity. This patient's weakness is unrelated to activity and the presence of upper/lower motor neuron signs is inconsistent with this diagnosis. Also, deep tendon reflexes are usually normal in patients with MG.

aneurysm

An aneurysm is an outward bulging, likened to a bubble or balloon, caused by a localized, abnormal, weak spot on a blood vessel wall.[1] Aneurysms may be a result of a hereditary condition or an acquired disease. Aneurysms can also be a nidus (starting point) for clot formation (thrombosis) and embolization. The word is from Greek: ἀνεύρυσμα, aneurysma, "dilation", from ἀνευρύνειν, aneurynein, "to dilate". As an aneurysm increases in size, the risk of rupture increases,[2] leading to uncontrolled bleeding. Although they may occur in any blood vessel, particularly lethal examples include aneurysms of the Circle of Willis in the brain, aortic aneurysms affecting the thoracic aorta, and abdominal aortic aneurysms. Aneurysms can arise in the heart itself following a heart attack, including both ventricular and atrial septal aneurysms. There are congenital atrial septal aneurysms, a rare heart defect.

arteriovenous fistula

An arteriovenous fistula is an atypical connection between an artery and vein that bypasses the capillary bed. This abnormality can cause tissues to receive an insufficient supply of blood and the nutrients it carries. AV fistulas can either be congenital or acquired. Congenital AV fistulas are uncommon. Acquired AV fistulas usually arise after a piercing injury to an artery and a vein that are located side-by-side. AV fistulas can be located anywhere in the body where an artery and a vein are in close proximity. Here, we focus on an AV fistula between the distal radial artery and the vena comitans. How do I know if I have an arteriovenous fistula? AV fistulas cause the skin to appear swollen or discolored. 50% of patients have some associated pain. Arterial blood has a higher pressure than venous blood, so abnormal connections will lead to bulging of the veins, which may resemble varicosities. The bulging of veins will allow more blood than normal to enter, resulting in a decrease in arterial blood pressure. The heart will detect this decrease in arterial blood pressure and respond by increasing stroke volume and cardiac output, which can lead to high-output congestive heart failure. A soft tissue bruit or vibration may be detected over the AV fistula via auscultation and ultrasound techniques. Ultrasound over the area will also reveal low resistance flow in the proximal artery waveform, as well as a distinctive speckled coloration on the color Doppler setting (1). What causes an arteriovenous fistula? Commonly, AV connections in vessels may acquire pathology after traumas, burns, infections, or neoplastic events (2). Patent congenital connections in arteries and veins from a joined stage in embryonic development may also manifest in patients in their 20s or 30s (1).The patient in the above video had a coronary intervention using a radial artery approach. Over the next 3 months, the patient developed pain in her distal wrist and paresthesia over the median nerve distribution. Since the time of the coronary intervention, the AV fistula was found to be enlarging, eventually measuring approximately 1.5-2 cm. The AV fistula was located between the distal radial artery and the vena comitans. There was a palpable thrill over the mass. This thrill was due to increased turbulence in the vein.

A 36-year-old woman, gravida 2, para 1, at 30 weeks' gestation comes to the physician for evaluation of increased urinary frequency. She has no history of major medical illness. Physical examination shows no abnormalities. Laboratory studies show an increased serum C-peptide concentration. Ultrasonography shows polyhydramnios and a large for gestational age fetus. Which of the following hormones is predominantly responsible for the observed laboratory changes in this patient?

An increased serum C-peptide concentration and ultrasound findings of polyhydramnios and macrosomia suggest a diagnosis of gestational diabetes. Hyperglycemia causes osmotic diuresis, which leads to increased urinary frequency. Human placental lactogen 71% Human placental lactogen causes pancreatic beta-cell hyperplasia and leads to an increase in insulin (and C-peptide) secretion as well as maternal insulin resistance. This ensures adequate glucose availability for the fetus. If maternal pancreatic function does not overcome insulin resistance, patients can develop gestational diabetes. In response to increased serum glucose concentrations, fetal production of insulin increases, which leads to increased fetal growth (macrosomia) as seen in this case.

Croup

An upper airway infection that blocks breathing and has a distinctive barking cough. Croup, also known as laryngotracheobronchitis, is a type of respiratory infection that is usually caused by a virus.[2] The infection leads to swelling inside the trachea, which interferes with normal breathing and produces the classic symptoms of "barking/brassy" cough, stridor, and a hoarse voice.[2] Fever and runny nose may also be present.[2] These symptoms may be mild, moderate, or severe.[3] Often it starts or is worse at night and normally lasts one to two days.

A 48-year-old patient comes to the physician because of a 4-day history fever, headaches, loss of appetite, and myalgia. Two weeks ago, the patient went on a camping trip to Wisconsin. His temperature is 39.5°C (103.1°F). Physical examination shows no rash or joint swelling. Laboratory studies show: Hemoglobin14.1 g/dLLeukocyte count3,800/mm3Platelet count85,000/mm3SerumAST68 U/LALT72 U/L Treatment with doxycycline is begun and the patient recovers. A paired acute and convalescent indirect fluorescent antibody test confirms that the patient had anaplasmosis. The patient's condition was most likely transmitted by which of the following vectors?

Anaplasmosis is a disease caused by the bacterium Anaplasma phagocytophilum. These bacteria are spread to people by tick bites primarily from the blacklegged tick (Ixodes scapularis) and the western blacklegged tick (Ixodes pacificus). People with anaplasmosis will often have fever, headache, chills, and muscle aches. The vector responsible for the patient's condition is also a vector for Lyme disease and babesiosis. Ixodes tick 66% Ixodes ticks are vectors for anaplasmosis, Lyme disease, and babesiosis. Ixodes scapularis is found in the northeast and midwest US (e.g., Wisconsin), while Ixodes pacificus is found primarily along the west coast of the US. Anaplasmosis is caused by the bacterium Anaplasma phagocytophilum and typically manifests with flu-like and gastrointestinal symptoms within 1-2 weeks after the bite of an infected tick. Typical laboratory findings include leukopenia, thrombocytopenia, and elevated transaminase levels, all of which are seen in this patient.

A 12-year-old boy is brought to the emergency department by his mother because of progressive shortness of breath, difficulty speaking, and diffuse, colicky abdominal pain for the past 3 hours. Yesterday he underwent a tooth extraction. His father and a paternal uncle have a history of repeated hospitalizations for upper airway and orofacial swelling. The patient takes no medications. His blood pressure is 112/62 mm Hg. Examination shows edematous swelling of the lips, tongue, arms, and legs; there is no rash. Administration of a drug targeting which of the following mechanisms of action is most appropriate for this patient?

Antagonist at bradykinin receptor 55% The autosomal dominant defect in patients with hereditary angioedema causes a deficiency of C1 inhibitor, which physiologically suppresses steps in the complement pathways and inactivates kallikrein. Because kallikrein normally converts kininogen to bradykinin, increased levels of kallikrein lead to a buildup of bradykinin, and eventual angioedema. First-line therapies are focused either on antagonizing the bradykinin-B2-receptor (icatibant), preventing further generation of bradykinin with kallikrein inhibitor therapy (ecallantide), or replacing the defective enzyme with intravenous injection of purified C1-INH. Agonist at alpha and beta-adrenergic receptors 17% Epinephrine, a dual α and β adrenergic agonist, is the first-line treatment for anaphylaxis associated with allergic or mast cell-mediated angioedema. Abdominal pain can also occur in some cases of anaphylaxis, most commonly with a known food allergen. However, anaphylaxis usually has an acute onset (i.e., within 30 minutes to 2 hours of exposure to an antigen), is rapidly progressive, and would present with additional system involvement (hypotension, pruritus, and urticaria). Epinephrine does not improve airway-associated symptoms in acute hereditary angioedema.

anterior fontanel

Anterior fontanelle is a diamond-shaped membrane-filled space located between the two frontal and two parietal bones of the developing fetal skull. It persists until approximately 18 months after birth. It is at the junction of the coronal suture and sagittal suture This is the junction where the 2 frontal and 2 parietal bones meet. The anterior fontanelle remains soft until about 18 months to 2 years of age. Doctors can assess if there is increased intracranial pressure by feeling the anterior fontanelle.

An investigator is studying the immunologic response to a Staphylococcus aureus toxin in a mouse model. Fourteen days after injecting mice with this toxin, he isolates antibodies against neutrophil proteinase 3 in their sera. A patient with high concentrations of these antibodies would most likely present with which of the following clinical features?

Antibodies against neutrophil proteinase 3 (PR3-ANCA/c-ANCA) are a type of antineutrophil cytoplasmic antibody (ANCA) that has a diffuse cytoplasmic staining pattern on immunofluorescence studies. These antibodies are most commonly found in patients with granulomatosis with polyangiitis. Antineutrophil cytoplasmic antibodies Abbreviation: ANCA An antibody against specific cytoplasmic antigens. An increase in ANCAs is a common finding in autoimmune vasculitides (e.g., p-ANCA in eosinophilic granulomatosis with polyangiitis, c-ANCA in granulomatosis with polyangiitis) Nasal mucosal ulcerations and hematuria 70% Nasal mucosal ulcerations and hematuria (due to underlying rapidly progressive glomerulonephritis) are features of granulomatosis with polyangiitis. This c-ANCA-positive vasculitis is idiopathic but can develop after respiratory infections. Additional manifestations include cough, dyspnea, and/or hemoptysis (due to lower respiratory tract inflammation), purpura, nodules, or ulcerations of the skin, and potentially life-threatening pericarditis or myocarditis. Diagnosis is confirmed with a biopsy of affected tissue that shows necrotizing granulomas of small and medium-sized vessels.

Metoclopramide

Antiemetic The antiemetic action of metoclopramide is due to its antagonist activity at D2 receptors in the chemoreceptor trigger zone in the central nervous system — this action prevents nausea and vomiting triggered by most stimuli.[31] At higher doses, 5-HT3 antagonist activity may also contribute to the antiemetic effect

Rifampin

Antitubercular Rifampicin, also known as rifampin, is an antibiotic used to treat several types of bacterial infections, including tuberculosis, Mycobacterium avium complex, leprosy, and Legionnaires' disease.[2] It is almost always used together with other antibiotics with two notable exceptions when given as a second-line treatment for latent TB and to prevent Haemophilus influenzae type b and meningococcal disease in people who have been exposed to those bacteria.[ Rifampicin inhibits bacterial DNA-dependent RNA synthesis by inhibiting bacterial DNA-dependent RNA polymerase.[4

A previously healthy 50-year-old woman is brought to the emergency department 30 minutes after she was observed having a seizure. On arrival, she is conscious and reports that she feels drowsy. An MRI of the brain shows a 4-cm, round, sharply demarcated mass. She undergoes resection of the mass. A photomicrograph of a section of the resected specimen is shown. This patient's mass is most likely derived from which of the following?

Arachnoid cells 59% Meningiomas arise from arachnoid cells of the arachnoid villi. They are usually benign tumors that manifest as a slow-growing sharply demarcated mass that is composed of spindle-shaped cells arranged in a whorled pattern. The symptoms they cause are due to compression of adjacent structures, e.g., contralateral spastic paresis due to compression of the motor cortex, personality changes due to compression of the frontal lobe, or focal seizures.

area postrema

Area in the medulla of the brain stem that is not isolated from chemicals in the blood. It is responsible for inducing a vomiting response when a toxic substance is present in the blood. Damage to the area postrema, caused primarily by lesioning or ablation, prevents the normal functions of the area postrema from taking place. This ablation is usually done surgically and for the purpose of discovering the exact effect of the area postrema on the rest of the body. Since the area postrema acts as an entry point to the brain for information from the sensory neurons of the stomach, intestines, liver, kidneys, heart, and other internal organs, a variety of physiological reflexes rely on the area postrema to transfer information. The area postrema acts to directly monitor the chemical status of the organism. Lesions of the area postrema are sometimes referred to as 'central vagotomy' because they eliminate the brain's ability to monitor the physiological status of the body through its vagus nerve.[12] These lesions thus serve to prevent the detection of poisons and consequently prevent the body's natural defenses from kicking in

A 55-year-old woman comes to the physician because of involuntary hand movements that improve with alcohol consumption. Physical examination shows bilateral hand tremors that worsen when the patient is asked to extend her arms out in front of her. The physician prescribes a medication that is associated with an increased risk of bronchospasms. This drug has which of the following immediate effects on the cardiovascular system? A postural tremor that improves with alcohol consumption is characteristic of essential tremor. Propranolol, a nonselective beta blocker that is typically the first-line drug for essential tremors, is associated with an increased risk of bronchospasms, which limits its use in patients with asthma or COPD.

As a nonselective beta blocker, propranolol has both beta 1 and beta 2 antagonistic effects. Beta-1 receptor blockade has a negative effect on chronotropy (lowers heart rate) and a negative effect on inotropy (reduces stroke volume), which causes a reflex increase in systemic vascular resistance (SVR) to preserve blood pressure. Beta-2 receptor blockade causes an additional increase in SVR. The overall reduction in blood pressure, if any, is mild because the reduction in cardiac output is mitigated by the increase in SVR.

A 27-year-old man comes to the physician because of a 1-day history of right-sided facial weakness and sound intolerance. Three days ago, he hit the right side of his head in a motor vehicle collision. He neither lost consciousness nor sought medical attention. Physical examination shows drooping of the mouth and ptosis on the right side. Sensation over the face is not impaired. Impedance audiometry shows an absence of the acoustic reflex in the right ear. Which of the following muscles is most likely paralyzed in this patient?

Asymmetric facial weakness and hyperacusis with an absent acoustic reflex indicate palsy of the facial nerve, which innervates structures derived from the 2nd branchial arch. An undisplaced temporal bone fracture can result in delayed-onset facial nerve palsy due to edema that leads to nerve compression.\ Stylohyoid 26% The stylohyoid muscle, which elevates the tongue and moves the hyoid bone backward during swallowing, develops from the 2nd branchial arch. Therefore, facial nerve palsy causes paralysis of this muscle. Other muscles that are derived from the 2nd branchial arch and thus innervated by the facial nerve include the facial muscles, platysma, posterior belly of the digastric, and stapedius. Stapedius involvement is demonstrated in this case by hyperacusis with absent acoustic reflex. Tensor tympani 56% The tensor tympani muscle, which dampens loud sounds such as chewing or thunder, is innervated by the mandibular branch of the trigeminal nerve. Therefore, a facial nerve palsy would not cause paralysis of this muscle.

A 60-year-old man is brought to the emergency department because of a 30-minute history of dizziness and shortness of breath. After establishing the diagnosis, treatment with a drug is administered. Shortly after administration, the patient develops severe left eye pain and decreased vision of the left eye, along with nausea and vomiting. Ophthalmologic examination shows a fixed, mid-dilated pupil and a narrowed anterior chamber of the left eye. The patient was most likely treated for which of the following conditions?

Atrioventricular block 45% The patient was most likely treated for atrioventricular block (AV block), which can present with shortness of breath and dizziness. Atropine (a muscarinic antagonist) is used to increase the heart rate in AV block due to its ability to reduce the effects of the parasympathetic nervous system on the SA node and the AV node. In the eye, atropine causes pupillary dilatation, which can result in narrowing of the anterior chamber angle and obstructed flow of aqueous humor between the posterior and anterior chamber, potentially leading to acute angle-closure glaucoma.

A 45-year-old woman comes to the physician because of fatigue, lightheadedness, dizziness upon standing, abdominal pain, and muscle pain over the past 6 months. She has also had an unintended weight loss of 5.8 kg (12.8 lb) over the past 3 years. She has had a history of hypoparathyroidism since she was a teenager. Her current medications include calcitriol and calcium carbonate. Her pulse is 85/min and blood pressure is 81/45 mm Hg. Physical examination shows tanned skin, as well as sparse axillary and pubic hair. Which of the following is the most likely cause of this patient's symptoms? This patient presents with the typical features of primary adrenal insufficiency: fatigue, orthostatic syncope (lightheadedness, dizziness upon standing), abdominal pain, myalgia, and weight loss (due to hypocortisolism), hypotension (due to hypoaldosteronism), sparse axillary and pubic hair (due to hypoandrogenism), and skin hyperpigmentation (due to increased ACTH levels).

Autoimmune destruction of the adrenal gland (autoimmune adrenalitis) is the most common cause of primary adrenal insufficiency, accounting for 80-90% of cases in the US. Autoimmune adrenalitis is often associated with other autoimmune endocrinopathies such as hypoparathyroidism, which is seen in this patient.

A 78-year-old woman is brought to the physician by her son because of progressive memory loss for the past year. She feels tired and can no longer concentrate on her morning crossword puzzles. She has gained 11.3 kg (25 lb) in the last year. Her father died from complications of Alzheimer disease. She has a history of drinking alcohol excessively but has not consumed alcohol for the past 10 years. Vital signs are within normal limits. She is oriented but has short-term memory deficits. Examination shows a normal gait and delayed relaxation of the achilles reflex bilaterally. Her skin is dry, and she has brittle nails. Which of the following is the most likely underlying etiology of this woman's memory loss?

Autoimmune thyroid disease 71% Hashimoto thyroiditis is the most common cause of hypothyroidism in the United States and is characterized by progressive destruction of thyroid tissue by antithyroid peroxidase antibodies and antithyroglobulin antibodies. Initially, it may manifest with transient hyperthyroidism, before progressing to hypothyroidism. Hypothyroidism results in a generalized decrease of the basal metabolic rate, which can present with weight gain, fatigue, dry skin and brittle nails, along with concentration and memory impairment.

femoral head necrosis

Avascular necrosis (AVN), also called osteonecrosis or bone infarction, is death of bone tissue due to interruption of the blood supply.[1] Early on, there may be no symptoms.[1] Gradually joint pain may develop which may limit the ability to move.[1] Complications may include collapse of the bone or nearby joint surface.[1] Risk factors include bone fractures, joint dislocations, alcoholism, and the use of high-dose steroids.[1] The condition may also occur without any clear reason.[1] The most commonly affected bone is the femur.[1] Other relatively common sites include the upper arm bone, knee, shoulder, and ankle.[1] Diagnosis is typically by medical imaging such as X-ray, CT scan, or MRI.[1] Rarely biopsy may be used.[1]

A 4-year-old boy is brought to the physician by his mother because of generalized weakness and difficulty walking for the past month. Laboratory studies show a hemoglobin concentration of 6.6 g/dL, mean corpuscular volume of 74 μm3, platelet count of 150,000/mm3, and serum total bilirubin of 2 mg/dl. An MRI of the spine shows low signal intensity in all vertebral bodies and a small epidural mass compressing the spinal canal at the level of L1. A CT scan of the head shows osteopenia with widening of the diploic spaces in the skull. A biopsy of the epidural mass shows erythroid colonies with an abundance of megakaryocytes and myeloid cells. Which of the following is the most likely diagnosis?

Beta-thalassemia 57% In patients with beta-thalassemia, defective hemoglobin chain synthesis results in ineffective hematopoiesis and subsequent chronic anemia. In a state of severe chronic anemia, extramedullary hematopoiesis often occurs as a compensatory mechanism. Extramedullary hematopoiesis most commonly involves the paravertebral areas, as seen in this patient, followed by the liver, spleen, and lymph nodes. Aplastic anemia 10% Aplastic anemia manifests with pancytopenia and normocytic anemia, unlike this patient with a normal platelet count and microcytic anemia. Moreover, aplastic anemia would not be associated with hyperbilirubinemia and it does not lead to extramedullary hematopoiesis. Pancytopenia A decrease in number of all cell lines in blood (i.e., erythrocytes, leukocytes, platelets).

An investigator is studying the function of different enzymes in various human cell types. A subset of normal human cells with an elevated intracellular concentration of bisphosphoglycerate mutase is harvested and isolated. Which of the following is most likely to be absent in these cells?

Bisphosphoglycerate mutase is an enzyme of the glycolytic pathway that is present only in RBCs and a specific type of placental cell. α-ketoglutarate dehydrogenase 36% The enzyme α-ketoglutarate dehydrogenase would not be found in RBCs because they lack mitochondria, which is where the TCA cycle takes place. The main biochemical pathways available in RBCs are the pentose-phosphate pathway, glycolysis, and glutathione reduction. 2,3-bisphosphoglycerate mutase is vital to RBCs for the conversion of 1,3-BPG, an intermediate in glycolysis, to 2,3-BPG, which binds to hemoglobin and causes a conformational change that results in oxygen release into local tissues.

A 68-year-old man is brought to the emergency department because of a severe headache, nausea, and vomiting for 30 minutes. Forty-five minutes ago, he fell and struck his head, resulting in loss of consciousness for 1 minute. After regaining consciousness, he felt well for 15 minutes before the headache began. On arrival, the patient becomes rigid and his eyes deviate to the right; he is incontinent of urine. Intravenous lorazepam is administered and the rigidity resolves. Which of the following is the most likely cause of the patient's condition?

Bleeding between dura mater and skull 73% Bleeding between the dura mater and the skull causes epidural hematoma. This diagnosis should be suspected in any patient with head trauma causing loss of consciousness followed by a lucid interval and subsequently renewed deterioration and signs of elevated ICP. The lucid interval represents the time required for the blood to accumulate in the epidural space to proportions great enough to compress the brain. Rupture of bridging veins 12% Rupture of bridging veins causes subdural hematoma (SDH). The onset of symptoms in patients with subdural hematoma depends on the size, location, and rate of growth of the SDH. In acute-onset SDH, the patient can also present with headache and signs of increased ICP immediately following head trauma. In rare cases, patients may also experience a lucid interval. However, the temporary recovery of consciousness is more commonly associated with a different diagnosis.

A 55-year-old man comes to the physician because of a 2-day history of severe perianal pain and bright red blood in his stool. Examination shows a bulging, red nodule at the rim of the anal opening. Which of the following arteries is the most likely source of blood to the mass found during examination?

Bright red blood in stool with a painful nodule at the rim of the anal opening suggests an external hemorrhoid. Hemorrhoids are swollen veins in your lower rectum. Internal hemorrhoids are usually painless, but tend to bleed. External hemorrhoids may cause pain. Hemorrhoids (HEM-uh-roids), also called piles, are swollen veins in your anus and lower rectum, similar to varicose veins Internal pudendal 67% The internal pudendal artery gives rise to the inferior rectal artery, which supplies the part of the anal canal below the pectinate line, where external hemorrhoids develop. An external hemorrhoid is often painful because the region below the pectinate line arises from ectoderm and is innervated by the inferior rectal nerve, which transmits somatic sensations such as pain. Internal hemorrhoids, which arise above the pectinate line, are supplied by the superior rectal artery, a branch of the inferior mesenteric artery). Unlike external hemorrhoids, internal hemorrhoids are typically painless because the region of the anal canal above the pectinate line arises from endoderm and is innervated by the inferior hypogastric plexus, which carries visceral sensation only. Dentate line Pectinate line An anatomical line that divides the upper and lower anal canal. Above the dentate line, the anal canal is supplied by the superior rectal artery and innervated by the inferior hypogastric plexus. Below the dentate line, the anal canal is supplied by the middle and inferior rectal arteries and innervated by the inferior rectal nerves.

An 84-year-old woman is brought by her caretaker to the physician because of a 2-day history of fever, severe headache, neck pain, and aversion to bright light. She appears uncomfortable. Her temperature is 38.5°C (101.3°F), pulse is 110/min, and blood pressure is 145/75 mm Hg. Physical examination shows involuntary flexion of the bilateral hips and knees with passive flexion of the neck. Cerebrospinal fluid analysis shows a leukocyte count of 1200/mm3 (76% segmented neutrophils, 24% lymphocytes), a protein concentration of 113 mg/dL, and a glucose concentration of 21 mg/dL. A CT scan of the brain shows leptomeningeal enhancement. Which of the following is the most appropriate initial pharmacotherapy?

Ceftriaxone, vancomycin, and ampicillin 79% Third generation cephalosporins (e.g., cefotaxime, ceftriaxone) are broad-spectrum antibiotics that are effective against most gram-negative and gram-positive bacteria with the exception of Enterococcus and Listeria. Since L. monocytogenes is an important cause of bacterial meningitis in patients > 60 years, empiric antibiotic therapy in this group should include ampicillin, an anti-listerial antibiotic. Ampicillin would also provide coverage against gram-positive and gram-negative bacteria, but its spectrum of activity is not as broad as 3rd generation cephalosporins. Vancomycin, which acts only against gram-positive bacteria, should be included in the antibiotic regimen because the incidence of penicllin- and cephalosporin-resistant pneumococcal infection has increased in recent years. Vancomycin, while effective in Listeria bacteremia, is not very effective against CNS Listeria infections. Therefore, ampicillin should be used despite the addition of vancomycin to the regimen. This patient has classic findings of bacterial meningitis: fever, confusion, Brudzinski sign, and CSF analysis showing ↑ cell count, ↑ protein, and ↓ glucose. The most common causes of community-acquired bacterial meningitis in patients older than 60 years are the gram-positive bacteria S. pneumoniae and L. monocytogenes, and the gram-negative bacteria H. influenzae and E. coli.

A 2900-g (6.4-lb) male newborn is delivered at term to a 29-year-old primigravid woman. His mother had no routine prenatal care. She reports that the pregnancy was uncomplicated apart from a 2-week episode of a low-grade fever and swollen lymph nodes during her early pregnancy. She has avoided all routine vaccinations because she believes that "natural immunity is better." The newborn is at the 35th percentile for height, 15th percentile for weight, and 90th percentile for head circumference. Fundoscopic examination shows inflammation of the choroid and the retina in both eyes. A CT scan of the head shows diffuse intracranial calcifications and mild ventriculomegaly. Prenatal avoidance of which of the following would have most likely prevented this newborn's condition?

Chorioretinitis, diffuse intracranial calcifications, and hydrocephalus (indicated by ventriculomegaly) in a newborn are characteristic of congenital toxoplasmosis. Undercooked pork 55% In the US, consumption of contaminated, undercooked meat is the most common cause of Toxoplasma gondii infection during pregnancy. While the likelihood of transplacental transmission is low in early pregnancy, the risk of symptomatic toxoplasmosis in the newborn is highest if infection occurs in the first trimester. Maternal toxoplasmosis is usually asymptomatic but may manifest with general symptoms, such as low-grade fever and cervical lymphadenopathy, both of which are reported here. Other sources of maternal infection include accidental ingestion of contaminated cat feces and drinking unpasteurized goat's milk.

A 68-year-old man with alcohol use disorder is brought to the physician by his sister for frequent falls and an unsteady gait over the past 2 months. He has not seen a physician in 10 years. He appears emaciated and inattentive. He is oriented to person only. Physical examination shows a wide-based gait with slow, short steps. Eye examination shows lateral gaze paralysis and horizontal nystagmus. One month later, he dies. Which of the following is the most likely finding on autopsy?

Confusion, ophthalmoplegia (lateral gaze paralysis), nystagmus, and ataxia in a malnourished patient with alcohol use disorder indicate Wernicke encephalopathy. Small vessel hemorrhage in mammillary bodies 67% Small vessel hemorrhage and necrosis in the mammillary bodies of the hypothalamus can be found in patients with Wernicke encephalopathy, a sequela of severe thiamine (vitamin B1) deficiency. Thiamine deficiency causes neuronal injury and axonal conduction impairment, which leads to a classic triad of confusion/memory loss, ataxia, and ophthalmoplegia. Alcohol use disorder, as seen in this patient, decreases intake, absorption, and hepatic storage of thiamine, and is the most important risk factor associated with Wernicke encephalopathy.

A 2-year-old girl is brought to the physician by her parents because of clumsiness and difficulty walking. She began to walk at 12 months and continues to have difficulty standing still without support. She also appears to have difficulty grabbing objects in front of her. Over the past year, she has had 5 episodes of sinusitis requiring antibiotic treatment and was hospitalized twice for bacterial pneumonia. Physical examination shows an unstable, narrow-based gait and several hyperpigmented skin patches. Serum studies show decreased levels of IgA and IgG and an increased level of alpha-fetoprotein. Over the next 5 years, which of the following complications is this patient most likely to develop?

Conjunctival telangiectasias 53% Conjunctival telangiectasias are a hallmark of the inherited syndrome ataxia telangiectasia (AT) and typically develop before the age of 5. Telangiectasias may also appear on other areas of exposed skin, particularly the face. The decreased serum levels of immunoglobulins predispose to infections, particularly of the respiratory tract. AT is due to an autosomal recessive mutation in the ATM gene, which assists in DNA break repair (nonhomologous end joining). This young patient presents with cerebellar ataxia and recurrent sinopulmonary infection along with laboratory evidence of decreased immune competency and elevated alpha-fetoprotein (AFP). This constellation of findings strongly suggests an underlying mutation in the ATM gene.

Cornyebacterium diptheriae

Corynebacterium diphtheriae infects the nasopharynx or skin. Toxigenic strains secrete a potent exotoxin which may cause diphtheria. The symptoms of diphtheria include pharyngitis, fever, swelling of the neck

A group of investigators are studying the effects of transcranial direct current stimulation (tDCS) on cognitive performance in patients with Alzheimer disease. A cohort of 50 patients with mild Alzheimer disease were randomized 1:1 to either tDCS or sham tDCS over the temporoparietal cortex. Both procedures were conducted so that patients experienced the same sensations while receiving treatment. After 1 week of observation during which no treatments were delivered, the two groups were switched. Neuropsychiatric testing was subsequently conducted to assess differences in recognition memory between the two groups. Which of the following best describes the study design?

Crossover 89% In a crossover study design, every participant receives all the interventions but at different times and/or in a different sequence. In the study described in this vignette, all the participants sequentially received tDCS and sham tDCS (i.e., the groups crossed over). A washout period between treatments is commonly included in crossover studies to enable any potentially confounding effects of the first treatment to wear off prior to starting the sequential treatment. Washout period A period of time during which a patient receives no treatment (e.g., medication) before starting a new therapy. E.g., used in clinical trials to help ensure that observed effects are due to the new therapy. Confounding A type of systematic error in which a third variable that has not been factored into the study (the confounder) has an association with both the dependent variable (outcome) as well as the independent variable (e.g., exposure to a risk factor). The confounder can suggest a causal association of the outcome with the independent variable, when, in fact, there is none.

A 48-year-old man is brought to the emergency department 20 minutes after being rescued from a house fire. He reports headache, metallic taste, abdominal pain, and nausea. He appears confused and agitated. His pulse is 125/min, respirations are 33/min, and blood pressure is 100/65 mm Hg. Pulse oximetry on room air shows an oxygen saturation of 98%. Physical examination shows a bright red color of the skin. His breath smells of bitter almonds. Hyperbaric oxygen therapy and appropriate pharmacotherapy are initiated. The expected beneficial effect of this drug is most likely due to which of the following mechanisms? This patient's headache, abdominal pain, altered mental status, tachycardia, shortness of breath, bright red skin tone, and breath smelling of bitter almonds with a normal pulse oximetry is most likely caused by cyanide exposure from the house fire. One approach in the treatment of cyanide toxicity includes giving nitrites (e.g., sodium nitrite).

Cyanide impairs the mitochondrial enzyme cytochrome c oxidase, leading to histotoxic hypoxia. Clinically, this manifests as headache, altered mental status, tachycardia, flushed skin, and almond-scented breath. Nitrites oxidize hemoglobin into methemoglobin, which binds cyanide, thereby allowing oxidative phosphorylation to resume and symptoms to improve. Other important treatments include hydroxocobalamin (which binds cyanide to form the harmless compound cyanocobalamin) and sodium thiosulfate (converts cyanide to thiocyanide, which is then safely excreted).

A 6-year-old boy is brought to the physician by his parents because of low performance in school since starting 1st grade 2 months ago. His mother reports that, according to his teachers, he always prefers to sit close to the blackboard. He continuously interrupts other children while they are talking and is unable to sit quietly through a classroom period. When he is asked by the teacher to stop, he bursts into tears and leaves the classroom. His parents say that "he has always been a bit slower than his peers." Physical examination shows pale skin, long and slender toes and fingers, and hunching of the back. Visual acuity is decreased bilaterally. His serum methionine concentration is increased. A deficiency of which of the following enzymes is the most likely cause of these findings?

Cystathionine synthase converts homocysteine and serine to cystathionine using vitamin B6 as a cofactor; a deficiency of this enzyme is one of the causes of homocystinuria. In homocystinuria caused by cystathionine synthase deficiency or decreased affinity of vitamin B6 for cystathionine synthase, homocysteine is converted back to methionine by methionine synthase, resulting in an increased serum methionine concentration, as seen here. The treatment for cystathionine synthase deficiency consists of a diet low in methionine and rich in cysteine, and supplementation of vitamin B6, B12, and folate.

A 55-year-old man is brought to the emergency department because of sudden-onset of difficulty speaking for 1 hour. He has hypertension, dyslipidemia, and type 2 diabetes mellitus. Current medications include atorvastatin, lisinopril, and metformin. He is 213 cm (7 ft) tall. On examination, the patient speaks slowly in simple sentences that consist of only one word. He seems to have difficulties understanding grammatically complex sentences and appears to be frustrated at the inability to speak fluently. He is able to follow commands. When another individual tries to enter the room, the physician asks the patient to "hold the door," and he does so immediately. Based on his symptoms, which of the following labeled areas in the schematic overview of a normal brain is most likely affected in this patient?

D 74% This patient's nonfluent, telegraphic speech pattern with constant repetition of one word, in combination with preserved comprehension of simple sentences and commands, is characteristic of Broca aphasia (expressive aphasia). An ischemic stroke involving area D, the Broca area, in the dominant (usually left) hemisphere results in expressive aphasia. Affected individuals typically appear frustrated by their speech deficits. Language comprehension is relatively intact as long as the Wernicke area in the temporal lobe is unaffected, although there may be difficulties understanding grammatically complex speech. Broca aphasia due to cerebral infarction may occur as an isolated finding or can be accompanied by contralateral, brachiofacial sensorimotor hemiparesis.

A 19-year-old man comes to the physician because of episodic tingling and numbness in his right hand for the past 3 weeks. He says that he cannot hold a pen very well and that this is affecting his handwriting. The symptoms started after he played in a football game. Physical examination shows decreased pinch strength in the right hand. Sensation to light touch is decreased over the right little finger and the medial palmar surface of the right hand. Radiologic evaluation in this patient is most likely to show a fracture in which of the following bones?

Decreased pinch strength and sensory deficits over the little finger and palmar surfaces of the medial aspect of the right hand are suggestive of ulnar nerve entrapment at the level of the wrist. G 41% Blunt trauma to the hook of hamate (e.g., fall on an outstretched hand) can lead to entrapment of the ulnar nerve, which is referred to as a distal ulnar lesion. In addition to paresthesias, numbness, weakness, and pain, this lesion can also cause claw hand deformity.

A 25-year-old woman comes to the physician because of a 4-month history of anxiety and weight loss. She also reports an inability to tolerate heat and intermittent heart racing for 2 months. She appears anxious. Her pulse is 108/min and blood pressure is 145/87 mm Hg. Examination shows a fine tremor of her outstretched hands. After confirmation of the diagnosis, the patient is scheduled for radioactive iodine ablation. At a follow-up visit 2 months after the procedure, she reports improved symptoms but new-onset double vision. Examination shows conjunctival injections, proptosis, and a lid lag. Slit-lamp examination shows mild corneal ulcerations. The patient is given an additional medication that improves her diplopia and proptosis. Which of the following mechanisms is most likely responsible for the improvement in this patient's ocular symptoms?

Decreased production of proinflammatory cytokines 64% Glucocorticoids such as prednisolone are the preferred therapy for moderate to severe Graves ophthalmopathy, which results from inflammation and soft tissue enlargement in the orbit. In Graves disease, TSH receptor antibodies stimulate orbital fibroblasts to secrete glycosaminoglycan (GAG) and the adipocytes to proliferate. GAGs are hyperosmolar, drawing fluid out into the interstitial space, which causes edema of the orbital tissues. GAG accumulation, edema, and increased adipocytes increase the orbital pressure and push the globe forwards, causing exophthalmos, compression of the extraocular muscles, ophthalmoplegia (inability to converge eyes, diplopia, difficulty reading), and ocular pain. Glucocorticoids improve these symptoms via their anti-inflammatory and immunosuppressive effects. Symptoms of thyrotoxicosis (e.g., anxiety, weight loss, heat intolerance, palpitations, tachycardia, tremor) together with ophthalmopathy (e.g., conjunctival injections, proptosis, lid lag, diplopia) are highly suggestive of Graves disease. Though several drugs are effective in treating symptoms of hyperthyroidism, only one addresses the underlying cause of Graves ophthalmopathy.

An investigator studying disorders of hemostasis performs gene expression profiling in a family with a specific type of bleeding disorder. These patients were found to have abnormally large von Willebrand factor (vWF) multimers in their blood. Genetic analysis shows that the underlying cause is a mutation in the ADAMTS13 gene. This mutation results in a deficiency of the encoded metalloprotease, which is responsible for cleavage of vWF. Which of the following additional laboratory findings is most likely in these patients?

Defects in hemostasis due to a gene mutation (in ADAMTS13) that causes impaired vWF cleavage is consistent with hereditary thrombotic thrombocytopenic purpura. Thrombotic thrombocytopenic purpura (TTP) is a blood disorder that results in blood clots forming in small blood vessels throughout the body. This results in a low platelet count, low red blood cells due to their breakdown, and often kidney, heart, and brain dysfunction. Fragmented erythrocytes

A 39-year-old man comes to the physician for evaluation of hearing loss. He reports difficulty hearing sounds like the beeping of the microwave or birds chirping, but can easily hear the pipe organ at church. He works as an aircraft marshaller. A Rinne test shows air conduction greater than bone conduction bilaterally. A Weber test does not lateralize. Which of the following is the most likely underlying cause of this patient's condition?

Destruction of the organ of Corti 64% Noise-induced hearing loss is caused by the destruction of cochlear hair cells, also known as the organ of Corti. In a normally functioning ear, sound waves are transmitted mechanically through the tympanic membrane, via the ossicles, and through the oval window to the perilymph-filled inner ear. The resulting vibrations are transmitted to the cochlear hair cells, from where neurologic impulses are transmitted via the vestibulocochlear nerve. Repeated exposure to sounds louder than 85 dB, like those this man experiences at work, or even an isolated exposure to extremely loud sounds greater than 140 dB, can damage the organ of Corti and result in subsequent sensorineural hearing loss that typically affects high-frequency hearing first. Bilateral high-frequency hearing loss in a patient with occupational noise exposure should raise concern for noise-induced hearing loss. The absence of lateralization on Weber tests suggests that both ears are equally affected, while the normal Rinne test suggests that the hearing loss is sensorineural in origin.

Two weeks after undergoing allogeneic stem cell transplant for multiple myeloma, a 55-year-old man develops a severely pruritic rash, abdominal cramps, and profuse diarrhea. He appears lethargic. Physical examination shows yellow sclerae. There is a generalized maculopapular rash on his face, trunk, and lower extremities, and desquamation of both soles. His serum alanine aminotransferase is 115 U/L, serum aspartate aminotransferase is 97 U/L, and serum total bilirubin is 2.7 mg/dL. Which of the following is the most likely underlying cause of this patient's condition?

Donor T cells in the graft 67% During allogeneic stem cell transplantation, donor T cells can perceive host tissue as a foreign antigen and mount an immune response, resulting in acute GvHD within 100 days of transplantation. It commonly affects the skin, mucosa, liver, and intestine; thus, patients present with maculopapular rash, diarrhea, jaundice, and hepatosplenomegaly, as observed in this patient. Acute graft-versus-host disease Abbreviation: Acute GvHD A complication of bone marrow transplant in which donor T lymphocytes react with the recipient's organs < 100 days after transplantation. Presentations include a pruritic maculopapular rash, nausea, vomiting, diarrhea, and hepatic dysfunction. Treatments include immunosuppressants, such as cyclosporine and corticosteroids (topical and systemic).

A 56-year-old man comes to the physician because of worsening double vision and drooping of the right eyelid for 2 days. He has also had frequent headaches over the past month. Physical examination shows right eye deviation laterally and inferiorly at rest. The right pupil is dilated and does not react to light or with accommodation. The patient's diplopia improves slightly on looking to the right. Which of the following is the most likely cause of this patient's findings?

Double vision, eyelid droop, lateral and inferior eye deviation at rest, and a dilated pupil that does not react to light point to oculomotor nerve (CN III) palsy, with both somatic and parasympathetic involvement. An aneurysm of the posterior communicating artery can cause compression of the nearby oculomotor nerve (CN III). Compressive lesions are characterized by pupillary involvement due to injury to the outer parasympathetic layer of CN III (which controls pupil dilation, light reactivity, and accommodation). Additional injury to the deeper somatic fibers of the nerve (which control extraocular muscles) is responsible for this patient's ptosis and "down and out" gaze.

A 62-year-old man with hypertension and atrial fibrillation is brought to the emergency department because of sudden onset visual changes. He stopped taking all of his medications 2 months ago. Blood pressure is 170/90 mm Hg and pulse is 108/min. He is alert and oriented to person but does not follow commands. His speech is fluent with normal cadence, but he uses frequent non-existent words and the content of his speech is nonsensical. He appears unaware of his speech impairment. He is unable to read a text out loud. Peripheral vision is diminished in the right upper visual fields bilaterally. This patient's presentation is most consistent with injury to which of the following labeled areas?

E The patient's presentation of Wernicke aphasia and right upper homonymous quadrantanopsia is most consistent with an ischemic injury to the superior temporal gyrus (region E) due to an ischemic stroke involving the inferior division of the left MCA. His visual field deficits are the result of the involvement of the inferior fascicle of the left optic radiations (Meyer loop) in the temporal lobe. His atrial fibrillation and medical noncompliance suggest a thromboembolic stroke as the underlying cause.

A 26-year-old man comes to the physician for a follow-up examination. Two weeks ago, he was treated in the emergency department for head trauma after being hit by a bicycle while crossing the street. Neurological examination shows decreased taste on the right anterior tongue. This patient's condition is most likely caused by damage to a cranial nerve that is also responsible for which of the following? Taste on the anterior two-thirds of the tongue is mediated by cranial nerve VII.

Eyelid closure Closing the eyelids is accomplished through the contraction of the orbicularis oculi muscle, which is, like all facial muscles, innervated by the facial nerve (CN VII). The chorda tympani branch of the facial nerve innervates the taste buds of the anterior two-thirds of the tongue. The facial nerve is also responsible for innervation of the lacrimal gland, submandibular and sublingual glands, and the stapedius muscle.

A 55-year-old man comes to the physician with a 3-month history of headache, periodic loss of vision, and easy bruising. Physical examination shows splenomegaly. His hemoglobin concentration is 13.8 g/dL, leukocyte count is 8000/mm3, and platelet count is 995,000/mm3. Bone marrow biopsy shows markedly increased megakaryocytes with hyperlobulated nuclei. Genetic analysis shows upregulation of the JAK-STAT genes. The pathway encoded by these genes is also physiologically responsible for signal transmission of which of the following hormones?

Easy bruising, periodic loss of vision, and splenomegaly in this patient with marked thrombocytosis and megakaryocytic hyperplasia are suggestive of essential thrombocythemia. The vast majority of patients with essential thrombocythemia have mutations involving the JAK-STAT signaling pathway. The hormones produced by the acidophil cells of the pituitary gland also transmit their signals via this pathway. Essential thrombocythemia Abbreviation: ET A myeloproliferative neoplasm marked by a massive elevation of thrombocytes (≥ 450,000/μL) that cannot be explained by reactive thrombocytosis. It may cause thromboembolic events, petechial bleeding, and vasomotor symptoms such as headache, visual disturbances, or acral paresthesias. In 50% of patients, JAK2 is mutated. Prolactin 45% Prolactin, which is produced in the acidophil cells of the anterior pituitary, exerts its effect via the JAK-STAT signaling pathway, a type of nonreceptor tyrosine kinase pathway. Nonreceptor tyrosine kinase pathways also mediate the effects of GH, erythropoietin, thrombopoietin, G-CSF, and various immune signaling molecules (e.g., IL-2, IFN-γ). In essential thrombocythemia, the diagnosis in this patient, it is thought that a mutation involving the JAK-STAT pathway permanently activates the thrombopoietin receptor, resulting in increased platelet production and thrombocytosis.

An 18-month-old boy is brought to the emergency department by his babysitter because of lethargy and fever for 3 hours. He has not fully recovered from a middle ear infection that started a few days ago. The parents cannot be reached for further information. His temperature is 39.1°C (102.3°F). Physical examination shows nuchal rigidity. A pathogen is isolated from the patient's cerebrospinal fluid that does not grow on regular blood agar plate but produces colonies when cocultured with Staphylococcus aureus. Vaccination against the causal pathogen of this patient's current disease is most likely to also prevent which of the following conditions?

Epiglottitis In unvaccinated children, epiglottitis is most commonly caused by Haemophilus influenzae type b (Hib). Prior to the introduction of routine Hib vaccination, this bacterium was also the leading cause of pediatric bacterial meningitis, pneumonia, empyema, pericarditis, bacteremia, and septic arthritis. Routine childhood vaccination has decreased the incidence of epiglottitis significantly. Unencapsulated strains (not covered by the Hib vaccine) are still common causes of otitis media, sinusitis, conjunctivitis, and pneumonia, especially in the pediatric population. The pathogen in question is likely Haemophilus influenzae. It requires factors X (hematin) and V (NAD) for growth, which are not found in regular sheep blood agar. Staphylococcus aureus produces NAD and lyses erythrocytes in the agar, which releases hematin into the surrounding growth medium and allows Haemophilus to grow within the hemolytic zone of S. aureus. This is known as the satellite phenomenon or satellitism. This child likely did not receive Hib vaccination.

excoriate

Excoriation of the skin refers to lesions on the surface of the skin, following a trauma. The blood and fluids that emerge from the surface form a thin crust, resulting in a skin lesion. There are two main types of lesions: primary and secondary.

During a clinical study examining the diffusion of gas between the alveolar compartment and the pulmonary capillary blood, men between the ages of 20 and 50 years are evaluated while they hold a sitting position. After inhaling a water-soluble gas that rapidly combines with hemoglobin, the concentration of the gas in the participant's exhaled air is measured and the diffusion capacity is calculated. Assuming that the concentration of the inhaled gas remains the same, which of the following is most likely to increase the flow of the gas across the alveolar membrane?

Exercise increases the surface area available for gas diffusion by recruiting additional lung zones not used during at-rest respiration. The increased cardiac output resulting from exercise also reduces capillary transit time, leading to a nearly two-fold increase in the diffusion capacity for carbon monoxide across the alveolar membrane. This effect is magnified with increased exercise intensity.

A 43-year-old woman comes to the physician because of tingling and weakness in her left arm for the past 2 days. An image of the brachial plexus is shown. Nerve conduction study shows decreased transmission of electrical impulses in the labeled structure. Physical examination is most likely to show impairment of which of the following movements? The structure marked in the image is the radial nerve.

Extension of the wrist and fingers 66% The radial nerve originates from the posterior cord of the brachial plexus (comprises cervical roots C5-T1) and provides motor innervation to the triceps and muscles in the posterior compartment of the forearm, including all of the wrist and hand extensors. Injury to the radial nerve, as seen in this diagram, would result in wrist drop as well as loss of sensation over the posterior forearm and dorsal hand.

A 70-year-old man is brought to the emergency department by staff of the group home where he resides because of worsening confusion for the past week. He has a history of major depressive disorder and had an ischemic stroke 4 months ago. Current medications are aspirin and sertraline. He is lethargic and disoriented. His pulse is 78/min, and blood pressure is 135/88 mm Hg. Physical examination shows moist oral mucosa, normal skin turgor, and no peripheral edema. While in the waiting room, he has a generalized, tonic-clonic seizure. Laboratory studies show a serum sodium of 119 mEq/L and an elevated serum antidiuretic hormone concentration. Which of the following sets of additional laboratory findings is most likely in this patient? Serum osmolalityUrine sodiumSerum aldosteroneA↑normalnormalB↓↓↓C↑↓normalD↓↓↑E↓↑↑F↓↑↓

F 49% SIADH manifests with hypotonic hyponatremia and decreased serum aldosterone. ADH increases the expression of aquaporin-2 channels in the distal convoluted tubules and the collecting ducts of nephrons. This increases the resorption of solute-free water from urine into systemic circulation, which concentrates the urine (hypernatriuria), increases urine osmolality, dilutes serum (hyponatremia), and decreases serum osmolality. The resulting hyponatremia causes the osmotic shift of fluid into cells, which may cause cerebral edema, increased intracranial pressure, and, ultimately, confusion, lethargy, and seizures. Patients remain euvolemic (euvolemic hyponatremia) because transient plasma volume expansion increases ANP secretion, which decreases aldosterone secretion, leading to natriuresis and the inhibition of sodium reabsorption in the collecting duct.

A 26-year-old woman comes to the physician because of several days of fever, abdominal cramps, and diarrhea. She drank water from a stream 1 week ago while she was hiking in the woods. Abdominal examination shows increased bowel sounds. Stool analysis for ova and parasites shows flagellated multinucleated trophozoites. Further evaluation shows the presence of antibodies directed against the pathogen. Secretion of these antibodies most likely requires binding of which of the following?

Flagellated multinucleated trophozoites on stool analysis are consistent with giardiasis. The immune response to bacterial and protozoal intestinal infections relies on the production of secretory IgA, which is produced in Peyer patches in the intestinal mucosa. CD40 to CD40 ligand 66% The binding of the CD40 membrane receptor on B cells with CD40 ligand (CD40L) on the surface of T helper cells is the second step in immunoglobulin class switching and is necessary for the secretion of IgA in patients with Giardia lamblia infection. The first step in B cell activation is the binding of antigen-presenting MHC II on B cells to the CD4 receptor on T helper cells. CD40 ligand CD154, CD40L A cluster of a differentiation molecule that is encoded by a gene on chromosome Xq and upregulated on activated CD4 T-cells (T helper cells). CD40L on T-helper cells serves as a ligand for the CD40 molecule on B cells; binding of CD40L with CD40 causes immunoglobulin phenotype switching (i.e., the production of immunoglobulins other than IgM) by B cells CD40 A surface membrane receptor on B cells that binds to CD40L on the surface of CD4+ T helper cells. CD40-CD40L serves as the second signal for B cell activation and induces Ig class switching.

An investigator is conducting an experiment to develop a new drug against HIV infection. HIV-infected and uninfected CD4 T-cell lines are mixed in-vitro with a chemically modified synthetic peptide triazole complex. The complex is found to have a high binding affinity to the gp120 protein, irreversibly inhibiting its function and reducing CD4 T cell infection. Inhibition of which of the following steps of HIV infection is the most likely therapeutic effect of this experimental drug?

Glycoprotein 120 (gp120) is exposed on the surface of the HIV envelope. Attachment to host CD4 T-cells The HIV envelope glycoprotein 120 (gp120) binds to CD4 receptors on host T-helper cells and macrophages. Gp120 functions as a docking protein for the virus and is essential for infection of host cells. A drug that inhibits gp120 would inhibit the attachment of HIV virus to host CD4 T-cells.

A 23-year-old woman is brought to the emergency department 30 minutes after stepping on a piece of broken glass. Physical examination shows a 3-cm, ragged laceration on the plantar aspect of the left foot. The physician uses hydrogen peroxide to clean the wound. Which of the following is the most likely mechanism of action of this disinfectant? Hydrogen peroxide is an oxidant.

Formation of free radicals 93% Hydrogen peroxide oxidizes cell membrane lipids, intracellular proteins, and DNA by forming hydroxyl free radicals. It is used in wound disinfection and as a surface disinfectant and is active against bacteria, yeasts, fungi, viruses, and spores. Halogenation of nucleic acids 3% Iodine and iodophors (e.g., povidone-iodine and poloxamer-iodine) act as disinfectants by halogenating RNA, DNA, and proteins of bacteria, viruses, and fungi. This halogenation interferes with their pathogenicity. Hydrogen peroxide is not known to function as a halogenating agent.

`gastric mucosal cells

Four major types of secretory epithelial cells cover the surface of the stomach and extend down into gastric pits and glands: Mucous cells: secrete an alkaline mucus that protects the epithelium against shear stress and acid. Parietal cells: secrete hydrochloric acid. Chief cells: secrete pepsin, a proteolytic enzyme.

A 34-year-old man comes to the physician because of a 2-day history of progressively blurred vision. He also reports seeing flashing lights in his visual field. He does not have any pain. The patient has not been examined by a physician in several years. He appears emaciated. Examination shows right conjunctival injection. Visual acuity is 20/20 in the left eye and 20/100 in the right eye. Fundoscopic examination of the right eye is shown. His CD4+ T-lymphocyte count is 46/mm3. Which of the following is the most appropriate pharmacotherapy for this patient's eye condition?

Fundoscopy in this patient, whose depressed CD4+ T-lymphocyte count is almost certainly secondary to infection with the human immunodeficiency virus (HIV), shows fluffy, hemorrhagic, perivascular retinal lesions that radiate out from the optic disc (pizza-pie appearance). These symptoms and findings are characteristic of cytomegalovirus (CMV) retinitis. Valganciclovir 60% Valganciclovir is a first-line systemic therapy for CMV retinitis, a condition that can cause blurring, vision loss, floaters, and photopsia (flashing lights). It usually occurs unilaterally (right eye in this patient) but can affect the other eye if untreated. Other first-line systemic therapies include ganciclovir, foscarnet, and cidofovir. Patients with acutely sight-threatening disease (e.g., lesions < 1.5 mm from the fovea or adjacent to the optic nerve head) should also receive intravitreal therapy with ganciclovir or foscarnet. Other CMV infections that develop in patients with CD4+ T-cell count < 50/mm3 include CMV esophagitis, CMV colitis, CMV pneumonitis, and CMV encephalitis.

A 29-year-old man comes to the emergency department because of progressively worsening fatigue and shortness of breath for the past 2 weeks. His only medication is insulin. Examination shows elevated jugular venous distention and coarse crackles in both lungs. Despite appropriate life-saving measures, he dies. Gross examination of the heart at autopsy shows concentrically thickened myocardium and microscopic examination shows large cardiomyocytes with intracellular iron granules. Examination of the spinal cord shows atrophy of the lateral corticospinal tracts, spinocerebellar tracts, and dorsal columns. Which of the following is the most likely underlying cause of this patient's condition?

GAA trinucleotide repeat expansion on chromosome 9 49% A GAA trinucleotide repeat expansion on chromosome 9 results in defective frataxin production, which causes Friedreich ataxia. Frataxin is involved in the synthesis of iron-sulfur clusters, which are used in the mitochondrial respiratory chain. A deficiency of the protein causes iron to accumulate within cells, including in the heart, resulting hypertrophic cardiomyopathy, in pancreatic islets, which explains the patient's need for insulin therapy, and in the spinal cord, causing neurologic dysfunction. An SOD1 gene mutation on chromosome 21 is the underlying genetic pathology of familial amyotrophic lateral sclerosis (ALS). Although ALS can manifest with degeneration of the lateral corticospinal tracts (upper motor neuron), atrophy of the spinocerebellar tracts and dorsal columns would not be expected. In addition, ALS would also cause degeneration of the anterior horn cells (lower motor neuron), which is not seen here. Furthermore, ALS is not associated with diabetes mellitus or hypertrophic cardiomyopathy, and respiratory failure is the most common cause of death, not heart failure.

A 32-year-old woman comes to the physician because of a 3-month history of irregular menses, milky discharge from her nipples, fatigue, and weight gain. Menses occur at irregular 25-40-day intervals and last 1-2 days with minimal flow. 5 months ago, she was started on clozapine for treatment of schizophrenia. She has hypothyroidism but has not been taking levothyroxine over the past 6 months. Visual field examination show no abnormalities. Her serum thyroid-stimulating hormone is 17.0 μU/mL and serum prolactin is 85 ng/mL. Which of the following is the most likely explanation for the nipple discharge in this patient?

Galactorrhea and irregular menstrual cycles suggest hyperprolactinemia. Hypothyroidism 34% This patient has hypothyroidism and does not take her medications, possibly as a result of schizophrenia. Increased TSH in a hypothyroid patient suggests primary hypothyroidism. In primary hypothyroidism, decreased T3 and T4 levels stimulate the hypothalamus to release TRH (thyrotropin releasing hormone), which in turn increases TSH secretion by the anterior pituitary. In addition to increasing TSH production, excessive TRH also stimulates the lactotroph cells of the anterior pituitary to release prolactin, resulting in hyperprolactinemia.

A 72-year-old woman is brought to the physician by her daughter because of a 6-month history of worsening short-term memory deficits and social withdrawal. Treatment with galantamine is initiated. Two weeks later, the patient develops vomiting, mild crampy abdominal pain, and watery, nonbloody diarrhea. Which of the following is the most appropriate pharmacotherapy?

Galantamine is an acetylcholinesterase inhibitor (AChEI) used to treat Alzheimer disease. The most common adverse effects of AChEIs involve peristalsis and include nausea, vomiting, diarrhea, and muscle cramps, as seen in this patient. These symptoms are caused by repeated muscarinic receptor activation due to accumulation of ACh in the synaptic cleft. If this effect is not reversed, additional symptoms can develop, such as excessive lacrimation and bronchial secretion, as well as increased bowel and bladder activity, bronchospasm, and bradycardia. Atropine 47% Atropine is an antimuscarinic agent, which can reverse the effects of cholinergic toxicity by blocking ACh action in smooth muscle, secretory glands, and the CNS. Atropine is also used to treat acute symptomatic bradycardia or AV block, decrease salivation (e.g., in surgery), and reverse organophosphate poisoning (in combination with pralidoxime).

A 38-year-old woman comes to the physician because of a 1-month history of palpitations. She does not smoke or drink alcohol. Her pulse is 136/min and irregularly irregular. An ECG shows irregularly spaced QRS complexes with no distinct P waves. Treatment is started with a drug that slows atrioventricular node conduction velocity and prevents voltage-dependent calcium entry into myocytes. The patient is at greatest risk for which of the following adverse effects?

Gingival hyperplasia is a side effect of non-dihydropyridine calcium channel blockers such as verapamil. In patients with atrial fibrillation, non-dihydropyridine calcium channel blockers act as rate control agents through two distinct mechanisms. First, these agents slow down conduction of atrial electrical impulses through the AV node. Second, inhibition of calcium entry into ventricular myocytes reduces the ventricular rate. Verapamil, along with other CCBs like nifedipine and diltiazem, may cause gingival hyperplasia with long-term use. Other side effects include dose-dependent bradycardia, AV block, and hypotension.

A 70-year-old man is brought to the emergency department with painful discharge from his right ear with difficulty hearing for 3 days. His temperature is 39.5°C (103.1°F) and pulse is 120/minute. Physical examination shows mild facial asymmetry with the right corner of his mouth lagging behind the left when the patient smiles. There is severe ear pain when the right auricle is pulled superiorly. Otoscopic examination shows granulation tissue at the transition between the cartilaginous and the osseous part of the ear canal. Which of the following is most likely associated with this patient's condition?

Glucose intolerance There is a strong association between diabetes mellitus and the development of malignant otitis externa, which is most commonly caused by an infection with Pseudomonas aeruginosa. The specific connection between diabetes mellitus and malignant otitis externa is not fully understood but may be related to increased pH in diabetic cerumen. Elderly diabetic patients are especially at risk for this potentially fatal condition. Following blood cultures, high-dose IV ciprofloxacin should be initiated and cranial imaging performed. This patient has a history and physical exam findings consistent with malignant otitis externa given his severe ear pain, vital sign abnormalities (i.e., high fever, tachycardia), facial droop (indicating progression to osteomyelitis of the temporal bone with associated CN VII palsy), and the presence of granulation tissue in the ear canal.

A 53-year-old man comes to the physician because of fatigue, recurrent diarrhea, and an 8-kg (17.6-lb) weight loss over the past 6 months. He has a 4-month history of recurrent blistering rashes on different parts of his body that grow and develop into pruritic, crusty lesions before resolving spontaneously. Physical examination shows scaly lesions in different phases of healing with central, bronze-colored induration around the mouth, perineum, and lower extremities. Laboratory studies show: Hemoglobin 10.1 mg/dL Mean corpuscular volume 85 μm3 Mean corpuscular hemoglobin 30.0 pg/cell Serum Glucose 236 mg/dL Abdominal ultrasonography shows a 3-cm, solid mass located in the upper abdomen. This patient's mass is most likely derived from which of the following types of cells?

Glucose levels are abnormally high Pancreatic α-cells 71% This patient has a glucagonoma, a functional neuroendocrine tumor derived from pancreatic α-cells. These tumors secrete glucagon, which increases gluconeogenesis and glycogenolysis and thus results in hyperglycemia. Other features of glucagonoma include chronic diarrhea, weight loss, neuropsychiatric symptoms (e.g., depression, dementia, ataxia), deep vein thrombosis, normocytic anemia (due to anemia of chronic disease and the inhibitory effect of glucagon on erythropoiesis), and necrolytic migratory erythema. The pathophysiology that underlies the development of necrolytic migratory erythema is not well understood, but it may be related to direct action of glucagon on the skin, protein deficiency (because glucagon causes proteolysis), and/or a deficiency of zinc and fatty acids due to chronic diarrhea.

A 26-year-old primigravid woman at 25 weeks' gestation comes to the physician for a prenatal visit. She has no history of serious illness and her only medication is a daily prenatal vitamin. A 1-hour 50-g glucose challenge shows a glucose concentration of 167 mg/dL (N < 135). A 100-g oral glucose tolerance test shows glucose concentrations of 213 mg/dL (N < 180) and 165 mg/dL (N < 140) at 1 and 3 hours, respectively. If she does not receive adequate treatment for her condition, which of the following complications is her infant at greatest risk of developing?

Glucose readily crosses the placenta, meaning that elevated maternal glucose levels lead to elevated fetal blood glucose and, accordingly, cause physiologic changes that can lead to complications in the newborn infant. Islet cell hyperplasia 78% Hyperplasia of islet cells, the pancreatic cells responsible for producing insulin, can occur in utero in response to elevated maternal blood glucose levels, e.g., due to in gestational diabetes. This can cause infantile hyperinsulinemia and put the infant at risk for hypoglycemia once the maternal placental transfer of glucose ceases after birth.

A 1-year-old boy is brought to the physician by his mother because he has become increasingly pale over the past several months. He has otherwise been healthy. Apart from his maternal grandfather, who had a blood disorder and required frequent blood transfusions since birth, the rest of his family, including his parents and older sister, are healthy. Examination shows conjunctival pallor. Laboratory studies show: Hemoglobin 7.7 g/dL Mean corpuscular volume 64.8 μm3 Serum Iron 187 μg/dL Ferritin 306 ng/mL A bone marrow aspirate shows numerous ringed sideroblasts. The patient is most likely deficient in an enzyme responsible for which of the following reactions?

Glycine + succinyl-CoA → aminolevulinic acid This reaction, which is the first and rate-limiting step of heme synthesis, is catalyzed by δ-aminolevulinic acid synthase. A deficiency of this enzyme leads to insufficient heme production, which results in iron overload and perinuclear accumulation of iron-loaded mitochondria (i.e., ringed sideroblasts). A peripheral blood smear would show basophilic stippling. Sideroblastic anemia can be hereditary, as in this case, but can also be acquired as a result of heavy alcohol consumption, vitamin B6 deficiency, lead poisoning, copper deficiency, or myelodysplastic syndromes.

A 13-year-old girl is brought to the physician by her mother because of a 1-year history of worsening clumsiness. Initially, she swayed while walking; over the past 3 months, she has fallen 4 times. Ophthalmic examination shows a horizontal nystagmus. Proprioception and vibratory sensation are decreased in the distal extremities. Deep tendon reflexes are 1+ bilaterally. Further evaluation of the patient shows a genetic disorder involving an iron-binding mitochondrial protein encoded on chromosome 9. Which of the following findings is most likely to also be seen in this patient?

Hammer toes 48% Friedreich ataxia (FA) is characterized by the progressive degeneration of the spinocerebellar tract, lateral corticospinal tract, dorsal column, and dorsal root ganglia due to intramitochondrial accumulation of iron. Degeneration of the spinocerebellar tract can result in cerebellar signs such as ataxia, nystagmus, and dysarthria. Degeneration of the lateral corticospinal tract causes spasticity, which can result in deformities such as hammer toe, pes cavus, and kyphoscoliosis. Despite the presence of spasticity, patients often have decreased deep tendon reflexes due to the damage to dorsal root ganglia and the dorsal column. Dorsal column damage also causes impaired proprioception (which contributes to ataxia) as well as impaired vibration sense. Patients with FA will eventually develop diabetes mellitus and hypertrophic cardiomyopathy, which is the most common cause of death in FA.

A 22-year-old man comes to the physician because of a 2-week history of cough and decreased urination. The cough was initially nonproductive, but in the last few days he has coughed up small amounts of blood-tinged sputum with clots. He has not had any fevers, chills, or weight loss. He has smoked one pack of cigarettes daily for 5 years. Pulse is 115/min and blood pressure is 125/66 mm Hg. Physical examination shows dried blood around the lips. Serum studies show a creatinine of 2.9 mg/dL. Results of a serum antineutrophil cytoplasm antibody test are negative. A biopsy specimen of the kidney is most likely to show which of the following light microscopy findings?

Hemoptysis and rapidly deteriorating renal function are seen in Goodpasture syndrome, a type of pulmonary-renal syndrome. Fibrin crescents in Bowman space 59% In Goodpasture syndrome, a subtype of rapidly progressive glomerulonephritis, deposition of anti-glomerular basement membrane antibodies causes damage to the glomerular capillary wall, which allows plasma products such as fibrinogen and inflammatory mediators to escape from the glomerulus. This is characterized by crescent moon-shaped, fibrin-containing deposits within the Bowman capsule seen on light microscopy. Patients classically present with nephritic syndrome, oliguria or anuria, fatigue, and hemoptysis.

A 44-year-old man is brought to the emergency department 30 minutes after being involved in a high-speed motor vehicle accident as a restrained driver. His pulse is 145/min and blood pressure is 102/63 mm Hg. Physical examination shows dark bruising and discoloration over the right upper quadrant of his abdomen and right flank. The abdomen is guarded and markedly tender to palpation. A CT scan of the chest and abdomen shows lacerations and compression injuries in the right lobe of the liver. An emergency partial hepatectomy is performed. Ultrasonography on follow-up examination 6 months later shows that the liver has grown back to almost the original size. Microscopic examination of the newly formed liver tissue is most likely to show which of the following findings?

Hepatocytes, like the cells of many other glands, are stable cells that are capable of multiplying after becoming damaged. Normal hepatic tissue 68% Liver tissue has the unique ability to regenerate after resection or trauma through replication of remaining hepatocytes and repopulation from progenitor cells, such that regenerated areas will show normal hepatic tissue on microscopic examination. Stimulation of stable cells (e.g., injured hepatocytes) results in the transition from the G0 to G1 phase. Regeneration begins at the periportal zone, advances to the mid-zonal hepatocytes, then to the centrilobular zone, and ends at the central veins. After regeneration, biliary epithelial and endothelial cells proliferate and an extracellular matrix is produced by stellate cells. Through this process, the liver can restore itself to its full mass and preserve its essential metabolic functions.

A 67-year-old man is brought to the physician by his daughter because he frequently misplaces his personal belongings and becomes easily confused. His daughter mentions that his symptoms have progressively worsened for the past one year. On mental status examination, he is oriented to person, place, and time. He vividly recalls memories from his childhood but can only recall one of three objects presented to him after 5 minutes. His affect is normal. This patient's symptoms are most likely caused by damage to which of the following?

Hippocampus The earliest sites of neurodegeneration in Alzheimer disease are structures in the medial temporal lobe such as the hippocampus and parahippocampal cortex, which are critical for memory formation. Short-term memory loss is often the earliest sign of Alzheimer disease, the most common cause of progressive dementia in the United States.

An investigator is studying muscle tissue in high-performance athletes. He obtains blood samples from athletes before and after a workout session consisting of short, fast sprints. Which of the following findings is most likely upon evaluation of blood obtained after the workout session?

Humans use mainly anaerobic glycolysis to supply energy for short and strenuous exercises. Increased concentration of H+ 74% Anaerobic glycolysis is the main source for ATP creation when energy demand exceeds oxygen supply, e.g., in the case of strenuous exercise, as it does not require oxygen. Utilizing this pathway, glucose is metabolized into pyruvate, which is then converted by LDH into lactic acid, generating two ATP molecules. Lactic acid freely dissociates into lactate and H+. Thus, the concentration of H+ ions would increase and pH would be lowered in these athletes' blood samples. Decreased concentration of lactate 3% Anaerobic glycolysis is the main source for ATP creation when energy demand exceeds oxygen supply, e.g., in the case of strenuous exercise, as it does not require oxygen. Utilizing this pathway, glucose is metabolized into pyruvate, which is then converted into lactic acid and its conjugate base, lactate. Therefore, the concentration of lactate would increase, not decrease, in these athletes' blood samples.

A parapneumonic effusion is a type of pleural effusion that arises as a result of a pneumonia, lung abscess, or bronchiectasis. There are three types of parapneumonic effusions: uncomplicated effusions, complicated effusions, and empyema.

In a patient with COPD, shortness of breath and fever suggest superimposed pneumonia complicated by parapneumonic effusion, which is further supported by the results of pleural fluid analysis (a protein concentration of 4.0 g/dL and LDH of 180 U/L indicate exudative pleural effusion). Increased pulmonary capillary permeability 72% This patient's pleural effusion is due to increased pulmonary capillary permeability, most likely caused by inflammation from pneumonia (parapneumonic effusion). There are two main types of effusions, transudates and exudates. The distinction between the two is types is determined by Light criteria, a scoring system that evaluates three main characteristics of pleural fluid: the pleural fluid protein to serum protein ratio, the pleural fluid LDH to the serum LDH ratio, and pleural fluid LDH. This patient's pleural fluid LDH is more than two-thirds of the upper limit of the serum LDH reference range. It, therefore, fulfills Light criteria for exudative pleural effusion. Pleural effusion The presence of fluid in the pleural cavity. Classified as transudative (e.g., due to congestive heart failure, liver cirrhosis) or exudative (e.g., pneumonia, malignancies, pulmonary embolism), depending on the underlying cause. Typically manifests with dyspnea and a dry cough.

Hyperacusis is a hearing disorder that makes it hard to deal with everyday sounds. You might also hear it called sound or noise sensitivity. If you have it, certain sounds may seem unbearably loud even though people around you don't seem to notice them. Hyperacusis is rare

In addition to the hypertension, hyperlipidemia, and gingival hyperplasia seen in this patient, other adverse effects of this medication include hirsutism, nephrotoxicity, and neurotoxicity. Cyclosporine 57% Cyclosporine is an immunosuppressant drug that is commonly used for transplant rejection prophylaxis. It binds cyclophilin to form a complex that inhibits calcineurin. Inhibition of calcineurin blocks activation of the NFAT transcription factors, which prevents interleukin-2 transcription and, as a result, decreases T cell activation. Cyclosporine is the only calcineurin inhibitor to cause gingival hyperplasia, so it is most likely the drug that was used in this patient.

normal white blood cell count

In both forms of agranulocytosis, you have a dangerously low neutrophil count. Healthy levels in adults usually fall in the range of 1,500 to 8,000 neutrophils per microliter (mcL) of blood. In agranulocytosis, you have less than 500 per mcL.

A 3-year-old boy is brought to the physician for evaluation of a generalized, pruritic rash. The rash began during infancy and did not resolve despite initiating treatment with topical corticosteroids. Three months ago, he was treated for several asymptomatic soft tissue abscesses on his legs. He has been admitted to the hospital three times during the past two years for pneumonia. Physical examination shows a prominent forehead and a wide nasal bridge. Examination of the skin shows a diffuse eczematous rash and white plaques on the face, scalp, and shoulders. Laboratory studies show a leukocyte count of 6,000/mm3 with 25% eosinophils and a serum IgE concentration of 2,300 IU/mL (N = 0-380). Flow cytometry shows a deficiency of T helper 17 cells. The patient's increased susceptibility to infection is most likely due to which of the following?

In combination with his increased serum IgE, eosinophilia, and deficiency of Th17 cells, this patient's coarse facies (prominent forehead, wide nasal bridge), history of eczema, cold abscesses, and recurrent pneumonia are consistent with autosomal dominant hyperimmunoglobulin E syndrome (Job syndrome). Job syndrome Hyper-IgE syndrome A condition caused by a mutation in STAT3 that results in Th17 cell deficiency and, thus, impaired neutrophil recruitment to sites of infection. Characterized by increased serum IgE concentrations. Manifestations include coarse facial features, recurrent abscesses, retained primary teeth, and eczema. Th17 cells A subset of T helper cells that secretes IL-17 and plays an important role in neutrophil and macrophage recruitment to the site of infection. Impaired chemotaxis of neutrophils 42% Autosomal dominant hyperimmunoglobulin E syndrome (hyper-IgE syndrome) is caused by a mutation in the STAT3 gene, which leads to Th17 cell deficiency and decreased Th17-dependent production of IL-17. This results in impaired neutrophil chemotaxis, which causes decreased inflammation, cold abscesses (most commonly due to Staphylococcus aureus), and increased susceptibility to sinopulmonary infections (e.g., pneumonia). In addition to this patient's findings, hyper-IgE syndrome is also associated with other skeletal abnormalities (e.g., retained baby teeth, osteopenia/osteoporosis, pathologic fractures), and decreased IFN-γ.

A group of investigators discovers a novel monomeric enzyme that cleaves glutamate-valine bonds in a bacterial exotoxin. The substrate binding site of the enzyme is rich in aspartate. A sample of the enzyme is added to two serum samples containing the bacterial exotoxin. One sample is assigned a test condition while the other is maintained as the control. The averaged results of several trials comparing Vmax and Km between control serum and test serum are shown. Vmax (μmol/min)Km (mM)Control serum13.281.2Test serum28.880.9 Which of the following conditions in the test serum would best explain these findings?

Increased enzyme concentration 52% An increase in enzyme concentration increases the total number of active sites available to catalyze a reaction. It is the only method of increasing the maximum rate of reactions for an enzyme (Vmax) that has the unit of concentration/time. The Michaelis constant (Km) is the concentration of substrate at which the enzymatic rate is Vmax/2. Therefore, an increase in enzyme concentration does not change the Km of the enzyme. Km only changes with changes in affinity to the enzyme.

A 35-year-old woman, gravida 2, para 1, at 16 weeks' gestation comes to the office for a prenatal visit. She reports increased urinary frequency but otherwise feels well. Pregnancy and delivery of her first child were uncomplicated. Her vital signs are within normal limits. Pelvic examination shows a uterus consistent in size with a 16-week gestation. Urinalysis shows mild glucosuria. Laboratory studies show a non-fasting serum glucose concentration of 100 mg/dL. Which of the following is the most likely explanation for this patient's glucosuria?

Increased glomerular filtration rate is normal during pregnancy and leads to mild glucosuria even in patients with normal serum blood glucose concentrations. This change is caused by secretion of relaxin and progesterone during pregnancy, which causes significant vasodilation, decreasing systemic vascular resistance, increasing cardiac output, and increasing renal blood flow. Increased amounts of solutes enter the renal tubules and overload sodium/glucose cotransporters (SGLT2), which results in glucosuria.

One week after admission to the hospital for burns sustained in a building fire, a 24-year-old man undergoes evaluation for the feasibility of skin grafting. Physical examination shows third-degree burns on his entire torso, left arm, and head. Urine and blood cultures grow no organisms. A chest x-ray shows no abnormalities. Compared with a healthy adult, which of the following changes are most likely to be seen in this patient at this time?

Increased resting energy expenditure is a typical finding in patients one week after severe thermal injury. Severe burns, as seen in this patient, lead to a hypermetabolic state, which provides energy to the cells involved in wound healing and counters the significant heat and water loss that results from the loss of insulating cutaneous tissue. This postburn hypermetabolic response is characterized by two distinct phases. During the first phase (called the "Ebb phase," up to 48 hours postburn), cardiac output and metabolism are decreased. The second phase (called the "flow phase") is characterized by an increase in stress-induced hormones (e.g., catecholamines, glucocorticoids) and inflammatory cytokines (e.g., TNF-α), which lead to a gradual increase in metabolism, cardiac output, and body temperature until reaching a plateau (around one week postburn). Consequences of the hypermetabolic response include hyperglycemia, peripheral insulin resistance, increased muscle protein degradation, increased lipolysis, and significant weight loss.

Refractory schizophrenia

Patients with "treatment resistant" or "treatment refractory" disorders are those who do not respond to the usual first- and even second- and third-line treatments. Many people whose lives have been adversely affected by psychiatric illness struggle with a range of difficulties that may include.

A 1-month-old girl is brought to the physician for evaluation of a rash on her face that first appeared 3 days ago. She was delivered at term after an uncomplicated pregnancy. She is at the 25th percentile for length and 40th percentile for weight. Examination shows small perioral vesicles surrounded by erythema and honey-colored crusts. Laboratory studies show: At birth Day 30 Hemoglobin 18.0 g/dL 15.1 g/dL Leukocyte count 7,600/mm3 6,830/mm3 Segmented neutrophils 2% 3% Eosinophils 13% 10% Lymphocytes 60% 63% Monocytes 25% 24% Platelet count 220,000/mm3 223,000/mm3 Which of the following is the most likely diagnosis?

Infants with severe congenital neutropenia are prone to develop recurrent bacterial infections, including otitis media, respiratory infections, or oral and skin infections secondary to staphylococcal and streptococcal pathogens (such as impetigo). Laboratory findings reveal absolute neutropenia (often as low as < 200/μL) and relative monocytosis, as seen in this patient.

A 10-year-old boy is brought to the physician by his father, who is concerned because his son has been less interested in playing soccer with him recently. They used to play every weekend, but his son has started to tire easily and has complained of pain in the lower legs while running around on the soccer field. The boy has no personal or family history of serious illness. Cardiac examination shows a systolic ejection murmur best heard over the left sternal border that radiates to the left paravertebral region. An x-ray of the chest shows erosions of the posterior aspects of the 6th to 8th ribs. If left untreated, this patient is at greatest risk for which of the following?

Intermittent leg pain on exertion suggests inadequate perfusion of the lower extremities (intermittent claudication). The systolic ejection murmur described is likely due to aortic narrowing, and the erosions of the ribs suggest collateral circulation through the intercostal arteries (rib notching). Together, these findings indicate aortic coarctation. Erosion A morphologic descriptor used to describe several types of lesions. In dermatology, refers to a loss of a portion of or all of the epidermis (with an intact basement membrane). In gastroenterology, refers to a superficial mucosal lesion with intact muscularis mucosae. In dentistry, refers to a loss of tooth enamel. Rib notching A radiographic sign where internal thoracic and intercostal arteries dilate and develop to collaterals in patients with aortic narrowing (e.g., coarctation of the aorta). The increased perfusion leads to pressure atrophy and resorption of the neighboring ribs, which may visualize in chest x-rays as inferior rib notching. Claudication A cramping pain induced by exercise. Caused by arterial insufficiency and subsequent ischemia, typically in the lower legs. Aortic valve stenosis — or aortic stenosis — occurs when the heart's aortic valve narrows. This narrowing prevents the valve from opening fully, which reduces or blocks blood flow from your heart into the main artery to your body (aorta) and onward to the rest of your body Aortic stenosis is most commonly caused by age-related progressive calcification (>50% of cases), with a mean age of 65 to 70 years. Another major cause of aortic stenosis is the calcification of a congenital bicuspid aortic valve or, more rarely, congenital unicuspid aortic valve. Those with unicuspid aortic valve typically need intervention when very young, often as a newborn. While those with congenital bicuspid aortic valve make up 30-40% of those presenting during adulthood and [15] typically presenting earlier (ages 40+ to 50+) than those with tricuspid aortic valves (65+) Intracranial hemorrhage 45% Patients with untreated aortic coarctation usually develop secondary hypertension, resulting in an increased risk of intracranial hemorrhage. Moreover, aortic coarctation is associated with intracranial aneurysm formation (due to hypertension and/or genetic risk factors), which increases the risk of hemorrhage due to aneurysm rupture. Other complications of aortic coarctation include aortic dissection, coronary artery disease, ischemic stroke, endocarditis, and heart failure. Coarctation of the aorta A congenital heart defect that involves the narrowing of the aorta at the aortic isthmus. Frequently associated with other congenital heart defects (e.g., bicuspid aortic valve, VSD and/or PDA) and Turner syndrome

A 75-year-old man comes to the physician because of a 2-month history of intermittent bright red blood in his stool, progressive fatigue, and a 5-kg (11-lb) weight loss. He appears thin and fatigued. Physical examination shows conjunctival pallor. Hemoglobin concentration is 7.5 g/dL and MCV is 77 μm3. Results of fecal occult blood testing are positive. A colonoscopy shows a large, friable mass in the anal canal proximal to the pectinate line. Primary metastasis to which of the following lymph nodes is most likely in this patient?

Internal iliac 51% The internal iliac lymph nodes receive lymphatic drainage from the upper part of the anal canal, proximal to the pectinate line. The internal iliac lymph nodes would therefore be the most likely site of lymphatic metastasis in this patient with a mass in the proximal anal canal.

A 68-year-old man is brought to the emergency department because of right-sided weakness for 2 hours. He has hypertension, dyslipidemia, and type 2 diabetes. Current medications include hydrochlorothiazide, metoprolol, amlodipine, pravastatin, and metformin. His pulse is 87/min and blood pressure is 164/98 mm Hg. Neurological examination shows right-sided weakness, facial droop, and hyperreflexia. Sensation is intact. Which of the following is the most likely cause of these findings?

Lipohyalinosis of penetrating vessels 58% Lipohyalinotic thickening of penetrating cerebral vessels (e.g., lenticulostriate arteries) is the most likely cause of this patient's pure motor lacunar infarct; obliteration of the lumina of the poorly-collateralized lenticulostriate arteries leads to the infarction of the genu and anterior ⅔ of the posterior limb of the internal capsule, where motor fibers are densely packed. Chronic, uncontrolled hypertension, diabetes mellitus, smoking, and hyperlipidemia are risk factors for lipohyalinosis. Lacunar infarcts Lacunar stroke A subcortical stroke that is primarily associated with hypertension and diabetes mellitus. Lipohyalinotic thickening of vessel walls results in the occlusion of small, penetrating arteries that supply the subcortical regions of the brain (internal capsule, pons, thalamus, putamen, and caudate). Typically presents as specific lacunar syndromes that are characterized by the absence of cortical signs (e.g., aphasia, hemianopsia, agnosia, apraxia).

A 42-year-old man comes to the physician for a follow-up examination. His blood pressure was 146/91 mm Hg at his appointment 1 month ago; subsequent home blood pressure measurements have ranged from 135/83 mm Hg to 156/96 mm Hg. His blood pressure today is 141/85 mm Hg. Physical examination shows no abnormalities. Pharmacotherapy with lisinopril is initiated. Administration of this drug is most likely to result in decreased activity of which of the following sections of a normal adrenal gland?

Lisinopril is an angiotensin-converting enzyme (ACE) inhibitor that blocks the conversion of angiotensin I to angiotensin II. Angiotensin II is the primary regulator of aldosterone production in the adrenal cortex. The zona glomerulosa of the adrenal cortex is the outermost layer of adrenal parenchyma and lies directly beneath the adrenal capsule (A). The cells of the zona glomerulosa are stimulated by angiotensin II to produce mineralocorticoids, most importantly aldosterone. Inhibition of ACE by lisinopril leads to decreased availability of angiotensin II, in turn resulting in decreased activity of the zona glomerulosa. The response of the zona glomerulosa to angiotensin II is attenuated by low serum potassium or high serum sodium levels.

A 6-day-old female infant is brought to the emergency department because of poor feeding and irritability for two days. She was born at 39 weeks' gestation, and the pregnancy and delivery were uncomplicated. Her temperature is 39.2°C (102.6°F). She appears lethargic and makes occasional twitching movements in both upper extremities. The anterior fontanelle is soft and full. A lumbar puncture is performed and analysis of the cerebrospinal fluid shows increased protein and decreased glucose. Cerebrospinal fluid culture shows gram-positive, intracellular rods with tumbling motility. Infection of a healthy adult with the pathogen affecting this infant would most likely present with which of the following clinical conditions

Listeria monocytogenes most commonly causes self-limited gastroenteritis in healthy adults. It classically is accompanied by a history of dairy product or deli meat consumption (it grows well at refrigeration temperatures). Although typically a self-limiting condition in healthy adults, Listeria infection is more concerning in infants, the elderly, and those with compromised immune systems. These high-risk individuals should be treated with ampicillin.

A 61-year-old woman comes to the physician for a follow-up examination 1 week after undergoing right-sided radical mastectomy and axillary lymph node dissection for breast cancer. She says that she has been unable to comb her hair with her right hand since the surgery. Physical examination shows shoulder asymmetry. She is unable to abduct her right arm above 90 degrees. When she pushes against a wall, there is protrusion of the medial aspect of the right scapula. Injury to which of the following nerves is the most likely cause of this patient's condition?

Long thoracic nerve 81% This patient has evidence of a long thoracic nerve injury, which is a common complication of axillary lymph node dissection. The long thoracic nerve innervates the serratus anterior muscle; injury to the nerve typically results in medial winging of the scapula and impaired abduction of the arm beyond 90°.

parietal cells

Parietal cells (also known as oxyntic cells) are epithelial cells in the stomach that secrete hydrochloric acid (HCl) and intrinsic factor. These cells are located in the gastric glands found in the lining of the fundus and cardia regions of the stomach.

A 25-year-old man is brought to the emergency department 30 minutes after he was involved in a motorcycle collision. Physical examination shows a deep laceration on the volar surface of the distal left forearm. Neurological examination shows loss of abduction and opposition of the left thumb. The radial and ulnar pulses are palpable. Based on these findings, which of the following nerves is most likely injured in this patient?

Median nerve 77% Laceration of the median nerve at the wrist causes loss of thumb abduction and opposition, as seen in this patient. Additional deficits include reduced or absent sensation in the thumb, index, middle, and radial-sided ring fingers. The laceration could be through the median nerve proper or through the recurrent branch of the median nerve. The presence of sensory disturbances would be the determining factor, as the recurrent branch solely provides motor innervation. If this same nerve were injured more proximally (e.g., at or above the elbow), examination would show the hand of benediction sign.

A 62-year-old woman is brought to the emergency department because of the sudden onset of severe left eye pain, blurred vision, nausea, and vomiting. She has had an upper respiratory tract infection for the past 2 days and has been taking phenylephrine to control her symptoms. Examination shows a rock-hard, injected left globe and a fixed, mid-dilated pupil on the left. Gonioscopy shows that the iris meets the cornea at an angle of 10° (N = 20-45°). Systemic pharmacotherapy is initiated with a drug that decreases the secretion of aqueous humor from the ciliary epithelium. The next day, her serum potassium concentration is slightly decreased. Which of the following additional adverse effects is most likely to occur in this patient?

Metabolic acidosis 33% Acetazolamide, a carbonic anhydrase inhibitor, improves the symptoms of glaucoma by decreasing aqueous humor synthesis by the ciliary epithelium. It is a first-line treatment for acute primary angle-closure glaucoma, which can be precipitated by mydriasis due to darkness, fear, stress, or certain medications (e.g., phenylephrine). However, acetazolamide can impair bicarbonate reabsorption in the proximal tubule, which may cause non-anion gap metabolic acidosis as a result of type 2 renal tubular acidosis. Other important adverse effects of acetazolamide include hypokalemia, sulfa allergy, and hyperammonemia with paresthesias.

A 54-year-old man comes to the physician for a follow-up examination after presenting with elevated blood pressures on both arms at a routine visit 1 month ago. He feels well and takes no medications. He is 178 cm (5 ft 10 in) tall and weighs 99 kg (218 lb); BMI is 31 kg/m2. His pulse is 76/min, and blood pressure is 148/85 mm Hg on the right arm and 152/87 mm Hg on the left arm. Physical examination and laboratory studies show no abnormalities. The physician recommends lifestyle modifications in combination with treatment with hydrochlorothiazide. From which of the following embryological tissues does the site of action of this drug arise?

Metanephric blastema 64% The metanephric blastema is part of the metanephros, the last of the three embryonic kidneys, which arises during the 5th week of embryonic development. Induced by the ureteric bud, the metanephric blastema differentiates into the glomerulus, proximal convoluted tubule, loop of Henle, and the distal convoluted tubule, which is the site of action of hydrochlorothiazide.

A 21-year-old man is admitted to the intensive care unit for respiratory failure requiring mechanical ventilation. His minute ventilation is calculated to be 7.0 L/min, and his alveolar ventilation is calculated to be 5.1 L/min. Which of the following is most likely to decrease the difference between minute ventilation and alveolar ventilation?

Minute ventilation is the volume of air that a person breathes per minute. Alveolar ventilation is the volume of air that reaches the alveoli per minute. Decreasing the physiologic dead space 73% The physiologic dead space consists of the volume of air in the conducting airways (i.e., anatomic dead space) and the sum of the volumes of alveoli that do not participate in gas exchange (i.e., alveolar dead space). Decreasing the physiologic dead space reduces the difference between minute ventilation (tidal volume x respiratory rate) and alveolar ventilation ([tidal volume - physiologic dead space volume] x respiratory rate).

An investigator is studying brachial artery reactivity in women with suspected coronary heart disease. The brachial artery diameter is measured via ultrasound before and after intra-arterial injection of acetylcholine. An increase of 7% in the vascular diameter is noted. The release of which of the following is most likely responsible for the observed effect?

Nitric oxide from endothelial cells 87% The release of nitric oxide (an endothelial-derived relaxation factor) from endothelial cells is caused by acetylcholine and is responsible for the vasodilatory effect seen here. The binding of acetylcholine to its receptors on endothelial cells results in the activation of nitric oxide synthase, which catalyzes the production of nitric oxide from L-arginine. Nitric oxide, in turn, activates guanylyl cyclase, which increases cGMP synthesis. The end result of this cascade is arterial smooth muscle relaxation and subsequent dilation of the vessel.

A 69-year-old man undergoes modified radical neck dissection for an oropharyngeal tumor. During the procedure, he requires multiple blood transfusions. Four hours after the surgery, examination shows that the right and left pupils do not constrict when a light is shone into the left eye. When light is shone into the right eye, both pupils constrict. Fundoscopic examination shows no abnormalities. Which of the following is the most likely location of the lesion?

Optic nerve A relative afferent pupillary defect (RAPD) typically occurs because of unilateral damage to the retina and/or optic nerve (CN II). In this patient damage to the optic nerve (CN II) due to ischemic optic neuropathy has resulted in RAPD. Complete damage to CN II would also result in loss of vision on the affected side. The patient presents with a relative afferent pupillary defect (RAPD). Given his need for multiple blood transfusions during surgery, the patient presumably had an episode of intraoperative blood loss. This blood loss could have resulted in hypotension and subsequent ischemic damage to a CNS structure involved in the pupillary response.

A 3-day-old boy is brought to the emergency department because of a 4-hour history of somnolence, poor feeding, and one episode of vomiting. He is exclusively breastfed. His serum glucose concentration is 88 mg/dL and his serum ammonia concentration is 850 μmol/L (N<90). Urinalysis shows an increased orotic acid to creatinine ratio. Urinary ketones are absent. This patient is most likely deficient in an enzyme that catalyzes which of the following reactions?

Orotic acid is an intermediary in the pyrimidine biosynthetic pathway, which is connected to the urea cycle via a common intermediate. Ornithine and carbamoyl phosphate to citrulline 68% Production of citrulline from ornithine and carbamoyl phosphate is a reaction in the urea cycle catalyzed by ornithine transcarbamylase (OTC). Deficiency of this enzyme is the most common urea cycle disorder. It causes a buildup of ammonia and carbamoyl phosphate, which is converted to orotic acid. This patient's orotic aciduria and hyperammonemia make OTC deficiency the most likely diagnosis. Affected individuals usually present in infancy or early childhood and typically become symptomatic after beginning to nurse or bottle feed. Both breast milk and formula contain amino acids that would normally be broken down in the urea cycle.

A 55-year-old woman comes to the physician because of a 4-month history of a painless lump on her neck. Examination shows a hard nodule on the left side of her neck. A fine-needle aspiration biopsy shows well-differentiated cuboidal cells arranged spherically around colloid. She undergoes thyroidectomy. Histopathological examination of the surgical specimen shows invasion of the thyroid capsule and blood vessels. Which of the following cellular events is most likely involved in the pathogenesis of this patient's condition

PAX8-PPAR gamma gene rearrangement 36% PAX8-PPAR gamma gene rearrangement is involved in the pathogenesis of follicular thyroid adenoma and follicular thyroid cancer, the second most common type of thyroid cancer. Unlike benign thyroid adenomas, follicular thyroid cancer is more likely to cause early invasion of the thyroid capsule and vasculature, making it the most likely diagnosis here.

A 78-year-old man comes to the physician because of difficulty swallowing and regurgitation of undigested food for the past several months. The patient reports that his symptoms have gotten progressively worse. Physical examination shows severe halitosis and a gurgling sound after he swallows a small amount of water. Results of fluoroscopic barium esophagography are shown. The nerve responsible for motor innervation of the affected muscle is also responsible for which of the following functions?

Parasympathetic innervation of the AV node 61% The vagus nerve innervates the inferior pharyngeal constrictor, which is the muscle affected in Zenker diverticulum. Parasympathetic fibers that innervate both the SA node (right vagus nerve) and the AV node (left vagus nerve) also originate from the vagus nerve. Additional functions of the vagus nerve include initiation of the cough reflex, innervation of taste receptors of the supraglottis, and innervation of chemo and baroreceptors in the aortic arch.

Parinuad's Syndrome

Parinaud's syndrome is an inability to move the eyes up and down. It is caused by compression of the vertical gaze center at the rostral interstitial nucleus of medial longitudinal fasciculus (riMLF). The eyes lose the ability to move upward and down. It is a group of abnormalities of eye movement and pupil dysfunction. It is caused by lesions of the upper brain stem and is named for Henri Parinaud[6][7] (1844-1905), considered to be the father of French ophthalmology. Parinaud's syndrome results from injury, either direct or compressive, to the dorsal midbrain.

A 2-month-old boy is brought to the pediatrician for a routine check-up. His mother says he is feeding well and has no concerns. He is at the 85th percentile for height and 82nd percentile for weight. Immunizations are up-to-date. Results of serum hepatitis B surface IgG antibody testing are positive. Which of the following best explains this patient's hepatitis B virus status?

Passive immunity This infant has positive hepatitis B surface IgG antibodies (anti-HBs) acquired via passive immunity from the mother. Since the mother produced anti-HBs, she must have a history of HBV infection or vaccination. IgG is the most common type of antibody and can indicate prior infection or immunization. Spontaneous recovery occurs in approximately 95% of acute HBV infections in adults. However, up to 90% of pediatric infections and vertically transmitted infections progress to chronic HBV. The presence of anti-HBs is not specific for active HBV infection and this patient is too young to have already had active infection with spontaneous recovery. A different scenario is more likely to explain the presence of anti-HBs IgG in this patient. Window period The window period in HBV infection is defined as the time when anti-HBs is still undetectable despite active HBV infection (serology may be positive for anti-HBc IgM, anti-HBe). This patient's positive anti-HBs is not consistent with the window period.

purpuric

Pertaining to bruising of the skin.

A 38-year-old woman comes to the physician because of a 4-month history of crampy abdominal pain, recurrent watery diarrhea, and a 2.5-kg (5.5-lb) weight loss. Her husband has noticed that after meals, her face and neck sometimes become red, and she develops shortness of breath and starts wheezing. Examination shows a grade 3/6 systolic murmur heard best at the left lower sternal border. The abdomen is soft, and there is mild tenderness to palpation with no guarding or rebound. Without treatment, this patient is at greatest risk of developing which of the following conditions?

Pigmented dermatitis 30% Dermatitis is a characteristic feature of pellagra, or niacin deficiency; other key findings include dementia and diarrhea (i.e., the "three Ds"). Dermatitis in patients with pellagra is characterized by a symmetrical, pigmented, scaly thickening of the skin in sun-exposed areas, such as the limbs and around the neck (casal necklace). The increased metabolism of serotonin in carcinoid tumors depletes the amount of tryptophan precursors available for other organic compounds, including niacin. A lack of niacin leads to decreased production of the coenzymes NAD+ and NADP+, which are essential in redox reactions throughout cellular metabolism. Diagnosis is confirmed via 24-hour urine testing for 5-hydroxyindoleacetic acid (5-HIAA), the end product of serotonin metabolism. Watery diarrhea, crampy abdominal pain, episodic cutaneous flushing triggered by eating or emotional events, asthma-like attacks, and a murmur indicative of tricuspid regurgitation are highly suggestive of carcinoid syndrome. The neuroendocrine tumor is highly metabolically active and secretes excess amounts of serotonin.

A 13-year-old boy is brought to the physician because of a 4-month history of worsening dizziness, nausea, and feeling clumsy. An MRI of the brain shows a well-demarcated, 4-cm cystic mass in the posterior fossa. The patient undergoes complete surgical resection of the mass. Pathologic examination of the surgical specimen shows parallel bundles of cells with eosinophilic, corkscrew-like processes. Which of the following is the most likely diagnosis?

Pilocytic astrocytoma 75% Pilocytic astrocytoma is the most common pediatric primary brain tumor. It typically occurs in the cerebellum (posterior cranial fossa), but can also occur in supratentorial regions such as the optic nerve (optic glioma) and the cerebral hemispheres. Tumors in the posterior cranial fossa can present with dizziness, nausea, and clumsiness due to obstructive hydrocephalus and/or cerebellar damage. On imaging, a pilocytic astrocytoma typically appears as a well-demarcated cystic mass with a brightly enhancing nodule in the wall of the cyst.

A 37-year-old woman, gravida 4, para 3, at 35 weeks' gestation is admitted to the hospital in active labor. Her three children were delivered by Cesarean section. One hour after vaginal delivery, the placenta is not delivered. Manual separation of the placenta leads to profuse vaginal bleeding. Her pulse is 122/min and blood pressure is 90/67 mm Hg. A firm, nontender uterine fundus is palpated at the level of the umbilicus. Hemoglobin is 8.3 g/dL and platelet count is 220,000/mm3. Activated partial thromboplastin time and prothrombin time are within normal limits. Which of the following is the most likely underlying mechanism of this patient's postpartum bleeding?

Placenta accreta occurs when the placenta is directly adherent to the myometrium instead of the decidua basalis but does not invade or penetrate the myometrium (i.e., placenta increta). Although the exact etiology is unknown, previous C-sections are the most important risk factor. Clinically, patients present with delayed placental detachment and massive, life-threatening postpartum hemorrhage at the time of attempted manual separation of the placenta, as seen in this patient.

A 15-year-old girl comes to the physician because of a 2-month history of progressive fatigue and weakness. She also reports recurrent headaches for 2 years, which have increased in severity and frequency. Her blood pressure is 185/95 mm Hg. Serum studies show a morning renin activity of 130 ng/mL per hour (N=1-4), a morning aldosterone concentration of 60 ng/dL (N=5-30), and a potassium concentration of 2.9 mEq/L. Further evaluation is most likely to show which of the following?

Pleomorphic modified smooth muscle cells in the renal cortex 55% The presence of pleomorphic smooth muscle cells in the renal cortex in combination with elevated renin and aldosterone concentrations is highly indicative of a juxtaglomerular tumor. Excessive renin production by these tumor cells stimulates aldosterone release. Consequently, patients exhibit symptoms of secondary hyperaldosteronism. The findings of hypertension and elevated morning renin and aldosterone concentrations, in a patient presenting with fatigue, weakness, and headache, is diagnostic of secondary hyperaldosteronism. Secondary hyperaldosteronism A condition characterized by increased serum aldosterone concentrations secondary to increased activity of renin. Manifestations include hypertension and symptoms of hypokalemia (headaches, fatigue, muscle weakness).

A 2-year-old girl who emigrated from Pakistan 2 weeks ago is brought to the emergency department because of lower limb weakness for one-day. One week ago, she had a 3-day episode of flu-like symptoms that resolved without treatment. She has not yet received any routine childhood vaccinations. Deep tendon reflexes are 1+ in the right lower extremity and absent in the left lower extremity. Analysis of cerebrospinal fluid shows a leukocyte count of 38 cells/mm3 (68% lymphocytes), a protein concentration of 49 mg/dL, and a glucose concentration of 60 mg/dL. Which of the following is the most likely diagnosis in this patient?

Poliomyelitis 66% Poliomyelitis, which is caused by the poliovirus, manifests as acute lower motor neuron weakness that begins asymmetrically in the lower extremities and ascends over hours to days. Patients can have pro-dromal flu-like symptoms before the onset of weakness. CSF analysis in poliomyelitis is typically consistent with viral infection (lymphocytic pleocytosis, normal or slightly increased protein levels, and normal glucose levels), as seen here. Diaphragmatic involvement ultimately results in respiratory failure and death. In this unvaccinated migrant child, the most likely cause of acute flaccid paralysis (weakness with decreased deep tendon reflexes) is anterior horn cell destruction.

An asymptomatic 35-year-old woman comes to the physician for HIV testing after her husband tested positive for HIV infection 2 weeks ago. She has no history of serious illness and her only medication is an oral contraceptive. Serum studies show antibodies against HIV-1. Therapy with elvitegravir, emtricitabine, and tenofovir is initiated. The physician also prescribes a new drug, cobicistat, which does not have antiretroviral activity. The patient is advised to use barrier contraception instead of oral contraceptive therapy while she is receiving cobicistat because this drug inhibits the hepatic metabolism of oral contraceptives and thereby increases the risk of vascular complications. Which of the following is the most likely reason for prescribing cobicistat?

Potentiation describes an interaction between two drugs in which a drug that has no therapeutic action by itself enhances the therapeutic action of the other drug. In this patient, cobicistat, which has no antiretroviral therapeutic action, inhibits CYP3A4 and, therefore, decreases the breakdown of the antiretroviral drugs. Consequently, the therapeutic substance remains in the body longer and therapeutic levels can be maintained with lower doses. Maintaining steady drug levels decreases the risk of developing viral drug resistance. Ritonavir, a protease inhibitor that also acts as a CYP450 inhibitor, is another drug classically used for pharmacokinetic enhancement in HIV therapy. Other examples of potentiation are the coadministration of decarboxylase inhibitor carbidopa and levodopa (in Parkinson disease) or β-lactamase inhibitor clavulanate and amoxicillin.

A healthy 31-year-old woman comes to the physician because she is trying to conceive. She is currently timing the frequency of intercourse with at-home ovulation test kits. An increase in the levels of which of the following is the best indicator that ovulation has occurred?

Progesterone 47% Towards the end of the follicular phase of the menstrual cycle, a peak in estrogen levels triggers an LH surge, which induces ovulation. The corpus luteum then begins to produce progesterone. An increase in progesterone levels is therefore an indicator that ovulation has occurred. Corpus luteum A yellow structure within the ovary that is formed from the ruptured Graafian follicle after ovulation. The corpus luteum, which secretes progesterone and inhibin A, degenerates 8-12 days after ovulation in non-pregnant women. If fertilization occurs, the corpus luteum persists for at least 2 more months due to the effect of placental human chorionic gonadotropin. Hemorrhage within the corpus luteum can result in a luteal cyst.

A 44-year-old male is brought to the physician by his father. Over the past year, the patient has become progressively forgetful and withdrawn. He frequently has trouble remembering names of acquaintances, and has been requiring increasing amounts of assistance with getting dressed, cooking, and personal hygiene. He was diagnosed with a genetic disorder during infancy. Physical examination shows prominent epicanthal folds, low-set small ears, and a protruding tongue. Mental examination shows significant deficits in short- and long-term memory. This patient's cognitive symptoms are most likely the result of which of the following neuropathologic changes?

Prominent epicanthal folds, low-set small ears, and a protruding tongue are signs of trisomy 21. This patient's presentation of cognitive and memory impairments are consistent with a condition that manifests early in patients with this genetic disorder. Extracellular accumulation of amyloid plaques 80% Accumulation of extracellular amyloid plaques and intracellular neurofibrillary tangles are observed in Alzheimer disease (AD). A majority of patients with trisomy 21 (Down syndrome) develop early-onset AD since an extra copy of the amyloid precursor protein (APP) gene (located on chromosome 21) leads to higher absolute levels of APP expression and deposition of the encoded amyloid-beta peptide in the brain as well as the cerebral vasculature. Mutations in the APP gene are associated with early-onset familial AD.

A 29-year-old woman comes to the physician for intermittent episodes of sharp chest pain and palpitations. She appears nervous. Her pulse is 115/min and irregularly irregular, and blood pressure is 139/86 mmHg. Examination shows a fine tremor on both hands and digital swelling; the extremities are warm. There is retraction of the right upper eyelid. Which of the following is the most appropriate next step in the management of this patient?

Propranolol 65% Propranolol is indicated for the immediate control of symptoms in patients with hyperthyroidism. This patient with anxiety, tremor, warm skin, digital swelling, retraction of the eyelid, and irregular heartbeats most likely has hyperthyroidism. Thyroid hormones affect cardiac myocytes directly and also stimulate proliferation of beta-adrenergic receptors, which can cause cardiac arrhythmias as seen in this patient. Beta blockers decrease the adrenergic tone and therefore improve tachycardia, hypertension, muscle weakness, tremor, and neuropsychiatric symptoms. Propranolol, in high doses, also decreases the peripheral conversion of T4 to T3 by inhibiting the 5'-monodeiodinase enzyme. The condition underlying this patient's arrhythmia causes a state of adrenergic hyperactivity.

A 58-year-old man with chronic obstructive pulmonary disease and hypertension comes to the physician because of shortness of breath 3 days after starting propranolol. His temperature is 36.7°C (98.1°F), pulse is 64/min, respirations are 20/min, and blood pressure is 138/88 mm Hg. Auscultation of the lungs shows diffuse expiratory wheezes. In addition to discontinuing the propranolol, which of the following drugs should be administered?

Propranolol is a non-selective beta blocker. Beta-2 blockade can trigger bronchoconstriction, particularly in patients with COPD or asthma. Albuterol 59% Albuterol is a short-acting beta-2 adrenergic agonist that induces relaxation of bronchial smooth muscle cells. A patient presenting with signs of acute exacerbation of COPD (e.g., shortness of breath, tachypnea, and expiratory wheezes) shortly after starting propranolol, most likely has drug-induced bronchospasm. Albuterol is typically used as a first-line bronchodilator in patients with acute exacerbation of COPD or asthma and would counteract the effect of propranolol seen in this patient.

A 27-year-old man comes to the physician because of a 4-month history of recurrent episodes of headaches, palpitations, and sweating. He was adopted shortly after birth and does not know his biological parents. His pulse is 103/min and blood pressure is 160/105 mm Hg. Physical examination shows multiple soft, yellow papules on the tip of the tongue. There is a 2-cm, firm thyroid nodule. He has long and slender upper extremities, and his elbows and knees can be hyperextended. The most likely cause of this patient's condition is a mutation in which of the following genes?

RET 68% Multiple endocrine neoplasia type 2, which includes MEN 2A and MEN 2B, results from a gain-of-function mutation of the RET proto-oncogene. RET encodes a receptor tyrosine kinase that plays a key role in cell growth, differentiation, and survival. Medullary thyroid cancer (which develops in almost all patients) and pheochromocytoma are associated with both MEN 2A and MEN 2B. MEN 2A is associated with primary hyperparathyroidism, and MEN 2B is associated with marfanoid body habitus and mucosal neuromas. Marfanoid habitus A collection of clinical features including tall stature, long limbs (with an increased limb length to body height ratio), overcrowded dentition with a high-arched palate, arachnodactyly (long, slender fingers), and joint hypermobility. Marfanoid habitus can be seen in Ehlers-Danlos syndrome, multiple endocrine neoplasia type 2b, Marfan syndrome, and homocystinuria.

A 47-year-old woman comes to the physician because of a 2-month history of a lump on her neck and a 1-week history of hoarseness. Examination shows a 3-cm, firm, nontender nodule on the anterior neck. Further evaluation confirms a thyroid malignancy, and she undergoes thyroidectomy. Histopathologic examination of the surgical specimen shows lymphatic invasion. Genetic analysis shows RET/PTC chromosomal rearrangement. Microscopic examination of the surgical specimen is most likely to also show which of the following?

RET/PTC rearrangement is often seen in papillary thyroid cancer and results in the production of a constitutively activated tyrosine kinase. RET/PTC rearrangement is often seen in papillary thyroid cancer and results in the production of a constitutively activated tyrosine kinase. Papillary thyroid carcinoma Papillary thyroid cancer A well-differentiated malignant neoplasia of the thyroid gland and the most common type of thyroid cancer, characterized by early lymphatic metastasis. Tyrosine kinase A group of > 90 enzymes that regulate cell signaling, growth, proliferation, differentiation, and adhesion by adding high energy phosphate groups to tyrosine residues of proteins. Several tyrosine kinases are proto-oncogenes (e.g., ALK, JAK2, RET). Calcified spherules and large oval cells with empty-appearing nuclei 39% Calcified spherules (psammoma bodies) and large oval cells with empty-appearing nuclei (Orphan Annie eyes nuclei) would most likely be seen on histopathology of thyroid tissue from this patient. These features, along with nuclear grooves, are characteristic of papillary thyroid cancer, which accounts for ∼ 80% of all thyroid cancers and is typically a well-differentiated, slow-growing tumor. In contrast to follicular thyroid cancer, which spreads hematogenously, papillary thyroid cancers spread by early lymphatic invasion. Psammoma body A group of concentric, intracellular calcium collections seen in certain neoplasms such as papillary carcinoma of thyroid, papillary serous endometrial carcinoma, mesothelioma, meningioma, and serous papillary cystadenocarcinoma of ovary and endometrium. Can also appear in chronically inflamed tissue.

A 24-year-old man comes to the physician because his vision has worsened rapidly over the last 2 months. His maternal uncle lost his vision suddenly over a period of 3 months at 26 years of age. The patient's wife and 1-year-old son have normal vision. Funduscopic examination of the patient shows bilateral circumpapillary telangiectasia. Genetic testing shows a missense mutation in one of the genes of the electron transport chain complexes. The probability that this patient's son will be affected by the same disease is closest to which of the following?

Rapidly worsening vision, circumpapillary telangiectasia on fundoscopy, and a mutation affecting the mitochondrial electron transport chain are diagnostic of Leber hereditary optic neuropathy. Leber hereditary optic neuropathy Leber optic atrophy Abbreviation: LHON An inherited form of optic neuropathy passed from mother to child by mitochondrial DNA (mitochondrial inheritance). Typically presents as subacute visual changes in one or both eyes, usually in young adults. Funduscopic exam typically shows circumpapillary telangectasia. 63% Leber hereditary optic neuropathy typically presents with acute monocular vision loss in a young adult followed by contralateral vision loss in the following weeks or months. A family history of similar ocular disease is common. The condition is passed exclusively from mother to child (as are all diseases inherited via mitochondrial DNA) since mitochondria from sperm cells are not transmitted during fertilization. Offspring of affected males, therefore, are at the same risk as the general population (about 1 in 50,000). Other conditions that demonstrate mitochondrial inheritance include mitochondrial myopathies (e.g., MERRF or MELAS syndrome). Mitochondrial inheritance Mitochondrial inheritance implies transmission of the disease only through the mother. All the offsprings of affected females may express the phenotype. Variable expression is observed due to heteroplasmy.

A 1-year-old girl is brought to the physician for follow-up examination 1 week after admission to the hospital for bacterial pneumonia. She has had multiple episodes of purulent otitis media and infectious diarrhea since the age of 6 months. She underwent treatment for oral thrush 1 month ago. There is no family history of serious illness. Her height and weight are both below the 10th percentile. Physical examination shows no visible tonsils and slightly decreased breath sounds in the left lower lobe. Laboratory studies show increased deoxyadenosine concentration in both the serum and urine. An x-ray of the chest shows an absent thymic shadow. Which of the following additional findings is most likely in this patient?

Recurrent infections (bacterial and fungal), failure to thrive, absent tonsils, and absent thymus shadow on x-ray suggest severe combined immunodeficiency (SCID). Decreased circulating T cells This patient's severe combined immunodeficiency (SCID) is most likely due to an autosomal recessive adenosine deaminase deficiency (the second most common cause of SCID), which results in disrupted purine metabolism and accumulation of metabolites (including deoxyadenosine and dATP) that are toxic to maturing lymphocytes. Accumulation of dATP also inhibits the function of ribonucleotide reductase, which is essential for the generation of deoxynucleotides, thereby impairing the proliferation of lymphocytes. These effects on lymphocytes proliferation and maturation lead to a decreased number of circulating T cells, B cells, and NK cells, resulting in both cellular and humoral immunodeficiency. Clinically, patients with SCID have no tonsils, no palpable lymph nodes, and a rudimentary thymus.

rhadomyosarcoma

Rhabdomyosarcoma (RMS) is a type of sarcoma made up of cells that normally develop into skeletal (voluntary) muscles. These are muscles that we control to move parts of our body. Well before birth, cells called rhabdomyoblasts (which will eventually form skeletal muscles) begin to form. These are the cells that can develop into RMS. Because this is a cancer of very early forms of muscle cells, it is much more common in children, although it does sometimes occur in adults.

A 13-year-old boy is brought to the emergency department because of a 2-day history of fever, headache, and irritability. He shares a room with his 7-year-old brother, who does not have any symptoms. The patient appears weak and lethargic. His temperature is 39.1°C (102.4°F) and blood pressure is 99/60 mm Hg. Physical examination shows several purple spots over the trunk and extremities. A lumbar puncture is performed. Gram stain of the cerebrospinal fluid shows numerous gram-negative diplococci. Administration of which of the following is most likely to prevent infection of the patient's brother at this time? The constellation of high fever, headache, irritability, and CSF findings (gram-negative diplococci) is diagnostic of meningococcal meningitis. Post-exposure chemoprophylaxis (PEP) is important to prevent infection and disease transmission.

Rifampin is a mainstay of prophylaxis against N. meningitidis for people in close contact with an infected individual. The patient's brother, as well as any other close contacts, should receive a 2-day course of this drug. In scenarios in which rifampin is contraindicated (e.g. hypersensitivity or concomitant HIV protease inhibitor administration) or unavailable, a single dose of intravenous ceftriaxone can be used for the same purpose. Oral ciprofloxacin, although effective as a prophylactic agent against N. meningitidis in adults, is contraindicated in the patient's 7-year-old brother.

An otherwise healthy 26-year-old man comes to the physician for medication counseling after recently being diagnosed with schizophrenia. Risperidone therapy is initiated. This patient is at increased risk for which of the following adverse effects?

Risperidone An atypical antipsychotic used to treat schizophrenia, bipolar disorder, aggression in children with autism disorder, and tics in people with Tourette syndrome. A specific adverse effect associated with risperidone is hyperprolactinemia, possibly leading to gynecomastia, lactation, irregular menses, and infertility. Second generation antipsychotics Atypical antipsychotics Abbreviation: SGAs A class of medications that act through postsynaptic blockade of dopamine D₂ receptors. They also act on 5-HT₂, α-adrenergic, and H₁ receptors. Commonly used to treat psychosis but also used as chemical sedatives. Less likely to cause extrapyramidal adverse effects than typical antipsychotics, but more likely to cause metabolic disturbances (e.g., weight gain, hyperglycemia). Examples include olanzapine, risperidone, quetiapine, and aripiprazole. Risperidone is an atypical antipsychotic with high affinity for dopamine D2 receptors Gynecomastia 79% Risperidone acts as a D2 receptor antagonist, thereby disrupting the inhibitory effect of dopamine within the tuberoinfundibular pathway. The subsequent increase in prolactin secretion from the anterior pituitary gland causes hyperprolactinemia, which stimulates the proliferation of the mammary gland. Patients subsequently develop gynecomastia and galactorrhea. Hyperprolactinemia also inhibits the release of GnRH from the hypothalamus, causing hypogonadotropic hypogonadism; this manifests in males with sexual dysfunction (erectile dysfunction, infertility).

A 14-year-old girl is brought to the physician by her father because of fever, chills, abdominal pain, and profuse non-bloody diarrhea. Her symptoms began one week ago, when she had several days of low-grade fever and constipation. She returned from Indonesia 2 weeks ago, where she spent the summer with her grandparents. Her temperature is 39.3°C (102.8°F). Examination shows diffuse abdominal tenderness and mild hepatosplenomegaly. There is a faint salmon-colored maculopapular rash on her trunk and abdomen. Which of the following is the most likely causal organism?

Salmonella typhi 73% Salmonella typhi is most likely to have caused this patient's symptoms, who has recently traveled to an endemic region. Typhoid infection is characterized by a fluctuating fever and malaise followed by profound fatigue, high fever, and relative bradycardia. Initial constipation can be followed by profuse diarrhea (pea-soup diarrhea), a salmon-colored maculopapular rash (rose spots), and hepatosplenomegaly, usually in the second week after onset. Salmonella typhi is transmitted via the fecal-oral route, with the bacteria initially entering the host through intestinal Peyer patches. After a period of replication, the organism enters the bloodstream and can seed several organs (e.g., the liver and spleen).

A 34-year-old man comes to the physician for evaluation of a rash on the elbows for several months. A biopsy of the affected area shows a thinned stratum granulosum as well as retained nuclei and spongiotic clusters of neutrophils in the stratum corneum. This patient's skin findings are most likely associated with which of the following conditions?

Seronegative spondylarthropathy 56% Psoriatic arthritis is a seronegative spondyloarthropathy seen in up to ⅓ of patients with psoriasis. Apart from skin involvement, common findings in psoriatic arthritis include asymmetric arthritis, nail lesions, inflamed, sausage-like fingers (dactylitis), and pencil-in-cup deformity of the distal interphalangeal joints (DIPs) on x-ray. Psoriatic arthritis A seronegative inflammatory arthritis found in patients with psoriasis. In 60-80% of patients, psoriasis precedes the arthritis; in 15-20% of patients, arthritis appears before psoriasis. Other common symptoms include enthesitis particularly where the Achilles tendon attaches to the calcaneus, dactylitis, and nail changes Seronegative spondyloarthropathy Seronegative arthritides A group of chronic inflammatory arthritic diseases that are negative for rheumatoid factor (RF) and have a strong genetic association with HLA-B27. The most important diseases in this group are ankylosing spondylitis, reactive arthritis, and psoriatic arthritis. Men are disproportionately affected, with symptom onset generally occurring before the age of 45. The cardinal sign is slowly progressive pain in the lower back and sacroiliac joints (especially at night). Asymmetrical oligoarthritis and enthesopathy are also common.

A 49-year-old man comes to the physician because of severe, shooting pain in his lower back for the past 2 weeks. The pain radiates down the back of both legs and started after he lifted a concrete manhole cover from the ground. Physical examination shows decreased sensation to light touch bilaterally over the lateral thigh area and lateral calf bilaterally. Patellar reflex is decreased on both sides. The passive raising of either the right or left leg beyond 30 degrees triggers a shooting pain down the leg past the knee. Which of the following is the most likely underlying cause of this patient's current condition?

Severe lower back pain radiating down the legs together with decreased sensation to touch, decreased reflexes, and positive Lasègue sign (pain when raising legs passively) is suggestive of lumbar radiculopathy. The association between heavy lifting and the onset of symptoms suggests an acute, mechanical mechanism of injury. Herniation of nucleus pulposus into vertebral canal 91% The nucleus pulposus is the gelatinous core of the intervertebral disc and is surrounded by a dense outer ring, the annulus fibrosus. Compression, tension, and shear and torque stresses on the spinal disc (as with heavy lifting) together with degenerative changes (e.g., dehydration) can lead to a tear in the annulus fibrosus, resulting in herniation of the nucleus pulposus into the vertebral canal with subsequent compression of the spinal cord and/or nerve roots. This nerve impingement causes symptoms of radiculopathy, which differ depending on the level of herniation. Nucleus pulposus The gelatinous core of the vertebral disc. Derived from the notochord and allows the disc to withstand compression and torsion. Degeneration of the outer fibrous annulus of the vertebral disc can cause extrusion of the nucleus pulposus and compression of a spinal nerve root. Lumbar radiculopathy Lumbosacral radiculopathy Damage to one or more nerve roots from L1 to S1, causing pain in the lower back and hip that radiates down the back of the thigh into the leg. The pain may be described as tingling, numbing, and/or shooting and may be associated with paresthesias.

A 56-year-old male comes to the physician because of a 2-month history of excessive sleepiness. He reports that he has been sleeping for an average of 10 to 12 hours at night and needs to take multiple naps during the day. Six months ago, he was diagnosed with small cell lung carcinoma and underwent prophylactic cranial irradiation. This patient's symptoms are most likely caused by damage to which of the following structures?

Suprachiasmatic nucleus 83% The suprachiasmatic nucleus lies in the hypothalamus and plays an important role in the circadian rhythm pathway. It receives input (light) from the retina. When there is decreased input, the suprachiasmatic nucleus releases norepinephrine, which stimulates the release of sleep-inducing melatonin from the pineal gland. Damage to this pathway (e.g., due to metastasis or radiation) can cause hypersomnolence or sleep-wake disturbances. This structure is responsible for the central regulation of the circadian rhythm.

A 44-year-old woman with recurrent urinary tract infections is brought to the emergency department by ambulance after sudden onset of severe headache 30 minutes ago. She has a history of occasional, mild headaches in the morning. There is no other history of serious illness. Both her father and her paternal grandmother died of chronic kidney disease. Her temperature is 37.2°C (99.1°F) and blood pressure is 145/90 mm Hg. Physical examination shows neck stiffness. When her hip is flexed, she is unable to fully extend her knee because of pain. Lumbar puncture performed 12 hours after headache onset yields 10 mL of yellow-colored fluid with no leukocytes. Which of the following is the most likely predisposing factor for this patient's current condition?

Signs of arterial hypertension (e.g., morning headaches), frequent UTIs, and a family history of renal failure are concerning for autosomal dominant polycystic kidney disease (ADPKD), whereas a positive Kernig's sign indicates meningeal or nerve root irritation. Saccular aneurysm 74% Patients with ADPKD have an increased risk of developing saccular aneurysms. This patient presented with symptoms of a subarachnoid hemorrhage resulting from rupture of one of these aneurysms. The sudden onset of a severe headache ("worst headache of my life") should raise suspicion for SAH. Other signs of SAH include fever, xanthochromia (yellow-colored CSF), and meningismus (neck stiffness and Kernig's sign) caused by irritation of the meninges by blood. Bacterial infection 16% While this patient has meningismus, she does not have a fever or leukocytes on CSF suggestive of meningitis. She is also not in a high-risk group for contracting meningitis (unvaccinated, college age and living in close quarters, or immunocompromised). In addition, there is no association between polycystic kidney disease and meningitis.

An otherwise healthy 45-year-old man comes to the physician because of a painful ulcer on his tongue for 3 days. Examination shows a shallow, tender 5-mm wide ulcer on the lateral aspect of the tongue, adjacent to his left first molar. There is no induration surrounding the ulcer or cervical lymphadenopathy. A lesion of the cranial nerve responsible for the transmission of pain from this ulcer would most likely result in which of the following?

Since this patient's lesion is located opposite the first molar, it lies anterior to the sulcus terminalis, the structure separating the anterior two-thirds of the tongue from the posterior third. Somatic sensation (e.g., pain) from the anterior two-thirds of the tongue is transmitted by the lingual nerve, which is a branch of the mandibular division of the trigeminal nerve (CN V3). Difficulty chewing 42% A lesion of the mandibular branch of the trigeminal nerve (CN V3) results in difficulty chewing due to paralysis of the muscles of mastication (medial and lateral pterygoid, masseter, temporalis). CN V3 also innervates the mylohyoid muscle, anterior digastric belly, tensor veli palatini, and tensor tympani, and transmits sensory input from the lower third of the face and anterior wall of the external auditory canal. CN V3 is a part of both the afferent and efferent arms of the jaw jerk reflex.

An investigator is studying central nervous system neurotransmitters in mice. He injects the spinal cord with a protease that cleaves SNARE proteins, thereby blocking the release of neurotransmitters from the Renshaw cells in the spinal cord. Which of the following consequences would be expected?

Spastic paralysis 79% The mechanism of action of the substance described is consistent with tetanospasmin, which cleaves SNARE proteins (e.g., synaptobrevin, syntaxin) and thereby prevents the release of inhibitory neurotransmitters from Renshaw cells. Injection with protease would lead to uncontrolled activation of alpha motor neurons, causing spastic paralysis, trismus, risus sardonicus, and opisthotonus. Another neurotoxin that acts through the cleavage of SNARE proteins is botulinum toxin, which acts peripherally at the neuromuscular junctions and prevents the release of neurotransmitters into the synaptic cleft. The resulting clinical manifestation is a peripheral flaccid muscle paralysis that descends caudally. Some neurotoxins released by bacteria are proteolytic enzymes that cause a disruption of neuronal function. This particular neurotoxin also causes risus sardonicus and opisthotonus.

An investigator is studying the biology of human sperm cells. She isolates spermatogonia obtained on a testicular biopsy from a group of healthy male volunteers. She finds that the DNA of spermatogonia obtained from these men show a large number of TTAGGG sequence repeats. This finding can best be explained by increased activity of an enzyme with which of the following functions?

TTAGGG is the characteristic sequence of telomeres in eukaryotic cells RNA-dependent synthesis of DNA 46% RNA-dependent synthesis of DNA is a characteristic function of telomerase, a eukaryotic reverse transcriptase that carries its own RNA template from which complementary telomere DNA segments are synthesized at the 3′ ends of chromosomes. This function prevents loss of genetic material during DNA replication. High telomerase activity is normal in germ (including immature sperm cells) cells, stem cells, and embryonic cells. However, high telomerase activity is abnormal in somatic cells and can give rise to dysregulated cancer cell formation. Telomerase A eukaryotic reverse transcriptase that carries its own RNA template and adds DNA telomeres to 3′ ends of chromosomes. This function prevents loss of genetic material during DNA replication.

Thalssemia

Thalassemia (thal-uh-SEE-me-uh) is an inherited blood disorder that causes your body to have less hemoglobin than normal. Hemoglobin enables red blood cells to carry oxygen. Thalassemia can cause anemia, leaving you fatigued.

A 51-year-old man with alcohol use disorder comes to the physician because of a fever and productive cough. An x-ray of the chest shows a right lower lobe consolidation and a diagnosis of aspiration pneumonia is made. The physician prescribes a drug that blocks peptide transfer by binding to the 50S ribosomal subunit. Which of the following drugs was most likely prescribed?

The drug in question is a bacteriostatic antibiotic frequently used to cover anaerobic bacteria. Clindamycin 58% Clindamycin is a bacteriostatic antibiotic that inhibits bacterial protein synthesis by preventing peptide translocation at the 50S (large) ribosomal subunit. It targets anaerobic organisms and is commonly used for the treatment of aspiration pneumonia and lung abscess, which are typically polymicrobial. Clostridioides difficile colitis is a potential complication of clindamycin.

Romberg test

The Romberg test is a test of the body's sense of positioning (proprioception), which requires healthy functioning of the dorsal columns of the spinal cord.[1] The Romberg test is used to investigate the cause of loss of motor coordination (ataxia). A positive Romberg test suggests that the ataxia is sensory in nature, that is, depending on loss of proprioception. If a patient is ataxic and Romberg's test is not positive, it suggests that ataxia is cerebellar in nature, that is, depending on localized cerebellar dysfunction instead. Ask the subject to stand erect with feet together and eyes closed. Stand close by as a precaution in order to stop the person from falling over and hurting themself. Watch the movement of the body in relation to a perpendicular object behind the subject (corner of the room, door, window etc.). A positive sign is noted when a swaying, sometimes irregular swaying and even toppling over occurs. The essential feature is that the patient becomes more unsteady with eyes closed. The essential features of the test are as follows: the subject stands with feet together, eyes open and hands by the sides. the subject closes the eyes while the examiner observes for a full minute. Because the examiner is trying to elicit whether the patient falls when the eyes are closed, it is advisable to stand ready to catch the falling patient. For large subjects, a strong assistant is recommended. Romberg's test is positive if the patient falls while the eyes are closed. Swaying is not a positive sign as it shows proprioceptive correction.[4] Patients with a positive result are said to demonstrate Romberg's sign or Rombergism. They can also be described as Romberg's positive. The basis of this test is that balance comes from the combination of several neurological systems, namely proprioception, vestibular input, and vision. If any two of these systems are working the person should be able to demonstrate a fair degree of balance. The key to the test is that vision is taken away by asking the patient to close their eyes. This leaves only two of the three systems remaining and if there is a vestibular disorder (labyrinthine) or a sensory disorder (proprioceptive dysfunction) the patient will become much more unbalanced.

A 28-year-old primigravid woman at 39 weeks gestation is admitted to the hospital in active labor. On examination, the cervix is 100% effaced and 10 cm dilated. After 5 minutes of pushing, there is a prolonged deceleration of the fetal heart rate to 90/min. A decision to perform an episiotomy is made to expedite vaginal delivery. The anesthesiologist locates the ischial spines by palpating the posterolateral vaginal sidewall and administers an anesthetic. Three minutes later, pinching the posterior vulva does not provoke pain. The anesthetized nerve most likely also supplies which of the following structures?

The anesthesiologist has performed a pudendal nerve block to reduce sensation from the posterior perineum, vulva, and lower vagina. External anal sphincter 66% The external anal sphincter, along with the external urethral sphincter, levator ani, bulbospongiosus, and ischiocavernosus muscles are motor targets of the pudendal nerve. Sensory innervation is to the perineum, clitoris, labia, and anal canal. Pudendal nerve block is sometimes used to provide anesthesia during surgical procedures involving the perineum (e.g., episiotomy) or during the second stage of labor if epidural anesthesia is no longer feasible. The ischial spine is usually used as a landmark for injection to locate the pudendal nerve, which runs in close proximity to this bony prominence.

A 61-year-old woman comes to the physician because of a 1-week history of dizziness, nausea, vomiting, and repeated falls. Neurologic examination shows past-pointing on a finger-nose test. She has a broad-based gait. Ophthalmologic exam shows rhythmic leftward movement of the globes. A serum antibody assay is positive for anti-Yo antibodies directed at proteins expressed by Purkinje cells. This patient's condition is most likely associated with which of the following tumors?

The anti-Yo antibody, or Purkinje cell cytoplasmic antibody type 1, is an autoantigen that causes paraneoplastic cerebellar degeneration (PCD). These antibodies are associated with a tumor that may be caused by amplification or overexpression of the gene that encodes human epidermal growth factor receptor 2. Breast cancer 25% Breast cancer is the most common cause of anti-Yo antibody-associated PCD in women. PCD is a rare syndrome that usually occurs as the first manifestation of the malignancy and presents with symptoms of cerebellar dysfunction (e.g., dizziness, ataxia, nystagmus, dysmetria). Other causes of anti-Yo antibody-associated PCD are malignant tumors of the ovary, endometrium, and fallopian tubes.

An investigator is studying a drug that acts on a G protein-coupled receptor in the pituitary gland. Binding of the drug to this receptor leads to increased production of inositol triphosphate (IP3) in the basophilic cells of the anterior pituitary. Administration of this drug every 90 minutes is most likely to be beneficial in the treatment of which of the following conditions?

The drug mimics a hormone that has an agonistic effect on this G protein-coupled receptor when it is secreted in a pulsatile fashion. Anovulatory infertility 67% Pulsatile administration of gonadotropin-releasing hormone (GnRH) is the first-line treatment for women with anovulatory infertility due to hypogonadotropic amenorrhea. GnRH, which is secreted from the hypothalamus, acts on a G protein-coupled receptor in the basophilic cells of the anterior pituitary gland, increasing the production of IP3. When administered in a pulsatile fashion, GnRH increases the secretion of FSH and LH from the anterior pituitary and thus induces ovulation.

A 57-year-old man comes to the physician for a follow-up evaluation of chronic, retrosternal chest pain. The pain is worse at night and after heavy meals. He has taken oral pantoprazole for several months without any relief of his symptoms. Upper endoscopy shows ulcerations in the distal esophagus and a proximally dislocated Z-line. A biopsy of the distal esophagus shows mature columnar epithelium with goblet cells. Which of the following microscopic findings underlies the same pathomechanism as the cellular changes seen in this patient?

The biopsy findings in this patient (columnar epithelium with goblet cells) confirm a diagnosis of Barrett esophagus, which is characterized by metaplasia from squamous epithelium to intestinal columnar epithelium as a result of chronic gastric acid exposure. Barrett's esophagus A condition in which the esophageal squamous epithelium is replaced by columnar epithelium and goblet cells (intestinal metaplasia) due to longterm reflux esophagitis. Considered a precancerous condition for esophageal adenocarcinoma. Squamous epithelium in the bladder 43% The bladder is normally lined by transitional epithelium. Chronic irritation of the bladder wall (e.g., from Schistosoma infection, urinary calculi, or indwelling catheters) can result in a transformation of transitional to squamous epithelium. Cellular transformation due to chronic stress (metaplasia) occurs when stem cells reprogram to replace one mature cell type with another that adapts to the chronic stress. Early metaplastic changes are reversible if exposure to the irritant stops. However, dysplastic changes can occur if chronic irritation persists. Dysplasia A disordered growth of epithelium characterized by abnormally frequent mitotic figures and loss of cell orientation and uniformity (size, shape). Can progress to irreversible malignancy but reversible if not all epithelial layers are affected (known as carcinoma in situ). Urothelium Urothelial cell epithelium, Transitional epithelium A type of stratified epithelium that lines much of the urinary tract, including the renal pelvis, the ureters, and the bladder. Tissue composed of transitional epithelium can stretch in response to distension. The human esophagus has a mucous membrane consisting of a tough stratified squamous epithelium

A 41-year-old man with a history of hypertension and hyperlipidemia is brought to the emergency department by his wife for difficulty breathing after choking on food at dinner. He is unconscious and pulseless on arrival. Despite appropriate life-saving measures, he dies. Examination of the heart shows a necrotic, pale yellow plaque in the left circumflex artery. Similar lesions are most likely to be found in which of the following locations?

The coronary lesion described on this patient's autopsy is an atherosclerotic plaque. These plaques typically appear in vessels with high levels of endothelial strain. Atherosclerosis Atherosclerotic plaque The formation of lipid, cholesterol, and/or calcium-laden plaques within the tunica intima of the arterial wall, which can restrict blood flow. Rupture can cause intraluminal thrombosis that results in myocardial infarction, unstable angina, and/or ischemic stroke. Abdominal aorta 62% The abdominal aorta is the most common site for the development of atherosclerotic plaques, followed by the coronary, popliteal, and internal carotid arteries. Turbulent, high-pressure flow in these vessels places chronic strain on the endothelium. The abdominal aorta lacks vasa vasorum below the level of the renal arteries and is therefore particularly susceptible to endothelial injury.

Neuropil

The dense tangle of axonal and dendritic branches, and the synapses between them, that lies between neuronal cell bodies in the gray matter of the brain and spinal cord.

Prior to undergoing a total knee arthroplasty, a 62-year-old man with coronary artery disease undergoes diagnostic cardiac catheterization. The catheter is inserted via the femoral artery and then advanced to the ascending aorta. Pressure tracing of the catheter is shown. The peak indicated by the arrow is most likely caused by which of the following?

The event that leads to the smaller peak, so-called the "incisure" or "dicrotic notch," is itself caused by the equalization of pressure between the left ventricle and the ascending aorta.

An investigator is studying the effect of a high-lipid diet on glucose metabolism in Wistar rats. The experimental rat group is fed a high-lipid diet while the control group is fed a low-lipid diet. Two months after initiation of the experiment, the rats in both groups are injected with insulin and serum glucose measurements are obtained. Compared to the control group, the high-lipid diet group has a significantly higher average serum glucose after receiving insulin. Which of the following intracellular changes is most likely involved in the pathogenesis of this finding?

The experimental group of rats on a high-lipid diet shows higher insulin resistance (i.e., decreased response of serum glucose after receiving insulin) when compared to the control group. Increased activity of serine kinases 46% Activation of serine kinases is a key molecular mechanism in the development of insulin resistance. These kinases inactivate insulin receptor substrate (IRS) protein function through phosphorylation, reducing the action of phosphatidylinositide 3-kinases, thereby blocking downstream signaling of insulin receptors. In addition, serine kinases are able to phosphorylate serine residues on the insulin receptor itself, which also impairs insulin signal transduction. The interruption of insulin receptor signaling prevents the expression of GLUT4 receptors on cell surfaces, attenuating insulin's ability to decrease serum glucose levels. Free fatty acids are known to promote phosphorylation of IRS by serine kinases. Further factors known to increase the activity of intracellular serine kinases include catecholamines, glucocorticoids, and TNF-alpha.

A 58-year-old man comes to the physician for evaluation of increasing swelling in his left cheek and recurrent oral ulcers for 1 year. He has smoked a pack of cigarettes daily for 25 years. Examination shows a mild, nontender swelling above the angle of the left jaw. An ultrasound-guided biopsy of the mass confirms the diagnosis of parotid adenoid cystic carcinoma. A left-sided total parotidectomy is performed. This patient is at greatest risk for which of the following complications?

The facial nerve runs through the parotid gland and is most likely to be damaged during a procedure such as total parotidectomy. Inability to wrinkle the left side of the forehead 45% Unilateral inability to wrinkle the forehead is a sign of ipsilateral peripheral facial nerve palsy, the most common complication of parotidectomy. Injury to the facial nerve during parotidectomy results in an impairment of the ipsilateral muscles of facial expression, including the muscles responsible for eyelid and forehead movements. Hyperacusis of the left ear 28% Hyperacusis may be a symptom of facial nerve dysfunction (e.g., Bell palsy) due to associated weakness of the stapedius muscle. The nerve to the stapedius branches off of the facial nerve before the facial nerve exits the skull, which means that stapedius dysfunction cannot be a complication of parotidectomy.

A 3-week-old boy is brought to the physician for the evaluation of poor feeding and recurrent episodes of vomiting. He was delivered at term after an uncomplicated pregnancy. He is at the 5th percentile for length and weight. Physical examination shows generalized hypotonia. Urinalysis shows increased propionic acid concentration. The finding on urinalysis is best explained by the breakdown of which of the following substances?

The failure of the breakdown of this substance can result in maple syrup urine disease. Branched-chain amino acids 65% Propionyl-CoA is generated by the catabolism of the amino acids threonine, methionine, isoleucine, and valine. Of these amino acids, isoleucine and valine are branched-chain amino acids. Propionyl-CoA is also generated by the catabolism of odd-chain fatty acids, cholesterol, and the pyrimidines thymine and uracil. Propionyl-CoA carboxylase converts propionyl-CoA to methylmalonyl-CoA, which is converted to succinyl-CoA by methylmalonyl-CoA mutase. Succinyl-CoA generates oxaloacetate via the TCA cycle, which can be used to generate glucose (gluconeogenesis). A deficiency of propionyl-CoA carboxylase or methylmalonyl-CoA mutase can, therefore, cause symptoms of hypoglycemia (e.g., muscular hypotonia, vomiting, and lethargy), failure to thrive, and HAGMA due to the accumulation of propionic acid, which is subsequently excreted in urine. High anion gap metabolic acidosis Abbreviation: HAGMA A metabolic acidosis caused by an increase in the serum concentration of organic acids (normal: 10-16 mmol/L). These organic acids can be endogenous (e.g., lactic acid, ketoacids) or exogenous (e.g., glycolic acid in ethylene glycol poisoning, formic acid in methanol poisoning).

A 33-year-old man is evaluated by paramedics after being found unconscious outside of his home. He has no palpable pulses. Physical examination shows erythematous marks in a fern-leaf pattern on his lower extremities. An ECG shows ventricular fibrillation. Which of the following is the most likely cause of this patient's findings? The fern-leaf pattern on the patient's lower extremities is caused by local hyperemia. Hyperemia: an excess of blood in the vessels supplying an organ or other part of the body. Lichtenberg figures are fern-like patterns that may appear on the skin of lightning strike victims that disappear in 24 hours.[9] A lightning strike can also create a large Lichtenberg figure in grass surrounding the point struck. These are sometimes found on golf courses or in grassy meadows.[10] Branching root-shaped "fulgurite" mineral deposits may also be created as sand and soil is fused into glassy tubes by the intense heat of the current.

The fern-leaf pattern seen on this patient's lower extremities are Lichtenberg figures, which are pathognomonic of a lightning strike. In about 30% of cases, a lightning strike is fatal; the most common cause of death is cardiac arrest due to a fatal arrhythmia, such as ventricular fibrillation or asystole. Superficial burns and metallization of the skin may also occur after a lightning strike injury.

A 12-month-old boy is brought to the physician by his mother for a well-child examination. He was delivered at term after an uncomplicated pregnancy. His mother says he is breastfeeding well. He is at the 50th percentile for height and 65th percentile for weight. Physical examination shows no abnormalities. Urinalysis shows 3+ reducing substances. Compared to a healthy infant, giving this patient apple juice to drink will result in increased activity of which of the following enzymes?

The finding of a reducing substance in the urine of a healthy, asymptomatic infant suggests a diagnosis of essential fructosuria. Essential fructosuria Fructokinase deficiency A benign disorder of fructose metabolism caused by an autosomal recessive gene defect that results in fructokinase deficiency. Elevated fructose levels may be detected in urine. Hexokinase 48% Patients with essential fructosuria have a deficiency of fructokinase and thus cannot convert fructose to fructose-1-phosphate. Hexokinase (found in adipocytes and myocytes) becomes the primary enzyme responsible for fructose metabolism and converts it to fructose-6-phosphate, which can be used for glycolysis or converted to glucose-6-phosphate by glucose-6-phosphate isomerase. Fructose that is not converted by hexokinase is excreted in urine, which leads to the presence of reducing substances.

A 56-year-old man comes to the physician for a follow-up examination one week after a chest x-ray showed a solitary pulmonary nodule. He has no history of major medical illness. He has smoked 1 pack of cigarettes daily for the past 30 years. Physical examination shows no abnormalities. A tuberculin skin test is negative. A CT scan of the chest shows a 2.1-cm well-circumscribed, calcified nodule in the periphery of the right lower lung field. A CT-guided biopsy of the lesion is performed. Histological examination of the biopsy specimen shows regions of disorganized hyaline cartilage interspersed with myxoid regions and clefts of ciliated epithelium. Which of the following is the most likely diagnosis?

The findings in this patient result from the most common cause of benign solitary pulmonary nodules. Pulmonary hamartoma 60% Pulmonary hamartoma (PH) is the most common cause of a benign solitary pulmonary nodule. This diagnosis is further supported by the tumor size (PH are typically 1-3 cm in diameter), the benign appearance on imaging (round, well-circumscribed, calcified mass; 90% of PH are in peripheral lung areas), and the histology findings (typical features of PH include cartilage lobules that calcify or undergo osseous changes and fibromyxoid tissue with clefts of ciliated epithelium). PHs are usually asymptomatic and only occasionally cause symptoms by obstructing a bronchus (e.g., atelectasis, pneumonitis, cough, expectoration, chest pain). Hamartoma A benign mass composed of mature cells that are native to the tissue of origin but have abnormal tissue organization. Has a low potential to undergo malignant transformation.

A 59-year-old man comes to the physician because of a 6-month history of progressive blurry vision in both eyes. His vision has not improved with the use of reading glasses. His blood pressure is 155/98 mm Hg. Physical examination shows his visual acuity to be 20/80 in the right eye and 20/60 in the left eye. A photograph of the fundoscopic examination of the right eye is shown. Which of the following is the most likely cause of this patient's symptoms?

The fundoscopic image shows neovascularization of the optic disc and the retinal vessels. Diabetic retinopathy 63% Diabetic retinopathy is a long-term complication of diabetes that occurs due to the microvascular ischemia and osmotic damage secondary to the chronic elevation in serum glucose. The molecular pathogenesis of these changes involves intraocular sorbitol accumulation, which consumes NADPH and leads to oxidative stress. This oxidative stress ultimately leads to damage of the retinal microvasculature. The release of vascular endothelial growth factor (VEGF) by dying cells promotes the neovascularization of the retina and optic disc that is characteristic of diabetic retinopathy. Diabetic retinopathy A vascular disease of the retina affecting individuals with long-standing diabetes mellitus. Diabetic retinopathy is classified into non-proliferative diabetic retinopathy, proliferative diabetic retinopathy, and diabetic maculopathy. Loss of vision is typically insidious and occurs in most patients 20-30 years after the onset of DM, but acute loss of vision may occasionally occur as a result of retinal detachment, vitreous hemorrhage, or macular edema.

A 75-year-old man comes to the physician because of a 2-week history of sharp, stabbing pain in the lower back that radiates to the back of his left leg. He also has had a loss of sensitivity around his buttocks and inner thighs as well as increased trouble urinating the last week. Two years ago, he was diagnosed with prostate cancer and was treated with radiation therapy. Neurologic examination shows reduced strength and reflexes in the left lower extremity; the right side is normal. The resting anal sphincter tone is normal but the squeeze tone is reduced. Which of the following is the most likely diagnosis?

The gradual development of asymmetrical lower limb weakness, reduced reflexes in both the knees and ankles, saddle anesthesia, difficulty voiding, a decreased anal tone, and severe back pain with radiation to the lower limb is the classic clinical picture of cauda equina syndrome. Cauda equina syndrome is typically caused by traumatic injury, disc herniation, or a tumor below the level of the L2 vertebra. Given the history of prostate cancer in this patient, spinal imaging is needed to rule out vertebral or epidural metastases.

A 45-year-old man is brought to the emergency department by ambulance after a motor vehicle collision. He is not responsive to verbal commands and is unable to provide any history. His pulse is 108/min and regular. Physical examination shows ecchymoses over the neck and back. Neurological examination indicates damage to the spinal cord at the level shown in the illustration. This patient's injury is most likely located at which of the following levels of the spinal cord?

The illustration shows a segment of the spinal cord that contains prominent lateral projections between the dorsal and ventral horns. These projections are lateral horns, which contain neuronal cell bodies of the sympathetic nervous system. T6 61% Cross-sections of the spinal cord show butterfly-shaped gray matter with surrounding white matter. Gray matter contains the cell bodies of neurons; white matter contains the axons of neurons as they travel between the brain and the rest of the body. In general, gray matter is divided into three regions: dorsal horns, ventral horns, and lateral horns. Dorsal and ventral horns are found throughout the spinal cord. Though lateral horns are present between segments T1-L3 and segments S2-S4, only segments T1-L3 have the prominent lateral projections between the dorsal and ventral horns as seen in the illustration. Thus, this patient's injury must be located within these segments.

A 15-year-old girl is brought to the physician by her mother for a 2-day history of abdominal pain, nausea, vomiting, diarrhea, and decreased appetite. Her last menstrual period was 3 weeks ago. Her temperature is 37.6°C (99.7°F). Abdominal examination shows tenderness to palpation with guarding in the right lower quadrant. Laboratory studies show a leukocyte count of 12,600/mm3. Which of the following is the most likely underlying cause of this patient's condition?

The incidence of this patient's condition peaks between 10-19 years of age. Lymphatic tissue hyperplasia 57% This patient most likely presents with acute appendicitis. Lymphatic tissue hyperplasia is the primary etiology of appendicitis in children; fecalith obstruction is the most common etiology in adults. Obstruction within the appendix results in mucus and fluids stasis, allowing bacterial overgrowth and ultimately causing inflammation of the organ. Patients with appendicitis typically present with abdominal pain (dull migratory periumbilical pain that progresses to sharp right lower quadrant pain), fever, loss of appetite, nausea, and leukocytosis, as seen in this patient.

A previously healthy 27-year-old man comes to the physician because of a 3-week history of anxiety, diarrhea, and a 4.1-kg (9-lb) weight loss. On questioning, he also reports that he noticed a painless mass on his left testicle 2 weeks ago. His pulse is 110/min and irregular and blood pressure is 150/70 mm Hg. Examination shows diaphoresis and a fine tremor of the outstretched fingers. Testicular examination shows a 3-cm, firm, nontender mass on the left scrotum that does not transilluminate. This patient's underlying condition is most likely to be associated with which of the following findings?

The incidental finding of a testicular mass that does not transilluminate should raise suspicion for testicular cancer. Additionally, the patient's symptoms (anxiety, diarrhea, and weight loss) and other physical examination findings (tachyarrhythmia, hypertension with widened pulse pressure, diaphoresis, fine tremor) are classic features of hyperthyroidism. Paraneoplastic hyperthyroidism is a possible complication of testicular choriocarcinomas. Testicular choriocarcinoma A rare germ cell tumor composed of trophoblastic cells. Manifests with a testicular mass and highly increased serum human chorionic gonadotropin concentrations, which can cause gynecomastia and symptoms of hyperthyroidism. Positive urine pregnancy test 35% A positive urine pregnancy test indicates the presence of elevated human chorionic gonadotropin (hCG) in the patient's urine. Testicular tumors can produce elevated circulating levels of hCG. Because the alpha subunits of hCG and thyroid-stimulating hormone (TSH) are identical and the beta subunits of these hormones are structurally very similar, hCG is able to weakly stimulate TSH receptors, resulting in the release of thyroid hormones. In rare cases, conditions with extremely high levels of hCG (e.g., choriocarcinoma, as is the likely diagnosis in this man, or gestational trophoblastic disease in women) may be associated with paraneoplastic hyperthyroidism.

A 2-year-old boy is brought to the physician by his mother for evaluation of recurrent infections and easy bruising. He has been hospitalized 3 times for severe skin and respiratory infections, which responded to treatment with antibiotics. Examination shows sparse silvery hair. The skin is hypopigmented and there are diffuse petechiae. Laboratory studies show a hemoglobin concentration of 8 g/dL, leukocyte count of 3000/mm3, and platelet count of 45,000/mm3. A peripheral blood smear shows giant cytoplasmic granules in granulocytes and platelets. Which of the following is the most likely underlying cause of this patient's symptoms?

The infant's blood smear shows giant granules in granulocytes and platelets, a characteristic finding of Chediak-Higashi syndrome (CHS). This condition is a rare, autosomal recessive disorder characterized by recurrent pyogenic infections (involving the skin, respiratory tract, and mucous membranes), partial albinism with various degree of skin hypopigmentation and silvery-white hair, pancytopenia, and mild coagulation defects that result in easy bruising and mucosal bleeding in early childhood. Chédiak-Higashi syndrome A rare congenital immunodeficiency syndrome that is caused by an autosomal recessive defect in the lysosomal trafficking regulator gene (LYST). This mutation results in a defect in neutrophil chemotaxis/microtubule polymerization. Clinical features include recurrent pyogenic infections, oculocutaneous albinism, mild bleeding diathesis, and progressive neuropathies. Patients eventually enter a fatal accelerated phase (hemophagocytic lymphohistiocytosis). CHS is a disease causing impaired bacteriolysis due to failure of phagolysosome formation.[8] As a result of disordered intracellular trafficking there is impaired lysosome degranulation with phagosomes, so phagocytosed bacteria are not destroyed by the lysosome's enzymes.[citation needed] In addition, secretion of lytic secretory granules by cytotoxic T cells is affected.[citation needed] Defective lysosomal trafficking regulator gene 67% A loss of function of the lysosomal trafficking regulator gene (LYST) is the underlying pathophysiology of Chediak-Higashi syndrome (CHS). It causes a fusion defect in granules of all cells, resulting in giant intracellular granules. In addition, a microtubule dysfunction impairs fusion of phagosomes and lysosomes, which affects the normal immune response of neutrophils and monocytes. CHS patients who survive to adolescence or longer (∼ 10%) develop severe defects of the peripheral and central nervous system (e.g., neuropathies, intellectual decline, dementia, ataxia).

An investigator is studying the effects of influenza virus on human lung tissue. Biopsy specimens of lung parenchyma are obtained from patients recovering from influenza pneumonia and healthy control subjects. Compared to the lung tissue from the healthy control subjects, the lung tissue from the affected patients is most likely to show which of the following findings on histopathologic examination?

The influenza virus exhibits tropism for a cell type that is capable of regenerating alveoli after lung injury. Viral tropism is the ability of a given virus to productively infect a particular cell (cellular tropism), tissue (tissue tropism) or host species (host tropism). Increased type II pneumocytes 65% Alveoli consist of type I pneumocytes, which are thin, squamous cells that form the majority of the alveolar surface, and type II pneumocytes, which are cuboidal cells that form clusters in alveolar tissue. Type II pneumocytes proliferate in response to lung damage (e.g., from influenza infection) and can differentiate into type I pneumocytes, which are unable to replicate. Type II pneumocytes also secrete surfactant, which decreases surface tension and prevents alveolar collapse.

A 42-year-old man comes to the physician for the evaluation of episodic headaches involving both temples for 5 months. The patient has been taking acetaminophen, but it has not provided relief. He has also had double vision. Ophthalmic examination shows impaired peripheral vision bilaterally. Contrast MRI of the head shows a 14 x 10 x 8-mm intrasellar mass. Further evaluation is most likely to show which of the following findings?

The intrasellar mass seen on this patient's MRI scan is likely a pituitary adenoma, which accounts for his headaches and bitemporal hemianopia. The most common type of pituitary adenomas is a prolactinoma. Impotence 54% Impotence is common in men with hyperprolactinemia, which causes hypogonadotropic hypogonadism. Prolactin inhibits the synthesis and release of GnRH, which results in low levels of FSH and LH secretion. Low levels of LH lead to low production of testosterone in testicular Leydig cells. In men, the lack of testosterone can cause erectile dysfunction, decreased libido, reduced testicular volume, impaired fertility, and loss of axillary hair. Pituitary macroadenomas may cause headaches and bitemporal hemianopia due to compression of the surrounding structures (e.g., optic chiasm).

A 42-year-old woman comes to the physician because of right flank pain that started 3 days following a procedure. Her vital signs are within normal limits. Physical examination shows right costovertebral angle tenderness. An intravenous pyelogram shows a dilated renal pelvis and ureter on the right with a lack of contrast proximal to the ureterovesical junction. This patient most likely recently underwent which of the following procedures?

The intravenous pyelogram results describe a ureteral stricture located close to a structure that is commonly ligated in this procedure. Intravenous urography Intravenous pyelography, Excretory urogram, Anterograde pyelography A diagnostic procedure that involves intravenous contrast agent and x-rays to provide images of the genitourinary tract. Especially useful for assessing the upper urinary tract. Hysterectomy 73% The lower third of the ureter (pelvic ureter) lies just below the uterine artery at the cardinal ligament. Because the uterine arteries are ligated during hysterectomy, the ureters are at risk of being injured. Although any abdominopelvic surgery is a risk factor for ureteric injuries, hysterectomies are the most frequent cause. The ureters lie below the ovarian arteries within the infundibulopelvic ligament and are also susceptible to injury during ovarian surgeries (e.g., oophorectomy).

A male newborn is evaluated 24 hours after delivery for high-pitched crying, poor feeding, rhinorrhea, and low-grade fever. He was born at 40 weeks' gestation at 2514 g (5.54 lb) to a 28-year-old woman, gravida 3, para 2, by an uncomplicated cesarean section. Apgar scores were 8 and 9 at 1 and 5 minutes, respectively. The mother did not receive prenatal care. The infant's temperature is 38.0°C (100.4°F), pulse is 170/min, and blood pressure is 71/39 mm Hg. Examination shows hyperreflexia, tremors, and an excessive startle response. These symptoms are mostly likely due to maternal use of which of the following

The irritability, poor feeding, evidence of autonomic dysfunction (diaphoresis, hyperthermia, tremors), and hyperreflexia in this neonate is consistent with neonatal abstinence syndrome. Mu receptor agonist 70% This infant presents with symptoms of neonatal abstinence syndrome, which suggests maternal use of opioids (mu receptor agonist) during pregnancy. In infants with mild opiate withdrawal symptoms, supportive care (e.g., swaddling, IV fluids, quiet room) is preferred. For infants with more severe symptoms, as seen in this infant, administration of oral morphine is recommended to treat the withdrawal. Neonatal abstinence syndrome Etiology: maternal drug abuse during pregnancy Clinical features High-pitched cry, irritability Muscle tone and movement disorders Poor feeding, vomiting, diarrhea, failure to thrive Fever and sweating

A 57-year-old man is brought to the emergency department by his wife 20 minutes after having had a seizure. He has had recurrent headaches and dizziness for the past 2 weeks. An MRI of the brain shows multiple, round, well-demarcated lesions in the brain parenchyma at the junction between gray and white matter. This patient's brain lesions are most likely comprised of cells that originate from which of the following organs?

The junction between the gray and white matter is the typical location for brain metastases, the most common type of intracranial tumors, which visualize as round, well-demarcated intraparenchymal lesions on MRI. Lung 68% Primary cancers of the lung (particularly small cell lung cancer and adenocarcinoma of the lung) are the most common origin of brain metastases. The 2nd most common origin of brain metastases are cancers of the breast followed by melanoma and cancers of the kidney and colon. Metastatic disease accounts for approx. 50% of all brain tumors.

A 32-year-old man comes to the physician because of a 2-day history of a tingling sensation in his right forearm. He reports that his symptoms started after he lifted heavy weights at the gym. Physical examination shows loss of sensation on the lateral side of the right forearm. Sensation over the thumb is intact. Range of motion of the neck is normal. His symptoms do not worsen with axial compression or distraction of the neck. Further examination of this patient is most likely to show weakness of which of the following actions?

The lateral cutaneous nerve of the forearm, which innervates the lateral surface of the forearm, is a branch of the musculocutaneous nerve. Elbow flexion 52% Elbow flexion is controlled by the biceps brachii and brachialis muscles, which are innervated by the musculocutaneous nerve. Musculocutaneous nerve injury can result from lifting heavy weights as seen in this case. This injury also impairs forearm supination and the biceps tendon reflex. The normal range of motion in the neck and negative Spurling sign indicate that this patient has a peripheral nerve lesion.

A 55-year-old woman with a 1-year history of left-sided tinnitus is diagnosed with a tumor of the left jugular fossa. Sialometry shows decreased production of saliva from the left parotid gland. The finding on sialometry is best explained by a lesion of the nerve that is also responsible for which of the following

The most common tumor of the jugular foramen is a paraganglioma (glomus jugulare), which typically presents with pulsatile tinnitus. The parotid gland is innervated by the tympanic branch of the glossopharyngeal nerve (CN IX), which exits the skull at the jugular foramen, posterior to the jugular fossa. Afferent limb of the gag reflex 57% The afferent sensory limb of the gag reflex is formed by the glossopharyngeal nerve (CN IX) and the efferent motor limb formed by the vagus nerve (CN X). Upon irritation of the posterior pharyngeal wall, the reflex elicits elevation of the soft palate and bilateral contraction of the pharyngeal muscles. Equilibrium and balance 2% The vestibular division of the vestibulocochlear nerve (CN VIII) carries information from the inner ear to the brain stem. A CN VIII lesion would explain this patient's tinnitus. However, CN VIII travels through the internal acoustic meatus in the temporal bone rather than through the jugular foramen, and a lesion of this nerve would not affect saliva production in the parotid gland.

A 65-year-old man comes to the physician because of double vision that began this morning. He has hypertension and type 2 diabetes mellitus. He has smoked two packs of cigarettes daily for 40 years. His current medications include lisinopril, metformin, and insulin. Physical examination shows the right eye is abducted and depressed with slight intorsion. Visual acuity is 20/20 in both eyes. Extraocular movements of the left eye are normal. Serum studies show a hemoglobin A1c of 11.5%. Which of the following additional findings is most likely in this patient?

The most likely cause of an isolated oculomotor nerve palsy in a patient with uncontrolled diabetes mellitus is diabetic mononeuropathy, which occurs as a result of microangiopathic ischemia. Upper eyelid droop on the right eye 55% Ptosis occurs in patients with oculomotor nerve palsy because levator palpebrae superioris is innervated by the oculomotor nerve. Whereas patients with oculomotor nerve palsy due to compression usually show dilated pupils, those with diabetic mononeuropathy normally do not have pupillary involvement because the microangiopathy typically involves the vasa nervorum within the oculomotor nerve, while the pial blood vessels (which supply the superficial parasympathetic fibers) are usually spared.

A 57-year-old man comes to the physician because of sudden-onset fever, malaise, and pain and swelling of his wrists and ankles that began a week ago. One month ago, he was started on hydralazine for adjunctive treatment of hypertension. His temperature is 37.8°C (100°F). Examination shows swelling, tenderness, warmth, and erythema of both wrists and ankles; range of motion is limited. Further evaluation is most likely to show an increased level of which of the following autoantibodies?

The onset of lupus-like symptoms (fever, malaise, polyarthritis) soon after starting hydralazine therapy indicates drug-induced lupus erythematosus. Idiopathic systemic lupus erythematosus can present similarly but it is less common in men (90% of patients with idiopathic SLE are females) and is not associated with hydralazine therapy. Anti-histone 74% Anti-histone antibodies are present in most cases of drug-induced lupus erythematosus (DILE). Since anti-histone antibodies are a subtype of antinuclear antibody, ANA will also be positive. Other common causes of DILE include methyldopa, sulfa drugs, isoniazid, procainamide, and phenytoin. DILE can have many of the features of idiopathic SLE such as fever, arthritis, malar rash, and serositis. However, unlike idiopathic SLE, DILE does not affect the oral mucosa, brain, or kidneys and is typically self-limiting after discontinuation of the offending drug.

Three days after admission to the hospital following a motor vehicle accident, a 45-year-old woman develops a fever. A central venous catheter was placed on the day of admission for the treatment of severe hypotension. Her temperature is 39.2°C (102.5°F). Examination shows erythema surrounding the catheter insertion site at the right internal jugular vein. Blood cultures show gram-positive, catalase-positive cocci and there is a low minimum inhibitory concentration of novobiocin. Which of the following is the most appropriate pharmacotherapy?

The organism found in this patient's blood culture is Staphylococcus epidermidis, which is a common cause of intravascular catheter infection. Vancomycin 80% Staphylococcus epidermidis is a gram-positive, catalase-positive, coagulase-negative bacteria that is sensitive to novobiocin. S. epidermidis is a common source of catheter-related bloodstream infections and sepsis. As part of the normal skin flora, it creates biofilms on catheters via intercellular adhesins, which lead to cell aggregation. Because these biofilms are resistant to antibiotic treatment, the catheter must be removed in case of infection. Most coagulase-negative bacteria like S. epidermidis are methicillin-resistant (> 90%). Therefore, vancomycin is the recommended empiric antibiotic agent for catheter-related bloodstream infection.

A 38-year-old man with chronic hepatitis C comes to the physician because of a 10-day history of darkening of his skin and painless blisters. He started working as a landscaper 2 weeks ago. He drinks 2 beers every night and occasionally more on the weekends. Examination shows bullae and oozing erosions in different stages of healing on his arms, dorsal hands, and face. There are atrophic white scars and patches of hyperpigmented skin on the arms and face. This patient's skin findings are most likely associated with increased concentration of which of the following?

The painless blisters on sun-exposed areas, which developed shortly after the patient started a job involving outdoor work, indicate photosensitivity. Along with his other skin findings, alcohol consumption, and chronic hepatitis C infection, this history suggests porphyria cutanea tarda (PCT). Uroporphyrinogen III 62% Increased plasma and urine porphyrins, including uroporphyrinogen III, are diagnostic of porphyria cutanea tarda (PCT) and are caused by a defect in uroporphyrinogen decarboxylase in the heme synthesis pathway. PCT is the most common porphyria and typically manifests in adults 40-70 years of age with blistering cutaneous photosensitivity (due to accumulation of porphyrins in the skin), as seen in this patient. The pigmentation changes and scarring in this patient likely developed from previous PCT lesions. Porphyria cutanea tarda Abbreviation: PCT A condition caused by defective uroporphyrinogen-III decarboxylase (UROD) which leads to the accumulation of uroporphyrinogen in the skin and chronic photosensitivity with blistering and hyperpigmentation. Can be acquired (type I) or inherited (autosomal dominant; type II).

A 13-year-old girl is brought to the physician by her father because of a 1-month history of pain in her right knee. She is a competitive volleyball player and has missed several games recently due to pain. Examination shows swelling distal to the right knee joint on the anterior surface of the proximal tibia; there is no overlying warmth or deformity. Extension of the right knee against resistance is painful. Which of the following structures is attached to the affected anterior tibial area?

The patellar ligament connects the patella to the tibial tuberosity and allows for full extension of the knee via the quadriceps muscle. Osgood-Schlatter disease arises due to overuse of the quadriceps muscle during a period of growth (adolescence), which puts excessive strain on the attachment of the patellar ligament at the tibial tuberosity, resulting in painful inflammation and avascular necrosis. Patients present with pain during quadriceps contraction

A 38-year-old man comes to the physician because of recurrent palpitations over the past 2 months. He says that he was diagnosed with a minor heart defect during childhood but does not remember the details and has not had regular medical follow-up examinations since. Physical examination shows a heart murmur. Cardiac catheterization is performed and hemodynamic results show: Oxygen saturation (%)Pressure (mm Hg)Right atrium753Right ventricle7525/4Pulmonary trunk7522/10Left atrium9910Left ventricle99165/45Ascending Aorta99155/70 Which of the following is the most likely diagnosis?

The patient has a widened pulse pressure in the ascending aorta.'' Aortic valve regurgitation 41% Aortic valve regurgitation causes a backward flow from the aorta during diastole. This backflow decreases diastolic pressure in the aorta (N = 60-90 mm Hg) and increases diastolic pressure in the left ventricle (N = 5-12 mm Hg). This regurgitant blood combines with blood from the left atrium to fill the left ventricle, which results in an increased preload and a subsequent increase in stroke volume. As a result, the systolic pressure in both the ascending aorta (N = 90-140 mm Hg) and in the left ventricle (N = 90-140 mm Hg) is increased. Oxygen saturation and right heart pressures are normal in aortic regurgitation. If left untreated, this condition leads to eccentric hypertrophy of the left ventricle and left heart failure.

A 49-year-old man is brought to the emergency department after being discovered unconscious in a field near the county fair. Several empty bottles of vodka were found near him. On arrival, he is mumbling incoherently. He is malodorous and appears disheveled. Serum studies show: Na+ 150 mEq/L K+ 3.3 mEq/L Cl- 115 mEq/L HCO3- 13 mEq/L Urea nitrogen 30 mg/dL Glucose 75 mg/dL Creatinine 1.4 mg/dL Lactic acid 6 mmol/L (N < 2) Which of the following changes to enzyme activity best explains this patient's laboratory findings?

The patient presents with altered mental status and lactic acidosis, most likely due to ethanol intoxication. Lactic acidosis occurs when ethanol metabolism results in a high hepatic NADH/NAD ratio, diverting pyruvate metabolism towards lactate and inhibiting gluconeogenesis. The excess lactate is consequently exported from the liver. Stop drinking alcohol. Chronic drinking can increase the buildup of lactic acid. Decreased activity of pyruvate dehydrogenase 73% The increased lactic acid levels in this patient suggest that his body has shifted from aerobic cellular respiration, the first step of which involves the transformation of pyruvate to acetyl-CoA by pyruvate dehydrogenase complex (PDC), to anaerobic cellular respiration that includes conversion of pyruvate to lactate. Alcohol dehydrogenase breaks down ethanol by reducing NAD+ to NADH. This increased ratio of NADH/NAD+ inhibits the citric acid cycle, which results in an accumulation of acetyl-CoA. NADH and acetyl-CoA are both pyruvate dehydrogenase reaction products and thus inhibit PDC activity. NAD+ is replenished by the reduction of pyruvate to lactate by lactate dehydrogenase, which has led to the lactic acidosis seen in this patient.

A 66-year-old woman comes to the physician because of a 1-week history of pruritic blister formation. Physical examination shows multiple 1-3 cm bullae on the palms, soles, lower legs, and inguinal folds. Gentle rubbing of the skin does not result in sloughing of the epidermis. Immunofluorescence studies of a perilesional skin biopsy specimen are most likely to show deposition of antibodies in which of the following areas?

The patient presents with large, pruritic bullae on the palms, soles, lower legs, and inguinal folds, sparing the mouth. Nikolsky sign is negative. These clinical features are typical of bullous pemphigoid, an autoimmune blistering disease, most common in older adults. At the dermoepidermal junction 79% Direct immunofluorescence in bullous pemphigoid reveals linear deposition of IgG and C3 at the dermo-epidermal junction. The bullae are formed due to an immune reaction caused by autoantibodies directed against bullous pemphigoid antigen 1 and bullous pemphigoid antigen 2 (components of the hemidesmosome). Microscopic examination of a blistering lesion usually reveals the presence of eosinophils with the separation of tissue layers subepidermally causing tense bullae. Bullous pemphigoid Abbreviation: BP An autoimmune, blistering skin disorder caused by circulating antibodies against the BP antigen on hemidesmosomes. Tense subepidermal blisters develop and can become hemorrhagic or turbid. Peak incidence in people above 60 years of age. Recurrent lesions typically occur in skin flexures (e.g., groin, axilla) and over the abdomen; mucous membranes are usually spared.

A 75-year-old woman is brought to the emergency department because of a 6-hour history of severe headache, nausea, and 1 episode of vomiting. On arrival, she is lethargic and oriented only to self. Her pulse is 50/min, respirations are 8/min and shallow, and blood pressure is 150/96 mm Hg. Examination shows medial deviation of the left globe and ecchymoses of the upper eyelids. Appropriate pharmacotherapy with an intravenous drug is initiated, and the patient's urine output subsequently increases. The patient is at greatest risk of which of the following adverse effects?

The patient presents with nausea, vomiting, headache, and a decreased level of consciousness. Examination shows the Cushing triad (bradycardia, respiratory depression, and hypertension), cranial nerve VI palsy, and spontaneous bruising of the upper eyelids. These are classic features of elevated intracranial pressure (ICP), which can be treated with mannitol, an osmotic diuretic. Pulmonary edema 46% Mannitol causes an increase in plasma osmolality, resulting in osmotic shifts of water into the intravascular space. This effect decreases cerebral edema but can lead to iatrogenic volume overload and pulmonary edema. Risk factors for this side effect of mannitol include very high doses and a low glomerular filtration rate.

A 29-year-old man comes to the physician for worsening restlessness over the past several days. Three weeks ago, he was started on trifluoperazine for the treatment of schizophrenia. He reports that, since then, he has often felt compelled to pace around his house and is unable to sit or stand still. He is switched to an alternative antipsychotic medication. Four weeks later, the patient reports improvement of his symptoms but says that he has developed increased drowsiness, blurred vision, and dry mouth. The patient was most likely switched to which of the following drugs?

The patient was initially started on a high-potency typical antipsychotic (trifluoperazine) and developed akathisia, an extrapyramidal symptom. He was then switched to a drug that is associated with fewer extrapyramidal symptoms but has more antihistaminergic (drowsiness) and anticholinergic (blurred vision, and dry mouth) side effects, most likely a low-potency typical antipsychotic. Akathisia A movement disorder characterized by restlessness (e.g., an inability to sit or stand still and a constant urge to move). Affected people typically fidget, pace, and rock on their feet. Common causes include adverse effects of antipsychotic medication and drug withdrawal. Extrapyramidal syndrome A collection of movement disorders that are typically seen due to disruption of pathways in the basal ganglia. Symptoms include bradykinesia, rigidity, dystonia, athetosis, chorea, ballismus, akathisia, tics, and tremors. Chlorpromazine 66% Chlorpromazine is a low-potency typical antipsychotic that blocks D2 receptors. Extrapyramidal symptoms are less commonly seen with chlorpromazine than with high-potency typical antipsychotics. However, the drug is associated with anticholinergic and antihistaminergic side effects, as observed in this patient. Additional α-1 receptor blockade effects may cause orthostatic hypotension. Haloperidol 9% Haloperidol is a high-potency typical antipsychotic drug that blocks D2 receptors. Unlike the drug given to this patient, haloperidol has low anticholinergic and antihistaminergic activity and carries a higher risk of development of extrapyramidal symptoms.

66-year-old man is brought to the emergency department 1 hour after the abrupt onset of painless loss of vision in his left eye. Over the last several years, he has noticed increased blurring of vision; he says the blurring has made it difficult to read, but he can read better if he holds the book below or above eye level. He has smoked 1 pack of cigarettes daily for 40 years. Fundoscopic examination shows subretinal fluid and small hemorrhage with grayish-green discoloration in the macular area in the left eye, and multiple drusen in the right eye with retinal pigment epithelial changes. Which of the following is the most appropriate pharmacotherapy for this patient's eye condition?

The patient's acute, painless, unilateral loss of central vision (central scotoma), together with his history of gradual loss of central vision and fundoscopic findings of a subretinal hemorrhage and grayish-green retinal discoloration, are characteristic of wet age-related macular degeneration. Ranibizumab Ranibizumab is a vascular endothelial growth factor (VEGF) inhibitor. VEGF release in wet age-related macular degeneration (wet AMD or neovascular AMD) causes growth of new, friable blood vessels in the choriocapillaris, which are prone to bleeding and leaking. VEGF inhibitors delivered intravitreally are the first-line treatment for wet AMD and have become the mainstay of treatment due to their efficacy and safety profile.

A 38-year-old woman comes to the physician because of a 3-week history of involuntary movements of her extremities. One year ago, she was fired from her position as an elementary school teacher because she had stopped preparing lessons and was frequently absent without notice. She now lives with her mother. She appears emaciated and malodorous. Examination shows rapid, nonrepetitive jerks of her limbs and face that frequently end with the patient covering her face and yawning. She has an unsteady gait. Genetic testing shows a mutation on chromosome 4. This patient's condition is most likely associated with increased levels of which of the following substances?

The patient's apparent changes in personality, mood, and/or cognitive function (recent termination of her job, lack of personal hygiene) and the finding of chorea on examination are consistent with Huntington disease (HD). HD is an autosomal dominant progressive neurodegenerative disorder caused by a trinucleotide expansion of CAG in the Huntington gene on chromosome 4p. Dopamine 48% Increased dopaminergic transmission has been observed in the early phases of Huntington disease leading to hyperkinetic movements, as seen in this patient. Huntington disease involves selective degeneration of medium spiny neurons in the caudate and putamen, preferentially affecting the indirect pathway of the basal ganglia in the early stages of the disease. In the late phase of HD, there is dopamine depletion, which leads to the development of bradykinesia and rigidity.

A 31-year-old woman comes to the physician because of a 5-month history of intermittent flank pain. Over the past 2 years, she has had five urinary tract infections. Her blood pressure is 150/88 mm Hg. Physical examination shows bilateral, nontender upper abdominal masses. Serum studies show a urea nitrogen concentration of 29 mg/dL and a creatinine concentration of 1.4 mg/dL. Renal ultrasonography shows bilaterally enlarged kidneys with multiple parenchymal anechoic masses. Which of the following is the most likely diagnosis?

The patient's condition is associated with cerebral berry aneurysms. Autosomal dominant polycystic kidney disease 83% ADPKD commonly presents in young adulthood with renal insufficiency, self-limited hematuria, and hypertension; abdomen, flank, or back pain is often the first clinical indication. The condition is the most common inherited kidney disorder and is caused by mutations in PKD1 and PKD2. Cysts appear as multiple parenchymal anechoic masses on ultrasound; coupled with an appropriate family history and physical exam, ultrasound is adequate to establish the diagnosis in most cases.

A 16-year-old girl is brought to the physician because of a 1-month history of fever, headaches, and profound fatigue. Her temperature is 38.2°C (100.8°F). Examination shows splenomegaly. Laboratory studies show: Leukocyte count13,000/mm3 (15% atypical lymphocytes)SerumAlanine aminotransferase (ALT)90 U/LAspartate aminotransferase (AST)60 U/LHeterophile antibody assaynegativeEBV viral capsid antigen (VCA) antibodiesnegativeHIV antibodynegative In an immunocompromised host, the causal organism of this patient's symptoms would most likely cause which of the following conditions?

The patient's constitutional symptoms, fever, splenomegaly, and the finding of atypical lymphocytosis with ≥ 10% lymphocytes and mild elevation of transaminases are characteristic of mononucleosis. The absence of exudative pharyngitis and lymphadenopathy, in combination with the negative monospot test (heterophile antibody assay) and negative EBV VCA antibodies, make cytomegalovirus (CMV) the most likely causal pathogen. Linear ulcers near the lower esophageal sphincter 61% Although CMV is asymptomatic in the majority of cases involving immunocompetent individuals, it can cause CMV mononucleosis in up to 10% of the population. In contrast, CMV infection in immunocompromised individuals can cause CMV esophagitis, manifesting as linear ulcers near the lower esophageal sphincter and many other complications, including CMV pneumonia, CMV retinitis, and CMV encephalitis (all of which are AIDS-defining conditions).

A 12-year-old girl is brought to the physician by her mother because she has been waking up multiple times at night to go to the bathroom even though she avoids drinking large amounts of water close to bedtime. She has no significant medical history apart from 3 episodes of lower urinary tract infections treated with nitrofurantoin in the past 2 years. Her family emigrated from Nigeria 10 years ago. Physical examination shows no abnormalities. Laboratory studies show: Hemoglobin14.2 g/dLMCV92 fLReticulocytes1.5%SerumOsmolality290 mOsmol/kg H2OUrineLeukocytesnegativeNitritenegativeGlucosenegativeOsmolality130 mOsmol/kg H2O Hemoglobin electrophoresis shows: HbA56%HbS43%HbF1% This patient is at greatest risk for which of the following conditions?

The patient's hemoglobin electrophoresis shows hemoglobin S (HbS) and a decreased, although still present, hemoglobin A (HbA), which indicates sickle cell trait. Necrosis of the renal papillae 60% Sickle cell trait causes renal papillary necrosis secondary to transient sickling of RBCs in the renal capillaries, which results in microthrombotic infarctions and sloughing of necrotic papillae. The renal medulla is particularly susceptible to ischemia as the hypoxic and hyperosmolar environment promotes RBC sickling in the vasa recta, disrupting free water reabsorption and countercurrent exchange. Renal papillary necrosis leads to hyposthenuria (indicated by this patient's nocturia) and episodes of painless gross hematuria. Other complications associated with sickle cell trait include recurrent urinary tract infections, chronic kidney disease, and renal medullary carcinoma. Renal papillary necrosis Injury to the kidney that is caused by coagulative necrosis of the renal medullary pyramids and papillae. The etiology is often multifactorial but possible precipitating factors include pyelonephritis, obstruction of the urinary tract, sickle cell disease, and analgesic abuse.

A 69-year-old right-handed man comes to the physician for a routine health maintenance examination. On questioning, he has had some difficulty speaking for 3 months. During a conversation, he often has difficulty repeating what his interlocutor has said. He has hypertension and takes hydrochlorothiazide. His vital signs are within normal limits. The patient speaks fluently in full sentences and demonstrates normal comprehension. When asked to say the word "kindergarten," he replies with, "Sintelmarvin... no, that's wrong...kinterflargin, no that isn't it either...kantolargen...no? How about, kindergarten?" The most likely cause of the patient's symptoms is occlusion of a branch of which of the following arteries?

The patient's impaired repetition with paraphasia, together with preserved comprehension, is called conduction aphasia. Lesions in the supramarginal gyrus can cause this syndrome, which often presents without other neurologic symptoms. Left middle cerebral 72% In a right-handed person, the supramarginal gyrus is supplied by a branch of the left middle cerebral. A lesion to this area can disrupt the articulate fasciculata and result in conductive aphasia.

A 73-year-old woman is brought to the physician by her son because of a 2-year history of increasing forgetfulness. She does not remember her children's names; she cannot go outside the house unaccompanied because she forgets where she is and what she is supposed to do. Her son reports that over the past year, she has withdrawn herself from social gatherings and avoids making conversation. Mental status examination shows both impaired short- and long-term memory. She is oriented only to person. Her abstract reasoning is impaired. She appears indifferent about her memory lapses and says this is normal for someone her age. This patient's condition is most likely associated with decreased production of which of the following substances?

The patient's memory impairment, disorientation, and executive dysfunction suggest a diagnosis of Alzheimer disease (AD), the most common cause of dementia. Acetylcholine 83% Alzheimer disease is characterized by a degeneration of cholinergic neurons in the temporal lobe cortex, hippocampus, and nucleus basalis of Meynert, leading to impaired cholinergic transmission to the cortex and reduced levels of choline acetyltransferase. Acetylcholine (ACh) is generally synthesized in the nucleus basalis of Meynert. ACh levels would be increased in Parkinson disease and decreased in Alzheimer disease and Huntington disease.

An 81-year-old woman is brought to the physician by her son because of worsening forgetfulness and disorientation over the past 2 years. She has to be reminded of her grandchildren's names and frequently forgets her current address. She lives with her son. She has occasional episodes of urinary incontinence. She appears well nourished. Neurologic examination shows no abnormalities; her gait is normal. Mental status examination shows mild memory impairment. She is oriented to self and place, but not to time. Which of the following is the most appropriate pharmacotherapy?

The patient's mental status examination suggests mild dementia; the predominant symptoms of memory loss and disorientation are consistent with Alzheimer disease. Galantamine 77% Patients with mild-to-moderate Alzheimer disease (AD) show improved cognition and ability to perform everyday activities when treated with acetylcholinesterase inhibitors, such as galantamine, donepezil, or rivastigmine. These drugs induce an increase in the neurotransmitter acetylcholine, which is decreased in AD. If this patient's condition progresses to moderate-to-severe AD, memantine (an NMDA-receptor antagonist) will become indicated and may be given in combination with an acetylcholinesterase inhibitor. Thiamine 3% Thiamine is used in the treatment of Wernicke encephalopathy, a syndrome seen in patients with poor nutrition that is typically associated with alcohol use disorder. This patient does not have a history of excessive alcohol use, and her normal appearance does not suggest malnutrition (although it cannot be ruled out entirely, especially in an elderly patient). Moreover, patients with Wernicke encephalopathy also present with gait dysfunction and oculomotor dysfunction, such as nystagmus, lateral rectus palsy, or conjugate gaze palsy, none of which is seen in this patient.

A 28-year-old man comes to the physician because of a persistent tingling sensation in the right side of his face. The sensation began after he underwent an extraction of an impacted molar 2 weeks ago. Examination shows decreased sensation of the skin over the right side of the mandible, chin, and the anterior portion of the tongue. Taste sensation is preserved. The affected nerve exits the skull through which of the following openings?

The patient's symptoms suggest a lesion of the mandibular branch of the trigeminal nerve (CN V3). Foramen ovale 73% The mandibular branch of the trigeminal nerve exits the skull through the foramen ovale. A lesion of this nerve, as is most likely present in this patient, results in ipsilateral decreased sensation of the skin over the mandible, chin, parts of the auricle and the temple, the buccal mucosa, and the anterior ⅔ of the tongue. The mandibular branch also provides motor innervation to the muscles of mastication. The other structures that pass through the foramen ovale are the accessory meningeal artery, the lesser petrosal nerve, and the emissary veins connecting the cavernous sinus with the pterygoid plexus.

A 20-year-old man comes to the physician because of worsening gait unsteadiness and bilateral hearing loss for 1 month. He has had intermittent tingling sensations on both cheeks over this time period. He has no history of serious medical illness and takes no medications. Audiometry shows bilateral sensorineural hearing loss. Genetic evaluation shows a mutation of a tumor suppressor gene on chromosome 22 that encodes merlin. This patient is at increased risk for which of the following conditions?

The patient's unsteadiness, bilateral sensorineural hearing loss, and midface paresthesia are consistent with bilateral compression of CN V and CN VIII. This is usually caused by bilateral vestibular schwannomas, a hallmark feature of neurofibromatosis type 2 (NF-2). NF-2 is an autosomal dominant syndrome that it is caused by a mutation in the NF-2 gene on chromosome 22, which produces merlin, a tumor suppressor. Meningioma 52% Beside bilateral schwannomas, neurofibromatosis type 2 (NF2) often manifests with multiple cerebral and spinal tumors, as well as meningiomas. Most meningiomas are slow-growing and patients may therefore be asymptomatic for a prolonged period of time. If symptoms are present, they usually include general symptoms of CNS tumors (e.g., seizures), or more specific symptoms that depend on the tumor location (e.g., paraparesis and/or uncontrolled micturition if tumor arises from the falx cerebri).

During an experimental study on the mucosa of the gastrointestinal tract, an investigator finds that vagal stimulation regulates the activity of a cell type that is mainly found in the fundus and body of the stomach. A photomicrograph of a normal gastric mucosa is shown; the cell type under investigation is indicated by the arrow. Stimulation of the labeled cell is most likely to cause increased secretion of which of the following substances?

The photomicrograph shows a large eosinophilic cell with a circular nucleus (fried egg appearance), which is characteristic of parietal cells. These cells are typically located in the middle of the gastric glands. Intrinsic factor 59% Parietal cells are responsible for the secretion of intrinsic factor, which facilitates vitamin B12 transportation and absorption in the terminal ileum. Parietal cells also secrete HCl, which maintains the acidic environment of the stomach necessary for digestion. In addition, these cells play an important role in the homeostasis of gastric mucosa by secreting multiple growth factors. The activity of parietal cells is stimulated by acetylcholine (via vagal stimulation), histamine, and gastrin, and is inhibited by somatostatin, secretin, prostaglandins, CCK, and GIP.

A 55-year-old man comes to the physician because of a 7-month history of dull hip pain that worsens at night. Treatment with over-the-counter ibuprofen has not lessened the pain. He has not had any trauma to the hip. Examination shows swelling and marked tenderness to palpation over the right hip. An x-ray of the right hip shows an ill-defined lytic lesion with spiculated calcifications and soft tissue infiltration in the proximal femur. A photomicrograph of a biopsy specimen of the lesion is shown. Which of the following is the most likely diagnosis?

The photomicrograph shows atypical, irregularly shaped lobules of cartilage separated by fibrous bands. Chondrosarcoma 45% Chondrosarcoma is a malignant tumor that arises from chondrocytes. It is most common in men ≥ 50 years of age and is usually a slow-growing tumor. Common sites of involvement include the pelvis, ribs, proximal femur, and proximal humerus. Manifestations include insidious, dull, deep pain that worsens at night and localized swelling with tenderness to palpation, as seen in this patient. X-ray findings typically show a moth-eaten (lytic) appearance with popcorn-like (spiculated) calcifications. The diagnosis is confirmed by biopsy, which typically shows irregularly shaped lobules of cartilage, fibrous bands, and calcium deposits. Surgery, which may be curative depending on the grade, extension, and location of the tumor, is the treatment of choice. Chondrosarcoma A heterogeneous group of malignant tumors that arise from cartilaginous tissue and typically occur in patients > 50 years old. Usually occur in the pelvis, ribs, and metaphyses of long bones.

A 6-year-old boy is brought to the physician because of worsening fatigue for the past 4 weeks. Examination of the head and neck shows conjunctival pallor, grayish-brown spots on the irises, prominent epicanthal folds, and a broad nasal bridge. He has a single transverse palmar crease, and there are scattered petechiae over the chest. An ultrasound of the abdomen shows enlargement of the liver and spleen. Analysis of the bone marrow aspirate of this patient is most likely to show which of the following findings?

The physical findings of prominent epicanthal folds and scattered, grayish-brown spots on the irises (Brushfield spots) are suggestive of an underlying diagnosis of Down syndrome. CD10 positive cells 51% This patient has fatigue, pallor, petechiae, and hepatosplenomegaly caused by acute leukemia. Patients with Down syndrome have an increased risk of developing acute lymphocytic leukemia (ALL) and acute myelogenous leukemia. Given this patient's young age, B-cell ALL is the most likely diagnosis. A bone marrow aspirate from patients with this condition typically shows > 25% lymphoblasts that are positive for CD10, a marker of pre-B cells.

An investigator studying immune response administers a 0.5 mL intradermal injection of an autoclaved microorganism to a study volunteer. Four weeks later, there is a 12-mm, indurated, hypopigmented patch over the site of injection. Which of the following is the most likely explanation for the observed skin finding?

The physician has performed a lepromin test, which involves intradermally injecting a small amount of autoclaved Mycobacterium leprae (the causative organism of leprosy) and assessing the skin reaction after 3 to 4 weeks. An area of induration > 5 mm at that time is defined as a positive result. It indicates that the body is able to mount an immune response following exposure to the organism. Increased activity of CD4+ Th1 cells 73% A lepromin skin test is used to determine the type of leprosy a person has contracted, lepromatous leprosy (negative test) or tuberculoid leprosy (positive test). The test measures the ability of an individual to mount an effective cell-mediated immune response against M. leprae. During this immune response an increased number of CD4+ Th1 cells and, subsequently, cytotoxic T-cells are activated, leading to an inflammation reaction with granuloma formation (induration). The latter one characterizes the pathogenesis of tuberculoid leprosy and indicates its presence (positive test result). Because lepromatous leprosy is characterized by weak cell-mediated immunity, a lepromin skin test will be negative.

A 13-year-old boy is brought to the emergency department after being involved in a motor vehicle accident in which he was a restrained passenger. He is confused and appears anxious. His pulse is 131/min, respirations are 29/min, and blood pressure is 95/49 mm Hg. Physical examination shows ecchymosis over the upper abdomen, with tenderness to palpation over the left upper quadrant. There is no guarding or rigidity. Abdominal ultrasound shows free intraperitoneal fluid and a splenic rupture. Intravenous fluids and vasopressors are administered. A blood transfusion and exploratory laparotomy are scheduled. The patient's mother arrives and insists that her son should not receive a blood transfusion because he is a Jehovah's Witness. The physician proceeds with the blood transfusion regardless of the mother's wishes. The physician's behavior is an example of which of the following principles of medical ethics?

The physician's decision to perform blood transfusion is made in a situation that is life-threatening for the patient. Beneficence 75% This physician's actions demonstrate beneficence, which refers to acting in the best interest of the patient. Beneficence may conflict with patient or guardian autonomy. It is appropriate to treat a child whose guardian refuses lifesaving treatment for them. However, when adult patients refuse lifesaving treatment for themselves based on religious or other beliefs, and they understand the consequences of forgoing treatment, their decision should be accepted in order to respect patient autonomy. Nonmaleficence 18% Nonmaleficence is the medical ethical principle of avoiding patient injury or suffering ("Do no harm."). An example of nonmaleficence is deciding not to perform a procedure that might put a patient at risk. This principle may be outweighed in certain situations by the benefits of "harming" the patient (e.g., in most surgeries). Although there are potential risks associated with blood transfusions (e.g., transfusion reactions), this physician has decided to continue with the treatment, thereby not following the principle of nonmaleficence.

78-year-old right-handed man with hypertension and hyperlipidemia is brought to the emergency department for sudden onset of nausea and vertigo one hour ago. Physical examination shows 5/5 strength in all extremities. Sensation to light touch and pinprick is decreased in the right arm and leg. A CT scan of the brain shows an acute infarction in the distribution of the left posterior cerebral artery. Further evaluation of this patient is most likely to show which of the following findings?

The posterior cerebral artery supplies the occipital lobe. Right-sided homonymous hemianopia 68% The posterior cerebral artery supplies parts of the thalamus and the visual cortex within the occipital lobe. In addition to contralateral hemisensory loss due to lateral thalamic infarction (as seen in this patient), occlusion to the posterior cerebral artery causes damage to the visual cortex that results in contralateral homonymous hemianopia with macular sparing. Central vision is not impaired in posterior cerebral artery infarction since the corresponding cortical area of the macula is supplied by the middle cerebral artery.

An otherwise healthy 58-year-old man comes to the physician because of a 1-year history of episodic coughing whenever he cleans his left ear. There is no history of hearing loss, tinnitus, or vertigo. Stimulating his left ear canal with a cotton swab triggers a bout of coughing. The physician informs him that these symptoms are caused by hypersensitivity of a cranial nerve. A peripheral lesion of this nerve is most likely to manifest with which of the following findings on physical examination?

The posterior external auditory canal is innervated by the auricular branch of the vagus nerve (CN X). In some individuals, this nerve can trigger an oto-respiratory reflex, causing coughing. Ipsilateral vocal cord palsy 43% Ipsilateral vocal cord palsy would occur in a lesion of the recurrent laryngeal nerve, which is a branch off the vagus nerve (CN X). Paralysis of the vagus nerve could also lead to loss of the gag reflex, dysphagia, decreased taste from the supraglottic region, as well as ipsilateral paralysis of the soft palate, leading to deviation of the uvula away from the side of the lesion. Ipsilateral deviation of the uvula 28% Contralateral (not ipsilateral) deviation of the uvula would be caused by a lesion of the glossopharyngeal nerve (CN IX) or the vagus nerve (CN X) due to paralysis of the soft palate. CN IX palsy could also lead to dysphagia, loss of the gag reflex, and sensory loss over the soft palate, upper pharynx, and posterior third of the tongue.

A 7-year-old boy is brought to the physician for recurrent 3-4 minutes episodes of facial grimacing and staring over the past month. He is nonresponsive during these episodes and does not remember them afterward. He recalls a muddy taste in his mouth before the onset of symptoms. One week ago, his brother witnessed an episode where he woke up, stared, and made hand gestures. After the incident, he felt lethargic and confused. Examination shows no abnormalities. Which of the following is the most likely diagnosis?

The presence of a muddy taste preceding the onset of motor symptoms suggests that the pathology arises within the mesial temporal lobe. Complex partial seizure 45% This patient's presentation is most consistent with recurring complex partial seizures (focal seizures with impaired awareness), which result from abnormal electrical activity arising from the temporal lobe. The gustatory aura (muddy taste) marks the onset of the seizure. The ictal phase consists of his unresponsive staring and automatisms (e.g., facial grimacing and hand gestures). The postictal phase is characterized by lethargy, temporary confusion, and amnesia.

A 23-year-old woman comes to the physician because of an 8-month history of weakness and intermittent burning pain in her neck, shoulders, and arms. She was involved in a motor vehicle collision 1 year ago. Examination of the upper extremities shows absent reflexes, muscle weakness, and fasciculations bilaterally. Sensation to temperature and pain is absent; vibration and proprioception are preserved. The pupils are equal and reactive to light. Which of the following is the most likely diagnosis?

The presence of pain along with sensory deficits to pain and temperature in a cape-like distribution over the neck and upper extremities while sparing more distal sensory levels suggests injury to decussating fibers in the cervical and upper thoracic spinal cord. A history of motor vehicle collision and progressive symptoms point to a traumatic etiology. Syringomyelia 78% Post-traumatic syringomyelia can occur months to years after whiplash-type injury to the neck and presents with the classical findings of a central cord syndrome. Patients develop a symmetric cape-like distribution of pain and temperature sensory loss around the upper trunk, shoulders and upper limbs. Vibration and position sense are selectively spared because fibers of the posterior columns decussate higher up in the medulla. As syringomyelia progresses, lower motor neuron weakness develops symmetrically in the upper limbs (evident by areflexia and fasciculations in this patient) because the fluid-filled cavity begins to compress the anterior horn cells of the spinal cord at the level of the lesion. Syringomyelia is commonly associated with Chiari type I malformation.

An otherwise healthy 17-year-old girl comes to the physician because of multiple patches on her face, hands, abdomen, and feet that are lighter than the rest of her skin. The patches began to appear 3 years ago and have been gradually increasing in size since. There is no associated itchiness, redness, numbness, or pain. She emigrated from India 2 years ago. An image of the lesions on her face is shown. Which of the following is most likely involved in the pathogenesis of this patient's skin findings?

The presence of sharply demarcated, depigmented skin macules on otherwise normal skin in a patient with no further symptoms is highly suggestive of vitiligo. Autoimmune destruction of melanocytes Vitiligo is a common skin condition that is thought to be caused by autoimmune destruction of melanocytes. The areas of depigmentation commonly involve the face (e.g., perioral and periocular regions), neck, scalp, acral surfaces (e.g., hands), extensor surfaces, and genitalia. Vitiligo is generally a clinical diagnosis. It may be a sign of an autoimmune disorder (e.g., autoimmune thyroiditis, type 1 diabetes mellitus, pernicious anemia), and patients with vitiligo should be tested for thyroid dysfunction and markers of autoimmune disease (e.g., ANA).

A previously healthy 38-year-old man comes to the emergency department for evaluation of several hours of severe, right-sided, colicky flank pain that radiates to the groin. He drinks 2-3 energy drinks daily. An x-ray of the abdomen shows a 4-mm radiopaque mass in the right ureter. Microscopic examination of the urine shows biconcave, dumbbell-shaped crystals. Which of the following is most likely to prevent future episodes of flank pain?

The radiopaque mass in the area of the ureter and biconcave dumbbells on urinalysis confirm the diagnosis of calcium oxalate stones. Calcium oxalate stones A common type of kidney stone that can occur due to hypercalciuria, hyperoxaluria, and hypocitraturia. On urinalysis, they appear as biconcave dumbbells or bipyramidal envelopes. Radiopaque on imaging. Associated with low urinary pH. Preventive measures include dietary modification, urine alkalinization, and thiazide diuretics. Maintain adequate hydration 80% Inadequate hydration causes supersaturation of solutes in urine, which increases the risk of stone formation. Fluid intake of ≥ 2.5 L/day reduces the risk of recurrent nephrolithiasis because it decreases the urinary solute concentration and increases the frequency of urination. Adequate hydration prevents recurrence of all types of urinary stones. This patient should also be advised against excessive consumption of energy drinks, which often contain oxalate and contribute to the formation of calcium oxalate stones. Restrict dietary calcium intake 6% Restricting normal dietary calcium intake (approx. 1000 mg/day) enhances the intestinal absorption of oxalate. This increases urinary oxalate excretion and the risk of recurrent calcium oxalate stones. In addition, prolonged low dietary calcium can decrease bone mineral density and cause osteoporosis. However, supplemental calcium intake (above the normal daily requirements) should be restricted in patients with calcium stones.

An investigator is studying the metabolism of an experimental drug that is known to have first order kinetics. Immediately after administering an intravenous dose of the drug to a patient, the serum concentration is 60 U/L. 3 hours later, the serum concentration of the drug is 30 U/L. 9 hours after administration, the serum concentration of the drug is most likely to be which of the following?

The reduction of drug concentration from 60 U/L to 30 U/L indicates a half-life of 3 hours. 7.5 U/L 81% Medications with first order kinetics are eliminated at a rate directly proportional to their concentration. This medication has a half-life of 3 hours. After 9 hours, the concentration would have halved three times (60 U/L → 30 U/L → 15 U/L → 7.5 U/L). First order kinetics A chemical property of a drug in which the rate of drug metabolism (or elimination) is directly proportional to the plasma concentration of the drug (e.g., there is a constant fraction of drug being eliminated per unit of time).

A 62-year-old man is brought to the emergency department because of a 2-hour history of left-sided body weakness and involuntary urinary leakage. He has a 15-year history of type 2 diabetes mellitus and hyperlipidemia treated with metformin and atorvastatin. He reports having experienced a similar episode two years ago. Neurologic examination shows left-sided hemiparesis and decreased sensation. The neurologic abnormalities are more prominent in the left lower than in the left upper extremity. He is oriented to time, place, and person. Which of the following labeled vessels in the MR angiography image of a normal brain is most likely occluded in this patient?

The right anterior cerebral artery (structure A) supplies the right anteromedial cortex including the medial portion of the frontal and parietal lobes. ACA occlusion causes loss of contralateral motor function, particularly of the lower extremities, and contralateral sensory loss. The ACA also supplies the medial frontal micturition center, the part of the brain responsible for bladder control.

A 35-year-old man is brought to the emergency department 40 minutes after spilling hot oil over himself in a kitchen accident. Examination shows multiple tense blisters over the abdomen, anterior chest, and anterior and posterior aspects of the right upper extremity and right thigh. On deroofing the blisters, the skin underneath is tender, mottled, and sluggishly blanches with pressure. The skin over the left thigh is tender, erythematous, and shows quick capillary refill after blanching with pressure. Which of the following most closely approximates the body surface area affected by 2nd-degree burns in this patient?

The rule of nines divides the body into 11 regions (head, anterior chest, abdomen, upper back, lower back, arms x2, thighs x2, lower legs x2), each of which is attributed a value of 9% body surface area. This patient has deep 2nd-degree burns, which are characterized by blister formation with nonblanching underlying skin, on his chest, abdomen, right arm, and right thigh, which corresponds to 36% of total body surface area. He also has a 1st-degree burn on the left thigh.

A 25-year-old man is brought to the emergency department 30 minutes after he was involved in a motorcycle collision. He was not wearing a helmet. Physical examination shows left periorbital ecchymosis. A CT scan of the head shows a fracture of the greater wing of the left sphenoid bone with compression of the left superior orbital fissure. Physical examination of this patient is most likely to show which of the following findings?

The superior orbital fissure contains cranial nerves III, IV, V1, and VI. The superior orbital fissure contains cranial nerves III, IV, V1, and VI. Absent left corneal reflex 54% The corneal reflex requires uninterrupted signal transmission via the ophthalmic nerve (cranial nerve V1). Since this structure passes through the superior orbital fissure, it is likely to be disrupted in this patient with a compressive fracture through the superior orbital fissure. Without the afferent limb of the reflex pathway (cranial nerve V1), there is no motor response (cranial nerve VII).

A 35-year-old man comes to the emergency department with acute shortness of breath that developed after a 10-hour international flight. His pulse is 124/min and pulse oximetry on room air shows an oxygen saturation of 90%. He weighs 50-kg (110-lb). A diagnosis of pulmonary embolism is suspected and intravenous heparin is initiated. If the volume of distribution of heparin is equivalent to 60 mL/kg and the target peak plasma concentration is 0.5 units/mL, which of the following is the most appropriate loading dose for this patient?

The total volume of distribution for this patient is 60 mL/kg x 50 kg = 3,000 mL. 1,500 units 64% Loading dose is equal to (Cp x Vd) / F, where Cp = peak plasma concentration in units/mL, Vd = volume of distribution (only the total volume), and F = bioavailability. In this patient, the bioavailability of heparin is 1 as it is an intravenous medication. This yields (0.5 units/mL x 3,000 mL)/1 = 1,500 units.

A 74-year-old man comes to the physician because of a 5-month history of lower extremity weakness that improves with exercise. He has difficulty climbing up the stairs to his apartment. He started using a psyllium fiber supplement 2 months ago for constipation. He has smoked 2 packs of cigarettes daily for the past 45 years. Examination shows dry mucous membranes. Muscle strength in the lower extremities is decreased, and deep tendon reflexes are 1+ bilaterally. Repeated muscle tapping increases reflex activity. Further evaluation is most likely to show which of the following?

This patient presents with weakness of the lower legs that improves with exercise, in addition to autonomic dysfunction (e.g., dry mouth, constipation) and decreased reflexes. Together with a positive smoking history, these features suggest a diagnosis of Lambert-Eaton myasthenic syndrome.

A 51-year-old man comes to the physician because of recurrent episodes of dizziness, tinnitus, and hearing loss on the left side for 6 weeks. These episodes last for hours at a time and are associated with the sensation that the room is spinning. He has no history of major medical illness and takes no medications. Examination shows horizontal nystagmus to the left. Weber test shows lateralization to the right ear. The Rinne test is positive bilaterally. Which of the following is the most likely cause of this patient's symptoms?

The triad of vertigo, tinnitus, and sensorineural hearing loss (normal Rinne test, lateralization to the unaffected side) is characteristic of Ménière disease. Endolymphatic hydrops 48% Endolymphatic hydrops, which results in accumulation of endolymph within the membranous labyrinth, is the underlying mechanism of Ménière disease. In addition to vertigo, tinnitus, and sensorineural hearing loss, patients often have ear fullness and nystagmus that can be directed towards the affected side (irritative nystagmus) or the healthy side (reversed nystagmus). Ménière's disease (MD) is a disorder of the inner ear that is characterized by episodes of feeling like the world is spinning (vertigo), ringing in the ears (tinnitus), hearing loss, and a fullness in the ear.[3][4] Typically, only one ear is affected initially; however, over time both ears may become involved.[3] Episodes generally last from 20 minutes to a few hours.[5] The time between episodes varies.[3] The hearing loss and ringing in the ears can become constant over time.[4] The cause of Ménière's disease is unclear but likely involves both genetic and environmental factors.[1][3] A number of theories exist for why it occurs including constrictions in blood vessels, viral infections, and autoimmune reactions.[3] About 10% of cases run in families.[4] Symptoms are believed to occur as the result of increased fluid build up in the labyrinth of the inner ear.[3] Diagnosis is based on the symptoms and, frequently, a hearing test.[3] Other conditions that may produce similar symptoms include vestibular migraine and transient ischemic attack.[1]

A 61-year-old man comes to the physician because of a 3-month history of worsening exertional dyspnea and a persistent dry cough. For 37 years he has worked in a naval shipyard. He has smoked 1 pack of cigarettes daily for the past 40 years. Pulmonary examination shows fine bibasilar end-expiratory crackles. An x-ray of the chest shows diffuse bilateral infiltrates predominantly in the lower lobes and pleural reticulonodular opacities. A CT scan of the chest shows pleural plaques and subpleural linear opacities. The patient is most likely to develop which of the following conditions?

The x-ray findings of diffuse bilateral infiltrates predominantly in the lower lobes and pleural reticulonodular opacities, along with the CT finding of pleural plaques, is a classic radiographic presentation of asbestosis. Bronchogenic carcinoma 63% Bronchogenic carcinoma is the most common malignant pulmonary tumor in patients with asbestosis and the second most common carcinoma worldwide. This patient has two important risk factors, primarily a smoking history and a history of asbestos exposure. The first symptom of bronchogenic carcinoma is usually a cough, although hemoptysis, dyspnea, and chest pain can also occur. Asbestosis is the main cause of mesothelioma, which rarely occurs without a history of asbestos exposure. However, bronchogenic carcinoma is still vastly more common than mesothelioma, even in people with a history of asbestos exposure.

A previously healthy 46-year-old woman comes to the physician with a one-week history of productive cough and fatigue. Two weeks ago, she had fever, nasal congestion, rhinorrhea, and myalgias that resolved with supportive care. She has not traveled out of the United States. Pulmonary examination shows dullness to percussion and increased fremitus at the right middle lobe. An x-ray of the chest is shown. A sputum sample is most likely to show which of the following findings?

This patient's chest x-ray shows a large irregularly rounded cavity with an air-fluid level in the right middle lobe, indicating a lung abscess.

theca interna cells

Theca interna cells express receptors for luteinizing hormone (LH) to produce androstenedione, which via a few steps, gives the granulosa the precursor for estrogen manufacturing. After rupture of the mature ovarian follicle, the theca interna cells differentiate into the theca lutein cells of the corpus luteum. Theca lutein cells secrete androgens[1] and progesterone. Theca lutein cells are also known as small luteal cells Theca interna cells 54% Polycystic ovary syndrome (PCOS) is characterized by an increase in the secretion of pituitary luteinizing hormone (LH), which stimulates ovarian theca interna cells to produce androgens. The resulting hyperandrogenism accounts for the typical clinical features of PCOS: cutaneous manifestations (hirsutism, androgenic alopecia, and acne vulgaris), anovulation/oligoovulation (often resulting in infertility), and polycystic ovaries on ultrasound. A majority of affected women also have hyperinsulinism and/or insulin resistance, leading to symptoms such as acanthosis nigricans and overweight, which are seen in this patient. Polycystic ovary syndrome Stein-Leventhal syndrome, Hyperandrogenic anovulation Abbreviation: PCOS An endocrinologic disorder characterized by amenorrhea, infertility, obesity, virilization and presence of polycystic ovaries. Although the exact pathophysiology is unknown, it is thought to be due to reduced peripheral insulin sensitivity, causing excess androgen production.

An autopsy is performed on a 39-year-old man 5 days after he was found pulseless at his apartment by his neighbor. Examination of the brain shows liquefactive necrosis in the distribution of the right middle cerebral artery with surrounding edema. Immunophenotyping of a sample of the affected brain tissue shows numerous cells that express CD40 on their surface. On further histopathological evaluation, the morphology of these cells is not readily discernible with Nissl stain. These histological findings are most consistent with which of the following cell types?

These CD40 positive cells in the brain are derived from the mesoderm and are activated in response to tissue damage. The gross findings present on autopsy suggest an ischemic stroke as a cause of his death. Microglia are responsible for the removal of cellular debris and apoptotic cells in the brain parenchyma and can be seen within 3-5 days after an injury, such as in the case of the ischemic stroke seen in this patient. Following activation, microglia express CD40 and other markers, including CD11b, CD68, and CD45.

cadaveric

These donations (for transplants) are from individuals after they are pronounced clinically dead.

atrial fibrillation

This arrhythmia occurs due to multiple electrical impulses leaving the atrium. Arteriosclerosis and other diseases can cause scarring of the atrium. This scar tissue becomes "irritable" and begins to send out many impulses across the atria. The ECG tracing will not show a distinct P wave since the rate is extremely rapid (350-600). The P waves appear as an atrial wave or in other words, wavering lines with no distinct P wave visible (see figure below). The patient's pulse is irregular. Since the P waves come so rapidly and at irregular intervals, the ventricular response is irregular and so is the pulse usually.

A 6-year-old boy is brought to the physician because of a 2-week history of frequent episodes of unresponsiveness. During these episodes, he stares blankly, rhythmically nods his head, and does not respond to verbal stimulation for several seconds. Hyperventilation for 30 seconds precipitates an episode of unresponsiveness and head nodding that lasts for 7 seconds. He regains consciousness immediately afterward. An electroencephalogram shows 3-Hz spikes and waves. Which of the following best describes the mechanism of action of the most appropriate pharmacotherapy for this patient's condition?

This boy has childhood absence epilepsy. Absence seizures typically manifest with episodes of staring, inattentiveness, and head nodding and can be triggered by hyperventilation. Patients typically maintain muscle tone during seizures and do not experience a postictal phase. The diagnosis is confirmed with EEG findings of a 3-Hz spike and wave pattern. First-line therapy is ethosumide Blockade of thalamic T-type calcium channels 74% Blockade of T-type calcium channels in the thalamus is the mechanism of action of ethosuximide, which is the first-line therapy for seizure prophylaxis in patients with childhood absence epilepsy. T-type calcium channels are thought to be responsible for the rhythmic 3-Hz discharge seen on EEG during absence seizures.

A 51-year-old woman comes to the physician because of progressively worsening lower back pain. The pain radiates down the right leg to the lateral side of the foot. She has had no trauma, urinary incontinence, or fever. An MRI of the lumbar spine shows disc degeneration and herniation at the level of L5-S1. Which of the following is the most likely finding on physical examination?

Weak achilles tendon reflex A lumbar disc herniation between the vertebral bodies L5 and S1 is a common cause of S1 radiculopathy, which can manifest with a weak achilles tendon reflex due to compression of the S1 spinal nerve root. The S1 spinal nerve is also responsible for sensation of the lateral aspect of the calf and foot, which explains this patient's radiating pain, as well as for plantar flexion and eversion, which can result in impaired toe walking.

A 6-year-old boy is brought to the physician by his parents because of right lower extremity weakness, worsening headaches, abdominal pain, dark urine, and a 5-kg (11-lb) weight loss for the past 2 months. His teachers report that he has not been paying attention in class and his grades have been worsening. He has a history of infantile seizures. Physical examination shows a palpable abdominal mass and left costovertebral angle tenderness. Neurological exam shows decreased strength of the right lower limb. He has several acne-like angiofibromas around the nose and cheeks. Further evaluation is most likely to show which of the following?

This boy's combination of facial angiofibromas (adenoma sebaceum), signs of angiomyolipomas (abdominal mass, macrohematuria), a history of infantile seizures, and signs of a potential brain tumor (i.e., focal neurologic deficits indicated by decreased limb strength, declining performance, poor attention) indicates tuberous sclerosis. Subependymal giant cell astrocytoma Subependymal giant cell astrocytoma is a brain tumor associated with tuberous sclerosis (TSC). Other common manifestations of TSC that this patient could develop include shagreen patches, ash-leaf spots, benign cysts, and renal cell carcinoma. Tuberous sclerosis is a rare disease that causes tumors, or growths, in the brain and other organs. These growths can occur in the skin, kidneys, eyes, heart , or lungs . They are usually benign (non-cancerous). The first signs of tuberous sclerosis may occur at birth. Other people develop symptoms over time.

A 6-year-old boy is brought to the physician by his mother because of a 3-month history of episodic chest pain and shortness of breath on exertion. He is at the 99th percentile for height and 40th percentile for weight. Examination shows a high-arched palate, long and slender upper extremities, and elbows and knees that can be hyperextended. Cardiac examination shows a grade 2/6 late systolic, crescendo murmur with a midsystolic click. Over which of the following labeled areas is the murmur most likely to be heard best?

This boy's tall stature, long and slender arms, joint hypermobility (as evidenced by his hyperextensible elbows and knees), and high-arched palate are consistent with Marfan syndrome, an inherited connective tissue disorder associated with ocular, musculoskeletal, and cardiovascular abnormalities. Area F 65% Patients with Marfan syndrome can have mitral valve prolapse (MVP), which causes a late systolic, crescendo murmur with a midsystolic click. This murmur is best heard at the heart apex (mitral area, located at the left 5th intercostal space in the midclavicular line) when the patient is in the left lateral recumbent position. Other cardiovascular abnormalities associated with Marfan syndrome include aortic disease (e.g., aortic root dilation, aortic regurgitation, aortic aneurysm, aortic dissection) and berry aneurysms. Mitral valve prolapse Abbreviation: MVP A valvular lesion characterized by bulging of a mitral valve leaflet into the left atrium during systole. Most commonly idiopathic but can also be associated with connective tissue disorders (e.g., Marfan syndrome) or occur as a complication of infective endocarditis, acute rheumatic heart disease, or myocardial infarction. On auscultation (best heard over the apex), typically characterized as a late systolic crescendo murmur with a midsystolic click due to sudden tensing of the chordae tendinae.

A 6-year-old boy is brought to the emergency department because of worsening confusion for the last hour. He has had high-grade fever, productive cough, fatigue, and malaise for the past 2 days. He has not seen a physician in several years. His temperature is 38.9°C (102°F), pulse is 133/min, respirations are 33/min, and blood pressure is 86/48 mm Hg. He is lethargic and minimally responsive. Mucous membranes are dry. Pulmonary examination shows subcostal retractions and coarse crackles bilaterally. Laboratory studies show a hemoglobin concentration of 8.4 g/dL and a leukocyte count of 16,000/mm3. A peripheral blood smear shows sickled red blood cells. Which of the following pathogens is the most likely cause of this patient's current condition?

This boy, who has sickle cell disease, has developed altered mental status, fever, hypotension, and leukocytosis; these features are concerning for a diagnosis of sepsis. The sepsis in this patient is most likely secondary to pneumonia, which caused his productive cough, tachypnea, and coarse crackles. Patients with sickle cell disease have an increased risk of infection by encapsulated organisms. Streptococcus pneumoniae 68% The spleen is important for the opsonization and elimination of encapsulated bacteria. Patients with sickle cell disease often develop functional asplenia by the age of 2-4 years due to repeated episodes of splenic infarction (autosplenectomy) and are therefore at increased risk for infections by encapsulated pathogens such as Streptococcus pneumoniae, Neisseria meningitidis, Haemophilus influenzae type b and Salmonella species. Of all the encapsulated pathogens, Streptococcus pneumoniae is the most common cause of community-acquired bacterial pneumonia and sepsis in patients with sickle cell disease.

A 9-year-old boy is brought to the physician for evaluation of 2 months of progressive clumsiness, falls, and increased urinary frequency. Physical examination shows bilateral temporal visual field loss. An MRI of the head shows a small calcified suprasellar mass. The patient undergoes surgery with complete removal of the mass. Pathological examination of the specimen shows a lobular tumor composed of cysts filled with oily, brownish-yellow fluid. This mass is most likely derived from which of the following structures?

This child's bitemporal hemianopia and the MRI showing a calcified suprasellar mass are highly suggestive of craniopharyngioma, the most common supratentorial tumor in children. The histopathologic findings of a cystic mass filled with dark, oily fluid ("motor oil-like") confirm this diagnosis. Rathke pouch 84% Craniopharyngiomas are derived from remnants of the Rathke pouch, which is an ectodermal outpouching of the pharyngeal roof that develops into the anterior pituitary gland. Compression of the optic chiasm by the craniopharyngioma would cause bitemporal hemianopsia and possibly, clumsiness or frequent tripping due to the visual field defect. Destruction of the posterior pituitary by the tumor would result in central diabetes insipidus, which explains the increased urinary frequency in this patient.

A 23-year-old man comes to the physician because of severe daytime sleepiness and random episodes of falling asleep during the day for short periods of time. When he wakes up from these episodes, he feels refreshed. He sleeps 6-7 hours at night. He reports that sometimes he is unable to move for a few minutes when he first wakes up in the morning. Last week, he suddenly "lost control" of his legs and neck for about 30 seconds after his classmate had told him a joke. An analysis of this patient's cerebral spinal fluid is most likely to show which of the following findings?

This patient's excessive daytime sleepiness, sleep attacks, sleep paralysis, and episode of cataplexy indicate narcolepsy. Decreased orexin-A 84% Orexin (hypocretin) is a neuropeptide that is released to increase the activity of brain regions involved in wakefulness, including the raphe nuclei and tuberomammillary nucleus and locus coeruleus. Orexin is decreased in the CSF levels of patients with narcolepsy due to a loss of orexigenic neurons in the lateral hypothalamus (type-1 narcolepsy).

A previously healthy, 16-year-old boy is brought to the emergency department with persistent bleeding from his gums after an elective removal of an impacted tooth. Multiple gauze packs were applied with minimal effect. He has a history of easy bruising. His family history is unremarkable except for a maternal uncle who had a history of easy bruising and joint swelling. Laboratory studies show: Hematocrit36%Platelet count170,000/mm3Prothrombin time13 secPartial thromboplastin time65 secBleeding time5 min (N = 2-7) Peripheral blood smear shows normal-sized platelets. Which of the following is the most likely diagnosis?

This disease primarily affects males. Hemophilia 85% Excessive bleeding in an otherwise healthy young male who easily bruises, with a probable family history of the disease in a maternal uncle and isolated PTT elevation is highly suspicious for hemophilia. Because hemophilia is an X-linked recessive disorder, it affects males almost exclusively, and ⅔ of patients have a positive family history in a maternal male relative. The history of easy bruising and joint swelling (likely hemarthrosis) in his maternal uncle is characteristic of hemophilia. Inherited hemophilias cause isolated factor VIII, IX, or XI deficiency (all part of the intrinsic coagulation pathway), presenting with isolated PTT (which measures intrinsic pathway function) elevation on lab workup. Partial thromboplastin time Abbreviation: PTT An assay that evaluates the intrinsic and common pathways of the coagulation cascade (measures activity of factors XII, XI, X, IX, VIII, V, II, and I). Used to evaluate coagulation disorders and to monitor the effect of heparin. Normal range is 25-40 seconds. For patients with deficiencies or defects of the intrinsic clotting cascade, the PTT will be elevated. Disorders with an elevated PTT include: Hemophilia A (Factor VIII deficiency) Hemophilia B (Factor IX deficiency) Von Willebrand disease 12% Certain types of von Willebrand disease can present with findings similar to those seen in this patient (mucosal bleeding, normal or ↑ PTT). However, von Willebrand disease also causes increased bleeding time, and it is autosomal dominant, meaning that one of the patient's parents would likely have a history of bleeding diathesis as well. Hemorrhagic diathesis An abnormal and increased susceptibility to bleeding, often due to hypocoagulability.

A 29-year-old primigravid woman at 18 weeks' gestation comes to the physician for her first prenatal visit. She works as a paralegal and lives with her husband. Her current pregnancy was unexpected, and she did not take any prenatal medications or supplements. Physical examination shows a uterus 2 inches above the umbilicus. The concentration of α-fetoprotein in the maternal serum and concentrations of both α-fetoprotein and acetylcholinesterase in the amniotic fluid are elevated. Ultrasonography of the uterus shows an increased amniotic fluid volume. The fetus most likely has which of the following conditions?

This fetus's condition results from failure of closure of the rostral neuropore during embryogenesis. The increased amniotic fluid volume (i.e., polyhydramnios) seen here is due to impaired fetal swallowing. Anencephaly is a lethal neural tube defect (NTD) characterized by an open cranial vault and absence of the forebrain. The most important risk factor associated with NTDs is folate deficiency, either due to dietary shortage or the consumption of folic acid antagonists (e.g., valproate, carbamazepine). Because anencephaly is an open NTD that leaves the neural tube exposed, α-fetoprotein (AFP) and acetylcholinesterase (AChE) can diffuse freely into the amniotic fluid and maternal serum. If both AFP and AChE are elevated in the amniotic fluid, fetal NTDs can be diagnosed with high accuracy, even if ultrasonography is inconclusive.

A 4-year-old girl is brought to the pediatrician because of throat pain for 2 days. She had frequent episodes of oral thrush as a baby and has been treated for vaginal yeast infection twice in the past year. Physical examination shows white patches on the tongue and palate. Scraping off the patches reveals erythematous oral mucosa with pinpoint bleeding. Leukocyte count is within normal range. Exposure to nitroblue tetrazolium turns the patient's neutrophils dark blue. A deficiency in which of the following enzymes is the most likely cause of this patient's recurrent infections?

This girl has had multiple Candida infections but no history of frequent bacterial or viral infections. A normal nitroblue tetrazolium test shows intact ability to reduce the yellow dye to a dark blue formazan. Nitroblue tetrazolium dye reduction test A diagnostic test used to assess phagocyte function. Considered normal (positive) if incubated neutrophils are able to reduce the dye to a blue color. Considered abnormal (negative) if no blue color is produced, which indicates that there is a defect in the production of reactive oxygen species (e.g., due to NADPH oxidase deficiency in patients with chronic granulomatous disease). Myeloperoxidase deficiency An autosomal recessive condition in which deficient or absent myeloperoxidase predisposes to recurrent Candida infections; more severe immunocompromise is rare. Diagnosis can be made with a positive (normal) nitroblue tetrazolium test and/or immunohistochemical or genetic testing

One week after starting amoxicillin for sinusitis, a 4-year-old girl is brought to the emergency department with fever, rash, and myalgia. She has been hospitalized multiple times for recurrent streptococcal pneumonia and meningitis. She appears tired. Examination shows a diffuse urticarial rash. Her antibiotic is discontinued. Which of the following is the most likely underlying mechanism for her recurrent infections?

This girl's history of recurrent streptococcal infection and current hypersensitivity reaction is concerning for C3 deficiency. Impaired opsonization Complement factor C3 is a precursor for C3b, which plays a major role in opsonization of pathogens and immune complexes. C3 deficiency results in increased susceptibility to infections by encapsulated bacteria, such as Streptococcus pneumoniae and Neisseria meningitidis. Patients with C3 deficiency classically present with recurring severe upper respiratory infections (e.g., sinusitis), pneumonia, and meningitis, as seen in this patient.

A 52-year-old man is brought to the emergency department 30 minutes after his farmhand found him on the ground sweating profusely. On arrival, he is lethargic and unable to provide any history. His temperature is 37.5°C (99.5°F), pulse is 42/min, and blood pressure is 95/60 mm Hg. Physical examination shows diaphoresis and excessive salivation. The pupils are constricted. There is scattered expiratory wheezing throughout both lung fields. His clothes are soaked with vomit, urine, and feces. A drug with which of the following mechanisms of action is most appropriate for this patient?

This man's symptoms (fecal and urinary incontinence, miosis, bronchospasm, bradycardia, salivation, diaphoresis) suggest organophosphate poisoning. Muscarinic acetylcholine receptor antagonism 74% Atropine is a muscarinic acetylcholine receptor antagonist that is considered the antidote of choice for organophosphate poisoning, which is likely given this patient's symptoms of cholinergic crisis and history of farm work. Organophosphates are commonly used pesticides that cause irreversible inhibition of acetylcholinesterase, resulting in increased acetylcholine levels and both muscarinic and nicotinic receptor activation. Atropine competitively inhibits the effects of organophosphates at the muscarinic receptors.

A 3-week-old male infant is brought to the physician for follow-up. He was delivered at 30 weeks' gestation via cesarean section and was cyanotic at birth, requiring resuscitation and a neonatal intensive care unit hospitalization. His mother received no prenatal care; she has diabetes mellitus type II and hypertension. She was not tested for sexually transmitted infections during the pregnancy. The infant appears well. Ophthalmologic examination shows tortuous retinal vessels. There are well-demarcated areas of nonvascularized retina in the periphery. This patient's retinal findings are most likely a result of which of the following?

This infant's ophthalmologic examination revealed areas of nonvascularized retina with tortuous vessels, which are consistent with retinopathy of prematurity (ROP). Oxygen toxicity 65% This premature infant likely received supplemental oxygen at birth, which is a major contributing factor in the development of ROP. Higher arterial oxygen tension in a premature infant inhibits the production of VEGF, leading to apoptosis of endothelial cells and cessation of normal retinal vascular formation. Switching to room air results in relative hypoxia due to the lower oxygen concentration, increased metabolic demand of the growing retina, and poor vascular network, leading to the production of proangiogenic growth factors (e.g., VEGF) and uncontrolled neovascularization. The formation of new blood vessels can cause hemorrhages, form fibrovascular membranes, and ultimately lead to retinal detachment or blindness. Chlamydia infection 8% Mothers receiving routine prenatal care are screened for chlamydia infection to eliminate the risk of chlamydial conjunctivitis in the newborn. This mother's lack of prenatal care puts the infant at risk of this disease, which often manifests 1-2 weeks after birth. However, ocular findings include mucopurulent or watery discharge, eyelid swelling, and chemosis. This disease does not affect the retina. Syphilis infection 7% Syphilis is one of several congenital infections (TORCH infections) that can cause ocular abnormalities in affected infants. Congenital syphilis can manifest with uveitis, keratitis, glaucoma, and congenital cataracts. However, this infant's fundoscopy findings are highly indicative of ROP, and, although infection may worsen the course of ROP, it is not a contributing factor to the development of this condition.

A 48-year-old man comes to the emergency department because of a 2-hour history of severe left-sided colicky flank pain that radiates towards his groin. He has vomited twice. Last year, he was treated with ibuprofen for swelling and pain of his left toe. He drinks 4-5 beers most days of the week. Examination shows left costovertebral angle tenderness. An upright x-ray of the abdomen shows no abnormalities. A CT scan of the abdomen and pelvis shows an 9-mm stone in the proximal ureter on the left. Which of the following is most likely to be seen on urinalysis?

This is a radiolucent ureteral stone in a patient with a likely history of gout (pain of the left toe indicating podagra), which occurs when an excess of uric acid in the blood builds up in joints and is excerbated by alcohol consumption. The same process may also lead to the formation of uric acid stones under certain conditions. Rhomboid-shaped crystals 68% Rhomboid or needle-shaped crystals on urinalysis are characteristic of uric acid stones, which usually form in acidic urine as a result of uric acid precipitation in the renal tubules. Only uric acid forms radiolucent stones, which is most consistent with this patient's findings.

A 59-year-old man is brought to the physician by his wife for a psychiatric evaluation. Over the past 12 months, his behavior has become increasingly disruptive. His wife no longer brings him along shopping because he has attempted to grope a female cashier on 2 occasions. He has begun to address the mail carrier using a racial epithet. Three years later, the patient dies. Light microscopy of sections of the frontal and temporal lobes shows intracellular inclusions of transactive response DNA binding protein (TDP-43). These proteins are bound to a regulatory molecule that usually marks them for degradation. The regulatory molecule in question is most likely which of the following?

This man had frontotemporal dementia, which causes changes in personality and social behavior. The postmortem histopathologic findings of intracellular inclusions of TDP-43 in the frontal and temporal lobes confirms the diagnosis. Binding of the regulatory molecule in question to proteins such as TDP-43 normally results in transport of the proteins to the proteasome. Ubiquitin 79% Tagging with ubiquitin marks proteins for degradation by the ubiquitin-proteasome system (UPS). Dysfunction of the UPS is implicated in various neurodegenerative brain diseases, including frontotemporal dementia, as seen here, as well as Alzheimer disease, Parkinson disease, and Huntington disease.

A 68-year-old man is brought to the emergency department 30 minutes after the onset of uncontrollable jerking movements of his arms and legs followed by loss of consciousness. His wife says that he seemed confused this morning and had a headache. Immediately before the shaking episode, he said that he smelled rotten eggs. He is unresponsive. Cerebrospinal fluid (CSF) analysis shows a leukocyte count of 700/μL (70% lymphocytes), a glucose concentration of 60 mg/dL, and a protein concentration of 80 mg/dL. Despite appropriate lifesaving measures, the man dies. Which of the following is most likely to be found on postmortem examination of this patient?

This man had viral encephalitis, which manifests with altered mental status, seizures and lymphocytic pleocytosis with normal glucose concentration and mildly elevated protein concentration on CSF analysis. Necrosis of the temporal lobes 65% Necrosis of the temporal lobes is a relatively specific finding in herpes simplex encephalitis, which is most commonly caused by HSV-1. Damage to the temporal lobe can cause clinical features such as olfactory hallucinations and aphasia. Biopsy of the lesion would show intranuclear eosinophilic inclusions, which is characteristic of herpes virus infections. HSV encephalitis is fatal in up to 70% of cases if left untreated; intravenous acyclovir is the first-line therapy.

A 45-year-old man is brought to the emergency department following a motor vehicle collision. He reports right hip pain and numbness along the right thigh. Physical examination shows decreased sensation to light touch over a small area of the medial thigh. X-rays of the pelvis show a displaced pelvic ring fracture. Further evaluation of this patient is most likely to show which of the following findings?

This man has a pelvic ring fracture with decreased sensation over a small area of the right medial thigh. This is consistent with an obturator nerve injury. Pelvic ring fracture is a common cause of obturator nerve injury. In addition to providing sensory cutaneous innervation to a small area of the medial thigh, the obturator nerve also provides motor innervation to the hip adductor muscles in the medial compartment of the thigh. Thus, injury to the nerve can result in impaired adduction of the hip.

A 47-year-old woman comes to the physician because of progressive pain and stiffness in her hands and wrists for the past several months. Her hands are stiff in the morning; the stiffness improves as she starts her chores. Physical examination shows bilateral swelling and tenderness of the wrists, metacarpophalangeal joints, and proximal interphalangeal joints. Her range of motion is limited by pain. Laboratory studies show an increased erythrocyte sedimentation rate. This patient's condition is most likely associated with which of the following findings?

This middle-aged woman has classical symptoms of rheumatoid arthritis. She has morning stiffness that improves with activity and polyarticular arthritis (indicated by swelling and tenderness) that involves small joints but spares the DIPs. In addition to non-specific parameters (e.g., increased erythrocyte sedimentation rate), a classic serological finding in some patients is the rheumatoid factor. IgM antibodies against the Fc region of IgG 69% Rheumatoid factors (RF) are a group of autoantibodies, typically IgM, that target the Fc region of IgG. Although elevated RF is classically associated with RA, it has a low specificity, since RF elevation may be physiological or caused by other autoimmune and infectious diseases. The level of RF in serum typically increases over the course of RA.

A 2-day-old male newborn is brought to the physician because he became somnolent and felt cold after breastfeeding. Pregnancy and delivery were uncomplicated. He was born at 40 weeks' gestation and weighed 3538 g (7 lb 13 oz); he currently weighs 3311 g (7 lb 5 oz). Examination shows generalized hypotonia. Serum studies show an ammonia concentration of 150 μmol/L (N < 50). Fasting serum glucose concentration is normal. Accumulation of which of the following substances is most likely involved in the pathogenesis of this patient's neurological symptoms?

This neonate's hyperammonemia after feeding is most likely due to a urea cycle disorder. Ammonia is metabolized through the actions of glutamate dehydrogenase and glutamine synthetase. Glutamine 48% Hyperammonemia results in the accumulation of glutamine, which plays an important role in the development of hepatic encephalopathy. Upon activation, neurons in the brain release neurotransmitters (e.g., glutamate, GABA) that are normally taken up from the synaptic cleft and recycled by astrocytes. In states of hyperammonemia, excess ammonia crosses the blood-brain barrier and enters into astrocytes. The astrocytes detoxify excess ammonia through the actions of glutamate dehydrogenase and glutamine synthetase. The consequent production of excess glutamine creates an osmotic effect that draws water into astrocytes and causes cellular swelling and dysfunction, resulting in cerebral edema that manifests with neurological symptoms such as those seen here.

A 5-day-old boy is brought to the emergency department because of a 1-day history of poor feeding, irritability, and noisy breathing. The mother did not receive any prenatal care. His respirations are 26/min. Physical examination shows sunken fontanelles, tightly clenched fists, and erythema around the umbilical cord stump. Which of the following best describes the pathogenesis of the disease process in this patient?

This newborn has several risk factors (lack of maternal prenatal care, umbilical cord stump infection) that should raise concern for neonatal tetanus. Neonatal tetanus is caused by tetanospasmin, which is produced by ubiquitous Clostridium tetani spores that then contaminate wounds (e.g., umbilical cord). The toxin reaches the CNS via retrograde axonal transport and blocks the release of glycine and GABA from inhibitory interneurons (Renshaw cells). This causes uncontrolled activation of alpha motor neurons, which results in tonic and clonic spasms (difficulty opening the mouth and feeding due to trismus, clenched hands).

A 2980-g (6.6-lb) female newborn is brought to the emergency department by her mother because of worsening lethargy. The newborn was delivered at home 10 hours ago. The mother has had no prenatal care. The newborn's temperature is 39.7°C (103.5°F). Physical examination shows scleral icterus. Her leukocyte count is 36,000/mm3 (85% segmented neutrophils). An organism is isolated from the blood. When grown together on sheep agar, the isolated organism enlarges the area of clear hemolysis formed by Staphylococcus aureus. Which of the following is the most likely causal organism?

This newborn's fever, lethargy, and elevated leukocyte count are concerning for neonatal sepsis, which is often accompanied by jaundice and dehydration. The causal organism produces CAMP factor, a protein that enhances the effect of β-lysin of S. aureus, thereby enlarging the area of clear hemolysis. CAMP factor CAMP test A pore-forming protein produced by group B streptococcus (GBS, or S. agalactiae). Not essential for virulence but can be used to identify GBS, as it enlarges the hemolysis area in cultures formed by S. aureus. This factor should not be confused with cAMP (cyclic adenosine monophosphate). CAMP is an acronym for Christie Atkins and Munch Peterson, the authors of the test. Streptococcus agalactiae 81% Streptococcus agalactiae (i.e., group B streptococcus (GBS)) are β-hemolytic, gram-positive cocci that colonize the vaginal mucosa and are among the most common pathogens of both early- and late-onset neonatal sepsis. Pregnant women are routinely screened at 35-37 weeks of gestation with rectal and vaginal swabs. Transmission of GBS to neonates can be prevented via administration of intrapartum penicillin prophylaxis. This patient's lack of prenatal care increased her child's risk of GBS sepsis. Neonatal sepsis A type of sepsis that occurs within the first month of life. Most commonly caused by group B Streptococcus and E. coli. Early-onset neonatal sepsis (within 1 week of birth) is typically caused by vertical or perinatal transmission to the fetus. Late-onset neonatal sepsis (> 1 week after birth) is typically a nosocomial infection.

Two hours after undergoing allogeneic kidney transplantation for polycystic kidney disease, a 14-year-old girl has lower abdominal pain. Examination shows tenderness to palpation in the area the donor kidney was placed. Ultrasound of the donor kidney shows diffuse tissue edema. Serum creatinine begins to increase and dialysis is initiated. Which of the following is the most likely cause of this patient's symptoms?

This patient appears to be rejecting the donor kidney. When rejection occurs less than 48 hours after transplantation, it is termed hyperacute rejection. Preformed antibodies against class I HLA molecules Hyperacute graft rejection is mediated by preformed cytotoxic antibodies against donor class I HLA molecules or donor blood group antigens (type 2 hypersensitivity reactions). Antigen-antibody binding activates the complement system, leading to thrombus formation and graft ischemia. Patients present with graft swelling and deterioration of organ function within 48 hours of the transplant. Hyperacute rejections can be prevented by cross-matching and ABO group matching before organ transplantation.

A 53-year-old woman is brought to the physician by her husband for the evaluation of progressive memory loss, which he reports began approximately 2 weeks ago. During this time, she has had problems getting dressed and finding her way back home after running errands. She has also had several episodes of jerky, repetitive, twitching movements that resolved spontaneously. She is oriented only to person and place. She follows commands and speaks fluently. She is unable to read and has difficulty recognizing objects. Which of the following is the most likely underlying cause of this patient's symptoms?

This patient has Creutzfeldt-Jakob disease, which causes myoclonus (jerky, repetitive, twitching movements) and rapidly-progressive dementia associated with aphasia and agnosis. Mutant prion accumulation Creutzfeldt-Jakob disease (CJD) is a neurodegenerative disorder caused by cerebral accumulation of misfolded protein particles called prions. Normal prion proteins (PrPc) are found on the surface of cortical neurons and are protective against free radicals. Misfolded prion proteins (PrPSc) have more beta-sheets in their secondary structures, which changes their conformation. These misfolded proteins accumulate into plaques, causing neuronal cell death and spongiform encephalopathy. As there is no curative treatment for CJD, management is supportive and most patients die within 12 months of diagnosis.

A 62-year-old man comes to the physician because of tremors in both hands for the past few months. He has had difficulty buttoning his shirts and holding a cup of coffee without spilling its content. He has noticed that his symptoms improve after a glass of whiskey. His maternal uncle began to develop similar symptoms around the same age. He has bronchial asthma controlled with albuterol and fluticasone. Examination shows a low-amplitude tremor bilaterally when the arms are outstretched that worsens during the finger-to-nose test. Which of the following is the most appropriate pharmacotherapy in this patient?

This patient has a bilateral low-amplitude tremor that worsens with purposeful voluntary movement (e.g., finger-to-nose test). Improvement with alcohol and a family history of similar symptoms makes essential tremor the most likely diagnosis. Essential tremor The most common form of tremor. Manifests as a (usually bilateral) tremor of the hands (about 90%), head (about 30%), and/or voice (about 15%) with a frequency of 5-10 Hz/s. Worse with voluntary movements (e.g., reaching for a glass), stress, fatigue, and caffeine; improves with alcohol consumption and resolves at rest. Most frequently autosomal dominant but may also occur sporadically. Primidone 34% Primidone is the first-line pharmacotherapy for patients with essential tremor. Treatment should be offered to patients in whom the condition causes intermittent or persistent impairment. Although the severity of essential tremor typically remains stable over time, some patients experience disability from tremor progression. Primidone is also used as an anticonvulsant in the treatment of grand mal, psychomotor, and focal seizures. Its primary active metabolite is phenobarbital, which is responsible for its sedative effects as well as strong cytochrome P450 induction. Levodopa 13% Levodopa is used for the treatment of resting tremor due to Parkinson disease, not essential tremor. Although Parkinson disease also manifests with a low-amplitude tremor, the tremor is typically asymmetrical and improves rather than worsens with purposeful voluntary movement. Resting tremor Pill-rolling tremor A symptom of tremors that occur at rest (e.g., when the affected body part is fully relaxed and supported against gravity). Can also affect the chin, lips, and trunk. Worsens with emotional stress and is reduced with target-directed movement. Most commonly seen in Parkinson disease.

A 35-year-old woman comes to the physician because of a 1-day history of swelling and pain in the left leg. Two days ago, she returned from a business trip on a long-distance flight. She has alcohol use disorder. Physical examination shows a tender, swollen, and warm left calf. Serum studies show an increased homocysteine concentration and a methylmalonic acid concentration within the reference range. Further evaluation of this patient is most likely to show which of the following serum findings?

This patient has a deep vein thrombosis, which typically manifests with pain, swelling, and warmth of the leg. Risk factors for this condition include a history of prolonged immobilization (e.g., long-distance flights) and coagulopathies (e.g., from hyperhomocysteinemia). Decreased folate concentration 66% Folate serves as a coenzyme in several biochemical reactions, including the conversion of homocysteine to methionine. Since this reaction is also vitamin B12-dependent, a deficiency of either vitamin (e.g., from malnutrition due to alcoholism) results in the accumulation of homocysteine. Hyperhomocysteinemia leads to hypercoagulability and is therefore associated with thromboembolic events such as deep vein thrombosis. The underlying mechanisms include decreased endothelial antithrombotic activity and activation of procoagulatory factors (e.g., factor V, factor VII). A normal serum methylmalonic acid concentration, as seen here, indicates folate deficiency rather than vitamin B12 deficiency. Decreased cobalamin concentration 15% Cobalamin (vitamin B12) serves as a coenzyme for the conversion of homocysteine to methionine. Cobalamin deficiency therefore results in the accumulation of homocysteine. Hyperhomocysteinemia leads to hypercoagulability and is associated with thromboembolic events such as deep vein thrombosis. This patient's alcoholism puts her at risk of cobalamin deficiency due to malnutrition, however, since cobalamin is also required to convert methylmalonyl-CoA to succinyl-CoA, increased methylmalonic acid serum concentrations would be expected. Moreover, this patient lacks the neuropsychiatric symptoms associated with cobalamin deficiency (e.g., ataxia, neuropathy, cognitive decline).

A 65-year-old man is brought to the emergency department because of a 3-day history of increasing shortness of breath and chest pain. He has had a productive cough with foul-smelling sputum for 1 week. He has gastritis as well as advanced Parkinson disease and currently lives in an assisted-living community. He smoked one pack of cigarettes daily for 40 years but quit 5 years ago. He has a 30-year history of alcohol abuse but has not consumed any alcohol in the past 5 years. His temperature is 39.3°C (102.7°F), he is tachycardic and tachypneic and his oxygen saturation is 77% on room air. Auscultation of the lung shows rales and decreased breath sounds over the right upper lung field. Examination shows a resting tremor. Laboratory studies show: Hematocrit38%Leukocyte count17,000/mm3Platelet count210,000/mm3Lactic acid4.1 mmol/L (N=0.5-1.5) A x-ray of the chest shows infiltrates in the right upper lobe. Which of the following is the most significant predisposing factor for this patient's respiratory symptoms?

This patient has a high fever, respiratory compromise, cough with foul-smelling sputum, and infiltrates in the right lung, which is indicative of aspiration pneumonia. Parkinson disease 37% Parkinson disease predisposes patients to aspiration pneumonia by impairing the swallowing and/or cough mechanism in up to 80% of those with late-stage disease. Aspiration is most likely to occur in the dependent portions of the lung (the superior segment of right lower lobe and the posterior segment of the right upper lobe when supine or the apical and posterior right lower lobe when upright). These are often mixed infections involving organisms from the oral flora, particularly anaerobic bacteria (e.g., Peptostreptococcus, Fusobacterium, Provetella, or Bacteroides).

A 28-year-old woman comes to the physician because of a 4-day history of lower extremity numbness, weakness, and urinary incontinence. She has not had any trauma. Neurologic examination shows bilateral lower extremity weakness. Stroking the lateral side of the sole of the foot from the heel to the base of the small toe and medially to the base of the big toe elicits dorsiflexion of the big toe and fanning of the other toes. Further examination of this patient is most likely to show which of the following additional findings?

This patient has a positive Babinski sign (extension of the hallux with abduction of all toes), which indicates damage to the corticospinal tract (i.e., upper motor neurons). Spasticity 84% Muscle spasticity is due to hyperexcitable tonic reflexes and occurs when upper motor neuron (UMN) regulation of alpha motor neurons is lost, which leads to net disinhibition of the spinal reflexes. The Babinski reflex is a primitive reflex mediated by the extrapyramidal tracts. It is lost progressively as the pyramidal tracts gain functionality through increasing myelination. As such, a positive Babinski reflex is normal in children up to 1 year of age, after which time it is usually suppressed by the pyramidal tracts.

A 43-year-old woman comes to the physician because of a 1-day history of rash on the trunk and lower extremities. Three days ago, she visited a spa resort with multiple swimming pools and whirlpools. A friend of hers who also visited the spa has developed a similar rash. She does not smoke or drink alcohol and takes no medications. She appears well. Her vital signs are within normal limits. Examination shows multiple erythematous, excoriated papules and pustules over the trunk and upper thighs. The inflammation seen in this patient's condition most likely originated in which of the following parts of the skin?

This patient has a self-limited infection that typically manifests 8-48 hours after exposure to water contaminated by Pseudomonas aeruginosa. Hair follicles 69% Inflammation of the hair follicles is the cause of folliculitis. This patient's history of visiting a spa resort and developing a pruritic, papulopustular rash is highly suggestive of a diagnosis of hot tub folliculitis. This diagnosis is further supported by her friend developing similar symptoms. Though folliculitis is most commonly caused by S. aureus, hot tub folliculitis is characteristically caused by P. aeruginosa, which proliferates in wet and warm environments. This condition is typically self-limited and does not require antibiotic treatment.

A 31-year-old man comes to the physician because of a 4-week history of a painless lump near the left wrist and tingling pain over his left hand. Physical examination shows a transilluminating, rubbery, fixed, non-tender mass over the lateral volar aspect of the left wrist. There is decreased sensation to pinprick on the thumb, index finger, middle finger, and radial half of the ring finger of the left hand. The tingling pain is aggravated by tapping over the swelling. Which of the following adjacent structures is at risk of entrapment if this mass persists?

This patient has a transilluminating, firm mass over the wrist, which is most likely a ganglion cyst. Decreased sensation and paresthesia in the median nerve distribution along with a positive Tinel's sign indicate that the cyst is in a location where it is causing compression of the carpal tunnel (e.g., carpal tunnel syndrome). Flexor pollicis longus tendon 48% The flexor pollicis longus tendon travels through the carpal tunnel with the median nerve. The other structures that traverse the carpal tunnel are the four tendons of the flexor digitorum profundus and the four tendons of the flexor digitorum superficialis. Further growth of this patient's ganglion cyst could endanger any of the structures within the carpal tunnel.

A 1-year-old boy is brought to the physician for a well-child examination. He has no history of serious illness. His older sister had an eye disease that required removal of one eye at the age of 3 years. Examination shows inward deviation of the right eye. Indirect ophthalmoscopy shows a white reflex in the right eye and a red reflex in the left eye. The patient is at increased risk for which of the following conditions?

This patient has a unilateral white light reflex (leukocoria), which can be a sign of retinoblastoma. The family history of an eye condition that required enucleation strongly suggest a germline mutation in the retinoblastoma gene. Osteosarcoma 55% Patients with inherited retinoblastoma gene (Rb gene) mutations have a significantly increased risk of developing osteosarcomas. Usually, a germline mutation takes place in one of the alleles of the Rb gene only and development of retinoblastoma does not occur until a spontaneous mutation of the second, priorly functioning allele results in a non-functioning RB gene (two-hit hypothesis). Sporadic retinoblastomas, on the other hand, are caused by two spontaneous mutations in the same retinal cell, affecting both RB alleles.

A 27-year-old woman comes to the physician because of a 3-day history of a sore throat and fever. Her temperature is 38.5°C (101.3°F). Examination shows edematous oropharyngeal mucosa and enlarged tonsils with purulent exudate. There is tender cervical lymphadenopathy. If left untreated, which of the following conditions is most likely to occur in this patient?

This patient has an acute infection of the oropharynx with odynophagia, fever, pharyngeal edema, cervical lymphadenopathy, and tonsillar hypertrophy with exudates. These features suggest group A streptococcal (GAS) pharyngitis. Odynophagia A sensation of painful swallowing. Dilated cardiomyopathy 43% Dilated cardiomyopathy is a potential late and serious complication of acute rheumatic fever, a type II hypersensitivity reaction which can occur within 2-4 weeks after untreated group A Streptococcus infection. Rheumatic heart disease can manifest in several ways, including myocarditis, pericarditis, as well as valvular disease (mitral valve involvement is most common). The development of dilated cardiomyopathy is multifactorial and usually occurs secondary to valvular disease, but can also result directly from myocarditis. Dilated cardiomyopathy Congestive cardiomyopathy A disorder leading to dilatation and impaired contraction of one or both ventricles, typically resulting in heart failure. Idiopathic in up to 50% of cases and may be hereditary. Other important causes include myocarditis, ischemic heart disease, hypertension, pregnancy, and heavy, long-term alcohol consumption

A 55-year-old woman with type 2 diabetes mellitus comes to the physician for evaluation of worsening tingling of her feet at night for the last 6 months. Two years ago, she underwent retinal laser photocoagulation in both eyes. She admits to not adhering to her insulin regimen. Her blood pressure is 130/85 mm Hg while sitting and 118/70 mm Hg while standing. Examination shows decreased sense of vibration and proprioception in her toes and ankles bilaterally. Her serum hemoglobin A1C is 11%. Urine dipstick shows 2+ protein. Which of the following additional findings is most likely in this patient?

This patient has clinical and laboratory evidence of poorly-controlled diabetes mellitus, including elevated HbA1C, peripheral neuropathy, nephropathy, and retinopathy. The presence of postural hypotension in this patient suggests diabetic autonomic neuropathy, which is a subtype of diabetic neuropathy that results from poorly controlled diabetes and is characterized by damage to small fiber autonomic nerves. Incomplete bladder emptying 67% Genitourinary manifestations of diabetic autonomic neuropathy include incomplete bladder emptying, urinary retention, erectile dysfunction, retrograde ejaculation, and dyspareunia. The loss of afferent and efferent autonomic innervation of the bladder results in the inability to sense a full bladder and incomplete emptying, predisposing patients to overflow incontinence and recurrent UTI.

A 57-year-old woman comes to the physician because of a 2-week history of worsening epigastric pain that improves with meals. She has had similar pain of lesser intensity for the past 4 years. Physical examination shows no abnormalities. Upper endoscopy shows a 0.5-cm mucosal breach in the anterior duodenal bulb that extends into the submucosa. A biopsy specimen of the lesion shows hypertrophy of the Brunner glands. This patient is at the greatest risk for which of the following complications?

This patient has epigastric pain that improves with food intake, which suggests a duodenal ulcer. The diagnosis of duodenal ulcer is confirmed by endoscopic findings, including a mucosal breach in the anterior duodenum, and by the hypertrophic Brunner glands in the biopsy specimen. The location of a peptic ulcer partly determines its most likely complications. Perforation 56% Peptic ulcer perforation occurs when an ulcer erodes through all the layers of the bowel, allowing spillage of gastric or duodenal contents into surrounding structures. It is the second most common complication of chronic PUD, bleeding being the first. The most common location for peptic ulcer perforation is the anterior duodenum. Ulcers of the anterior duodenum usually perforate into the peritoneal cavity, in contrast to ulcers of the posterior duodenum, which tend to cause massive bleeding (e.g., hematemesis) from the adjacent gastroduodenal artery. PEPTIC ULCER DISEASEr4

A 76-year-old woman with hypertension and coronary artery disease is brought to the emergency department after the sudden onset of right-sided weakness. Her pulse is 83/min and blood pressure is 156/90 mm Hg. Neurological examination shows right-sided facial drooping and complete paralysis of the right upper and lower extremities. Tongue position is normal and she is able to swallow liquids without difficulty. Knee and ankle deep tendon reflexes are exaggerated on the right. Sensation to vibration, position, and light touch is normal bilaterally. She is oriented to person, place, and time, and is able to speak normally. Occlusion of which of the following vessels is the most likely cause of this patient's current symptoms?

This patient has features of a pure motor stroke without any sensory deficits or cortical signs (e.g., aphasia, agnosia, apraxia). The lesion is most likely located at the posterior limb of the internal capsule. Contralateral lenticulostriate artery 57% A pure motor stroke is caused by infarction in the posterior limb of the internal capsule, which contains the corticospinal and corticobulbar fibers that innervate the motor nuclei on the contralateral side. Therefore, a lesion at this site can cause contralateral hemiplegia as well as weakness in the lower half of the face because the region of the facial nerve nucleus that supplies the lower half of the face only receives unilateral corticobulbar innervation. Other cranial nerve nuclei as well as the region of the facial nerve nucleus that innervates the forehead receive bilateral corticobulbar innervation and are therefore spared. The most common cause of stroke in this region is a lacunar stroke of the contralateral lenticulostriate artery (a branch of the MCA).

A 27-year-old man comes to the emergency department because of nausea and repeated vomiting for the past 30 minutes. Two hours ago, he was at a summer barbecue gathering where he ate spicy, freshly grilled chicken that was fully cooked through with some reheated leftover lime basil rice. He has not had diarrhea. Physical examination shows a mildly tender abdomen with no rigidity or rebound tenderness. The next morning, his symptoms have resolved. Which of the following is the most likely cause of this patient's symptoms? KEY INFO

This patient has food poisoning caused by a spore-forming gram-positive rod. Ingestion of preformed cereulide toxin 87% Improperly refrigerated and reheated starch-containing food (e.g., rice), as seen here, is a common source of emetic Bacillus cereus food poisoning. B. cereus can form spores, which are heat-resistant and survive the cooking process. If the food is cooled too slowly or improperly refrigerated after cooking, these spores can germinate and produce cereulide, a heat-resistant emetic toxin that causes nausea and vomiting, or a heat-labile diarrheal toxin that causes diarrhea. Food poisoning caused by the ingestion of preformed enterotoxins (e.g., from B. cereus and S. aureus) leads to the rapid onset of symptoms within minutes to hours, as seen here. In comparison, food poisoning caused by bacteria that produce enterotoxins inside the intestines (e.g., enterotoxigenic E. coli, V. cholerae) has a delayed onset (hours to days after ingestion) and typically manifests with watery diarrhea. Foodborne illnesses that manifest with bloody diarrhea usually indicate invasion of the intestinal mucosa by the pathogen (e.g., Campylobacter, Shigella, nontyphoidal Salmonella).

A 26-year-old man is brought to the emergency department because of abdominal pain, dizziness, shortness of breath, and swelling and pruritus of the lips, tongue, and throat for 1 hour. The symptoms began minutes after he started eating a lobster dinner. It is determined that his symptoms are due to surface crosslinking of IgE. This immunologic event most likely caused the release of which of the following?

Tryptase In the immediate phase of anaphylactic reactions, surface crosslinking of IgE causes mast cell degranulation, which releases tryptase, histamine, heparin, and eosinophil chemotactic factors. Of these compounds, tryptase is a relatively specific marker of mast cell activation. Tryptase A proteinase released during mast cell degranulation. Used as a marker of mast cell activation, and can help identify a type 1 hypersensitivity reaction if high levels are measured in the serum. Commonly used to differentiate anaphylaxis from mimics such as hereditary angioedema when the diagnosis is uncertain. The patient's acute-onset abdominal pain, dizziness, shortness of breath, swelling, and pruritus after eating shellfish (a common allergen) are characteristic of an anaphylactic reaction. Anaphylaxis is a type I hypersensitivity reaction mediated by surface crosslinking of IgE on mast cells.

A 32-year-old man visits his primary care physician for a routine health maintenance examination. During the examination, he expresses concerns about not wanting to become a father. He has been sexually active and monogamous with his wife for the past 5 years, and they inconsistently use condoms for contraception. He tells the physician that he would like to undergo vasectomy. His wife is also a patient under the care of the physician and during her last appointment, she expressed concerns over being prescribed any drugs that could affect her fertility because she would like to conceive soon. Which of the following is the most appropriate action by the physician regarding this patient's wish to undergo vasectomy?

This patient has full decision-making capacity with the ability to evaluate the benefits and risks of the procedure. Explain the procedure's benefits, alternatives, and potential complications 87% This patient must understand the benefits, alternatives, and potential complications of undergoing a vasectomy in order to provide informed consent. This conversation should explore the patient's reasons for undergoing permanent sterilization. Although the wife's consent for this procedure is not required, the physician should encourage the patient to discuss his decision with his wife, whose wish to conceive in the near future could factor into the patient's decision.

A 62-year-old man comes to the physician for an annual health maintenance examination. He has a history of stable angina, gout, and hypertension. His medications include lisinopril and aspirin. He has smoked a pack of cigarettes daily for 20 years. He drinks 5-6 beers on the weekends. His blood pressure is 150/85 mm Hg. Laboratory studies show a total cholesterol of 276 mg/dL with an elevated low-density lipoprotein (LDL) concentration and low high-density lipoprotein (HDL) concentration. Administration of which of the following agents is the most appropriate next step in treatment?

This patient has hyperlipidemia and a history that suggests coronary artery disease (stable angina). HMG-CoA reductase inhibitor 93% HMG-CoA reductase inhibitors, more commonly referred to as statins, are the first-line treatment for hyperlipidemia. Statins inhibit cholesterol synthesis in the liver by inhibiting the conversion of HMG-CoA to mevalonate. Statins are the most effective drug for reducing LDL levels, and they also increase HDL and lower triglyceride levels. Common side effects of HMG-CoA reductase inhibitors include elevated LFTs and myalgias (especially when used concomitantly with fibrates).

A 63-year-old man comes to the physician for blurry vision and increased difficulty walking over the past month. He feels very fatigued after watering his garden but feels better after taking a nap. He has not had any recent illness. He has smoked one pack of cigarettes daily for 35 years. Examination shows drooping of the upper eyelids bilaterally and diminished motor strength in the upper extremities. Sensation to light touch and deep tendon reflexes are intact. An x-ray of the chest shows low lung volumes bilaterally. A drug with which of the following mechanisms of action is most appropriate for this patient?

This patient has ptosis as well as weakness that is worse with activity and improves with rest. These clinical features suggest a diagnosis of myasthenia gravis (MG) Inhibition of acetylcholinesterase 84% Cholinesterase inhibitors are used to manage the symptoms of myasthenia gravis (MG) because they increase the availability of endogenous acetylcholine (ACh) in the neuromuscular junction, which competes with the ACh receptor antibodies that would otherwise block ACh receptors. Pyridostigmine or alternatively neostigmine are the preferred drugs, since they only act on peripheral ACh receptors and do not cross the blood-brain barrier, which limits their adverse effects.

A 48-year-old woman comes to the physician for the evaluation of 24-hour blood pressure monitoring results. Over the last 3 months, she has had intermittent nausea, decreased appetite, and increasing weakness and fatigue during the day. She has been treated twice for kidney stones within the past year. Her current medications include lisinopril, amlodipine, and furosemide. She is 178 cm (5 ft 10 in) tall and weighs 97 kg (214 lb); BMI is 31 kg/m2. Her blood pressure is 152/98 mm Hg. Physical examination shows no abnormalities. Serum studies show: Na+141 mEq/LCl−101 mEq/LK+4.5 mEq/LHCO3−24 mEq/LCalcium12.9 mg/dLCreatinine1.0 mg/dL Twenty-four-hour blood pressure monitoring indicates elevated nocturnal blood pressure. Further evaluation is most likely to show which of the following findings?

This patient has resistant hypertension despite treatment with 3 antihypertensive drugs, which should raise concern for secondary hypertension. She also has nephrolithiasis (stones), nausea and decreased appetite (groans), fatigue, and weakness. Increased serum parathyroid hormone 61% Increased serum parathyroid hormone would be found in primary hyperparathyroidism, a known cause of secondary hypertension, especially in patients with hypercalcemia, as seen here. Hypercalcemia, occurring as a result of primary hyperparathyroidism or otherwise, is thought to cause hypertension by affecting vascular reactivity and interrupting day-night blood pressure regulation. The most common causes of hypercalcemia are hyperparathyroidism and malignancy.

A 36-year-old man comes to the physician for a 4-week history of swollen legs. He has difficulty putting on socks because of the swelling. Two years ago, he was diagnosed with sleep apnea. He takes no medications. He emigrated from Guatemala with his family when he was a child. He is 171 cm (5 ft 6 in) tall and weighs 115 kg (253 lb); BMI is 39 kg/m2. His pulse is 91/min and blood pressure is 135/82 mm Hg. Examination shows periorbital and bilateral lower extremity edema. SerumAlbumin3.1 g/dLTotal cholesterol312 mg/dLUrineBloodnegativeProtein+4RBC1-2/hpfRBC castnegativeFatty castsnumerous A renal biopsy is obtained. Which of the following is most likely to be seen under light microscopy of the patient's renal biopsy specimen?

This patient has several features of nephrotic syndrome, including swelling of the face and extremities, hypoalbuminemia, hypercholesterolemia, obesity, and heavy proteinuria (4+ on urine dipstick). The most likely diagnosis is the most common cause of nephrotic syndrome in Hispanic populations and is associated with obesity. Segmental sclerosis of the glomeruli 60% This patient's nephrotic syndrome is most likely due to focal segmental glomerulosclerosis (FSGS), which appears as segmental sclerosis and hyalinosis under light microscopy. FSGS can be primary (i.e., idiopathic), or secondary to another condition (e.g., obesity, heroin use, sickle cell disease, HIV). Without immunosuppressive treatment, FSGS usually leads to end-stage renal disease.

One month after undergoing surgical spinal fusion because of a traumatic spinal cord injury, a 68-year-old man comes to the physician because of lower abdominal pain. He last voided yesterday. Physical examination shows a suprapubic mass and decreased sensation below the umbilicus. Urodynamic studies show simultaneous contractions of the detrusor muscle and the internal urethral sphincter. Urinary catheterization drains 900 mL of urine from the bladder. Which of the following is the most appropriate pharmacotherapy for this patient's urinary symptoms?

This patient has signs and symptoms of spastic neurogenic bladder (detrusor sphincter dyssynergia) with urinary retention following spinal cord injury. Appropriate pharmacotherapy would involve inhibition of the detrusor and sphincter muscle. Prazosin 34% Alpha blockers such as prazosin are the most appropriate pharmacotherapy in patients with spastic neurogenic bladder because of their effects on smooth muscle relaxation of the neck of the bladder and the internal urethral sphincter, which can improve urine outflow. This patient likely had a spinal cord lesion near the level of T10 (decreased sensation below the umbilicus), which is now causing sympathetic stimulation of the urethral sphincter, as evidenced by the urodynamic studies. Alpha blockers are also a first-line therapy in urinary retention due to outflow tract obstruction caused by benign prostatic hyperplasia. The most common side effects of alpha blockers are postural hypotension and headaches. Spastic bladder Reflex bladder A bladder dysfunction caused by neurological damage (e.g., spinal injury, stroke, multiple sclerosis). Results in uncoordinated bladder contraction and external urinary sphincter relaxation (detrusor-sphincter dyssynergia). Manifests with an overactive bladder (e.g., urine leakage, urge to urinate) as well as urinary retention.

A 52-year-old woman is brought to the emergency department for a severe, sudden-onset headache, light-sensitivity, and neck stiffness that began 30 minutes ago. A CT scan of the head shows hyperdensity between the arachnoid mater and the pia mater. The patient undergoes an endovascular procedure. One week later, she falls as she is returning from the bathroom. Neurologic examination shows 3/5 strength in the right lower extremity and 5/5 in the left lower extremity. Treatment with which of the following drugs is most likely to have prevented the patient's current condition?

This patient has signs of acute cerebral ischemia (new-onset unilateral lower extremity weakness) 7 days after a subarachnoid hemorrhage. Blood in the subarachnoid space leads to the release of vasoactive substances; this can bring about vasospasm in nonbleeding vessels that manifests with focal neurologic deficits. Nimodipine 72% Oral nimodipine should be given after subarachnoid hemorrhage to prevent vasospasm, which can lead to ischemic stroke. This patient's sudden-onset neurologic deficit 7 days after SAH is a common presentation of vasospasm, which is a complication in ∼ 30% of SAH cases and is more common with aneurysms. Nimodipine has been shown to reduce morbidity and mortality associated with SAH, but the exact mechanism of benefit of nimodipine is unknown. Nimodipine A short-acting dihydropyridine calcium channel blocker with a half-life of < 2 hours. Commonly used to prevent secondary vasospasm after subarachnoid hemorrhage. Calcium channel blockers, calcium channel antagonists or calcium antagonists are a group of medications that disrupt the movement of calcium through calcium channels. Calcium channel blockers are used as antihypertensive drugs, i.e., as medications to decrease blood pressure in patients with hypertension.

A 67-year-old woman comes to the physician for chest tightness, shortness of breath, and lightheadedness. She has experienced these symptoms during the past 2 weeks while climbing stairs but feels better when she sits down. She had a cold 2 weeks ago but has otherwise been well. She appears short of breath. Her respirations are 21/min and blood pressure is 131/85 mmHg. On cardiovascular examination, a late systolic ejection murmur is heard best in the second right intercostal space. The lungs are clear to auscultation. Which of the following mechanisms is the most likely cause of this patient's current condition?

This patient is presenting with chest tightness consistent with angina. She also has a late ejection systolic murmur on examination, which should raise suspicion of aortic stenosis. Aortic valve stenosis Abbreviation: AS A valvular disease characterized by narrowing of the aortic valve, resulting in obstruction of the outflow of blood from the left ventricle into the aorta during systole. Increased left ventricular oxygen demand 64% The narrowed opening of a stenotic aortic valve during systole obstructs blood flow from the left ventricle (LV), which over time results in left ventricular hypertrophy (LVH). LVH subsequently results in increased LV oxygen demand, leading to myocardial ischemia, which manifests as angina especially during periods of strenuous exercise or stress. The reduction in cardiac output during exertion also causes transient cerebral hypoperfusion, which explains her presyncopal symptoms.

A 31-year-old woman with multiple sclerosis comes to the physician because of a 4-day history of cramps in her left leg. Physical examination shows flexion of the left hip and increased tone in the thigh muscles. A local anesthetic block of which of the following nerves would most likely improve this patient's condition the most?

This patient is showing signs of spasticity, a common clinical feature of multiple sclerosis, of the hip flexors. The nerve that innervates these muscles originates from spinal roots L2-L4 and lies below the inguinal ligament. Femoral nerve 76% Motor fibers of the femoral nerve (L2-L4 nerve roots) innervate the hip flexors. Selectively blocking the femoral nerve would improve hip flexor spasticity. One adverse effect, though, would be a loss of knee extension because the nerve also innervates the quadriceps femoris muscles. Since the femoral nerves also provide sensory-only fibers, another adverse effect would be a loss of feeling of her anteromedial thigh via blocking of the anterior cutaneous branches, as well as the medial lower leg and foot via a saphenous nerve deficit, respectively.

A 2-year-old boy is brought to the physician by his parents several weeks after the family immigrated from Russia. The parents are worried because the child appears to have trouble seeing and has not started walking. The child was born at home and has never been evaluated by a physician. During the pregnancy, the mother had a week of fever, myalgia, diffuse rash, and bilateral nontender cervical adenopathy after the family adopted a new cat. An MRI of the head is shown. Which of the following additional findings is most likely in this patient?

This patient likely developed congenital toxoplasmosis after his mother had a primary infection (fever, myalgia, and nontender cervical adenopathy after exposure to an infected cat). One of the cardinal manifestations of congenital toxoplasmosis is hydrocephalus, which manifests as dilation of the ventricles, as seen on MRI. Spasticity of bilateral lower extremities 52% Spasticity of the lower extremities is a common finding in advanced, chronic hydrocephalus due to stretching of the motor cortices and pyramidal tracts around the dilated ventricles. Hyperreflexia, another sign of upper motor neuron damage, can also be seen in many cases. Other symptoms of hydrocephalus in children include difficulty feeding, irritability, and macrocephaly.

A 39-year-old man is brought to the physician by his wife because of personality changes over the past year. He has become increasingly irritable, loud, aggressive, and impulsive. His wife also reports jerky movements of his limbs and trunk for the past few months. His father had dementia in his mid-40s, but the details of his condition are unclear. The patient appears restless. Examination shows irregular movements of the extremities and twitching of the face and tongue. Mental status examination shows impaired memory. This patient's condition is most likely associated with which of the following changes on MRI?

This patient most likely has Huntington disease (HD) based on the presence of chorea (e.g., involuntary limb and trunk movements), behavioral changes (aggression, impulsivity), and cognitive impairments (impaired memory). His father's dementia may have also been associated with HD, which would be consistent with the autosomal dominant inheritance pattern of the condition. Atrophy of the striatum 65% Atrophy of the striatum, specifically the caudate nucleus and putamen, is a classic finding on brain imaging in patients with Huntington disease (HD). HD is caused by CAG trinucleotide repeats in the huntingtin gene, which cause aggregates of abnormal huntingtin protein to accumulate, resulting in neuronal death and gliosis in the striatum. The disorder is diagnosed via genetic testing, with neuroimaging used to rule out other intracranial pathologies.

An otherwise healthy 25-year-old man comes to the physician because of a 2-day history of chills, palpitations, and worsening shortness of breath. The patient is 183 cm (6 ft) tall and weighs 70 kg (154 lbs); BMI is 21 kg/m2. His temperature is 39.5°C (103.1°F), pulse is 110/min, and blood pressure is 140/60 mm Hg. Examination of the extremities shows several nontender, nonblanching, erythematous macules on the palms and soles and painful nodules on the pads of the fingers and toes. Coarse crackles are heard over both lungs. Cardiac examination is shown. Results of initial blood cultures are pending. Echocardiography is most likely to show which of the following abnormalities?

This patient most likely has acute infective endocarditis (IE), which is indicated by a high fever, chills, palpitations, dyspnea, Janeway lesions (nontender, nonblanching, erythematous macules on the palms and soles), and Osler nodes (painful nodules on pads of the fingers and toes). The carotid pulsations heard in the audio clip indicate ventricular systole. There is a high-pitched, nonradiating, blowing, decrescendo early diastolic murmur, heard best at the left sternal border. The auscultatory findings in this patient are characteristic of aortic regurgitation (AR), which causes left ventricular end-diastolic volume overload as the left ventricle receives blood from the left atrium and the regurgitant aorta. This overload results in hyperdynamic circulation, which manifests as palpitations (especially in the left lateral decubitus position, when the enlarged left ventricle is closest to the chest wall), a widened pulse pressure, and dyspnea due to pulmonary edema resulting from left heart failure, as seen here. The most common cause of acute valvular AR is IE, which this patient has. AR due to valvular disease is heard best along the left sternal border, as seen in this patient, while AR due to aortic root disease (e.g., in Marfan syndrome) is heard best along the right sternal border. Other common causes of AR include aortic dissection, acute rheumatic fever, and congenital bicuspid aortic valve.

A 30-year-old woman comes to the physician because of numbness, fatigue, and blurry vision for 1 week. The symptoms are worse after a hot shower or bath. She had an episode of right arm weakness 2 years ago that resolved without intervention. She recently returned from a hiking trip in upstate New York. Her temperature is 37.1°C (100°F) and blood pressure is 100/66 mm Hg. Physical examination shows decreased sensation to light touch in the left hand, right thigh, and right flank. Strength is normal. There is left-sided photophobia and pupillary constriction in the left eye is decreased compared to the right eye. Which of the following best describes the pathogenesis of the disease process in this patient?

This patient presents with characteristic findings of multiple sclerosis including optic neuritis (impaired vision), Uhthoff's phenomenon (worsening of symptoms in response to increased body temperature), as well as episodes of sensory and motor deficits followed by periods of remission. Th1 cell-mediated nerve sheath damage 70% Multiple sclerosis is an immune-mediated demyelinating disease with a predilection for young, white females. Although the pathophysiology is not fully understood, it is believed that Th1 cells react to myelin basic proteins and initiate an inflammatory response, leading to focal demyelination of white matter in the brain and spinal cord. Typical clinical features include transient, isolated episodes of optic neuritis, diplopia, limb weakness, sensory deficits, and ataxia that fully or partially resolve in days to weeks.

A 51-year-old woman comes to the physician because of a 3-month history of fatigue, increased urinary frequency, and low back pain. She reports frequent passing of hard stools, despite using stool softeners. During this time, she has not been as involved with her weekly book club. Her family is concerned that she is depressed. She has no history of serious illness. She has smoked 1 pack of cigarettes daily for the past 20 years. Her pulse is 71/min and blood pressure is 150/90 mm Hg. Abdominal examination shows right costovertebral angle tenderness. The patient's symptoms are most likely caused by hyperplasia of which of the following?

This patient presents with classic features of hypercalcemia such as fatigue, hypertension, polyuria ("thrones"), abdominal and lower back pain ("groans, bones"), costovertebral angle tenderness (renal "stones"), constipation, and depression ("psychiatric overtones"). Chief cells in the parathyroid gland 60% Parathyroid chief cells are the site of parathyroid hormone (PTH) production and secretion. Under physiological conditions, parathyroid chief cells maintain calcium homeostasis through PTH-mediated increases in bone resorption, renal calcium absorption, and phosphate excretion. This patient's hypercalcemia is most likely caused by excessive production and secretion of parathyroid hormone (primary hyperparathyroidism). The most common causes of primary hyperparathyroidism include parathyroid adenoma (approx. 80% of cases) and hyperplasia (approx. 15% of cases).

A 3-year-old girl is brought to the emergency department because of abdominal pain and watery diarrhea for the past 2 days. This morning, her stool had a red tint. She and her parents visited a circus 1 week ago. The patient attends day care. Her immunizations are up-to-date. Her temperature is 38°C (100.4°F), pulse is 140/min, and blood pressure is 80/45 mm Hg. Abdominal examination shows a soft abdomen that is tender to palpation in the right lower quadrant with rebound. A stool culture grows Yersinia enterocolitica. Consumption of which of the following is most likely to cause the infection seen in this patient?

This patient presents with gastroenteritis, signs of pseudoappendicitis (right lower quadrant pain), and has stool cultures growing Yersinia enterocolitica. Humans are considered to be incidental hosts. Undercooked pork 53% Yersinia enterocolitica is usually transmitted via contaminated raw pork, unpasteurized milk products, unfiltered water, or pet feces. In day-care centers, where children might not be toilet trained or follow personal hygiene practices (e.g., handwashing), person-to-person fecal-oral transmission of Y. enterocolitica is possible. After an incubation period of typically 4-6 days, yersiniosis causes inflammatory diarrhea, nausea, low-grade fever, and possibly RLQ tenderness that may mimic appendicitis ("pseudoappendicitis").

A 41-year-old woman comes to the physician because of a 3-month history of anxiety, difficulty falling asleep, heat intolerance, and a 6-kg (13.2-lb) weight loss. The patient's nephew, who is studying medicine, mentioned that her symptoms might be caused by a condition that is due to somatic activating mutations of the genes for the TSH receptor. Examination shows warm, moist skin and a 2-cm, nontender, subcutaneous mass on the anterior neck. Which of the following findings would suggest a different etiology other than the one described by her nephew?

This patient presents with classic symptoms of hyperthyroidism. An activating mutation of the genes for the TSH receptor is the most common cause of toxic adenoma and toxic multinodular goiter, which are both causes of hyperthyroidism. One of the answer options is a finding seen only in Graves diseases and not in other causes of hyperthyroidism. Nonpitting edema 37% Graves disease is the most common cause of hyperthyroidism. Although the majority of symptoms are due to increased thyroid hormone levels, myxedema (nonpitting edema) and exophthalmos in patients with Graves disease are instead due to autoantibody-mediated activation of TSH receptors on peripheral tissue. TSH receptor antibodies stimulate dermal fibroblasts and periorbital adipocytes, thereby increasing glycosaminoglycan synthesis. Deposition of these mucopolysaccharides is the underlying pathogenesis of myxedema and exophthalmos, which are specific for hyperthyroidism caused by Graves disease.

A 34-year-old woman comes to the physician because of a 3-month history of fatigue and a 4.5-kg (10-lb) weight loss despite eating more than usual. Her pulse is 115/min and blood pressure is 140/60 mm Hg. Physical examination shows warm, moist skin, and a diffuse, non-tender swelling over the anterior neck. Ophthalmologic examination shows swelling of the eyelids and proptosis bilaterally. Which of the following is the most likely cause of this patient's symptoms?

This patient presents with clinical findings of hyperthyroidism due to Graves disease, including unintentional weight loss, warm, moist skin, tachycardia, hypertension, painless goiter, and ophthalmopathy. Thyrotropin receptor autoantibodies 64% Graves disease is caused by thyrotropin-receptor autoantibodies (TRAbs) that activate the TSH receptor. TRAbs induce hyperthyroidism by stimulating follicular epithelial cells to produce abnormally high amounts of triiodothyronine (T3) and thyroxine (T4). As seen in this patient, the metabolically active free T3 increases the basal metabolic rate and lipolysis, which manifests as weight loss, high blood pressure, and increased body temperature. TSH receptor antibodies also bind to orbital TSH receptors and stimulate adipocyte proliferation and orbital fibroblast secretion of glycosaminoglycans. Increased adipocyte count, osmotic muscle swelling, and muscle inflammation cause expansion of retro-orbital tissue, which manifests as exophthalmos and decreased ocular motility.

A 43-year-old man is brought to the emergency department by his wife because of a 1-hour history of confusion and strange behavior. She reports that he started behaving in an agitated manner shortly after eating some wild berries that they had picked during their camping trip. His temperature is 38.7°C (101.7°F). Physical examination shows warm, dry skin and dry mucous membranes. His pupils are dilated and minimally reactive to light. His bowel sounds are decreased. The patient is admitted and pharmacotherapy is initiated with a drug that eventually results in complete resolution of all of his symptoms. This patient was most likely administered which of the following drugs?

This patient presents with delirium, hyperthermia, mydriasis, decreased intestinal motility, and dry skin and mucus membranes. This constellation of symptoms is suggestive of anticholinergic syndrome, likely due to intoxication from belladonna alkaloids in the wild berries. Physostigmine 67% Physostigmine is a CNS-penetrating acetylcholinesterase inhibitor used to treat anticholinergic toxicity, the diagnosis in this case. The berries of the atropa belladonna plant contain atropine, which can cause anticholinergic syndrome, as seen in this patient. The first-line treatment is to reversibly inhibit acetylcholinesterase both centrally and peripherally with physostigmine.

A 63-year-old man comes to the physician because of fatigue and muscle cramps for 6 weeks. He also noticed several episodes of tingling around the mouth and in the fingers and toes. He has osteoarthritis of his knees and hypertension. Current medications include ibuprofen and ramipril. He has smoked one pack of cigarettes daily for 35 years. Tapping over the facial nerve area in front of the ear elicits twitching of the facial muscles on the same side of the face. His serum alkaline phosphatase activity is 66 U/L, serum phosphorus is 5.2 mg/dL, and serum creatinine is 1.1 mg/dL. An ECG shows sinus rhythm with a prolonged QT interval. Which of the following is the most likely underlying cause of this patient's symptoms?

This patient presents with fatigue, muscle cramps, paresthesias, Chvostek sign, and a prolonged QT interval, all of which are typical features of hypocalcemia. Chvostek's sign Contraction of the facial muscles elicited by tapping the facial nerve in the area of the cheek. It is a sign of nerve hyperexcitability usually seen in tetany. Destruction of parathyroid glands 41% This patient presents with hypocalcemia due to hypoparathyroidism, as indicated by the elevated phosphorus, normal alkaline phosphatase, and symptoms of hypocalcemia in the setting of normal kidney function. Decreased parathyroid hormone (PTH) activity leads to decreased release of calcium and phosphorus from the bone, as well as decreased calcium reabsorption and phosphorus secretion at the kidney. The most common cause of hypoparathyroidism in adults is surgery (e.g., thyroidectomy, parathyroidectomy), which this patient has no history of. Instead, his hypoparathyroidism is most likely due to autoimmune destruction of parathyroid glands (the second most common cause in adults). Other less common causes of hypoparathyroidism are congenital (e.g., DiGeorge syndrome), infiltration of the parathyroid gland (e.g., hemochromatosis, granulomas), and radiation-induced destruction.

A 53-year-old man comes to the physician because of a 3-month history of a nonpruritic rash, fatigue, and decreased urination. Physical examination shows multiple erythematous, purpuric papules on his trunk and extremities that do not blanch when pressed. Serum creatinine is elevated and urinalysis shows red blood cell casts and protein. Serum complement levels are decreased. Renal biopsy shows subendothelial immune complex deposits with granular immunofluorescence and tram-track basement membrane splitting. Further laboratory evaluation of this patient is most likely to show the presence of which of the following antibodies?

This patient presents with fatigue, palpable purpura, decreased complement levels, and an active urine sediment. The renal biopsy findings of subendothelial immune complex deposits and basement membrane tram tracking further support a diagnosis of membranoproliferative glomerulonephritis due to mixed cryoglobulinemia. Mixed cryoglobulinemia A collective term for type II and type III cryoglobulinemia. Type II cryoglobulins are monoclonal IgM and polyclonal IgG antibodies, while type III cryoglobulins are polyclonal IgM and polyclonal IgG antibodies. In vivo, these cryoglobulins form immune complexes, have rheumatoid-factor-like activity, and can activate the complement pathway. In vitro, these cryoglobulins precipitate slowly on cooling. Mixed cryoglobulinemia is associated with infections (especially hepatitis C), and autoimmune diseases (especially Sjogren's disease and SLE). Anti-hepatitis C antibodies Mixed cryoglobulinemia is strongly associated with hepatitis C infection. HCV antigen-immune complex formation triggers complement activation and inflammation of blood vessels, resulting in cryoglobulinemic vasculitis, which can manifest with the palpable purpura and membranoproliferative glomerulonephritis seen in this patient. Given that approx. 90% of cases of mixed cryoglobulinemia are associated with HCV, it is important to test patients with mixed cryoglobulinemia for hepatitis C to ensure that early therapy can be initiated.

A 79-year-old woman is brought to the emergency department because of a 1-hour history of hoarseness. Her pulse is 85/min and blood pressure is 142/90 mm Hg. Physical examination shows a diminished gag reflex and loss of pain and temperature over the left face and right trunk and extremities. There is no facial asymmetry and the tongue does not deviate when protruded. Her strength is 5/5 bilaterally in both the upper and lower extremities. Ophthalmologic examination shows involuntary, rhythmic ocular movements. There is left-sided miosis and ptosis. This patient's condition is most likely caused by occlusion of which of the following vessels?

This patient presents with features of damage to the nucleus ambiguus (hoarseness and diminished gag reflex), spinal trigeminal nucleus (loss of pain and temperature sensations in the face ipsilaterally), spinothalamic tract (loss of (loss of pain and temperature sensations in the contralateral trunk and extremities), vestibular nucleus (nystagmus), and hypothalamospinal tract (ipsilateral ptosis and miosis). This constellation of features suggests that the infarction is located in the lateral medulla. Posterior inferior cerebellar artery The lateral medulla is supplied by the posterior inferior cerebellar artery (PICA) and the lateral pons by the anterior inferior cerebellar artery (AICA). Both infarction of the lateral medulla (lateral medullary syndrome or Wallenburg syndrome) and lateral pons (lateral pontine syndrome) manifest with loss of pain and temperature sensations in one half of the face and the contralateral trunk and extremities, as well as vestibulocerebellar symptoms (e.g., nystagmus, vertigo, ataxia) and unilateral Horner syndrome. The two conditions can be differentiated based on the presence or absence of certain cranial nerve palsies. Lateral medullary syndrome also manifests with features of CN IX and CN X palsies as a result of nucleus ambiguus infarction, whereas lateral pontine syndrome manifests with features of cochlear nucleus infarction and CN VII palsy because of facial nucleus infarction.

A 67-year-old man is brought to the emergency department 1 hour after the sudden onset of left-sided body weakness and difficulty speaking. He has a history of hypertension and coronary artery disease. He has smoked one pack of cigarettes daily for 30 years. His temperature is 36°C (96.8°F), pulse is 100/min, respirations are 18/min, and blood pressure is 151/87 mm Hg. Physical examination shows deviation of the tongue to the right, left-sided hemiparesis, and decreased proprioception on the left side of the body. Sensation to pinprick and temperature is intact. This patient's condition is most likely caused by occlusion of which of the following labeled vessels in the schematic overview of the arterial supply to the brain?

This patient presents with features of infarction in the right corticospinal tract (left-sided hemiparesis), right medial lemniscus (left-sided loss of proprioception), and right hypoglossal nucleus (deviation of the tongue to the right and difficulty speaking due to dysarthria). C 48% This artery, which is formed by the union of branches from the vertebral artery and located in the midline on the ventral surface of the medulla and spinal cord, is the anterior spinal artery. Occlusion of the paramedian branches of the anterior spinal artery and/or vertebral artery can lead to medial medullary syndrome, which is caused by infarction of the corticospinal tract, medial lemniscus, and hypoglossal nucleus, as seen here. This patient's history of coronary heart disease, hypertension, and smoking history suggests a thrombotic stroke, which is caused by plaque rupture in arteriosclerotic arteries. After the exclusion of bleeding as a cause, treatment will most likely consist of reperfusion therapy, supportive care, and prevention of post-stroke complications.

A 40-year-old woman with HIV infection comes to the emergency department because of a 4-week history of progressively worsening fatigue and headache. Her temperature is 38.1°C (100.6°F). On mental status examination, the patient is somnolent and oriented only to person. Her CD4+ lymphocyte count is 80/mm3. Analysis of this patient's cerebrospinal fluid (CSF) shows a leukocyte count of 30/mm3 (60% lymphocytes), a protein concentration of 52 mg/dL, and a glucose concentration of 37 mg/dL. An India ink stain of the CSF is shown. Which of the following is the most appropriate pharmacotherapy for this patient's neurological symptoms?

This patient presents with fever, headache, and altered mental status, all of which suggests the presence of a CNS infection. Lymphocytic pleocytosis and the detection of a yeast with a prominent halo on Indian ink stain suggest meningitis due to infection with Cryptococcus neoformans. Amphotericin B and flucytosine The combination of intravenous amphotericin B and oral flucytosine is used to treat cryptococcal meningitis. This combination is administered during the induction phase to achieve rapid sterilization of the CSF, followed by maintenance therapy with fluconazole. Cryptococcal meningitis can develop following initial respiratory infection and may lead to death within weeks if left untreated.

A previously healthy 2-year-old boy is brought to the physician because of a 10-day history of unsteady gait, frequent falls, and twitching of the extremities. Physical examination shows bilateral saccadic eye movement in all directions and brief, involuntary muscle contractions of the trunk and limbs. There is an ill-defined, nontender mass in the upper right abdomen. He undergoes surgical resection of the tumor. Histopathologic examination of this mass is most likely to show which of the following?

This patient presents with gait ataxia and twitching of the eyes (opsoclonus) and extremities (myoclonus). Opsoclonus-myoclonus syndrome in a patient with an abdominal mass is highly suggestive of a diagnosis of neuroblastoma, the most common extracranial solid malignancy of childhood. Neuroblastoma A malignant neuroendocrine tumor of the sympathetic nervous system that originates from neural crest cells. Primarily affects children and infants. Composed of blastematous, stromal, and/or epithelial components. Diagnosed by increased urinary catecholamines. Small blue cells arranged in rosettes around a central neuropil 66% Small, blue cells arranged in rosettes around a central neuropil (Homer-Wright rosettes) are a histopathologic feature of neuroblastomas. To diagnose and monitor disease activity, urine can be tested for homovanillic acid and vanillylmandelic acid, which are produced when neuroblastoma cells metabolize catecholamines. In some cases, genetic analysis shows a deletion in chromosome 1p that amplifies the N-myc oncogene. Neuroblastomas can occur anywhere along the sympathetic chain, the majority being located in the abdomen, as seen in this patient. They become symptomatic due to the mass effect of the tumor or due to metastases (e.g., orbits, bones). This patient's opsoclonus-myoclonus ataxia is a paraneoplastic syndrome associated with neuroblastoma.

A 38-year-old woman undergoes hemithyroidectomy for treatment of localized, well-differentiated papillary thyroid carcinoma. The lesion is removed with clear margins. However, during the surgery, a structure lying directly adjacent to the superior thyroid artery at the upper pole of the thyroid lobe is damaged. This patient is most likely to experience which of the following symptoms?

Voice pitch limitation 72% Voice pitch limitation is caused by damage to the external branch of the superior laryngeal nerve (SLN), which occurs in up to 30% of thyroid dissections. The external branch of the SLN innervates the cricothyroid muscle, which is responsible for lengthening, thinning, and stiffening the vocal cords and increases the pitch of phonation. The cricothyroid muscle is the only tensor of the vocal cord, and its denervation results in an inability to increase the pitch of the voice or produce explosive sounds as well as easy voice fatigability. The external branch of the superior laryngeal nerve lies directly adjacent to the superior thyroid artery at the upper pole of the thyroid lobe. If the artery is ligated during surgery, it may be damaged.

While walking through a park with his wife, a 51-year-old man with type 2 diabetes mellitus develops nausea, sweating, pallor, and palpitations. For the past 3 weeks, he has been trying to lose weight and has adjusted his diet and activity level. He eats a low-carb diet and swims 3 times a week. The man returned home from a training session 2 hours ago. Current medications include basal insulin and metformin. Shortly before his wife returns from their car with his emergency medication kit, he becomes unconscious. Administration of which of the following is the most appropriate next step in treatment?

This patient presents with nausea, sweating, pallor, and palpitations. Given his history of type 2 diabetes mellitus and the recent change in his diet and exercise regimen (especially without having adjusted his insulin dosage), the symptoms are highly suggestive of severe acute hypoglycemia. Intramuscular glucagon 70% Intramuscular glucagon is the treatment of choice in a nonhospital setting for patients with severe hypoglycemia who are unconscious or unable to tolerate oral delivery of fast-acting carbohydrates. Glucagon elevates endogenous glucose production through hepatic glycogenolysis and gluconeogenesis. It achieves this by binding a hepatic G protein-coupled receptor, thereby activating a G protein that stimulates adenylate cyclase. Adenylate cyclase produces more cAMP, which increases protein kinase A (PKA) activity. PKA-mediated phosphorylation activates gluconeogenesis by stimulating FBPase-2 and inhibiting PFK-2. PKA also promotes glycogenolysis by activating glycogen phosphorylase and inactivating glycogen synthase via phosphorylation. The net effect is a rapid increase in serum glucose levels. In a hospital setting, intravenous dextrose would be the treatment of choice. Oral glucose 17% Oral glucose with fast-acting carbohydrates (such as glucose tablets, candy, or juice) is an important part of management for conscious patients with hypoglycemia. This patient, however, is unconscious. Therefore, orally administered glucose is not feasible.

A 58-year-old man comes to the physician because of a 4-day history of abdominal pain and vomiting. Initially, the vomitus was food that he had recently eaten, but it is now bilious. He has had similar complaints several times in the past 6 years. He has smoked 1 pack of cigarettes daily for the past 25 years and drinks 18 oz of whiskey daily. He is 160 cm (5 ft 3 in) tall and weighs 48 kg (105 lb); BMI is 19 kg/m2. His vital signs are within normal limits. Physical examination shows an epigastric mass. The remainder of the examination shows no abnormalities. Which of the following is the most likely diagnosis?

This patient presents with recurrent abdominal pain, vomiting, and a palpable epigastric mass, while his vital signs and physical examination are otherwise unremarkable. His history of alcohol use suggests an underlying condition that is often caused by chronic alcohol use. Pancreatic pseudocyst 55% A pancreatic pseudocyst, which is a complication of acute or chronic pancreatitis, is caused by leakage of pancreatic exocrine secretions from damaged ducts. This patient's history of recurrent epigastric pain, vomiting, and chronic alcohol consumption makes a diagnosis of pancreatitis highly likely. Patients with chronic pancreatitis and heavy alcohol use are often cachectic due to malnutrition. Pancreatic pseudocysts, when large enough, can manifest with a palpable epigastric mass and bilious vomiting (due to extrinsic compression of the distal duodenum) or non-bilious vomiting (due to gastric outlet obstruction). The diagnosis can be confirmed by imaging; a well-defined fluid collection would be seen in the vicinity of the pancreas.

A 3-year-old boy is brought to the physician for the evaluation of recurrent skin lesions. The lesions first appeared at the age of 3 months. He has also had several episodes of respiratory tract infections, enlarged lymph nodes, and recurrent fevers since birth. The boy attends daycare. His immunizations are up-to-date. He is at the 5th percentile for length and 10th percentile for weight. He appears ill. Temperature is 38°C (100.4°F). Examination shows several raised, erythematous lesions of different sizes over the face, neck, groin, and extremities; some are purulent. Bilateral cervical and axillary lymphadenopathy are present. Which of the following is the most likely underlying mechanism of this patient's symptoms?

This patient presents with recurring respiratory infections, lymphadenopathy, and inflammatory, pustular lesions, starting at a very young age. Together, these features suggest impaired functioning of phagocytic cells, making him especially susceptible to infection with catalase-positive organisms (e.g., S. aureus, E. coli, Aspergillus). Defective NADPH oxidase 48% The NADPH oxidase enzyme is responsible for the respiratory burst (oxidative burst) within phagolysosomes, a component of phagocytic cells, which causes a rapid release of reactive oxygen species (ROS) that can destroy bacteria. In patients with defective NADPH oxidase, also known as chronic granulomatous disease (CGD), this reaction does not occur. This leads to impaired function of phagocytic cells and makes patients susceptible to infection, as in this patient, usually with catalase-positive organisms. The nitroblue tetrazolium test and the dihydrorhodamine flow cytometry test are used to check for NADPH oxidase activity and to diagnose chronic granulomatous disease.

A 71-year-old man with hypertension and hyperlipidemia is brought to the emergency department after the sudden onset of slurred speech 1 hour ago. Physical examination shows drooling and hypernasal speech. His face is symmetric and strength is 5/5 in all extremities. This patient's presentation is most consistent with injury to which of the labeled areas of the primary motor cortex?

This patient presents with signs of an ischemic stroke. His slurred speech and drooling indicate pathological changes in the cortical area responsible for the central control of swallowing and vocalization. The motor homunculus represents all parts of the body arranged medially to laterally. A 46% This area of the motor homunculus corresponds with the motor innervation of the muscles of the throat involved in swallowing, as well as the tongue, mastication, and vocalization. Slurred speech and drooling are signs of injury to this area, most likely due to an ischemic stroke.

An 8-year-old boy is brought to the physician by his mother because of a 3-week history of irritability and frequent bed wetting. She also reports that he has been unable to look upward without tilting his head back for the past 2 months. He is at the 50th percentile for height and weight. His vital signs are within normal limits. Ophthalmological examination shows dilated pupils that are not reactive to light and bilateral optic disc swelling. Pubic hair development is Tanner stage 2. The most likely cause of this patient's condition is a tumor in which of the following locations?

This patient presents with vertical gaze palsy and signs of increased intracranial pressure (papilledema). These findings are suggestive of a brain tumor that is causing compression symptoms due to hydrocephalus. Signs of precocious puberty are due to the tumor cell line. Dorsal midbrain 40% Pinealoma is a childhood tumor of the dorsal midbrain that compresses the pretectal region, resulting in vertical gaze palsy. Because of the close proximity to the cerebral aqueduct, hydrocephalus (due to obstruction of cerebrospinal fluid outflow pathway) can also develop, which would explain this patient's increased intracranial pressure. The most common type of pinealoma in children is germinoma, which often secretes β-hCG, stimulating testosterone production and causing precocious puberty in boys.

A 55-year-old man comes to the physician because of episodic retrosternal chest pain and shortness of breath for the past 6 months. His symptoms occur when he takes long walks or climbs stairs but resolve promptly with rest. He has a history of chronic obstructive pulmonary disease, for which he takes ipratropium bromide. His pulse is 81/min and blood pressure is 153/82 mm Hg. Physical examination shows mild expiratory wheezing over both lungs. Additional treatment with a beta blocker is considered. Which of the following agents should be avoided in this patient?

This patient presents with symptoms of chronic stable angina and a history of COPD. Treatment with nonselective beta blockers should be avoided. Stable angina pectoris A type of angina that occurs upon exertion, mental stress, and/or exposure to cold and usually subsides within 20 minutes of rest or after administration of nitroglycerin. Occurs due to a mismatch in myocardial oxygen supply and oxygen demand from underlying coronary artery disease. Labetalol 60% Labetalol is a beta blocker that non-selectively inhibits both beta-1 receptors and beta-2 receptors. The negative inotropic and chronotropic effects of beta-1 receptor antagonism would decrease myocardial oxygen demand. This is beneficial in patients with angina. However, inhibition of beta-2 receptors would lead to bronchoconstriction, which would decrease FEV1 and worsen air trapping in patients with obstructive lung diseases such as COPD and asthma. Therefore, nonselective beta blockers (e.g., labetalol, propranolol, nadolol, pindolol, timolol, carvedilol) should be avoided in this group of patients.

A 61-year-old man with Alzheimer disease is brought to the emergency department 20 minutes after ingesting an unknown amount of his medications in a suicide attempt. He reports abdominal cramps, diarrhea, diaphoresis, and muscular weakness and spasms in his extremities. His temperature is 38.4°C (101.1°F), pulse is 51/min, respirations are 12/min and labored, and blood pressure is 88/56 mm Hg. Physical examination shows excessive salivation and tearing, and small pupils bilaterally. Treatment with atropine is initiated. Shortly after, most of his symptoms have resolved, but he continues to have muscular spasms. Administration of which of the following is the most appropriate next step in treatment of this patient?

This patient presents with symptoms of muscarinic acetylcholine excess (e.g., diarrhea, diaphoresis, bradycardia, bronchospasm, salivation, lacrimation, miosis) and nicotinic acetylcholine excess (e.g., muscle weakness). Atropine can only reverse the muscarinic effects. Pralidoxime 61% When taken in excess, medications for Alzheimer disease such as donepezil can result in symptoms similar to that of organophosphate poisoning. In such cases, pralidoxime is given to restore activity of acetylcholinesterase, which then catalyzes the breakdown of excessive acetylcholine to reverse symptoms. Atropine does not act on nicotinic cholinergic receptors and thus cannot treat symptoms of neuromuscular dysfunction, for which pralidoxime is still necessary. Pralidoxime has poor blood-brain barrier penetration and can lead to a transient worsening of acetylcholinesterase inhibition following administration, which is why atropine is typically administered first to treat CNS symptoms and prevent further acetylcholinesterase inhibition. Pancuronium 11% Pancuronium belongs to the class of nondepolarizing muscle relaxants that bind to acetylcholine receptors at the neuromuscular junction, which blocks nicotinic stimulation by acetylcholine. Even though administration of pancuronium would lead to a reversal of neuromuscular symptoms in this patient, muscular paralysis and possibly even respiratory impairment would result. The risks of pancuronium therefore outweigh its potential benefits, especially considering that safer alternatives are available.

A 40-year-old woman comes to the physician because of a 2-month history of increasingly frequent episodes of vertigo and feeling unsteady while walking. She reports that she has had episodic tinnitus in her right ear for the past 3 years. Neurologic examination shows hearing loss in the right ear. An MRI of the brain is shown. Pathologic examination of this patient's lesion is most likely to show which of the following?

This patient presents with symptoms of vestibulocochlear nerve dysfunction (imbalance, vertigo, tinnitus, hearing loss). The MRI shows a hyperintense lesion in the right cerebellopontine angle, which is consistent with a vestibular schwannoma Hypercellular areas of spindle cells and hypocellular areas of myxoid stroma 57% Hypercellular areas of spindle cells and hypocellular areas of myxoid stroma are characteristic findings of vestibular schwannoma, a tumor arising from the Schwann cells of the vestibulocochlear nerve (CN VIII). Schwann cells are of neural crest origin, which explains why vestibular schwannomas stain positive for S100.

An 8-year-old girl is brought to the emergency room for a 6-hour history of fever, sore throat, and difficulty swallowing. Physical examination shows pooling of oral secretions and inspiratory stridor. Lateral x-ray of the neck shows thickening of the epiglottis and aryepiglottic folds. Throat culture with chocolate agar shows small, gram-negative coccobacilli. The patient's brother is started on the recommended antibiotic for chemoprophylaxis. Which of the following is the primary mechanism of action of this drug?

This patient presents with the thumbprint sign (thickening of the epiglottis) on x-ray of the neck as well as classic clinical features (dysphagia, drooling, distress) of acute epiglottitis due to Haemophilus influenzae infection. The drug of choice for H. influenzae prophylaxis is rifampin. Inhibition of DNA-dependent RNA-polymerase Inhibition of prokaryotic DNA-dependent RNA polymerase is the mechanism of rifampin, the drug of choice for prophylaxis for Haemophilus influenzae. Side effects of rifampin include hepatotoxicity, development of red/orange bodily fluids, and cytochrome P450 enzyme induction. Rifampin may also be used for prophylaxis of meningococcal meningitis.

A 45-year-old woman comes to the physician because of fatigue, irregular menses, and recurrent respiratory infections for the past 6 months. Her blood pressure is 151/82 mm Hg. Physical examination shows a round face, thinning of the skin, and multiple bruises on her arms. Further studies confirm the diagnosis of an ACTH-secreting pituitary adenoma. This patient is at greatest risk for which of the following?

This patient presents with typical findings of hypercortisolism (e.g., moon facies, bruisable skin, irregular menstrual cycle, recurrent infections, hypertension) due to an ACTH-secreting pituitary adenoma (Cushing disease) Pathologic fracture 56% Cushing disease is associated with pathological fractures due to osteoporosis. Excess cortisol inhibits calcitriol and osteoblasts maturation while simultaneously increasing production of RANKL, which stimulates osteoclasts and results in increased levels of bone resorption. Other features of Cushing disease include cataracts, peptic ulcer disease, dyslipidemia, muscle atrophy/weakness, acne, hirsutism, and psychiatric conditions such as generalized anxiety disorder, major depressive disorder, and psychosis. Cushing disease ACTH-secreting pituitary adenoma A condition of increased ACTH production by a pituitary tumor leading to classic Cushing symptomatology, including fatigue, recurrent infections, glucose intolerance, lower extremity swelling, proximal muscle weakness, depression, central and neck obesity, easy bruising, striae.

A 58-year-old woman with HIV infection is brought to the emergency department because of a 2-week history of headaches, blurred vision, and confusion. Her current medications include antiretroviral therapy and trimethoprim-sulfamethoxazole. Neurological examination shows ataxia and memory impairment. Her CD4+ T-lymphocyte count is 90/mm3. Analysis of her cerebrospinal fluid analysis shows lymphocytic predominant pleocytosis, and PCR is positive for Epstein-Barr virus DNA. An MRI of the brain with contrast shows a solitary, weakly ring-enhancing lesion with well-defined borders involving the corpus callosum. Which of the following is the most likely diagnosis?

This patient with HIV and a low CD4 count presents with neurological impairment (ataxia, confusion, memory dysfunction). The likely diagnosis can be inferred from her MRI findings (a solitary ring-enhancing lesion) and CSF analysis result (EBV DNA in the CSF). Primary cerebral lymphoma 63% Primary cerebral lymphoma is a non-Hodgkin lymphoma with a much higher incidence in HIV/AIDS patients than in the general population. It has a strong association with EBV infection and is derived from B cells. The condition can present with focal neurological deficits, neuropsychiatric symptoms, signs of elevated intracranial pressure, and/or seizures. It also results in solitary ring-enhancing cerebral lesions on imaging. If findings on CSF are inconclusive, pathologic examination of a brain biopsy specimen is indicated to confirm the diagnosis prior to starting treatment with chemotherapy and/or radiation.

An 8-year-old girl is brought to the physician by her parents for the evaluation of an episode of unconsciousness while at the playground that morning. She was unconscious for about 15 seconds and did not shake, bite her tongue, or lose bowel or bladder control. Her grandfather died suddenly at the age of 29 of an unknown heart condition; her parents are both healthy. An ECG shows sinus rhythm and a QT interval corrected for heart rate (QTc) of 470 milliseconds. Laboratory studies are within normal limits. Which of the following is the most likely additional finding in this patient?

This patient with a family history of sudden death presents with syncope associated with exercise and a prolonged QTc (> 440 milliseconds) on ECG, both of which indicate the diagnosis of long QT syndrome (LQTS). There is no indication of electrolyte or endocrine abnormalities that could explain this patient's prolonged QT interval, making a congenital rather than acquired LQTS the most likely diagnosis. Long QT syndrome Abbreviation: LQTS A congenital or acquired heart condition in which the QT interval (i.e. ventricular depolarization and repolarization) is prolonged. Most patients remain asymptomatic until they present with syncope, seizures, or life-threatening arrhythmias including torsades de pointes and ventricular fibrillation Sensorineural hearing loss 67% Congenital long QT syndrome can be divided into Romano-Ward syndrome and Jervell and Lange-Nielson syndrome; the latter is associated with sensorineural deafness. Jervell and Lange-Nielson syndrome is autosomal recessive and more likely in this patient with an affected grandparent. This syndrome is caused by a mutation in the gene encoding outward-rectifying potassium channels, which are important in membrane repolarization in cardiac myocytes as well as neurons in the inner ear. Romano-Ward syndrome is autosomal dominant, and while it is more common, it is unlikely in a patient with unaffected parents.

A 38-year-old woman comes to the physician because of a 2-day history of a red, itchy, burning rash on her vulva. She has had three similar episodes over the last two years that have all self-resolved. Genitourinary examination shows a small area of erythema with an overlying cluster of vesicles on the inside surface of the vulva. Latent infection of which of the following is most likely responsible for this patient's recurrent symptoms?

This patient with acute onset of itchy, burning rash and grouped vesicles on genital examination has recurrent genital herpes simplex virus (HSV) infection. Sensory neurons Latency and reactivation in the dorsal root ganglion of host sensory neurons is the mechanism underlying the recurrent symptoms seen in herpes simplex virus 1 and 2 infection. Infection with HSV is generally acquired by direct contact with mucosal tissue or secretions of an infected person. After inoculation, the virus invades, spreads, and replicates in nerve cells, producing either asymptomatic infection or acute gingivostomatitis/genital herpes. Under certain conditions (e.g., immunodeficiency, stress, trauma), HSV may reactivate and multiply in the dorsal root ganglia of the trigeminal (HSV-1) or sacral (HSV-2) nerves, producing recurrent flares, as are seen in this patient.

A 59-year-old man with alcohol use disorder is brought to the emergency department by a friend because of progressively worsening forgetfulness and frequent falls. He appears disheveled. On mental status examination, he is confused and oriented only to person. Neurologic examination shows horizontal nystagmus on lateral gaze. He walks with wide-based, small steps and his gait is unsteady. An MRI of the brain is shown. Supplementation of which of the following is most likely to have prevented this patient's current condition?

This patient with alcohol use disorder presents with confusion, oculomotor dysfunction (e.g., nystagmus), gait ataxia, and a brain MRI showing hyperintensities in the thalamus. These signs and symptoms suggest Wernicke encephalopathy. Vitamin B1 81% Thiamine deficiency leads to neuronal injury and impaired axonal conduction that cause the symptoms of Wernicke encephalopathy (i.e., confusion, oculomotor dysfunction, and gait ataxia). Thiamine (vitamin B1) is a cofactor for critical enzymes involved in cerebral glucose and energy metabolism, which can be depleted in patients with alcohol use disorder and/or malnourishment. Early supplementation of thiamine would most likely have prevented this patient's condition. Vitamin B12 12% Cobalamin (vitamin B12) deficiency can be caused by severe malnutrition associated with alcohol use disorder. It can present clinically with megaloblastic anemia, subacute combined degeneration of the spinal cord, irritability, memory loss, and dementia. However, it does not manifest with the symptoms of Wernicke encephalopathy seen here. Although cobalamin is also used in the long-term treatment of patients with malnutrition and alcohol use disorder, supplementation would not have prevented this patient's condition.

A 31-year-old man comes to the physician because of worsening abdominal pain, an inability to concentrate at work, and a general lack of motivation over the past several months. He has a history of spontaneous passage of two kidney stones. His father and uncle underwent thyroidectomy before the age of 35 for thyroid cancer. Physical examination shows diffuse tenderness over the abdomen. Serum studies show: Na+ 142 mEq/L K+ 3.7 mEq/L Glucose (fasting) 83 mg/dL Ca2+ 12.3 mg/dL Albumin 4.1 g/dL Parathyroid hormone 350 pg/mL Further evaluation is most likely to show elevated levels of which of the following? A young patient with primary hyperparathyroidism (difficulty concentrating, lethargy, recurrent kidney stones, elevated PTH levels, and increased serum calcium) and a family history of thyroid cancer should raise suspicion for multiple endocrine neoplasia type 2A (MEN 2A) syndrome.

Urine metanephrines 56% Increased levels of catecholamine metabolites such as metanephrines in the urine or serum are diagnostic of pheochromocytoma, a catecholamine-secreting neuroendocrine tumor typically located in the adrenal medulla. Pheochromocytomas may be a sign of genetic syndromes such as MEN 2A and MEN 2B due to an autosomal dominant mutation of the RET gene.

A 35-year-old man comes to the Veterans Affairs hospital because of a 2-month history of anxiety. He recently returned from his third deployment to Iraq, where he served as a combat medic. He has had difficulty readjusting to civilian life. He works as a taxi driver but had to take a leave of absence because of difficulties with driving. Last week, he hit a stop sign because he swerved out of the way of a grocery bag that was in the street. He has difficulty sleeping because of nightmares about the deaths of some of the other soldiers in his unit and states, "it's my fault, I could have saved them. Please help me." Mental status examination shows a depressed mood and a restricted affect. There is no evidence of suicidal ideation. Which of the following is the most appropriate initial step in treatment?

This patient's 2-month history of anxiety, hypervigilance, nightmares, and feelings of sadness and guilt following a traumatic event (e.g., the deaths of his fellow soldiers) are highly suggestive of a diagnosis of post-traumatic stress disorder (PTSD). Trauma-focused cognitive behavioral therapy (CBT), with or without pharmacotherapy, is the best initial step to treat this patient's PTSD. This form of psychotherapy is effective both in the short-term and long-term for PTSD. CBT utilizes cognitive restructuring (i.e., thinking differently about the meanings imposed on events) and behavioral modifications (e.g., systemic desensitization, aversion therapy) to change disruptive thought patterns and improve functioning. The exposure therapy aspect of trauma-focused CBT can significantly improve symptoms such as avoidance, hypervigilance, and negative emotions as patients learn to manage their fears within a safe environment.

A 68-year-old woman comes to the emergency department because of abdominal pain for 3 days. Physical examination shows guarding and tenderness to palpation over the left lower abdomen. Test of the stool for occult blood is positive. A CT scan of the abdomen is shown. Which of the following mechanisms best explains the patient's imaging findings?

This patient's CT scan shows a horseshoe kidney. Her abdominal symptoms are likely unrelated to the renal pathology as a horseshoe kidney is often an incidental finding. Impaired organ ascent 67% Impaired organ ascent explains the presence of this horseshoe kidney at an inferior position relative to normal kidneys. The fused inferior poles of the horseshoe kidneys get caught under the inferior mesenteric artery, impeding the normal ascension of the kidneys to their retroperitoneal locations. However, this obstructed migration is generally benign, does not affect kidney function, and is not commonly associated with ischemia or abdominal pain. This radiographic finding is most likely incidental and not associated with this patient's current complaint.

A 63-year-old woman is brought to the emergency department because of a 2-day history of severe epigastric pain and nausea. She has a 20-year history of alcohol use disorder. Nine hours after admission, she becomes increasingly dyspneic and tachypneic. Pulse oximetry on supplemental oxygen shows an oxygen saturation of 81%. Physical examination shows diffuse lung crackles, marked epigastric tenderness, and a periumbilical hematoma. Laboratory studies show normal brain natriuretic peptide. An x-ray of the chest shows bilateral opacities in the lower lung fields. Which of the following pathomechanisms best explains this patient's pulmonary findings?

This patient's abdominal pain, history of alcohol use disorder, and Cullen sign (periumbilical hematoma) suggest acute pancreatitis. Patients with acute pancreatitis are at increased risk of developing severe complications such as acute respiratory distress syndrome. Cullen sign A physical examination finding typically associated with acute pancreatitis. Characterized by periumbilical ecchymosis and bluish-red discoloration. Can also be seen in any disease that causes retroperitoneal hemorrhage (e.g., ruptured AAA) Alveolocapillary membrane leakage 69% Acute pancreatitis produces a substantial systemic inflammatory response, resulting in increased membrane permeability and edema. Within the lungs, this permeability results in an exudative pleural effusion, leading to acute respiratory distress syndrome. This pathophysiology is further supported by this patient's x-ray findings, hypoxia, and dyspnea. Oxygen supplementation and treatment of this patient's pancreatitis will ultimately resolve her lung issues.

A 5-year-old boy is brought to the physician because of an irregular gait 3 days after receiving age-appropriate vaccinations. Examination of the lower extremities shows no redness or swelling. When the child stands on his right leg, his left leg drops and his pelvis tilts towards the left. Sensation to light touch is normal in both legs. This patient's symptoms are most likely due to the injection of the vaccine into which of the following locations?

This patient's abnormal gait is most likely caused by weakness of hip abductors due to iatrogenic nerve injury. Superomedial quadrant of the right buttock 59% The superior gluteal nerve lies in this region and innervates the gluteus medius, gluteus minimus, and tensor fascia latae, which prevent the pelvis from tilting to the contralateral side. Injury to the right superior gluteal nerve would, therefore, cause the pelvis to tilt towards the left side when bearing weight on the right leg (positive Trendelenburg sign). In order to compensate for this pelvic tilt, the patient would tilt the trunk to the right. Intramuscular injections are typically given in the superolateral quadrant of the buttock to avoid nerve injury.

A 26-year-old man comes to the emergency department for evaluation of burning with urination and purulent urethral discharge for the past 3 days. He is sexually active with multiple female partners. Several months ago he was diagnosed with urethritis caused by gram-negative diplococci and received antibiotic treatment with complete resolution of his symptoms. A Gram stain of the patient's urethral discharge shows gram-negative intracellular diplococci. Which of the following properties of the infecting organism most contributed to the pathogenesis of this patient's recurrent infection?

This patient's burning sensation with urination and Gram stain results (gram-negative intracellular diplococci) are consistent with gonococcal urethritis. Neisseria gonorrhoeae has developed several virulence factors that prevent host immune recognition. Variation of expressed pilus proteins 71% Pili are an important virulence factor that mediate the attachment of N. gonorrhoeae to the epithelial cells of a host. Anti-pilin antibodies that are directed against specific pilus proteins could prevent attachment of the pathogen. To circumvent this host immune response, N. gonorrhoeae utilizes DNA-recombination to create a high frequency of antigenic variation of expressed pilus proteins; these proteins thus are no longer recognized by the initial antibodies. This particular virulence factor has made it difficult to create a vaccine that targets the gonococcal pili.

Nissl stain

a neural stain that has an affinity for structures in neuron cell bodies A Nissl body, also known as Nissl substance and Nissl material, is a large granular body found in neurons.

A 61-year-old man with a history of type 1 diabetes mellitus and depression is brought to the emergency department because of increasing confusion and fever over the past 14 hours. Four days ago, he was prescribed metoclopramide by his physician for the treatment of diabetic gastroparesis. His other medications include insulin and paroxetine. His temperature is 39.9°C (103.8°F), pulse is 118/min, and blood pressure is 165/95 mm Hg. Physical examination shows profuse diaphoresis and flushed skin. There is generalized muscle rigidity and decreased deep tendon reflexes. His serum creatine kinase is 1250 U/L. Which of the following drugs is most likely to also cause this patient's current condition?

This patient's clinical features of hyperthermia, generalized muscle rigidity, altered mental status, autonomic instability (tachycardia, hypertension) in combination with elevated creatine kinase (indicating muscle breakdown) suggest neuroleptic malignant syndrome (NMS). Fluphenazine 53% High-potency antipsychotics such as fluphenazine are the most common cause of neuroleptic malignant syndrome (NMS), but low-potency or atypical antipsychotics, as well as antiemetics like metoclopramide (which was likely the triggering substance in this patient), have also been associated with NMS. While symptoms usually develop during the first 2 weeks of therapy, as seen in this patient, NMS may also develop after years of continuous therapy and without any associated increase of dose. Nonetheless, higher doses or switching antipsychotics are risk factors for the development of NMS.

A 53-year-old man comes to the physician because of a 3-month history of cough and progressively worsening difficulty walking up the stairs in his apartment. He has noticed that it is easier for him to climb the stairs after he has exercised. He has also had a 4.6-kg (10.1-lb) weight loss over the past 6 months. He has smoked one pack of cigarettes daily for 35 years. Examination shows dry mucous membranes. The pupils are equal and react sluggishly to light. Muscle strength in the proximal lower extremity is initially 3/5 but increases to 5/5 after repeated muscle tapping. His achilles reflex is 1+. Which of the following is the most likely diagnosis?

This patient's combination of a smoking history, chronic cough, and weight loss are concerning for a lung malignancy. Proximal muscle weakness that improves on repeated stimulation, combined with decreased deep tendon reflexes are suggestive of a paraneoplastic syndrome associated with lung cancer. Lambert-Eaton syndrome 88% Lambert-Eaton myasthenic syndrome (LEMS), which is associated with small-cell lung cancer in two-thirds of cases, is caused by circulating autoantibodies that block presynaptic voltage-gated calcium channels at the neuromuscular junction. This blockage impairs acetylcholine release which leads to reduced or absent deep tendon reflexes and proximal muscle weakness; the latter improves during exercise as repetitive nerve stimulation increases the amount of ACh in the synaptic cleft. Patients with LEMS may also develop symptoms of autonomic dysfunction such as a dry mouth, constipation, or impotence.

A 53-year-old woman with hypertension is brought to the emergency department 30 minutes after having a generalized, tonic-clonic seizure. She has had recurrent headaches and dizziness in the last 3 months. One year ago, she had diarrhea after a trip to Ecuador that resolved without treatment. She has not received any medical care in the last five years. She has smoked 1 pack of cigarettes daily for 20 years. Her temperature is 36°C (96.8°F) and blood pressure is 159/77mm Hg. Physical examination shows dysarthria and hyperreflexia. She is confused and oriented only to name and place. Four brain lesions are found in a CT scan of the head with contrast; one of the lesions is shown. Which of the following is most likely to have prevented this patient's condition?

This patient's combination of seizure, headache, dizziness, dysarthria, hyperreflexia, confusion, recent travel to South America, and ring-enhancing, cystic lesions on head CT scan indicates a diagnosis of neurocysticercosis. Avoidance of contaminated food 53% Avoidance of food contaminated by the eggs of Taenia solium reduces the risk of neurocysticercosis, which is the most common parasitic disease of the nervous system and the most important cause of adult-onset seizures in Asia, Africa, Central and South America. Neurocysticercosis is a fecal-oral transmitted disease that results from ingestion of eggs of Taenia solium, which can be found in food and water contaminated by the feces of people with intestinal taeniasis. Several strategies have been described to prevent the infection, which include washing food prior to consumption, proper hand hygiene, and avoidance of raw pork meat, as well as adequate freezing and cooking of food to destroy the parasite.

A previously healthy 41-year-old woman comes to the physician for the evaluation of recurrent episodes of palpitations and sweating over the past month. Her symptoms typically start after swimming practice and improve after drinking ice tea and eating some candy. She has also had a 5-kg (11-lb) weight gain over the past 3 months. She works as a nurse. Physical examination shows no abnormalities. Fasting serum studies show: Glucose38 mg/dLInsulin260 μU/mL (N=11-240)Proinsulin65 μU/mL (N <20% of total insulin)C-peptide5.0 ng/mL (N=0.8-3.1)Insulin secretagoguesabsent Which of the following is the most likely cause of her symptoms?

This patient's condition can be associated with MEN 1 syndrome. Multiple endocrine neoplasia type 1 A collection of endocrine neoplasias caused by an alteration in menin protein expression. Characterized by primary hyperparathyroidism, often in association with endocrine pancreatic tumors and/or pituitary adenomas Pancreatic β-cell tumor 70% Insulin is secreted by pancreatic β-cells and promotes cellular uptake of glucose. Thus, a pancreatic β-cell tumor (insulinoma) manifests as hypoglycemia, which would explain this patient's post-exertional autonomic symptoms (sweating and palpitations) that resolve after consuming sugary foods. These features, together with the documented hypoglycemia are known as Whipple triad, a characteristic feature of insulinoma. Insulin also stimulates lipogenesis and inhibits lipolysis, which explains the weight gain in this patient. As C-peptide is released during cleavage of insulin from proinsulin, the high levels of serum C-peptide and proinsulin on serum analysis confirm that the hyperinsulinemia is endogenous, making insulinoma the most likely diagnosis. Whipple's triad A triad that consists of 1) hypoglycemic symptoms (e.g., sweating, palpitations) that typically follow exertion or fasting; 2) documented hypoglycemia during an episode, and 3) relief of symptoms upon administration of glucose. Characteristically seen in patients with insulinoma.

A 54-year-old woman is brought to the emergency department because of a 2-hour history of nausea and retrosternal chest pain. She has a 15-year history of type 2 diabetes mellitus. Her current medications include atorvastatin, metformin, and lisinopril. She is diaphoretic. Her serum troponin level is 3.0 ng/mL (N: < 0.04). She undergoes cardiac catheterization. A photograph of coronary angiography performed prior to percutaneous coronary intervention is shown. An acute infarct associated with the finding on angiography is most likely to manifest with ST elevations in which of the following leads on ECG?

V1-V6 70% ST elevations typically develop in leads V1-V6 on ECG as a result of infarcts that involve the proximal left anterior descending artery (LAD). Leads V1-V2 correspond to the anteroseptal territory; leads V3-V4 correspond to the anteroapical territory; leads V5-V6 correspond to the anterolateral territory. In some cases, ST elevations in the lateral leads (leads I and aVL) can also develop.

A 62-year-old man is brought to the physician by his wife for increased forgetfulness and unsteady gait over the past 3 months. He is a journalist and has had difficulty concentrating on his writing. He also complains of urinary urgency recently. His temperature is 36.8°C (98.2°F) and blood pressure is 139/83 mm Hg. He is oriented only to person and place. He is able to recall 2 out of 3 words immediately and 1 out of 3 after five minutes. He has a slow, broad-based gait and takes short steps. Neurological examination is otherwise normal. Urinalysis is normal. Which of the following is the most likely diagnosis?

This patient's confusion, ataxic gait, and urinary symptoms suggest a cause of reversible dementia in older adults. Normal pressure hydrocephalus 78% Normal pressure hydrocephalus (NPH) manifests with gait instability, cognitive disturbance, and urinary incontinence - the classic "wet, wacky, and wobbly" triad. Gait changes are typically symmetrical, while cognitive impairment primarily takes the form of decreased attention span and concentration. NPH is an important diagnosis to consider in a patient with dementia, as it may be reversible if the hydrocephalus is addressed (e.g., with a lumbar puncture to remove CSF). In most patients > 60 years of age, the accumulation of CSF is idiopathic, but it may also occur due to secondary causes (e.g., inflammation, hemorrhage) that lead to obstruction of the subarachnoid villi that normally resorb CSF in the ventricular system.

A previously healthy 21-year-old man is brought to the emergency department 4 hours after the sudden onset of shortness of breath and pleuritic chest pain. He has smoked 1 pack of cigarettes daily for the past 3 years. He is 188 cm (6.2 ft) tall and weighs 70 kg (154 lb); BMI is 19.8 kg/m2. Physical examination shows decreased tactile fremitus and diminished breath sounds over the left lung. Which of the following is the most likely cause of this patient's symptoms?

This patient's current condition typically affects patients without pre-existing lung disease, particularly young male patients with an asthenic body habitus (tall and thin) and a history of smoking. Asthenia A term that can refer to: 1. An abnormal loss of strength. 2. A slender, tall body type.

A 4-month-old boy is brought to the physician because of a painful lesion on his right leg. Yesterday, he received all of his scheduled childhood immunizations, including the second dose of the diphtheria, tetanus, and acellular pertussis vaccine, which was administered in the right quadriceps. Physical examination shows a 2-cm ulcer with surrounding induration over the right anterolateral thigh. A skin biopsy of the lesion shows localized edema, skin necrosis, neutrophilic infiltration, and complement deposition. Which of the following is the most likely cause of his symptoms?

This patient's cutaneous reaction after receiving his second diphtheria and tetanus vaccine is consistent with an Arthus reaction. Arthus reaction A rare, localized, type III hypersensitivity reaction characterized by immune complex deposition in the walls of dermal blood vessels. Caused by an immune response via complement activation after intradermal injection of antigens (e.g., tetanus or diphtheria toxoid). Manifests with pain, swelling, erythema, and occasionally necrosis at the injection site. Immune complex deposition 35% Immune complex deposition describes the mechanism of Arthus reaction, a localized type III hypersensitivity reaction that can occur 12 - 36 hours after intradermal injection of an antigen in pre-sensitized individuals (circulating antigen-specific IgG). This child was most likely sensitized to an antigen in the DTaP vaccine when he received it for the first time at 2 months of age (routine immunization schedule). DTaP is administered intramuscularly but inadvertent intradermal injection often occurs. The circulating antibodies combine with the antigen to form immune complexes, which trigger localized vasculitis and fibrinoid necrosis by activating the classical complement pathway. An Arthus reaction is usually self-limiting.

A 34-year-old man comes to the physician because of foul-smelling diarrhea, fatigue, and bloating for 6 months. During this time, he has had a 5-kg (11-lb) weight loss without a change in diet. He has type 1 diabetes mellitus that is well-controlled with insulin. Examination shows conjunctival pallor and inflammation of the corners of the mouth. The abdomen is soft, and there is diffuse tenderness to palpation with no guarding or rebound. His hemoglobin concentration is 10.4 g/dL. The patient undergoes upper endoscopy. A photomicrograph of tissue from an intestinal biopsy is shown. Which of the following is most likely to improve this patient's symptoms?

This patient's duodenal biopsy shows villous flattening, elongated and hyperplastic crypts of Lieberkuhn, and lymphocytic infiltration of the lamina propria. Avoidance of certain types of cereal grains 72% Foul-smelling diarrhea, fatigue, bloating, unintentional weight loss, abdominal tenderness, and signs of iron deficiency anemia (low hemoglobin, conjunctival pallor, angular cheilitis) should raise suspicion for celiac disease, for which type 1 diabetes mellitus is a risk factor. The diagnosis is confirmed by the presence of characteristic histology findings together with positive celiac serology. While the disease can occur at any age, the age of onset follows a bimodal distribution; it most often affects infants and adults in the 3rd and 4th decade of life. Treatment consists of a lifelong gluten-free diet, including avoidance of certain types of cereal grains such as wheat, rye, barley, and spelt.

Three days after starting a new medication, a 66-year-old woman with resistant hypertension and hyperlipidemia comes to the doctor because of decreased urination. She denies dysuria, fevers, or chills. Blood pressure is 133/67 mm Hg. Physical examination shows no abnormalities. Compared to measurements from one week ago, the glomerular filtration rate is markedly decreased and plasma renin activity is elevated. A duplex ultrasound of the kidneys shows increased systolic flow velocity in both renal arteries when compared to the aorta. Which of the following drugs is the most likely cause of this patient's worsening renal function?

This patient's duplex ultrasound shows findings consistent with renal artery stenosis, which manifests with increased flow velocity from the narrowed artery. Enalapril 77% ACE inhibitors like enalapril inhibit efferent arteriole constriction, which can lead to a decrease in the GFR and acute kidney injury, as seen here. Patients with bilateral renal artery stenosis are especially susceptible to this effect, as the kidneys rely on efferent arteriole constriction to maintain the GFR. Angiotensin II receptor blockers (ARBs) work in a similar fashion. Therefore, both ARBs and ACEIs are relatively contraindicated in patients with bilateral renal stenosis.

A 32-year-old woman comes to the physician because of a 4-day history of low-grade fever, joint pain, and muscle aches. The day before the onset of her symptoms, she was severely sunburned on her face and arms during a hike with friends. She also reports being unusually fatigued over the past 3 months. Her only medication is a combined oral contraceptive pill. Her temperature is 37.9°C (100.2°F). Examination shows bilateral swelling and tenderness of the wrists and metacarpophalangeal joints. There are multiple nontender superficial ulcers on the oral mucosa. The detection of antibodies directed against which of the following is most specific for this patient's condition?

This patient's fever, arthritis, increased photosensitivity, and painless oral ulcers are highly suggestive of a diagnosis of systemic lupus erythematosus (SLE). Nuclear Sm proteins Antibodies to nuclear Sm proteins (i.e., anti-Smith antibodies) are a highly specific marker for SLE. However, they are only positive in 30% of patients, meaning they are not a very sensitive marker. For this reason, the diagnosis of SLE is always based on clinical features as well as immunologic test results. Another highly specific antibody for the diagnosis of SLE targets double-stranded DNA. Anti-dsDNA antibodies are present in 70% of patients with SLE (high sensitivity), correlate with disease activity, and are associated with lupus nephritis. Anti-Sm antibody Anti-Smith antibody An autoantibody against Smith antigens (nonhistone nuclear proteins) which are highly specific (but only 30% sensitive) for systemic lupus erythematosus. Systemic lupus erythematosus Abbreviation: SLE A multisystem autoimmune disease that particularly affects women of childbearing age and leads to chronic inflammatory reactions in a variety of organs, including the skin, kidney, and joints. Typical findings include fever and fatigue, a malar rash (facial "butterfly rash"), myalgia, and arthritis.

A 56-year-old man comes to the emergency department because of a 3-day history of severe epigastric pain that is radiating to his back and accompanied by nausea and vomiting. He has a history of alcohol use disorder. His blood pressure is 90/60 mm Hg and his pulse is 110/min. Physical examination shows diffuse abdominal tenderness and distention. Laboratory studies show: SerumLipase180 U/L (N < 50)Amylase150 U/LCreatinine2.5 mg/dLUrineSodium45 mEq/LOsmolality280 mOsmol/kg H2OEpithelial cell castsnumerous Laboratory studies from a recent office visit were within normal limits. This patient's condition is most likely to affect which of the following kidney structures first?

This patient's history, symptoms, and elevated lipase and amylase indicate acute pancreatitis, a condition that can lead to hypovolemia and associated prerenal acute kidney injury (AKI). Prolonged prerenal AKI and the resulting renal hypoxia can lead to intrinsic AKI, which is the likely diagnosis in a patient with elevated serum creatinine, urine Na+> 40 mEq/L, and urine osmolality < 350 mOsmol/kg. Certain renal structures are highly susceptible to ischemic injury because of their location. Straight segment of proximal tubule 39% The straight segment of proximal tubule (PST) and the distal straight tubule (aka thick ascending limb, or TAL) are both located in the outer renal medulla and are the nephron segments damaged first and most severely during renal hypoperfusion. In contrast to the renal cortex, in which the rate of blood flow is high to facilitate glomerular filtration and resorption, the rate of blood flow in the renal medulla is lower to facilitate concentration of the urine via osmotic gradients (i.e., medullary hypertonicity). Moreover, the PST and TAL carry out active transport of electrolytes, a process which requires oxygen and therefore renders them particularly vulnerable to ischemia following hypovolemic vasoconstriction. The presence of brownish (muddy) granular renal tubular epithelial cell casts in his urine is evidence of acute tubular necrosis, the cause of ∼ 85% of intrinsic AKIs and most commonly the result of ischemic changes to the PST and/or TAL.

A 69-year-old woman comes to the physician because of a 4-month history of cough with blood-tinged sputum and a 4.5-kg (10-lb) weight loss. She has smoked one pack of cigarettes daily for 38 years. Auscultation of the lungs shows wheezing in the right lung field. An x-ray of the chest shows an irregular lesion with a central cavity in the proximal right lung. A lung biopsy shows malignant cells that express desmoglein and stain positive for cytokeratin. Which of the following findings confers the worst prognosis in this patient?

This patient's history, the proximal lesion on x-ray, and biopsy findings are consistent with squamous cell carcinoma of the lung. Tumor staging provides the most important prognostic value for survival. Mediastinal invasion 80% Mediastinal invasion by cancer cells classifies a tumor as T4 per TNM classification system and would confer the worst prognosis in this patient. Besides tumor size and invasiveness (T), the TNM score includes the extent of lymph node involvement (N) and the presence of distant metastases (M), both of which are typically the most important prognostic factors for survival. Staging as a measure of tumor spread is generally more significant in determining the prognosis than tumor grading.

A 73-year-old woman is brought to the physician by her daughter for evaluation of impaired memory and word-finding difficulties for 2 years. She was recently asked to step down from her position as volunteer accountant for a local charity organization because she was no longer able to coordinate her tasks. She reports that she has become unable to taste or smell her food. Two years later, the patient dies. At autopsy, examination of the brain shows generalized cortical atrophy. A photomicrograph of a section of the brain is shown. The inclusions indicated by the arrows are predominantly composed of which of the following substances?

This patient's memory deficits, amnesic aphasia, hyposmia (and associated hypogeusia), and generalized cortical atrophy are suggestive of Alzheimer disease, the most common type of dementia Hyperphosphorylated tau 67% Hyperphosphorylated tau constitutes the greater part of neurofibrillary tangles, which are seen in the photomicrograph. Tau proteins are microtubule-associated proteins that play a central role in the preservation of cell structure. Activation of glycogen synthase kinase-3 and other protein kinases induces increased phosphorylation of tau which leads to cluster formation and development of tangles. These tangles impair neuronal function and eventually cause neuronal death, which manifests as gradual onset of dementia, as was seen in this patient. Amyloid-β 19% Amyloid-β, a derivative of amyloid precursor protein, accumulates in patients with Alzheimer disease and plays a major role of its underlying pathogenesis. This patient did have Alzheimer disease, but amyloid-β aggregates are found extracellularly in the form of senile plaques, not intracellularly as the inclusions seen in this image. Neurofibrillary tangle A collection of intracellular aggregates composed of hyperphosphorylated tau. Accumulation of neurofibrillary tangles impairs neuronal function and leads to cell death. Associated with Alzheimer disease; the severity of tangle formation correlates with the degree of dementia.

Prosopagnosia

a neurological condition characterized by the inability to recognize the faces of familiar people.

A 62-year-old woman with type 2 diabetes mellitus comes to the physician because of a 1-year history of progressively worsening paresthesias in her upper limbs. She has accidentally burned her fingers on hot dishes several times. She was involved in a motor vehicle collision 3 years ago. Neurologic examination shows absent temperature sensation with normal fine touch sensation over the upper extremities and chest. Without treatment, this patient is at increased risk of developing which of the following?

This patient's neurologic exam cape-like dissociated sensory loss typical of central cord syndrome, which is most likely the result of syringomyelia secondary to a whiplash injury. Central cord syndrome (CCS) is the most common form of cervical spinal cord injury. It is characterized by loss of motion and sensation in arms and hands. It usually results from trauma which causes damage to the neck, leading to major injury to the central corticospinal tract of the spinal cord.[1] The syndrome is more common in people over the age of 50 because osteoarthritis in the neck region causes weakening of the vertebrae. CCS most frequently occurs among older persons with cervical spondylosis,[2] however, it also may occur in younger individuals.[3] Syringomyelia is a generic term referring to a disorder in which a cyst or cavity forms within the spinal cord. Often, syringomyelia is used as a generic term before an etiology is determined. [3]This cyst, called a syrinx, can expand and elongate over time, destroying the spinal cord. The damage may result in loss of feeling, paralysis, weakness,[4] and stiffness in the back, shoulders, and extremities. Syringomyelia may also cause a loss of the ability to feel extremes of hot or cold, especially in the hands. It may also lead to a cape-like bilateral loss of pain and temperature sensation along the upper chest and arms. Each patient experiences a different combination of symptoms. These symptoms typically vary depending on the extent and, often more critically, on the location of the syrinx within the spinal cord. Drooping of the eyelid (ptosis) is part of the triad of Horner syndrome. This patient's presentation (dysesthetic pain, absent temperature sensation, normal fine touch sensation) and history of a motor vehicle accident indicate syringomyelia. Without treatment, expansion of the syrinx can damage descending hypothalamic fibers in the T1 to T4 segments, which carry sympathetic fibers from the hypothalamus to the intermediate horn, resulting in central Horner syndrome.

A 45-year-old woman comes to the physician for a health maintenance examination. Her father has a history of type 2 diabetes mellitus. She has gained 18-kg (40-lb) since the birth of her son 6 years ago. Her BMI is 31 kg/m2. Physical examination shows hyperpigmented plaques in the axillae and posterior neck. Laboratory studies show a hemoglobin A1c of 6.4% and an elevated serum insulin level. Which of the following factors is most likely involved in the pathogenesis of this patient's condition?

This patient's obesity, acanthosis nigricans (hyperpigmented plaques in the axillae and neck), elevated serum insulin, and elevated HbA1c are consistent with insulin resistance. Acanthosis nigricans A skin disorder characterized by patches of velvety hyperpigmentation in the axilla, groin, and/or neck. Skin tags can also develop in affected areas. Classified as benign (e.g., from endocrine or drug adverse effects) or malignant (e.g., as a paraneoplastic syndrome). Increased levels of serum fatty acids 54% Elevated plasma free fatty acids (which is likely in this patient given her obesity) decrease insulin-mediated glucose uptake into tissues, resulting in insulin resistance, hyperglycemia, and increased risk of type 2 diabetes. When insulin resistance first develops, the pancreas is able to compensate by increasing insulin secretion. Over the course of the disease, insulin resistance progresses and the ability of the pancreas to overcome this resistance declines, resulting in decreased endogenous insulin production and subsequent hyperglycemia.

A 13-year-old boy is brought to the physician by his parents because of several bumps on his fingers. He has no history of serious illness and is up-to-date on all immunizations. Physical examination shows several superficial, flesh-colored, hyperkeratotic papules along the dorsal aspect of the fingers with minimal surrounding hyperpigmentation. The remainder of the physical examination is normal. A virus with which of the following characteristics is most likely responsible for this patient's findings? EnvelopeGenome typeGenome shapeCapsid shapeAYesssRNAcircularhelicalBYesssRNAlinearhelicalCYesdsDNAlinearcomplexDYesdsDNAlinearicosahedralENodsDNAcircularicosahedralFNossRNAlinearicosahedral

This patient's physical examination findings suggest common warts, which are caused by HPV. E 37% HPV is a nonenveloped DNA virus with a circular, double-stranded genome and icosahedral capsid shape. Certain serotypes of HPV (particularly 1, 2, and 4) are the cause of common warts, which appear as skin-colored, scaly papules (sometimes with a cauliflower-like appearance) on the elbows, knees, fingers, and/or palms. This type of wart is most common in childhood and adolescence. Common warts are transmitted via close person-to-person contact, but other serotypes of HPV are sexually transmitted and can cause a variety of pathologies, including benign anogenital warts (serotypes 6 and 11) and cervical cancer (serotypes 16 and 18).

A 48-year-old man comes to the physician because of a 2-day history of fever, flank pain, and hematuria. He has chronic back pain, for which he has been taking meloxicam for the past 2 weeks. His temperature is 38.9°C (102°F). Physical examination shows a diffuse maculopapular rash over his trunk and extremities. Urinalysis shows 10-15 RBC/hpf, 20 WBC/hpf, and numerous eosinophils. Histologic examination of a kidney biopsy specimen is most likely to show which of the following findings?

This patient's presentation (2-day fever, flank pain, maculopapular rash) and urine abnormalities (hematuria, sterile pyuria, eosinophiluria) are consistent with a hypersensitivity reaction, most likely due to his NSAID intake (meloxicam) Interstitial T-cell infiltration 64% Diffuse interstitial T-cell and monocyte infiltration is seen in allergic interstitial nephritis (AIN), a hypersensitivity reaction that can occur days to weeks after the initiation of NSAIDs, such as meloxicam. Drugs that act as haptens (e.g., NSAIDs, antibiotics) have the ability to elicit an uncontrolled immune response and acute kidney injury. Hapten A small molecule that binds to bigger carriers and triggers a hypersensitivity reaction. Does not elicit an immune reaction on its own. Examples include NSAIDs, diuretics, allopurinol, phenytoin, and some antibiotics.

A 67-year-old man is brought to the emergency department because of the sudden onset of severe substernal chest pain at rest. He has a history of hypertension, type 2 diabetes mellitus, and alcohol use disorder. He is diaphoretic and appears anxious. The lungs are clear to auscultation. An ECG shows ST-segment elevations in leads I, aVL, V5, and V6. One hour later, he develops dyspnea and a productive cough with frothy sputum. Which of the following best describes the most likely underlying pathophysiology of this patient's dyspnea?

This patient's presenting signs and symptoms (chest pain and ST-segment elevations) suggest an acute anterolateral myocardial infarction, most likely involving the left circumflex artery. Transudation of plasma into the alveoli 67% Considering this patient's recent myocardial infarction, his clinical findings of dyspnea and cough are consistent with pulmonary edema as a result of acute left heart failure. Transudation of plasma into the alveoli is the underlying pathomechanism of the acute cardiogenic pulmonary edema. It is caused by a sudden elevation in pulmonary capillary pressure due to a decline in systolic and diastolic function, without accompanying change in the permeability of the pulmonary vascular endothelium.

A 62-year-old man comes to the physician because of a 5-day history of swelling in his left arm. Two months ago, he was diagnosed with a deep venous thrombosis in the left calf. He has had a 7-kg (15-lb) weight loss in the last 3 months. He has smoked 1 pack of cigarettes daily for the past 25 years. His only medication is warfarin. Physical examination shows warm edema of the left forearm with overlying erythema and a tender, palpable cord-like structure along the medial arm. His lungs are clear to auscultation bilaterally. Duplex sonography shows thrombosis of the left basilic and external jugular veins. Which of the following is the most appropriate next step to confirm the underlying diagnosis?

This patient's presents with a history of thrombosis and current superficial thrombophlebitis involving different anatomical locations, which is consistent with migratory thrombophlebitis. The presence of this sign should always raise suspicion for an underlying hypercoagulable state. CT scan of the abdomen 36% A CT scan of the abdomen is most likely to confirm the diagnosis of Trousseau syndrome, a condition that is characterized by recurrent venous thrombosis in different veins with evidence of simultaneous thrombophlebitis (warm, red, swollen skin with tender cord-like structures) as seen here. This patient's older age, smoking history, and weight loss raise concern for a malignancy-induced hypercoagulable state. Trousseau syndrome is most often seen in the setting of a visceral malignancy (classically, pancreatic adenocarcinoma) due to the production of procoagulants by the malignant cells. Thrombophlebitis migrans A form of superficial thrombophlebitis that is recurrent and migratory (i.e., occurring in various locations, resolving, then appearing elsewhere). Referred to as Trousseau syndrome if related to underlying malignancy (most commonly pancreatic adenocarcinoma and bronchial carcinoma). It is also a finding of thromboangiitis obliterans and Behçet disease.

A 32-year-old woman with a recurrent vesicular genital rash comes to the physician because of a 3-day history of a painful, pruritic rash that began on the extremities and has spread to her trunk. Her only medication is acyclovir. Her temperature is 38.1°C (100.6°F). Examination of the skin shows several reddish-purple papules and macules, some of which have a dusky center with a lighter ring around them. Which of the following is the most likely diagnosis?

This patient's rash is most commonly associated with herpes simplex infections and is characterized by target lesions (macules with a dusky center surrounded by a lighter ring).

One week after being involved in a bicycling accident, a 32-year-old woman comes to the physician because of intermittent double vision. She reports worsening of symptoms when she tries to type on her computer or while buttoning her shirts. Physical examination shows a slight right-sided head tilt. Her left eye is deviated outwards and upwards, which becomes even more prominent when she attempts left eye adduction. This patient's symptoms are most likely due to impaired innervation to which of the following muscles?

This patient's right-sided head tilt, outward and upward deviated eye, and history of blunt trauma to the head suggest palsy of the left trochlear nerve (CN IV). Superior oblique 73% The superior oblique muscle is innervated by the trochlear nerve (CN IV) and is responsible for internal rotation (incyclotorsion) and depression of the eye during adduction. Impairment of this muscle manifests with vertical and torsional diplopia that is worsened in downgaze; expected examination findings include ipsilateral external rotation (excyclotorsion) and upward deviation (hypertropia) of the affected eye that is greater when the gaze is directed downward and horizontally towards the contralateral eye, as well as during ipsilateral tilting of the head. The contralateral tilting of the head in this patient may be conscious or unconscious and serves as a compensatory mechanism.

A 69-year-old man is brought to the emergency department by his wife because of fever, cough, diarrhea, and confusion for 2 days. He recently returned from a cruise to the Caribbean. He has a history of chronic obstructive pulmonary disease. He has smoked one pack of cigarettes daily for 40 years. His temperature is 39.1°C (102.4°F), pulse is 83/min, and blood pressure is 111/65 mm Hg. He is confused and oriented only to person. Physical examination shows coarse crackles throughout both lung fields. His serum sodium concentration is 125 mEq/L. Culture of the most likely causal organism would require which of the following mediums?

This patient's risk factors (age, recent travel on a cruise ship, history of smoking, and chronic obstructive pulmonary disease) and symptoms (fever, relative bradycardia, nonproductive cough, diarrhea, confusion, hyponatremia) are characteristic of Legionnaires' disease, which is caused by Legionella pneumophila. Charcoal yeast extract agar 77% Charcoal yeast extract agar (buffered with cysteine and iron) is the medium used to culture and identify Legionella pneumophila, the likely cause of this patient's symptoms. Urinary antigen tests and polymerase chain reaction are additional methods of identifying the L. pneumophila pathogen.

A 29-year-old man comes to the physician for a follow-up examination. He has had numbness and tingling of his legs for the past 10 days. Three months ago, he was diagnosed with pulmonary tuberculosis and started on antituberculosis therapy. Examination shows dry scaly lips and cracks at the corner of the mouth. Sensation to pinprick and light touch is decreased over the lower extremities. His hemoglobin concentration is 10.4 g/dL and mean corpuscular volume is 76 μm3. Administration of which of the following is most likely to have prevented this patient's current symptoms?

This patient's sensory neuropathy and microcytic anemia are the result of isoniazid toxicity Pyridoxine Since pyridoxine (vitamin B6) is an essential cofactor in the synthesis of neurotransmitters and heme, patients with a chronic pyridoxine deficiency typically present with sensory peripheral neuropathy and hypochromic, microcytic anemia (sideroblastic anemia), as seen here. While vitamin B6 deficiency is rare in the general population, it is commonly seen in association with isoniazid (INH) use in antituberculosis treatment, because INH competes with pyridoxine in neurotransmitter synthesis. To prevent peripheral neuropathy, supplemental pyridoxine should therefore be added to every treatment regimen that includes INH.

A 51-year-old man comes to the physician because of a 4-day history of fever and cough productive of foul-smelling, dark red, gelatinous sputum. He has smoked 1 pack of cigarettes daily for 30 years and drinks two 12-oz bottles of beer daily. An x-ray of the chest shows a cavity with air-fluid levels in the right lower lobe. Sputum culture grows gram-negative rods. Which of the following virulence factors is most likely involved in the pathogenesis of this patient's condition?

This patient's sputum culture with gram-negative rods and history of foul-smelling, dark red, gelatinous sputum (currant jelly sputum) is consistent with a pulmonary infection caused by Klebsiella pneumoniae. Capsular polysaccharide 75% Klebsiella pneumoniae possess a capsular polysaccharide that acts as an antiphagocytic virulence factor and allows bacteria to evade host immune defenses. This pathogen is part of the natural flora of the gastrointestinal tract. In patients who are immunocompromised and patients who have an increased risk for aspiration events (e.g., due to chronic alcohol use), aspiration of Klebsiella can lead to destruction of alveoli, resulting in bloody sputum with a currant jelly appearance.

A 34-year-old woman comes to the physician for evaluation of a breast lump she noticed 2 days ago while showering. She has no history of major illness. Her mother died of ovarian cancer at age 38, and her sister was diagnosed with breast cancer at age 33. Examination shows a 1.5-cm, nontender, mobile mass in the upper outer quadrant of the left breast. Mammography shows pleomorphic calcifications. Biopsy of the mass shows invasive ductal carcinoma. The underlying cause of this patient's condition is most likely a mutation of a gene involved in which of the following cellular events?

This patient's strong family history of early-onset breast and ovarian cancer is concerning for a mutation of the BRCA tumor suppressor gene(s). BRCA mutation BRCA+ An autosomal-dominant inherited gene mutation (BRCA1 or BRCA2) of a tumor suppressor gene that codes for a DNA repair protein. Associated with an increased risk of breast cancer (∼ 70%) and ovarian cancer as well as, to a lesser extent, colon, pancreas, stomach and prostate cancer. BRCA is an abbreviation for BReast CAncer. Repair of double-stranded DNA breaks 59% The BRCA gene encodes for a DNA repair protein that conducts homologous end joining, a mechanism that repairs double-stranded DNA breaks. If BRCA is mutated and dsDNA breaks are not repaired, other DNA mutations can occur that increase the risk of cancer. Thus, BRCA is a tumor suppressor gene. As with other tumor suppressor genes, both alleles must have loss of function for the risk of cancer to be significantly increased.

A 5-day-old boy is brought to the emergency department by his mother because of a 2-day history of difficulty feeding and multiple episodes of his lips turning blue. He was born at home via spontaneous vaginal delivery and Apgar scores were 7 and 8 at 1 and 5 minutes, respectively. Physical examination shows grunting and moderate intercostal and subcostal retractions. Echocardiography shows a single vessel exiting from the heart. Which of the following is the most likely underlying cause of this patient's condition?

This patient's symptoms (poor feeding, cyanosis, respiratory distress) in early infancy and echocardiography results are consistent with persistent truncus arteriosus. Persistent truncus arteriosus A congenital heart defect caused by failed separation of the aorta and pulmonary artery during development. Characterized by a single trunk that receives output from both ventricles. Results in cyanosis, respiratory distress, and heart failure within the first few weeks of life. Failure of neural crest cell migration 55% During cardiac development, failure of neural crest cell migration leads to a lack of aorticopulmonary septum formation, resulting in a single outflow vessel that receives output from both the right and left ventricles; this condition is known as persistent truncus arteriosus. The mixing of oxygen-rich blood from the left ventricle with the oxygen-poor blood from the right ventricle leads to a decrease in overall oxygenation of the blood going into the systemic circulation, causing cyanosis, as seen in this patient with episodic blue lips. This patient also has poor feeding, grunting, and intercostal and subcostal retractions, which are concerning for respiratory distress caused by pulmonary congestion secondary to an increase in overall blood flow to the lungs.

A 29-year-old woman, gravida 1, para 0, at 36 weeks' gestation is brought to the emergency department after an episode of dizziness and vomiting followed by loss of consciousness lasting 1 minute. She reports that her symptoms started after lying down on her back to rest, as she felt tired during yoga class. Her pregnancy has been uncomplicated. On arrival, she is diaphoretic and pale. Her pulse is 115/min and blood pressure is 90/58 mm Hg. On examination, the patient is lying in the supine position with a fundal height of 36 cm. There is a prolonged fetal heart rate deceleration to 80/min. Which of the following is the most appropriate action to reverse this patient's symptoms in the future? KEY INFO ATTENDING TIP LA

This patient's symptoms are consistent with supine hypotensive syndrome, which frequently occurs due to compression of the inferior vena cava associated with the anatomical changes involved in pregnancy. Supine hypotensive syndrome A pregnancy complication caused by compression of the inferior vena cava (less commonly also the abdominal aorta) by the enlarged uterus in women lying in a supine position. Can occur after approx. 20 weeks' gestation and presents with symptoms of reduced preload/hypovolemic shock and fetal bradycardia. Placing the mother in the left lateral position usually alleviates symptoms. Lying in the left lateral decubitus position 74% The left lateral decubitus position is generally recommended for pregnant women when lying down, especially in the third trimester, because this position shifts the uterus away from and thus avoids compression of the inferior vena cava. Compression of the inferior vena cava is the cause of supine hypotensive syndrome, which results in compromised venous return and leads to decreased cardiac output, fetal hypoxia, and prolonged decelerations on cardiotocography, as seen here.

A 32-year-old woman comes to the physician because of a 4-day history of headache, depressed mood, insomnia due to vivid dreams, and fatigue. Since yesterday, she has also had nausea and has noticed an unusual "electric shock-like" sensation on her skin. She has had major depression for 6 years and multiple sclerosis for 4 years. Her medications are natalizumab and paroxetine, which she stopped taking 1 week ago after having a positive pregnancy test. She appears irritable. Vital signs are within normal limits. Examination shows flat affect, gait instability, tremor, and muscle pain. Muscle strength is normal. An MRI of the brain shows multiple old periventricular white matter lesions. Which of the following is the most likely explanation for the patient's current symptoms?

This patient's symptoms are most likely related to her stopping her medications. Antidepressant discontinuation syndrome 66% Abrupt cessation of antidepressant drugs without appropriate tapering, as seen here, can lead to adverse effects in as few as 1-4 days. This patient's presentation (i.e., headache, depressed mood, fatigue, insomnia, nausea, sensory disturbances, irritability, ataxia, tremor, and myalgia) is consistent with antidepressant discontinuation syndrome (ADS), a known side effect of sudden discontinuation of SSRIs. Paroxetine, in particular, is associated with an increased risk of ADS. Treatment consists of restarting antidepressant therapy at the original dose before beginning to taper slowly for at least 2-4 weeks. The main clinical features of antidepressant discontinuation syndrome can be remembered with the mnemonic FINISH: Flu-like symptoms, Insomnia, Nausea, Instability, Sensory disturbances, and Hyperarousal.

An 8-year-old girl is brought to the physician by her parents for a 10-month history of disturbing dreams and daytime sleepiness. She has difficulty falling asleep and says she sometimes sees ghosts just before falling asleep at night. She has had a 7-kg (15-lb) weight gain during this period despite no changes in appetite. She is alert and oriented, and neurologic examination is unremarkable. During physical examination, she spontaneously collapses after the physician drops a heavy book, producing a loud noise. She remains conscious after the collapse. Polysomnography with electroencephalogram is most likely to show which of the following?

This patient's symptoms of visual hypnagogic hallucinations (hallucinations while falling asleep), excessive daytime sleepiness, abrupt loss of postural tone in response to a frightening stimulus (cataplexy), and weight gain for over 10 months make narcolepsy the most likely diagnosis. Cataplexy is a sudden and transient episode of muscle weakness accompanied by full conscious awareness, typically triggered by emotions such as laughing, crying, or terror.[1] Cataplexy affects approximately 70% of people who have narcolepsy,[2] and is caused by an autoimmune destruction of hypothalamic neurons that produce the neuropeptide hypocretin (also called orexin), which regulates arousal and has a role in stabilization of the transition between wake and sleep states. Rapid onset of beta waves 47% EEG beta waves are seen during the wakeful state and during REM sleep. The diagnosis of narcolepsy can be confirmed by the rapid onset of beta waves (i.e., shortened REM sleep latency) during polysomnography or by decreased levels of hypocretin in CSF. Narcolepsy is treated with good sleep hygiene measures, daytime stimulant therapy (e.g., amphetamines, modafinil), and nighttime sodium oxybate therapy. REM sleep Rapid eye movement sleep A stage of deep sleep that lasts 10-30 minutes, characterized by predominant beta waves on EEG. During REM sleep there is a high waking threshold, maximum muscle relaxation with simultaneously rapid eye movement (REM), activation of autonomic functions, and high dream activity. Narcolepsy A neurological disorder that disrupts the circadian sleep-wake rhythm. Often begins in adolescence with short diurnal sleep attacks. Primary narcolepsy is either due to orexin deficiency (type 1) or idiopathic (type 2). Secondary narcolepsy may occur as a result of metabolic or structural brain damage or as a symptom of other genetic syndromes. The characteristic symptoms of narcolepsy are excessive daytime sleepiness, cataplexy, hallucinations upon waking or falling asleep, and sleep paralysis. Rapid onset of beta waves 47% EEG beta waves are seen during the wakeful state and during REM sleep. The diagnosis of narcolepsy can be confirmed by the rapid onset of beta waves (i.e., shortened REM sleep latency) during polysomnography or by decreased levels of hypocretin in CSF. Narcolepsy is treated with good sleep hygiene measures, daytime stimulant therapy (e.g., amphetamines, modafinil), and nighttime sodium oxybate therapy Sleep hygiene A set of practices and habits that are conducive to good nighttime sleep. Includes avoiding stimulants close to bedtime, limiting naps to 30 minutes, aerobic exercise during the day, avoiding bright lights before bedtime, establishing a bedtime routine, and going to sleep the same time each night. Sodium oxybate A sodium salt of gamma hydroxybutyric acid used to treat narcolepsy.

A 23-year-old man comes to the physician because of a 1-week history of anorexia, nausea, upper abdominal pain, and dark-colored urine. He recently returned from a trip to Belize. His temperature is 38.3°C (100.9°F). Physical examination shows scleral icterus and tender hepatomegaly. Serum studies show markedly elevated transaminases and anti-hepatitis A virus antibodies. In this patient, infected cells express more MHC class I molecules on their surfaces. This cellular event is primarily induced by a molecule that is also responsible for which of the following changes?

This patient's symptoms, together with his recent travel history, the findings of the physical examination, and serum studies (especially anti-HAV antibodies) suggest acute infection with hepatitis A virus (HAV). Virus-infected cells produce Interferon α and Interferon β, which upregulate MHC I expression to facilitate the recognition and destruction of infected cells. They also downregulate protein synthesis to prevent viral replication. Increased breakdown of intracellular RNA Besides promoting the expression of MHC class I proteins on the surface of virus-infected cells, interferon α and interferon β induce the activation of various enzymes that promote the antiviral state of the cell, including ribonuclease L. Ribonuclease L is an enzyme that mediates the degradation of cellular and viral RNA and thereby inhibits viral protein synthesis. The enzyme is only activated upon binding to double-stranded RNA, which ensures that breakdown of intracellular RNA occurs preferentially in virus-infected cells.

A 43-year-old man comes to the emergency department with nausea, abdominal discomfort, diarrhea, and progressive perioral numbness for the past 24 hours. 3 days ago, he underwent a total thyroidectomy for treatment of papillary thyroid cancer. His only medication is a multivitamin supplement. He appears fatigued. While measuring the patient's blood pressure, the nurse observes a spasm in the patient's hand. Physical examination shows a well-healing surgical wound on the neck. Which of the following ECG findings are most likely in this patient?

This patient's tetany (Trousseau sign), paresthesias (perioral tingling), abdominal discomfort, and diarrhea indicate acute hypocalcemia. Because these symptoms have appeared 2 days after thyroidectomy, they are most likely due to iatrogenic hypoparathyroidism. QT prolongation 57% QT prolongation is seen in hypocalcemia due to slower calcium influx during phase 2 of the myocardial action potential and lengthening of the ST segment, whereas the T wave remains unaffected. Hypoparathyroidism, caused by trauma during thyroidectomy or parathyroidectomy, is the most common cause of hypocalcemia. Other, less common causes include vitamin D deficiency, PTH resistance, increased phosphate binding (rhabdomyolysis or tumor lysis syndrome), and hypomagnesemia.

A previously healthy 29-year-old man comes to the emergency department because of burning with urination for several days. He has also had pain in the right ankle for 3 days and pain and swelling in the left knee for 1 day. Two weeks ago, he had several days of fever and bloody diarrhea, for which he was treated with antibiotics. Examination shows a small left knee effusion and bilateral conjunctival injection. Which of the following is the most likely additional finding in this patient?

This patient's urethritis (dysuria), asymmetric oligoarticular arthritis, and conjunctivitis comprise the classic triad of reactive arthritis, a condition commonly triggered by diarrheal illness. Reactive arthritis Reiter syndrome An immune-mediated condition that typically occurs 1-4 weeks after a bacterial infection of the gastrointestinal or genitourinary tract. Particularly common after infection with Shigella, Yersinia, Salmonella, Campylobacter, or Chlamydia. The characteristic triad of symptoms includes arthritis, conjunctivitis, and urethritis. Tenderness at the insertion of the Achilles tendon Enthesitis causes tenderness at the insertion of the Achilles tendon, another common manifestation of reactive arthritis. Other features of reactive arthritis include sacroiliitis, dactylitis, oral lesions, and keratoderma blennorrhagicum. This patient likely had an acute gastrointestinal infection due to Shigella, Yersinia, Salmonella, or Campylobacter. Patients with reactive arthritis following an enteric infection can develop aseptic urethritis, which is likely the case in this patient. Enthesitis An inflammation of the enthesis (the point where a tendon attaches to a bone). Typically seen in patients who are HLA-B27 positive (e.g., ankylosing spondylitis, psoriatic arthritis, enthesitis-associated juvenile idiopathic arthritis) as a manifestation of reactive arthritis. Commonly affected sites are the spine, Achilles tendon, supraspinatus tendon, and iliac crest.

An investigator is studying biomolecular mechanisms in human cells. A radioactive isotope that is unable to cross into organelles is introduced into a sample of cells. The cells are then fragmented via centrifugation and the isotope-containing components are isolated. Which of the following reactions is most likely to be present in this cell component?

This reaction takes place in the cytoplasm. Glucose-6-phosphate to 6-phosphogluconolactone 32% The formation of 6-phosphogluconolactone from glucose-6-phosphate by the enzyme glucose-6-phosphate dehydrogenase is the first step in the HMP shunt. HMP shunt reactions occur exclusively in the cytoplasm. Other metabolic processes that occur exclusively in the cytoplasm are glycolysis, nucleotide synthesis, nonsecretory protein synthesis, and fatty acid synthesis. The synthesis of secretory proteins occurs in the rough endoplasmic reticulum and the synthesis of cholesterol and steroids occurs in the smooth endoplasmic reticulum. Fatty acid beta-oxidation, Krebs cycle, oxidative phosphorylation, and ketogenesis are processes that occur exclusively in the mitochondria. Certain metabolic processes such as heme production and the urea cycle occur in both the mitochondria and cytoplasm. Gluconeogenesis is a process that occurs in the cytoplasm, mitochondria, and smooth endoplasmic reticulum. Hexose monophosphate shunt Pentose phosphate pathway A cytoplasmic metabolic pathway that degrades abundant glucose-6-phosphate, thereby providing NADPH as well as ribose for nucleotide synthesis. Consists of an oxidative and nonoxidative phase.

A 45-year-old woman comes to the physician because of fatigue, abdominal cramps, watery diarrhea, and a weight loss of 4 kg (8.8 lb) over the last 4 months. She has recently avoided drinking alcohol and eating spicy food because it worsens her diarrhea and causes episodes of heart palpitations and reddening of the face and neck. She takes lisinopril for hypertension. Her temperature is 36.5°C (97.7°F), pulse is 98/min, and blood pressure is 149/90 mm Hg. The abdomen is soft, and there is mild tenderness to palpation with no guarding or rebound. Laboratory studies show an increased urine 5-hydroxyindoleacetic acid concentration. Further evaluation of this patient will most likely show which of the following?

This woman has carcinoid syndrome, which manifests with abdominal cramps, diarrhea, and episodic cutaneous flushing and palpitations triggered by food ingestion. The diagnosis is confirmed by laboratory testing that shows an increased urine concentration of 5-hydroxyindoleacetic acid, which is a product of serotonin metabolism. Pulmonic valve stenosis 57% Pulmonic valve stenosis is a manifestation of carcinoid heart disease, a condition that eventually affects > 50% of patients with carcinoid syndrome. Carcinoid heart disease typically develops as a result of chronic exposure of the heart to serotonin from liver metastases. Serotonin enters the inferior vena cava and reaches the right side of the heart, where it induces valvular and myocardial fibrosis. This can lead to pulmonary valve stenosis, tricuspid insufficiency, and right-sided heart failure. Carcinoid syndrome A condition caused by serotonin secretion from extraintestinal neuroendocrine tumors. Causes diarrhea, flushing, and wheezing.

A 24-year-old woman comes to the physician because of bothersome hair growth on her face and abdomen over the past 8 years. She does not take any medications. She is 163 cm (5 ft 4 in) tall and weighs 85 kg (187 lb); BMI is 32 kg/m2. Physical examination shows coarse dark hair on the upper lip and periumbilical and periareolar skin. Her external genitalia appear normal. Her serum follicle-stimulating hormone, luteinizing hormone, and testosterone are within the reference range. A urine pregnancy test is negative. Which of the following is the most appropriate pharmacotherapy for this patient's condition at this time?

This woman presents with slow progression of excessive male pattern hair growth (hirsutism) with normal gonadotropins and testosterone levels without signs of further virilization. This combination is highly suggestive of idiopathic hirsutism (IH). Oral contraceptive 52% Oral contraceptives are recommended as the first-line therapy for women with hirsutism due to their ability to reduce circulating androgen levels. Estrogen-progestin contraceptives reduce androgen levels by suppression of circulating luteinizing hormone (LH) and stimulation of sex hormone-binding globulin (SHBG) levels. Leuprolide 5% Leuprolide, a gonadotropin-releasing hormone receptor agonist (GnRH agonist), inhibits ovarian androgen secretion and can therefore suppress male pattern hair growth. However, this therapy is not considered first-line management of idiopathic hirsutism and has a risk of estrogen deficiency.

A 28-year-old man comes to the emergency department because of an excruciating headache that started 30 minutes ago. The pain is located around the right eye and it awoke him from sleep. Over the past two weeks, he has had similar headaches around the same time of the day, and he reports pacing around restlessly during these episodes. Physical examination shows conjunctival injection and tearing of the right eye. In addition to supplemental oxygen therapy, administration of which of the following drugs is most likely to provide acute relief in this patient?

This young man has recurrent attacks of severe unilateral headache that are associated with ipsilateral cranial autonomic symptoms (conjunctival injection, lacrimation), which is indicative of cluster headache. Sumatriptan 73% Triptans like sumatriptan are the first-line pharmacological treatment for acute attacks of cluster headache. They abort the headache and provide resolution of the conjunctival injection associated with these attacks. The exact mechanism is unclear, but it is thought to involve the vasoconstriction of cranial blood vessels in response to stimulation of 5-hydroxytryptamine (5-HT) receptors. Inhalation of 100% oxygen is also effective in terminating acute episodes of cluster headache and has a rapid onset of action. Prednisone 4% High-dose steroid therapy is the treatment of choice for giant cell arteritis (GCA), which can present with severe unilateral headache, as seen in this patient. However, autonomic symptoms do not occur, and symptoms such as jaw claudication and scalp tenderness, which are not seen in this patient, are usually present. More importantly, GCA usually occurs in elderly individuals (peak incidence: 70-79 years) and is rare in patients younger than 50.

A 23-year-old man comes to the physician for frequent and painful urination. He has also had progressive painful swelling of his right knee over the past week. He is sexually active with two female partners and uses condoms inconsistently. His mother has an autoimmune disease that involves a malar rash. Examination shows conjunctivitis bilaterally. The right knee is warm, erythematous, and tender to touch; range of motion is limited. Laboratory studies show an erythrocyte sedimentation rate of 62 mm/h. Urinalysis shows WBCs. Further evaluation of this patient is most likely to reveal which of the following?

This young patient has a triad of arthritis, urethritis, and conjunctivitis, which is highly suggestive of reactive arthritis. HLA-B27 positive genotype 73% Reactive arthritis occurs in genetically susceptible individuals in response to an infection with Chlamydia, Shigella, or Campylobacter, among other organisms. An HLA-B27 genotype is present in over two-thirds of patients with reactive arthritis. Those with reactive arthritis and a positive HLA-B27 genotype should be screened for other diseases associated with the same genotype, such as ankylosing spondylitis and acute anterior uveitis. Given this patient's sexual history, he should also be screened for HIV, which is associated with a higher risk of reactive arthritis.

A previously healthy 42-year-old woman comes to the physician because of a 7-month history of diffuse weakness. There is no cervical or axillary lymphadenopathy. Cardiopulmonary and abdominal examination shows no abnormalities. A lateral x-ray of the chest shows an anterior mediastinal mass. Further evaluation of this patient is most likely to show which of the following?

Thymoma is the most common type of anterior mediastinal mass. Acetylcholine receptor antibodies 74% Acetylcholine receptor antibody detection is used to diagnose myasthenia gravis, which is present in about 30% of patients with thymoma and may present with diffuse weakness. Thymic lesions, including thymoma, are the most common underlying cause of anterior mediastinal masses, accounting for ∼ 50% of cases. The peak incidence of thymomas is between 40-60 years of age. Patients with thymoma may be asymptomatic, or they may present with chest pain, dyspnea, and cough.

A 55-year-old man with a history of repeated hospitalization for chronic pancreatitis comes to the physician because of difficulty walking and standing steadily. Neurological examination shows an unsteady, broad-based gait, distal muscle weakness, decreased deep tendon reflexes, and an abnormal Romberg test. His hemoglobin concentration is 11.9 g/dL, mean corpuscular volume is 89/μm3, and serum lactate dehydrogenase is 215 U/L. His serum haptoglobin is slightly decreased. A deficiency of which of the following substances is the most likely cause of this patient's findings?

Tocopherol 44% Tocopherol (vitamin E) deficiency classically presents with hemolytic anemia and symptoms of posterior column and spinocerebellar tract dysfunction (ataxia, loss of proprioception, and diminished reflexes). It is an uncommon condition in developed countries, but specific populations, such as patients with fat malabsorption disorders (e.g., chronic pancreatitis, abetalipoproteinemia, children with cystic fibrosis) are at significant risk. Vitamin E deficiency is harmful to neurons and red blood cells specifically as they are particularly sensitive to the oxidative damage that results from the loss of vitamin E antioxidative effects. Folate 5% Folate (vitamin B9) deficiency typically occurs in individuals with a history of excessive alcohol use or patients taking folate antagonists such as sulfonamides or methotrexate. However, glossitis is the main clinical finding, whereas neurological symptoms, as seen in this patient, are not typically present. Also, patients with folate deficiency have megaloblastic anemia, not the hemolytic anemia seen in this patient. Cobalamin 27% Cobalamin (vitamin B12) deficiency causes subacute combined degeneration, which may manifest with neurological signs and symptoms similar to the ones seen in this patient, including ataxia and muscle weakness. However, cobalamin deficiency typically also manifests with symmetrical paresthesias of the extremities and dementia. In addition, it causes megaloblastic anemia. Finally, common risk factors for this vitamin deficiency include restricted diets, lack of intrinsic factor, or terminal ileum disease (e.g., Crohn disease), none of which are present in this patient.

mechanism of cyanide poisoning cyanide binds ferric ion (Fe3+) in the mitochondrial cytochrome complex IV, which inhibits oxidative phosphorylation this causes the cell to convert to anaerobic metabolism, causing accumulation of lactic acid and a metabolic acidosis results in functional hypoxia, as cells, cannot use oxygen Physical exam cardiovascular initially tachycardic and hypertensive then bradycardic and hypotensive almond breath odor pulmonary edema flushing "cherry-red" skin due to high venous oxyhemoglobin concentration cyanosis irritant dermatitis if skin is exposed to cyanide nausea and vomiting if cyanide is ingested

Treatment Management approach supplemental oxygen is not useful in this case all contaminated clothes should be removed all wounds from cyanide exposure should be cleaned antidotes bind to cyanide, induce methemoglobinemia (which provides an alternative binding site for cyanide), or act as sulfur donors (which detoxifies cyanide and transforms it to thiocyanate) First-line hydroxocobalamin, a precursor to vitamin B12 combines with cyanide to form cyanocobalamin, which is renally excreted thiosulfate thiosulfates convert cyanide to thiocyanide, which can be renally excreted Second-line nitrates nitrates convert hemoglobin to methemoglobin, which will bind to cyanide contraindicated in those with concomitant carbon monoxide toxicity Other treatments activated charcoal

A 24-year-old woman in graduate school comes to the physician for recurrent headaches. The headaches are unilateral, throbbing, and usually preceded by blurring of vision. The symptoms last between 12 and 48 hours and are only relieved by lying down in a dark room. She has approximately two headaches per month and has missed several days of class because of the symptoms. Physical examination is unremarkable. The patient is prescribed an abortive therapy that acts by inducing cerebral vasoconstriction. Which of the following is the most likely mechanism of action of this drug?

Triptans are the standard abortive therapy for primary migraines. Activation of 5-HT1 receptors 72% Selective activation of 5-HT1 receptors on cerebral blood vessels and sensory fibers of the trigeminal nerve is the mechanism of action of triptans, which are considered first-line abortive therapy for the treatment of acute migraine headaches and cluster headache attacks. Triptans induce vasoconstriction by inhibiting the release of vasoactive peptide and reduce neurogenic inflammation associated with migraine. They are especially effective if taken early at the onset of an attack. Adverse effects of triptans include coronary vasospasm, serotonin syndrome, and paresthesias. Inhibition of 5-HT and NE reuptake 11% Inhibition of 5-HT and norepinephrine (NE) reuptake is the mechanism of action of amitriptyline, a tricyclic antidepressant used for the prophylaxis of migraine headaches. Vasoconstriction has not been identified as one of amitriptyline's effects and it is not used as an abortive therapy. Inhibition of β1- and β2-adrenergic receptors 8% Inhibition of β1 receptors and β2 receptors is the mechanism of action of non-selective β-blockers (e.g., propranolol), which can be used for the long-term prophylaxis of migraine headaches. β-blockers inhibit norepinephrine release, which stabilizes neuronal membranes, reducing their activity and excitability. However, vasoconstriction of intracranial vessels has not been described among their effects in the treatment of migraine.

A 2720-g (6-lb) female newborn is delivered at term to a 39-year-old woman, gravida 3, para 2. Examination in the delivery room shows micrognathia, prominent occiput with flattened nasal bridge, and pointy low-set ears. The eyes are upward slanting with small palpebral fissures. The fists are clenched with fingers tightly flexed. The index finger overlaps the third finger and the fifth finger overlaps the fourth. A 3/6 holosystolic murmur is heard at the lower left sternal border. The nipples are widely spaced and the feet have prominent heels and convex, rounded soles. Which of the following is the most likely cause of these findings?

Trisomy 18 70% This infant's features are consistent with trisomy 18 (Edwards syndrome). Classic features include a characteristic craniofacial morphology (micrognathia, prominent occiput, and pointy, low-set ears), clenched fists with flexion contractures of the fingers, and rocker-bottom feet (convex foot deformity with prominent heel). The infant also presents with other features that are commonly associated with the syndrome, such as a congenital heart defect (holosystolic murmur suggestive of VSD) and low birth weight (suggestive of intrauterine growth retardation). Second trimester quadruple screening test would have shown ↓ free estriol, ↓ AFP, ↓ Inhibin A, and ↓ β-HCG. Unconjugated estriol Free estriol Abbreviation: uE3 A weak form of estrogen. During pregnancy, estriol is the major estrogen because fetal dehydroepiandrosterone enters maternal serum and is converted by the placenta to unconjugated estriol, which is then rapidly conjugated by the liver to a water-soluble form that can be excreted in urine. Decreased levels of unconjugated estriol are associated with Down syndrome, Edward syndrome, molar pregnancy, and fetal demise. Trisomy 21 10% Trisomy 21 (Down syndrome) is the most common chromosomal disorder seen in live births. Infants with trisomy 21 are usually diagnosed based on a combination of classic clinical features including: brachycephaly, dysmorphic facial features (e.g., epicanthus, protuberant tongue, flattened nasal bridge, hypertelorism), a single transverse palmar crease, and a sandal gap deformity. This child does exhibit some features of Down syndrome, (e.g., flattened nasal bridge, upward slanting eyes with small palpebral fissures, and presence of VSD), but the absence of all other typical features as well as the presence of additional features (micrognathia, rocker-bottom feet) makes this diagnosis highly unlikely.

A 34-year-old man comes to the physician because of palpitations, shortness of breath, diarrhea, and abdominal cramps for 2 months. Physical examination shows cutaneous flushing of the face. Auscultation of the chest shows bilateral wheezing. A 24-hour urine collection shows increased 5-hydroxyindoleacetic acid (5-HIAA) concentration. A contrast-enhanced CT scan of the abdomen shows an intestinal tumor with extensive metastasis to the liver. A diagnosis of an inoperable disease is made and the patient is started on treatment with octreotide. Six weeks later, the patient's symptoms have improved except for his abdominal pain and frequent loose stools. The physician suggests enrolling the patient in a trial to test additional treatment with a new drug that has been shown to improve symptoms in other patients with the same condition. The expected beneficial effect of this new drug is most likely caused by inhibition of which of the following? Palpitations, dyspnea with wheezing, flushing, and diarrhea in a patient with a metastatic intestinal tumor and increased urinary 5-HIAA is suggestive of carcinoid syndrome. Telotristat is used as adjunctive therapy in patients with carcinoid tumors.

Tryptophan hydroxylase is an enzyme that converts tryptophan to serotonin. The hallmark of carcinoid tumors is the excess production of serotonin, which is primarily responsible for the gastrointestinal symptoms (e.g., diarrhea) in carcinoid syndrome. Telotristat, an inhibitor of tryptophan hydroxylase, has recently been approved as adjunctive therapy for patients with carcinoid tumors whose diarrhea is not controlled with somatostatin analogs (e.g., octreotide

Oligodendrocytes

Type of glial cell in the CNS that wrap axons in a myelin sheath.

A 55-year-old woman comes to the physician because of a 2-week history of painful swelling on the right side of her face. The pain worsens when she eats. Examination of the face shows a right-sided, firm swelling that is tender to palpation. Oral examination shows no abnormalities. Ultrasonography shows a stone located in a duct that runs anterior to the masseter muscle and passes through the buccinator muscle. Sialoendoscopy is performed to remove the stone. At which of the following sites is the endoscope most likely to be inserted during the procedure?

Ultrasonography shows that the stone is located in the parotid duct. Lateral to the second upper molar tooth 67% The parotid duct emerges from the anterior edge of the parotid gland, turns medially at the anterior edge of the masseter muscle, and, after piercing through the buccinator muscle, enters the oral cavity lateral to the second upper molar. Sialolithiasis, which affects the parotid gland or duct in approx. 20% of cases, typically manifests with acute pain while eating and tender swelling of the affected gland, both of which are seen here. Risk factors include dehydration, certain medications (e.g., anticholinergics), and trauma. Sialolithiasis is usually treated conservatively, e.g., with NSAIDs and stimulation of salivary flow by gland massage or warm compresses. In severe cases or if conservative treatment fails, salivary stones can be removed via sialoendoscopy or open surgical procedures. Sialolithiasis Salivary stones The formation of concrements in the salivary ducts or parenchyma of the glands, most commonly in the submandibular duct. Patients usually present with pain and swelling in the affected gland, typically right before and during eating. More often affects men.

An investigator studying hormone synthesis and transport uses immunocytochemical techniques to localize a carrier protein in the central nervous system of an experimental animal. The investigator finds that this protein is synthesized together with a specific hormone from a composite precursor. The protein is involved in the transport of the hormone from the supraoptic and paraventricular nuclei to its destination. The hormone transported by these carrier proteins is most likely responsible for which of the following functions?

Upregulation of renal aquaporin-2 channels 79% Neurophysins are carrier proteins that transport the hormones oxytocin and vasopressin to the posterior pituitary gland from the paraventricular nucleus and supraoptic nucleus of the hypothalamus. Vasopressin, also named antidiuretic hormone (ADH), is released into circulation from the posterior pituitary gland in response to an elevated plasma osmolality and has two primary functions. First, it increases the amount of water reabsorbed via upregulation of renal aquaporin-2 channels (V2-receptors). Second, vasopressin constricts arterioles, increasing peripheral vascular resistance (V1-receptors). The hormone is released from the posterior pituitary gland in response to serum hypertonicity.

Bradykinin

a powerful vasodilator that increases capillary permeability and constricts smooth muscle Bradykinin An inflammatory mediator that causes increased vascular permeability, vascular dilation, bronchoconstriction, and pain.

A 23-year-old man comes to the emergency department because of palpitations, dizziness, and substernal chest pain for three hours. The day prior, he was at a friend's wedding, where he consumed seven glasses of wine. The patient appears diaphoretic. His pulse is 220/min and blood pressure is 120/84 mm Hg. Based on the patient's findings on electrocardiography, the physician diagnoses atrial fibrillation with rapid ventricular response and administers verapamil for rate control. Ten minutes later, the patient is unresponsive and loses consciousness. Despite resuscitative efforts, the patient dies. Histopathologic examination of the heart at autopsy shows an accessory atrioventricular conduction pathway. Electrocardiography prior to the onset of this patient's symptoms would most likely have shown which of the following findings?

Wolff-Parkinson-White syndrome Abbreviation: WPW A congenital condition characterized by intermittent tachycardia and a ventricular pre-excitation pattern on ECG, which both arise from an accessory pathway, known as the "bundle of Kent," that connects the atria and ventricles, bypassing the AV node. Typical resting ECG findings include a shortened PR interval and a "slurred" upstroke in the QRS complex, called a "delta wave." Slurred upstroke of the QRS complex 76% A slurred upstroke of the QRS complex (i.e., delta wave) and shortened PR interval are the characteristic ECG findings of WPW syndrome. These findings are caused by an accessory atrioventricular pathway (i.e., bundle of Kent) that bypasses the AV node. Atrial fibrillation occurs in about 20% of patients with WPW syndrome and should be treated with rhythm control agents (e.g., procainamide). AV nodal blocking agents (e.g., adenosine, beta-blockers, nondihydropyridine calcium channel blockers such as verapamil) are contraindicated since they increase conduction of atrial impulses through the accessory pathway. This pathway has a short refractory period, allowing frequent conduction to the ventricles and induction of fatal arrhythmias (e.g., ventricular fibrillation), which has occurred here. QRS complex Abbreviation: QRS A component of the ECG waveform that represents the electrical conduction in the ventricles and corresponds approximately to ventricular systole.

Nitroblue tetrazolium test

a qualitative test to determine defects in NADPH oxidase; employed in detection of chronic granulomatous disease

saccadic eye movement

a rapid, jerky movement from one fixation to the next

area postrema

a brainstem area, in which the blood-brain barrier is more permeable, that triggers vomiting in response to the detection of circulating toxins Damage to the area postrema, caused primarily by lesioning or ablation, prevents the normal functions of the area postrema from taking place. This ablation is usually done surgically and for the purpose of discovering the exact effect of the area postrema on the rest of the body. Since the area postrema acts as an entry point to the brain for information from the sensory neurons of the stomach, intestines, liver, kidneys, heart, and other internal organs, a variety of physiological reflexes rely on the area postrema to transfer information. The area postrema acts to directly monitor the chemical status of the organism. Lesions of the area postrema are sometimes referred to as 'central vagotomy' because they eliminate the brain's ability to monitor the physiological status of the body through its vagus nerve.[12] These lesions thus serve to prevent the detection of poisons and consequently prevent the body's natural defenses from kicking in. In one example, experiments done by Bernstein et al. on rats indicated that the area postrema lesions prevented the detection of lithium chloride, which can become toxic at high concentrations. Since the rats could not detect the chemical, they were not able to employ a psychological procedure known as taste aversion conditioning, causing the rat to continuously ingest the lithium-paired saccharin solution. These findings indicate that rats with area postrema lesions do not acquire the normal conditioned taste aversions when lithium chloride is used as the unconditioned stimulus.

Triptans

a class of drugs commonly used to treat migraines, they work by causing dilated (enlarged) blood vessels to constrict (become smaller) Triptans are used for the treatment of severe migraine attacks or those that do not respond to NSAIDs[6] or other over-the-counter drugs.[7] Triptans are a mid-line treatment suit Triptans are selective 5-hydroxytryptamine (5-HT) receptor agonists with high affinity for 5-HT1B and 5-HT1D receptors. Stimulation of the 5-HT1B receptors on smooth muscle cells of blood vessels causes cranial vasoconstriction. Triptans are the standard abortive therapy for primary migraines.

Multiple Endocrine Neoplasia

a group of disorders characterized by functioning tumors in more than one endocrine gland problems with not using MEDMOS. The term multiple endocrine neoplasia encompasses several distinct syndromes featuring tumors of endocrine glands, each with its own characteristic pattern.

megakaryocyte

a large bone marrow cell from which platelets are derived A megakaryocyte (mega- + karyo- + -cyte, "large-nucleus cell") is a large bone marrow cell with a lobated nucleus responsible for the production of blood thrombocytes (platelets), which are necessary for normal blood clotting.

myeloperoxidase

a lysosomal enzyme that combines peroxide with Cl to produce bleach (HOCl)

abductor pollicis longus

abducts and extends thumb In human anatomy, the abductor pollicis longus (APL) is one of the extrinsic muscles of the hand. As the name implies, its major function is to abduct the thumb at the wrist. Its tendon forms the anterior border of the anatomical snuffbox.

anosmia

absence of the sense of smell

zona reticularis

androgens

diabetes insipidus

antidiuretic hormone is not secreted adequately, or the kidney is resistant to its effect

Non-caseating granulomas

associated with Crohn's disease. Other classic findings: serpinginous ulceration, cobblestone appearance of mucosa, and transmural inflammatory infiltrate - leads to narrowing of involved segment (causes the classic "string sign" on barium studies). These strictures can lead to intestinal obstruction.

pancytopenia

deficiency of all types of blood cells Pancytopenia is a condition that occurs when a person has low counts for all three types of blood cells: red blood cells, white blood cells, and platelets. Pancytopenia is usually due to a problem with the bone marrow that produces the blood cells. However, there can be several different underlying causes.

lid lag

hyperthyoidism The failure of the normal downward following movement of the upper eyelids on looking downward

groin

inguinal region

median nerve

innervates pronator teres

Putamen

large subcortical structure, part of the basal ganglia The putamen is a large structure located within the brain. It is involved in a very complex feedback loop that prepares and aids in movement of the limbs.

supine position

lying on back, facing upward

Subscapularis

medially rotates arm Lift off test

dysarthic speech

motor speech disorder. The muscles of the mouth, face, and respiratory system may become weak, slow, or paralyzed after brain injury; slurred, soft, hoarse, rapid "mumble" quality, limited movement, abnormal intonation, changing vocals, breathiness

nuchal rigidity

neck stiffness Nuchal rigidity is an inability to flex the neck forward due to rigidity of the neck muscles

allogenic

originating within another

diaphoresis

profuse sweating

gliosis

prolifertion/hypertrophy of the glial cells reaction to CNS injury astrocytes undergo major changes

excercise stress test

study that evaluates cardiac function during physical stress by riding a bike or walking on a treadmill

Kulchitsky cells

small cell carcinoma

C fibers

small nerve fibers, poorly myelinated or unmyelinated. slow conduction rate. post ganglionic fibers of sympathetic system. exteroceptors for pain, temp, and touch.

petechiae

small, pinpoint hemorrhages

A 19-year-old girl comes to the physician for evaluation after a minor motor vehicle collision. While driving down a residential street, a young boy ran out in front of her, chasing after a ball. She applied the brakes of her vehicle and avoided hitting the boy, but then she suddenly experienced generalized weakness that rendered her unable to operate the vehicle and collided at low speed with a parked car. One minute later, she recovered her strength. She was uninjured. She has had several similar episodes of transient generalized weakness over the past month, once during an argument with her mother and another time while watching her favorite comedy movie. She has also had excessive daytime sleepiness for 18 months despite 9 hours of sleep nightly and 2 daily naps. She has fallen asleep in class several times. She often sees intensely bright colors as she is falling asleep. During this time, she is often unable to move; this inability to move is very distressing to her. Which of the following is the most appropriate nighttime pharmacotherapy for this patient?

sodium oxybate 44% Sodium oxybate is the preferred drug for severe cataplexy in narcolepsy. It lowers the number of cataplexy attacks and improves nighttime sleep, thereby reducing excessive daytime sleepiness. Considering it is a powerful, rapidly acting sedative, sodium oxybate is taken at night. Caution should be exercised in the prescription of sodium oxybate, as it has a high potential for abuse. Overdose can cause respiratory depression, coma, and death. A symptom of narcolepsy present in 80% of cases. Characterized by sudden muscle weakness in a fully conscious patient that are triggered by strong emotions (e.g., patients may experience buckling of the knees upon laughing). Self-resolves in < 2 minutes. Amphetamine 4% Amphetamine is a recommended treatment option for patients with narcolepsy who have persistent excessive daytime sleepiness despite lifestyle modifications (e.g., > 7.5 hours of sleep per night and scheduled daytime naps). However, amphetamines do not treat cataplexy, which is this patient's most disabling symptom. In addition, amphetamines are stimulants and should not be taken at night.

episiotomy

surgical incision of the perineum to enlarge the vagina and so facilitate delivery during childbirth

Pacinian corpuscles

vibration, pressure receptors located in deep skin joints and ligamenys egg shaped A alpha large myelinated fibers


Conjuntos de estudio relacionados

MGMT 351 Chapter 4 Organizational Environment

View Set

chapter 6 organization and management

View Set

Fluid Volume Deficit: Hypovolemia

View Set

Research Methods: Ch's 4-6, & 15

View Set

Health Assessment in Nursing, chapter 1

View Set